Site Loader

Π‘ΠΎΠ΄Π΅Ρ€ΠΆΠ°Π½ΠΈΠ΅

ΠŸΠΎΡ‚Π΅Π½Ρ†ΠΈΠ°Π» элСктричСского поля — ΠžΡΠ½ΠΎΠ²Ρ‹ элСктроники

Β Β 

Π’ зависимости ΠΎΡ‚ количСства зарядов ΠΈ ΠΈΡ… Π²Π΅Π»ΠΈΡ‡ΠΈΠ½Ρ‹ измСняСтся энСргия элСктричСского поля, создаваСмого этими зарядами. ΠžΡ‡Π΅Π²ΠΈΠ΄Π½ΠΎ, Ρ‡Ρ‚ΠΎ Π²Π΅Π»ΠΈΡ‡ΠΈΠ½Π° энСргии элСктричСского поля, ΠΎΠ±Ρ€Π°Π·ΠΎΠ²Π°Π½Π½ΠΎΠ³ΠΎ ΠΎΠ΄Π½ΠΈΠΌ ‘зарядом, Π±ΡƒΠ΄Π΅Ρ‚ ΠΎΡ‚Π»ΠΈΡ‡Π°Ρ‚ΡŒΡΡ ΠΎΡ‚ Π²Π΅Π»ΠΈΡ‡ΠΈΠ½Ρ‹ энСргии поля, ΠΎΠ±Ρ€Π°Π·ΠΎΠ²Π°Π½Π½ΠΎΠ³ΠΎ двумя ΠΈΠ»ΠΈ трСмя Ρ‚Π°ΠΊΠΈΠΌΠΈ ΠΆΠ΅ зарядами.

Π’ ΠΏΡ€Π°ΠΊΡ‚ΠΈΠΊΠ΅ ΠΎΡ‡Π΅Π½ΡŒ часто приходится ΡΡ€Π°Π²Π½ΠΈΠ²Π°Ρ‚ΡŒ Ρ€Π°Π·Π»ΠΈΡ‡Π½Ρ‹Π΅ ΠΏΠΎ Π²Π΅Π»ΠΈΡ‡ΠΈΠ½Π΅ поля. Π­Ρ‚ΠΎ сравнСниС производится ΠΏΠΎ дСйствиям ΠΏΠΎΠ»Π΅ΠΉ Π½Π° Π΅Π΄ΠΈΠ½ΠΈΡ‡Π½Ρ‹ΠΉ ΠΏΠΎΠ»ΠΎΠΆΠΈΡ‚Π΅Π»ΡŒΠ½Ρ‹ΠΉ заряд (Ρ‚Π°ΠΊ Π½Π°Π·Ρ‹Π²Π°Π΅ΠΌΡ‹ΠΉ ΠΏΡ€ΠΎΠ±Π½Ρ‹ΠΉ заряд). Поясним это.

Β 

ΠžΠΏΡ€Π΅Π΄Π΅Π»Π΅Π½ΠΈΠ΅:Β Β Π•Π΄ΠΈΠ½ΠΈΡ‡Π½Ρ‹ΠΌ называСтся заряд, Π²Π΅Π»ΠΈΡ‡ΠΈΠ½Π° ΠΊΠΎΡ‚ΠΎΡ€ΠΎΠ³ΠΎ Ρ€Π°Π²Π½Π° ΠΎΠ΄Π½ΠΎΠΉ Π΅Π΄ΠΈΠ½ΠΈΡ†Π΅ заряда.

Β 

ΠŸΡƒΡΡ‚ΡŒ, Π½Π°ΠΏΡ€ΠΈΠΌΠ΅Ρ€, ΠΏΠΎΠ»Π΅ ΠΎΠ±Ρ€Π°Π·ΠΎΠ²Π°Π½ΠΎ Π½Π΅ΠΊΠΎΡ‚ΠΎΡ€Ρ‹ΠΌ ΠΏΠΎΠ»ΠΎΠΆΠΈΡ‚Π΅Π»ΡŒΠ½Ρ‹ΠΌ зарядом. Π§Ρ‚ΠΎΠ±Ρ‹ внСсти Π² ΠΊΠ°ΠΊΡƒΡŽ-Ρ‚ΠΎ Ρ‚ΠΎΡ‡ΠΊΡƒ этого поля Π΅Π΄ΠΈΠ½ΠΈΡ‡Π½Ρ‹ΠΉ ΠΏΠΎΠ»ΠΎΠΆΠΈΡ‚Π΅Π»ΡŒΠ½Ρ‹ΠΉ заряд, Π½Π΅ΠΎΠ±Ρ…ΠΎΠ΄ΠΈΠΌΠΎ Π·Π°Ρ‚Ρ€Π°Ρ‚ΠΈΡ‚ΡŒ ΠΎΠΏΡ€Π΅Π΄Π΅Π»Π΅Π½Π½ΡƒΡŽ Ρ€Π°Π±ΠΎΡ‚Ρƒ Π½Π° ΠΏΡ€Π΅ΠΎΠ΄ΠΎΠ»Π΅Π½ΠΈΠ΅ силы отталкивания ΠΌΠ΅ΠΆΠ΄Ρƒ основным ΠΈ Π΅Π΄ΠΈΠ½ΠΈΡ‡Π½Ρ‹ΠΌ зарядами. Π’Π΅Π»ΠΈΡ‡ΠΈΠ½Π° ΠΏΠΎΡ‚Π΅Π½Ρ†ΠΈΠ°Π»ΡŒΠ½ΠΎΠΉ энСргии поля ΠΏΡ€ΠΈ этом возрастаСт.

ΠŸΠΎΠΏΡ€ΠΎΠ±ΡƒΠ΅ΠΌ Ρ‚Π΅ΠΏΠ΅Ρ€ΡŒ внСсти Π΅Π΄ΠΈΠ½ΠΈΡ‡Π½Ρ‹ΠΉ заряд Π² Π΄Ρ€ΡƒΠ³ΠΎΠ΅ ΠΏΠΎΠ»Π΅, ΠΎΠ±Ρ€Π°Π·ΠΎΠ²Π°Π½Π½ΠΎΠ΅ Π² Π΄Π²Π° Ρ€Π°Π·Π° большим элСктричСским зарядом. ΠžΡ‡Π΅Π²ΠΈΠ΄Π½ΠΎ, Ρ‡Ρ‚ΠΎ ΠΏΡ€ΠΈ этом придСтся Π·Π°Ρ‚Ρ€Π°Ρ‚ΠΈΡ‚ΡŒ Π±ΠΎΠ»ΡŒΡˆΡƒΡŽ Ρ€Π°Π±ΠΎΡ‚Ρƒ, Ρ‡Π΅ΠΌ Π² ΠΏΠ΅Ρ€Π²ΠΎΠΌ случаС. Π‘Π»Π΅Π΄ΠΎΠ²Π°Ρ‚Π΅Π»ΡŒΠ½ΠΎ, ΠΈ ΠΏΠΎΡ‚Π΅Π½Ρ†ΠΈΠ°Π»ΡŒΠ½Π°Ρ энСргия поля возрастСт большС, Ρ‡Π΅ΠΌ Π² ΠΏΠ΅Ρ€Π²ΠΎΠΌ случаС.

Π’ элСктротСхникС для характСристики поля вводится ΡΠΏΠ΅Ρ†ΠΈΠ°Π»ΡŒΠ½ΠΎΠ΅ понятиС β€” элСктричСский ΠΏΠΎΡ‚Π΅Π½Ρ†ΠΈΠ°Π».

ΠžΠΏΡ€Π΅Π΄Π΅Π»Π΅Π½ΠΈΠ΅; ЭлСктричСский ΠΏΠΎΡ‚Π΅Π½Ρ†ΠΈΠ°Π» Π½Π΅ΠΊΠΎΡ‚ΠΎΡ€ΠΎΠΉ Ρ‚ΠΎΡ‡ΠΊΠΈ поля числСнно Ρ€Π°Π²Π΅Π½ Ρ€Π°Π±ΠΎΡ‚Π΅, Π·Π°Ρ‚Ρ€Π°Ρ‡ΠΈΠ²Π°Π΅ΠΌΠΎΠΉ ΠΏΡ€ΠΈ внСсСнии Π΅Π΄ΠΈΠ½ΠΈΡ‡Π½ΠΎΠ³ΠΎ ΠΏΠΎΠ»ΠΎΠΆΠΈΡ‚Π΅Π»ΡŒΠ½ΠΎΠ³ΠΎ заряда ΠΈΠ·-Π·Π° ΠΏΡ€Π΅Π΄Π΅Π»ΠΎΠ² поля Π² Π΄Π°Π½Π½ΡƒΡŽ Ρ‚ΠΎΡ‡ΠΊΡƒ.

Π˜Π·ΠΌΠ΅Ρ€ΡΠ΅Ρ‚ΡΡ ΠΏΠΎΡ‚Π΅Π½Ρ†ΠΈΠ°Π» элСктричСского поля Π² Π²ΠΎΠ»ΡŒΡ‚Π°Ρ…. Π’Π°ΠΊΠΎΠ΅ Π½Π°Π·Π²Π°Π½ΠΈΠ΅ Π΅Π΄ΠΈΠ½ΠΈΡ†Ρ‹ для измСрСния ΠΏΠΎΡ‚Π΅Π½Ρ†ΠΈΠ°Π»Π° Π΄Π°Π½ΠΎ ΠΏΠΎ ΠΈΠΌΠ΅Π½ΠΈ ΠΈΡ‚Π°Π»ΡŒΡΠ½ΡΠΊΠΎΠ³ΠΎ Ρ„ΠΈΠ·ΠΈΠΊΠ° АлСссандро Π’ΠΎΠ»ΡŒΡ‚Π° (1745β€”1827), ΠΎΡ‚ΠΊΡ€Ρ‹Π²ΡˆΠ΅Π³ΠΎ Π·Π°ΠΊΠΎΠ½ взаимодСйствия элСктричСских Ρ‚ΠΎΠΊΠΎΠ² ΠΈ ΠΏΡ€Π΅Π΄Π»ΠΎΠΆΠΈΠ²ΡˆΠ΅Π³ΠΎ ΠΏΠ΅Ρ€Π²ΡƒΡŽ Π³ΠΈΠΏΠΎΡ‚Π΅Π·Ρƒ для объяснСния ΠΌΠ°Π³Π½ΠΈΡ‚Π½Ρ‹Ρ… свойств вСщСства.

Π₯арактСристика поля с ΠΏΠΎΠΌΠΎΡ‰ΡŒΡŽ элСктричСского ΠΏΠΎΡ‚Π΅Π½Ρ†ΠΈΠ°Π»Π° ΠΎΡ‡Π΅Π½ΡŒ ΡƒΠ΄ΠΎΠ±Π½Π°. Она позволяСт ΡΡ€Π°Π²Π½ΠΈΠ²Π°Ρ‚ΡŒ Π½Π΅ Ρ‚ΠΎΠ»ΡŒΠΊΠΎ Ρ€Π°Π·Π»ΠΈΡ‡Π½Ρ‹Π΅ элСктричСскиС поля, Π½ΠΎ ΠΈ ΠΎΡ‚Π΄Π΅Π»ΡŒΠ½Ρ‹Π΅ Ρ‚ΠΎΡ‡ΠΊΠΈ ΠΎΠ΄Π½ΠΎΠ³ΠΎ ΠΈ Ρ‚ΠΎΠ³ΠΎ ΠΆΠ΅ поля. ВмСсто Ρ‚ΠΎΠ³ΠΎ, Π½Π°ΠΏΡ€ΠΈΠΌΠ΅Ρ€, Ρ‡Ρ‚ΠΎΠ±Ρ‹ Π³ΠΎΠ²ΠΎΡ€ΠΈΡ‚ΡŒ Β«ΡˆΠ°Ρ€ А наэлСктризован Π±ΠΎΠ»Π΅Π΅ сильно, Ρ‡Π΅ΠΌ ΡˆΠ°Ρ€ Π‘Β», ΠΌΠΎΠΆΠ½ΠΎ ΡΠΊΠ°Π·Π°Ρ‚ΡŒ: Β«ΠΏΠΎΡ‚Π΅Π½Ρ†ΠΈΠ°Π» ΡˆΠ°Ρ€Π° А Π²Ρ‹ΡˆΠ΅ ΠΏΠΎΡ‚Π΅Π½Ρ†ΠΈΠ°Π»Π° ΡˆΠ°Ρ€Π° Π‘Β». ΠŸΠΎΡ‚Π΅Π½Ρ†ΠΈΠ°Π» Ρ‚ΠΎΡ‡ΠΊΠΈ поля ΠΎΠ±Ρ‹Ρ‡Π½ΠΎ обозначаСтся Π±ΡƒΠΊΠ²ΠΎΠΉ Ο†.

ЭлСктричСскоС ΠΏΠΎΠ»Π΅ ΠΌΠΎΠΆΠ΅Ρ‚ ΡΠΎΠ·Π΄Π°Π²Π°Ρ‚ΡŒΡΡ Π½Π΅ Ρ‚ΠΎΠ»ΡŒΠΊΠΎ ΠΏΠΎΠ»ΠΎΠΆΠΈΡ‚Π΅Π»ΡŒΠ½Ρ‹ΠΌ ΠΈΠ»ΠΈ ΠΎΡ‚Ρ€ΠΈΡ†Π°Ρ‚Π΅Π»ΡŒΠ½Ρ‹ΠΌ зарядом, Π½ΠΎ ΠΈ ΠΈΡ… ΡΠΎΠ²ΠΎΠΊΡƒΠΏΠ½ΠΎΡΡ‚ΡŒΡŽ. Π’ Ρ‚Π°ΠΊΠΎΠΌ ΠΏΠΎΠ»Π΅ ΠΎΡ‚Π΄Π΅Π»ΡŒΠ½Ρ‹Π΅ Ρ‚ΠΎΡ‡ΠΊΠΈ ΠΌΠΎΠ³ΡƒΡ‚ ΠΈΠΌΠ΅Ρ‚ΡŒ ΠΊΠ°ΠΊ ΠΎΡ‚Ρ€ΠΈΡ†Π°Ρ‚Π΅Π»ΡŒΠ½Ρ‹Π΅, Ρ‚Π°ΠΊ ΠΈ ΠΏΠΎΠ»ΠΎΠΆΠΈΡ‚Π΅Π»ΡŒΠ½Ρ‹Π΅ ΠΏΠΎΡ‚Π΅Π½Ρ†ΠΈΠ°Π»Ρ‹. Π§Ρ‚ΠΎΠ±Ρ‹ Π² этом случаС ΡΡ€Π°Π²Π½ΠΈΠ²Π°Ρ‚ΡŒ ΠΏΠΎΡ‚Π΅Π½Ρ†ΠΈΠ°Π»Ρ‹ Ρ€Π°Π·Π»ΠΈΡ‡Π½Ρ‹Ρ… Ρ‚ΠΎΡ‡Π΅ΠΊ, Π²Π²Π΅Π»ΠΈ условноС понятиС ΠΎ Ρ‚ΠΎΡ‡ΠΊΠ΅ с Π½ΡƒΠ»Π΅Π²Ρ‹ΠΌ ΠΏΠΎΡ‚Π΅Π½Ρ†ΠΈΠ°Π»ΠΎΠΌ, Ρ‚. Π΅. стали ΡΡ‡ΠΈΡ‚Π°Ρ‚ΡŒ, Ρ‡Ρ‚ΠΎ ΠΎΠ΄Π½Π° ΠΈΠ· Ρ‚ΠΎΡ‡Π΅ΠΊ (ΠΈΠ»ΠΈ нСсколько Ρ‚ΠΎΡ‡Π΅ΠΊ) ΠΈΠΌΠ΅Π΅Ρ‚ ΠΏΠΎΡ‚Π΅Π½Ρ†ΠΈΠ°Π», Ρ€Π°Π²Π½Ρ‹ΠΉ Π½ΡƒΠ»ΡŽ. ΠŸΠΎΡ‚Π΅Π½Ρ†ΠΈΠ°Π»Ρ‹ ΠΎΡΡ‚Π°Π»ΡŒΠ½Ρ‹Ρ… Ρ‚ΠΎΡ‡Π΅ΠΊ поля ΠΎΠΏΡ€Π΅Π΄Π΅Π»ΡΡŽΡ‚ΡΡ ΠΎΡ‚Π½ΠΎΡΠΈΡ‚Π΅Π»ΡŒΠ½ΠΎ Ρ‚ΠΎΡ‡ΠΊΠΈ Π½ΡƒΠ»Π΅Π²ΠΎΠ³ΠΎ ΠΏΠΎΡ‚Π΅Π½Ρ†ΠΈΠ°Π»Π°. Π­Ρ‚ΠΎΡ‚ ΠΌΠ΅Ρ‚ΠΎΠ΄ Π°Π½Π°Π»ΠΎΠ³ΠΈΡ‡Π΅Π½ ΠΌΠ΅Ρ‚ΠΎΠ΄Ρƒ измСрСния Ρ‚Π΅ΠΌΠΏΠ΅Ρ€Π°Ρ‚ΡƒΡ€. Π’Π°ΠΌ Ρ‚Π°ΠΊΠΆΠ΅ опрСдСлСнная Ρ‚Π΅ΠΌΠΏΠ΅Ρ€Π°Ρ‚ΡƒΡ€Π° (Ρ‚Π΅ΠΌΠΏΠ΅Ρ€Π°Ρ‚ΡƒΡ€Π° Ρ‚Π°ΡŽΡ‰Π΅Π³ΠΎ льда) принимаСтся Π·Π° Π½ΡƒΠ»Π΅Π²ΡƒΡŽ Ρ‚ΠΎΡ‡ΠΊΡƒ ΠΈ ΠΏΠΎ ΠΎΡ‚Π½ΠΎΡˆΠ΅Π½ΠΈΡŽ ΠΊ Π½Π΅ΠΉ опрСдСляСтся Ρ‚Π΅ΠΌΠΏΠ΅Ρ€Π°Ρ‚ΡƒΡ€Π° Π΄Ρ€ΡƒΠ³ΠΈΡ… Ρ‚Π΅Π».

Π’ элСктротСхникС условно ΡΡ‡ΠΈΡ‚Π°ΡŽΡ‚, Ρ‡Ρ‚ΠΎ Π½ΡƒΠ»Π΅Π²ΠΎΠΉ ΠΏΠΎΡ‚Π΅Π½Ρ†ΠΈΠ°Π» ΠΈΠΌΠ΅Π΅Ρ‚ ΠΏΠΎΠ²Π΅Ρ€Ρ…Π½ΠΎΡΡ‚ΡŒ Π·Π΅ΠΌΠ»ΠΈ.

Если ΠΏΠΎΡ‚Π΅Π½Ρ†ΠΈΠ°Π» Π² Π΄Π°Π½Π½ΠΎΠΉ Ρ‚ΠΎΡ‡ΠΊΠ΅ Π²Ρ‹ΡˆΠ΅ ΠΏΠΎΡ‚Π΅Π½Ρ†ΠΈΠ°Π»Π° Π·Π΅ΠΌΠ»ΠΈ, Ρ‚ΠΎ ΠΌΡ‹ Π³ΠΎΠ²ΠΎΡ€ΠΈΠΌ, Ρ‡Ρ‚ΠΎ Ρ‚ΠΎΡ‡ΠΊΠ° ΠΎΠ±Π»Π°Π΄Π°Π΅Ρ‚ ΠΏΠΎΠ»ΠΎΠΆΠΈΡ‚Π΅Π»ΡŒΠ½Ρ‹ΠΌ ΠΏΠΎΡ‚Π΅Π½Ρ†ΠΈΠ°Π»ΠΎΠΌ. Если ΠΆΠ΅, Π½Π°ΠΎΠ±ΠΎΡ€ΠΎΡ‚, ΠΏΠΎΡ‚Π΅Π½Ρ†ΠΈΠ°Π» Ρ‚ΠΎΡ‡ΠΊΠΈ Π½ΠΈΠΆΠ΅ ΠΏΠΎΡ‚Π΅Π½Ρ†ΠΈΠ°Π»Π° Π·Π΅ΠΌΠ»ΠΈ, Ρ‚ΠΎ Ρ‚ΠΎΡ‡ΠΊΠ° ΠΎΠ±Π»Π°Π΄Π°Π΅Ρ‚ ΠΎΡ‚Ρ€ΠΈΡ†Π°Ρ‚Π΅Π»ΡŒΠ½Ρ‹ΠΌ ΠΏΠΎΡ‚Π΅Π½Ρ†ΠΈΠ°Π»ΠΎΠΌ.

Π˜Π·ΠΌΠ΅Ρ€ΡΡ ΠΏΠΎΡ‚Π΅Π½Ρ†ΠΈΠ°Π»Ρ‹ Ρ€Π°Π·Π»ΠΈΡ‡Π½Ρ‹Ρ… Ρ‚ΠΎΡ‡Π΅ΠΊ элСктричСского поля ΠΎΡ‚Π½ΠΎΡΠΈΡ‚Π΅Π»ΡŒΠ½ΠΎ Π·Π΅ΠΌΠ»ΠΈ, ΠΌΠΎΠΆΠ½ΠΎ ΡƒΠ±Π΅Π΄ΠΈΡ‚ΡŒΡΡ Π² Ρ‚ΠΎΠΌ, Ρ‡Ρ‚ΠΎ ΠΎΠ½ΠΈ Π½Π΅ΠΎΠ΄ΠΈΠ½Π°ΠΊΠΎΠ²Ρ‹. Π—Π½Π°Ρ‡ΠΈΡ‚, ΠΌΠ΅ΠΆΠ΄Ρƒ ΠΎΡ‚Π΄Π΅Π»ΡŒΠ½Ρ‹ΠΌΠΈ Ρ‚ΠΎΡ‡ΠΊΠ°ΠΌΠΈ ΠΌΠΎΠΆΠ΅Ρ‚ Π±Ρ‹Ρ‚ΡŒ нСкоторая Ρ€Π°Π·Π½ΠΎΡΡ‚ΡŒ ΠΏΠΎΡ‚Π΅Π½Ρ†ΠΈΠ°Π»ΠΎΠ².

ΠžΠΏΡ€Π΅Π΄Π΅Π»Π΅Π½ΠΈΠ΅:Β Β Π Π°Π·Π½ΠΎΡΡ‚ΡŒ ΠΏΠΎΡ‚Π΅Π½Ρ†ΠΈΠ°Π»ΠΎΠ² ΠΌΠ΅ΠΆΠ΄Ρƒ двумя Ρ‚ΠΎΡ‡ΠΊΠ°ΠΌΠΈ элСктричСского поля называСтся напряТСниСм. НапряТСниС, Ρ‚Π°ΠΊ ΠΆΠ΅ ΠΊΠ°ΠΊ ΠΈ ΠΏΠΎΡ‚Π΅Π½Ρ†ΠΈΠ°Π», измСряСтся Π² Π²ΠΎΠ»ΡŒΡ‚Π°Ρ….

Π‘ΠΊΠ°Π·Π°Π½Π½ΠΎΠ΅ поясним ΠΏΡ€ΠΈΠΌΠ΅Ρ€ΠΎΠΌ.

На рис. 1 ΠΌΡ‹ условно ΠΏΠΎΠΊΠ°Π·Π°Π»ΠΈ Ρ‡Π΅Ρ‚Ρ‹Ρ€Π΅ Ρ‚ΠΎΡ‡ΠΊΠΈ: А—с ΠΏΠΎΡ‚Π΅Π½Ρ†ΠΈΠ°Π»ΠΎΠΌ + 20 Π², Π‘ β€” с ΠΏΠΎΡ‚Π΅Π½Ρ†ΠΈΠ°Π»ΠΎΠΌ +40 Π², Π’ β€” с Π½ΡƒΠ»Π΅Π²Ρ‹ΠΌ ΠΏΠΎΡ‚Π΅Π½Ρ†ΠΈΠ°Π»ΠΎΠΌ (зСмля) ΠΈ Π“ β€” с ΠΏΠΎΡ‚Π΅Π½Ρ†ΠΈΠ°Π»ΠΎΠΌβ€”15 Π².

Рисунок 1. Π Π°Π·Π½ΠΎΡΡ‚ΡŒ ΠΏΠΎΡ‚Π΅Π½Ρ†ΠΈΠ°Π»ΠΎΠ² ΠΌΠ΅ΠΆΠ΄Ρƒ Ρ€Π°Π·Π»ΠΈΡ‡Π½Ρ‹ΠΌΠΈ Ρ‚ΠΎΡ‡ΠΊΠ°ΠΌΠΈ элСктричСского поля

Β 

Π Π°Π·Π½ΠΎΡΡ‚ΡŒ ΠΏΠΎΡ‚Π΅Π½Ρ†ΠΈΠ°Π»ΠΎΠ² ΠΌΠ΅ΠΆΠ΄Ρƒ Ρ‚ΠΎΡ‡ΠΊΠ°ΠΌΠΈ Π‘ ΠΈ А =40β€”20=20 Π²;

Π Π°Π·Π½ΠΎΡΡ‚ΡŒ ΠΏΠΎΡ‚Π΅Π½Ρ†ΠΈΠ°Π»ΠΎΠ² ΠΌΠ΅ΠΆΠ΄Ρƒ Ρ‚ΠΎΡ‡ΠΊΠ°ΠΌΠΈ А ΠΈ Π’ =20β€” 0=20 Π²;

Π Π°Π·Π½ΠΎΡΡ‚ΡŒ ΠΏΠΎΡ‚Π΅Π½Ρ†ΠΈΠ°Π»ΠΎΠ² ΠΌΠ΅ΠΆΠ΄Ρƒ Ρ‚ΠΎΡ‡ΠΊΠ°ΠΌΠΈ Π‘ ΠΈ Π’ =40β€” 0=40 Π²;

Π Π°Π·Π½ΠΎΡΡ‚ΡŒ ΠΏΠΎΡ‚Π΅Π½Ρ†ΠΈΠ°Π»ΠΎΠ² ΠΌΠ΅ΠΆΠ΄Ρƒ Ρ‚ΠΎΡ‡ΠΊΠ°ΠΌΠΈ А ΠΈ Π“=20β€”(β€”15) =35 Π².

ΠŸΠΎΡ‚Π΅Π½Ρ†ΠΈΠ°Π» Ρ‚ΠΎΡ‡ΠΊΠΈ Π‘ Π²Ρ‹ΡˆΠ΅ ΠΏΠΎΡ‚Π΅Π½Ρ†ΠΈΠ°Π»ΠΎΠ² Ρ‚ΠΎΡ‡Π΅ΠΊ А, Π’ ΠΈ Π“. ΠŸΠΎΡ‚Π΅Π½Ρ†ΠΈΠ°Π» Ρ‚ΠΎΡ‡ΠΊΠΈ А Π²Ρ‹ΡˆΠ΅ ΠΏΠΎΡ‚Π΅Π½Ρ†ΠΈΠ°Π»ΠΎΠ² Ρ‚ΠΎΡ‡Π΅ΠΊ Π’ ΠΈ Π“, Π½ΠΎ Π½ΠΈΠΆΠ΅ ΠΏΠΎΡ‚Π΅Π½Ρ†ΠΈΠ°Π»Π° Ρ‚ΠΎΡ‡ΠΊΠΈ Π‘. ΠŸΠΎΡ‚Π΅Π½Ρ†ΠΈΠ°Π» Ρ‚ΠΎΡ‡ΠΊΠΈ Π’ Π½ΠΈΠΆΠ΅ ΠΏΠΎΡ‚Π΅Π½Ρ†ΠΈΠ°Π»ΠΎΠ² Ρ‚ΠΎΡ‡Π΅ΠΊ А ΠΈ Π‘, Π½ΠΎ Π²Ρ‹ΡˆΠ΅ ΠΏΠΎΡ‚Π΅Π½Ρ†ΠΈΠ°Π»Π° Ρ‚ΠΎΡ‡ΠΊΠΈ Π“.

Π‘Π»Π΅Π΄ΡƒΠ΅Ρ‚ ΠΎΠ±Ρ€Π°Ρ‚ΠΈΡ‚ΡŒ Π²Π½ΠΈΠΌΠ°Π½ΠΈΠ΅ Π½Π° Ρ‚ΠΎ, Ρ‡Ρ‚ΠΎ Ρ‚ΠΎΡ‡ΠΊΠΈ ΠΎΡ‚Ρ€ΠΈΡ†Π°Ρ‚Π΅Π»ΡŒΠ½ΠΎΠ³ΠΎ ΠΏΠΎΡ‚Π΅Π½Ρ†ΠΈΠ°Π»Π° ΠΈΠΌΠ΅ΡŽΡ‚ Π±ΠΎΠ»Π΅Π΅ Π½ΠΈΠ·ΠΊΠΈΠΉ ΠΏΠΎΡ‚Π΅Π½Ρ†ΠΈΠ°Π», Ρ‡Π΅ΠΌ Ρ‚ΠΎΠ½ΠΊΠΈ Π½ΡƒΠ»Π΅Π²ΠΎΠ³ΠΎ ΠΏΠΎΡ‚Π΅Π½Ρ†ΠΈΠ°Π»Π°.

МоТно ΠΈ ΠΈΠ½Π°Ρ‡Π΅ ΠΎΠΏΡ€Π΅Π΄Π΅Π»ΠΈΡ‚ΡŒ напряТСниС ΠΌΠ΅ΠΆΠ΄Ρƒ двумя Ρ‚ΠΎΡ‡ΠΊΠ°ΠΌΠΈ. Для этого рассмотрим Π΄Π²Π΅ Ρ‚ΠΎΡ‡ΠΊΠΈ А ΠΈ Π‘ элСктричСского поля.

Допустим, Ρ‡Ρ‚ΠΎ ΠΏΠΎΡ‚Π΅Π½Ρ†ΠΈΠ°Π» Ρ‚ΠΎΡ‡ΠΊΠΈ А равСн φА потСнциал Ρ‚ΠΎΡ‡ΠΊΠΈ Π‘ Ρ€Π°Π²Π΅Π½Β Ο†Π‘. ΠŸΠΎΡ‚Π΅Π½Ρ†ΠΈΠ°Π» Ρ‚ΠΎΡ‡ΠΊΠΈ А (ΠΈΠ»ΠΈ Π‘) опрСдСляСтся Ρ‚ΠΎΠΉ Ρ€Π°Π±ΠΎΡ‚ΠΎΠΉ, ΠΊΠΎΡ‚ΠΎΡ€ΡƒΡŽ Π½Π΅ΠΎΠ±Ρ…ΠΎΠ΄ΠΈΠΌΠΎ Π·Π°Ρ‚Ρ€Π°Ρ‚ΠΈΡ‚ΡŒ Π½Π° пСрСнос Π΅Π΄ΠΈΠ½ΠΈΡ‡Π½ΠΎΠ³ΠΎ ΠΏΠΎΠ»ΠΎΠΆΠΈΡ‚Π΅Π»ΡŒΠ½ΠΎΠ³ΠΎ заряда ΠΈΠ·-Π·Π° ΠΏΡ€Π΅Π΄Π΅Π»ΠΎΠ² поля Π² Ρ‚ΠΎΡ‡ΠΊΡƒ А (ΠΈΠ»ΠΈ Π‘). Если для пСрСноса Π΅Π΄ΠΈΠ½ΠΈΡ‡Π½ΠΎΠ³ΠΎ ΠΏΠΎΠ»ΠΎΠΆΠΈΡ‚Π΅Π»ΡŒΠ½ΠΎΠ³ΠΎ заряда ΠΈΠ·-Π·Π° ΠΏΡ€Π΅Π΄Π΅Π»Π° поля Π² Ρ‚ΠΎΡ‡ΠΊΡƒ А ΠΈ Π² Ρ‚ΠΎΡ‡ΠΊΡƒ Π‘ трСбуСтся Π·Π°Ρ‚Ρ€Π°Ρ‚ΠΈΡ‚ΡŒ Ρ€Π°Π·Π»ΠΈΡ‡Π½ΡƒΡŽ ΠΏΠΎ Π²Π΅Π»ΠΈΡ‡ΠΈΠ½Π΅ Ρ€Π°Π±ΠΎΡ‚Ρƒ, то φА нС Ρ€Π°Π²Π½ΠΎΒ Ο†Π‘Β ΠΈ ΠΌΠ΅ΠΆΠ΄Ρƒ Ρ‚ΠΎΡ‡ΠΊΠ°ΠΌΠΈ А ΠΈ Π‘ сущСствуСт нСкоторая Ρ€Π°Π·Π½ΠΎΡΡ‚ΡŒ ΠΏΠΎΡ‚Π΅Π½Ρ†ΠΈΠ°Π»ΠΎΠ², ΠΈΠ»ΠΈ напряТСниС. Π­Ρ‚ΠΎ напряТСниС опрСдСляСтся Ρ€Π°Π·Π½ΠΎΡΡ‚ΡŒΡŽΒ 

φА —Β Ο†Π‘Β Ρ‚. Π΅. Ρ€Π°Π±ΠΎΡ‚ΠΎΠΉ, ΡΠΎΠ²Π΅Ρ€ΡˆΠ°Π΅ΠΌΠΎΠΉ силами поля ΠΏΡ€ΠΈ пСрСносС Π΅Π΄ΠΈΠ½ΠΈΡ‡Π½ΠΎΠ³ΠΎ ΠΏΠΎΠ»ΠΎΠΆΠΈΡ‚Π΅Π»ΡŒΠ½ΠΎΠ³ΠΎ заряда ΠΈΠ· Ρ‚ΠΎΡ‡ΠΊΠΈ А Π² Ρ‚ΠΎΡ‡ΠΊΡƒ Π‘.

ΠŸΠžΠΠ ΠΠ’Π˜Π›ΠΠ‘Π¬ БВАВЬЯ? ΠŸΠžΠ”Π•Π›Π˜Π‘Π¬ Π‘ Π”Π Π£Π—Π¬Π―ΠœΠ˜ Π’ Π‘ΠžΠ¦Π˜ΠΠ›Π¬ΠΠ«Π₯ Π‘Π•Π’Π―Π₯!

ΠŸΠΎΡ…ΠΎΠΆΠΈΠ΅ ΠΌΠ°Ρ‚Π΅Ρ€ΠΈΠ°Π»Ρ‹:

Π”ΠΎΠ±Π°Π²ΠΈΡ‚ΡŒ ΠΊΠΎΠΌΠΌΠ΅Π½Ρ‚Π°Ρ€ΠΈΠΉ

ΠŸΠΎΡ‚Π΅Π½Ρ†ΠΈΠ°Π», Ρ€Π°Π±ΠΎΡ‚Π° элСктростатичСского поля. ΠŸΠΎΡ‚Π΅Π½Ρ†ΠΈΠ°Π»ΡŒΠ½Π°Ρ энСргия, Ρ€Π°Π·Π½ΠΎΡΡ‚ΡŒ ΠΏΠΎΡ‚Π΅Π½Ρ†ΠΈΠ°Π»ΠΎΠ², ΠΏΡ€ΠΈΠ½Ρ†ΠΈΠΏ супСрпозиции. ВСсты, Ρ„ΠΎΡ€ΠΌΡƒΠ»Ρ‹

ВСстированиС ΠΎΠ½Π»Π°ΠΉΠ½

Π Π°Π±ΠΎΡ‚Π° элСктростатичСского поля

Рассмотрим ΡΠΈΡ‚ΡƒΠ°Ρ†ΠΈΡŽ: заряд q0 ΠΏΠΎΠΏΠ°Π΄Π°Π΅Ρ‚ Π² элСктростатичСскоС ΠΏΠΎΠ»Π΅. Π­Ρ‚ΠΎ элСктростатичСскоС ΠΏΠΎΠ»Π΅ Ρ‚ΠΎΠΆΠ΅ создаСтся ΠΊΠ°ΠΊΠΈΠΌ-Ρ‚ΠΎ заряТСнным Ρ‚Π΅Π»ΠΎΠΌ ΠΈΠ»ΠΈ систСмой Ρ‚Π΅Π», Π½ΠΎ нас это Π½Π΅ интСрСсуСт. На заряд q0 со стороны поля дСйствуСт сила, которая ΠΌΠΎΠΆΠ΅Ρ‚ ΡΠΎΠ²Π΅Ρ€ΡˆΠ°Ρ‚ΡŒ Ρ€Π°Π±ΠΎΡ‚Ρƒ ΠΈ ΠΏΠ΅Ρ€Π΅ΠΌΠ΅Ρ‰Π°Ρ‚ΡŒ этот заряд Π² ΠΏΠΎΠ»Π΅.


Π Π°Π±ΠΎΡ‚Π° элСктростатичСского поля Π½Π΅ зависит ΠΎΡ‚ Ρ‚Ρ€Π°Π΅ΠΊΡ‚ΠΎΡ€ΠΈΠΈ. Π Π°Π±ΠΎΡ‚Π° поля ΠΏΡ€ΠΈ ΠΏΠ΅Ρ€Π΅ΠΌΠ΅Ρ‰Π΅Π½ΠΈΠΈ заряда ΠΏΠΎ Π·Π°ΠΌΠΊΠ½ΡƒΡ‚ΠΎΠΉ Ρ‚Ρ€Π°Π΅ΠΊΡ‚ΠΎΡ€ΠΈΠΈ Ρ€Π°Π²Π½Π° Π½ΡƒΠ»ΡŽ. По этой ΠΏΡ€ΠΈΡ‡ΠΈΠ½Π΅ силы элСктростатичСского поля Π½Π°Π·Ρ‹Π²Π°ΡŽΡ‚ΡΡ консСрвативными, Π° само ΠΏΠΎΠ»Π΅ называСтся ΠΏΠΎΡ‚Π΅Π½Ρ†ΠΈΠ°Π»ΡŒΠ½Ρ‹ΠΌ.

ΠŸΠΎΡ‚Π΅Π½Ρ†ΠΈΠ°Π»

БистСма «Π·Π°Ρ€ΡΠ΄ — элСктростатичСскоС ΠΏΠΎΠ»Π΅» ΠΈΠ»ΠΈ «Π·Π°Ρ€ΡΠ΄ — заряд» ΠΎΠ±Π»Π°Π΄Π°Π΅Ρ‚ ΠΏΠΎΡ‚Π΅Π½Ρ†ΠΈΠ°Π»ΡŒΠ½ΠΎΠΉ энСргиСй, ΠΏΠΎΠ΄ΠΎΠ±Π½ΠΎ Ρ‚ΠΎΠΌΡƒ, ΠΊΠ°ΠΊ систСма «Π³Ρ€Π°Π²ΠΈΡ‚Π°Ρ†ΠΈΠΎΠ½Π½ΠΎΠ΅ ΠΏΠΎΠ»Π΅ — Ρ‚Π΅Π»ΠΎ» ΠΎΠ±Π»Π°Π΄Π°Π΅Ρ‚ ΠΏΠΎΡ‚Π΅Π½Ρ†ΠΈΠ°Π»ΡŒΠ½ΠΎΠΉ энСргиСй.

ЀизичСская скалярная Π²Π΅Π»ΠΈΡ‡ΠΈΠ½Π°, Ρ…Π°Ρ€Π°ΠΊΡ‚Π΅Ρ€ΠΈΠ·ΡƒΡŽΡ‰Π°Ρ энСргСтичСскоС состояниС поля называСтся ΠΏΠΎΡ‚Π΅Π½Ρ†ΠΈΠ°Π»ΠΎΠΌ Π΄Π°Π½Π½ΠΎΠΉ Ρ‚ΠΎΡ‡ΠΊΠΈ поля. Π’ ΠΏΠΎΠ»Π΅ помСщаСтся заряд q, ΠΎΠ½ ΠΎΠ±Π»Π°Π΄Π°Π΅Ρ‚ ΠΏΠΎΡ‚Π΅Π½Ρ†ΠΈΠ°Π»ΡŒΠ½ΠΎΠΉ энСргиСй W. ΠŸΠΎΡ‚Π΅Π½Ρ†ΠΈΠ°Π» — это характСристика элСктростатичСского поля.

Вспомним ΠΏΠΎΡ‚Π΅Π½Ρ†ΠΈΠ°Π»ΡŒΠ½ΡƒΡŽ ΡΠ½Π΅Ρ€Π³ΠΈΡŽ Π² ΠΌΠ΅Ρ…Π°Π½ΠΈΠΊΠ΅. ΠŸΠΎΡ‚Π΅Π½Ρ†ΠΈΠ°Π»ΡŒΠ½Π°Ρ энСргия Ρ€Π°Π²Π½Π° Π½ΡƒΠ»ΡŽ, ΠΊΠΎΠ³Π΄Π° Ρ‚Π΅Π»ΠΎ находится Π½Π° Π·Π΅ΠΌΠ»Π΅. А ΠΊΠΎΠ³Π΄Π° Ρ‚Π΅Π»ΠΎ ΠΏΠΎΠ΄Π½ΠΈΠΌΠ°ΡŽΡ‚ Π½Π° Π½Π΅ΠΊΠΎΡ‚ΠΎΡ€ΡƒΡŽ высоту, Ρ‚ΠΎ говорят, Ρ‡Ρ‚ΠΎ Ρ‚Π΅Π»ΠΎ ΠΎΠ±Π»Π°Π΄Π°Π΅Ρ‚ ΠΏΠΎΡ‚Π΅Π½Ρ†ΠΈΠ°Π»ΡŒΠ½ΠΎΠΉ энСргиСй.

ΠšΠ°ΡΠ°Ρ‚Π΅Π»ΡŒΠ½ΠΎ ΠΏΠΎΡ‚Π΅Π½Ρ†ΠΈΠ°Π»ΡŒΠ½ΠΎΠΉ энСргии Π² элСктричСствС, Ρ‚ΠΎ здСсь Π½Π΅Ρ‚ Π½ΡƒΠ»Π΅Π²ΠΎΠ³ΠΎ уровня ΠΏΠΎΡ‚Π΅Π½Ρ†ΠΈΠ°Π»ΡŒΠ½ΠΎΠΉ энСргии. Π•Π³ΠΎ Π²Ρ‹Π±ΠΈΡ€Π°ΡŽΡ‚ ΠΏΡ€ΠΎΠΈΠ·Π²ΠΎΠ»ΡŒΠ½ΠΎ. ΠŸΠΎΡΡ‚ΠΎΠΌΡƒ ΠΏΠΎΡ‚Π΅Π½Ρ†ΠΈΠ°Π» являСтся ΠΎΡ‚Π½ΠΎΡΠΈΡ‚Π΅Π»ΡŒΠ½ΠΎΠΉ физичСской Π²Π΅Π»ΠΈΡ‡ΠΈΠ½ΠΎΠΉ.

Π’ ΠΌΠ΅Ρ…Π°Π½ΠΈΠΊΠ΅ Ρ‚Π΅Π»Π° стрСмятся Π·Π°Π½ΡΡ‚ΡŒ ΠΏΠΎΠ»ΠΎΠΆΠ΅Π½ΠΈΠ΅ с наимСньшСй ΠΏΠΎΡ‚Π΅Π½Ρ†ΠΈΠ°Π»ΡŒΠ½ΠΎΠΉ энСргиСй. Π’ элСктричСствС ΠΆΠ΅ ΠΏΠΎΠ΄ дСйствиСм сил поля ΠΏΠΎΠ»ΠΎΠΆΠΈΡ‚Π΅Π»ΡŒΠ½ΠΎ заряТСнноС Ρ‚Π΅Π»ΠΎ стрСмится пСрСмСстится ΠΈΠ· Ρ‚ΠΎΡ‡ΠΊΠΈ с Π±ΠΎΠ»Π΅Π΅ высоким ΠΏΠΎΡ‚Π΅Π½Ρ†ΠΈΠ°Π»ΠΎΠΌ Π² Ρ‚ΠΎΡ‡ΠΊΡƒ с Π±ΠΎΠ»Π΅Π΅ Π½ΠΈΠ·ΠΊΠΈΠΌ ΠΏΠΎΡ‚Π΅Π½Ρ†ΠΈΠ°Π»ΠΎΠΌ, Π° ΠΎΡ‚Ρ€ΠΈΡ†Π°Ρ‚Π΅Π»ΡŒΠ½ΠΎ заряТСнноС Ρ‚Π΅Π»ΠΎ — Π½Π°ΠΎΠ±ΠΎΡ€ΠΎΡ‚.

ΠŸΠΎΡ‚Π΅Π½Ρ†ΠΈΠ°Π»ΡŒΠ½Π°Ρ энСргия поля — это Ρ€Π°Π±ΠΎΡ‚Π°, ΠΊΠΎΡ‚ΠΎΡ€ΡƒΡŽ выполняСт элСктростатичСская сила ΠΏΡ€ΠΈ ΠΏΠ΅Ρ€Π΅ΠΌΠ΅Ρ‰Π΅Π½ΠΈΠΈ заряда ΠΈΠ· Π΄Π°Π½Π½ΠΎΠΉ Ρ‚ΠΎΡ‡ΠΊΠΈ поля Π² Ρ‚ΠΎΡ‡ΠΊΡƒ с Π½ΡƒΠ»Π΅Π²Ρ‹ΠΌ ΠΏΠΎΡ‚Π΅Π½Ρ†ΠΈΠ°Π»ΠΎΠΌ.

Рассмотрим частный случай, ΠΊΠΎΠ³Π΄Π° элСктростатичСскоС ΠΏΠΎΠ»Π΅ создаСтся элСктричСским зарядом Q. Для исслСдования ΠΏΠΎΡ‚Π΅Π½Ρ†ΠΈΠ°Π»Π° Ρ‚Π°ΠΊΠΎΠ³ΠΎ поля Π½Π΅Ρ‚ нСобходимости Π² Π½Π΅Π³ΠΎ Π²Π½ΠΎΡΠΈΡ‚ΡŒ заряд q. МоТно Π²Ρ‹ΡΡ‡ΠΈΡ‚Π°Ρ‚ΡŒ ΠΏΠΎΡ‚Π΅Π½Ρ†ΠΈΠ°Π» любой Ρ‚ΠΎΡ‡ΠΊΠΈ Ρ‚Π°ΠΊΠΎΠ³ΠΎ поля, находящСйся Π½Π° расстоянии r ΠΎΡ‚ заряда Q.

ДиэлСктричСская ΠΏΡ€ΠΎΠ½ΠΈΡ†Π°Π΅ΠΌΠΎΡΡ‚ΡŒ срСды ΠΈΠΌΠ΅Π΅Ρ‚ извСстноС Π·Π½Π°Ρ‡Π΅Π½ΠΈΠ΅ (Ρ‚Π°Π±Π»ΠΈΡ‡Π½ΠΎΠ΅), Ρ…Π°Ρ€Π°ΠΊΡ‚Π΅Ρ€ΠΈΠ·ΡƒΠ΅Ρ‚ срСду, Π² ΠΊΠΎΡ‚ΠΎΡ€ΠΎΠΉ сущСствуСт ΠΏΠΎΠ»Π΅. Для Π²ΠΎΠ·Π΄ΡƒΡ…Π° ΠΎΠ½Π° Ρ€Π°Π²Π½Π° Π΅Π΄ΠΈΠ½ΠΈΡ†Π΅.

Π Π°Π·Π½ΠΎΡΡ‚ΡŒ ΠΏΠΎΡ‚Π΅Π½Ρ†ΠΈΠ°Π»ΠΎΠ²

Π Π°Π±ΠΎΡ‚Π° поля ΠΏΠΎ ΠΏΠ΅Ρ€Π΅ΠΌΠ΅Ρ‰Π΅Π½ΠΈΡŽ заряда ΠΈΠ· ΠΎΠ΄Π½ΠΎΠΉ Ρ‚ΠΎΡ‡ΠΊΠΈ Π² Π΄Ρ€ΡƒΠ³ΡƒΡŽ, называСтся Ρ€Π°Π·Π½ΠΎΡΡ‚ΡŒΡŽ ΠΏΠΎΡ‚Π΅Π½Ρ†ΠΈΠ°Π»ΠΎΠ²

Π­Ρ‚Ρƒ Ρ„ΠΎΡ€ΠΌΡƒΠ»Ρƒ ΠΌΠΎΠΆΠ½ΠΎ ΠΏΡ€Π΅Π΄ΡΡ‚Π°Π²ΠΈΡ‚ΡŒ Π² ΠΈΠ½ΠΎΠΌ Π²ΠΈΠ΄Π΅

Π­ΠΊΠ²ΠΈΠΏΠΎΡ‚Π΅Π½Ρ†ΠΈΠ°Π»ΡŒΠ½Π°Ρ ΠΏΠΎΠ²Π΅Ρ€Ρ…Π½ΠΎΡΡ‚ΡŒ (линия) — ΠΏΠΎΠ²Π΅Ρ€Ρ…Π½ΠΎΡΡ‚ΡŒ Ρ€Π°Π²Π½ΠΎΠ³ΠΎ ΠΏΠΎΡ‚Π΅Π½Ρ†ΠΈΠ°Π»Π°. Π Π°Π±ΠΎΡ‚Π° ΠΏΠΎ ΠΏΠ΅Ρ€Π΅ΠΌΠ΅Ρ‰Π΅Π½ΠΈΡŽ заряда вдоль ΡΠΊΠ²ΠΈΠΏΠΎΡ‚Π΅Π½Ρ†ΠΈΠ°Π»ΡŒΠ½ΠΎΠΉ повСрхности Ρ€Π°Π²Π½Π° Π½ΡƒΠ»ΡŽ.

НапряТСниС

Π Π°Π·Π½ΠΎΡΡ‚ΡŒ ΠΏΠΎΡ‚Π΅Π½Ρ†ΠΈΠ°Π»ΠΎΠ² Π½Π°Π·Ρ‹Π²Π°ΡŽΡ‚ Π΅Ρ‰Π΅ элСктричСским напряТСниСм ΠΏΡ€ΠΈ условии, Ρ‡Ρ‚ΠΎ сторонниС силы Π½Π΅ Π΄Π΅ΠΉΡΡ‚Π²ΡƒΡŽΡ‚ ΠΈΠ»ΠΈ ΠΈΡ… дСйствиСм ΠΌΠΎΠΆΠ½ΠΎ ΠΏΡ€Π΅Π½Π΅Π±Ρ€Π΅Ρ‡ΡŒ.

НапряТСниС ΠΌΠ΅ΠΆΠ΄Ρƒ двумя Ρ‚ΠΎΡ‡ΠΊΠ°ΠΌΠΈ Π² ΠΎΠ΄Π½ΠΎΡ€ΠΎΠ΄Π½ΠΎΠΌ элСктричСском ΠΏΠΎΠ»Π΅, располоТСнными ΠΏΠΎ ΠΎΠ΄Π½ΠΎΠΉ Π»ΠΈΠ½ΠΈΠΈ напряТСнности, Ρ€Π°Π²Π½ΠΎ ΠΏΡ€ΠΎΠΈΠ·Π²Π΅Π΄Π΅Π½ΠΈΡŽ модуля Π²Π΅ΠΊΡ‚ΠΎΡ€Π° напряТСнности поля Π½Π° расстояниС ΠΌΠ΅ΠΆΠ΄Ρƒ этими Ρ‚ΠΎΡ‡ΠΊΠ°ΠΌΠΈ.

ΠžΡ‚ Π²Π΅Π»ΠΈΡ‡ΠΈΠ½Ρ‹ напряТСния зависит Ρ‚ΠΎΠΊ Π² Ρ†Π΅ΠΏΠΈ ΠΈ энСргия заряТСнной частицы.

ΠŸΡ€ΠΈΠ½Ρ†ΠΈΠΏ супСрпозиции

ΠŸΠΎΡ‚Π΅Π½Ρ†ΠΈΠ°Π» поля, созданного нСсколькими зарядами, Ρ€Π°Π²Π΅Π½ алгСбраичСской (с ΡƒΡ‡Π΅Ρ‚ΠΎΠΌ Π·Π½Π°ΠΊΠ° ΠΏΠΎΡ‚Π΅Π½Ρ†ΠΈΠ°Π»Π°) суммС ΠΏΠΎΡ‚Π΅Π½Ρ†ΠΈΠ°Π»ΠΎΠ² ΠΏΠΎΠ»Π΅ΠΉ ΠΊΠ°ΠΆΠ΄ΠΎΠ³ΠΎ поля Π² ΠΎΡ‚Π΄Π΅Π»ΡŒΠ½ΠΎΡΡ‚ΠΈ

ΠŸΡ€ΠΈ Ρ€Π΅ΡˆΠ΅Π½ΠΈΠΈ Π·Π°Π΄Π°Ρ‡ Π²ΠΎΠ·Π½ΠΈΠΊΠ°Π΅Ρ‚ ΠΌΠ½ΠΎΠ³ΠΎ ΠΏΡƒΡ‚Π°Π½ΠΈΡ†Ρ‹ ΠΏΡ€ΠΈ ΠΎΠΏΡ€Π΅Π΄Π΅Π»Π΅Π½ΠΈΠΈ Π·Π½Π°ΠΊΠ° ΠΏΠΎΡ‚Π΅Π½Ρ†ΠΈΠ°Π»Π°, разности ΠΏΠΎΡ‚Π΅Π½Ρ†ΠΈΠ°Π»ΠΎΠ², Ρ€Π°Π±ΠΎΡ‚Ρ‹.

На рисункС ΠΈΠ·ΠΎΠ±Ρ€Π°ΠΆΠ΅Π½Ρ‹ Π»ΠΈΠ½ΠΈΠΈ напряТСнности. Π’ ΠΊΠ°ΠΊΠΎΠΉ Ρ‚ΠΎΡ‡ΠΊΠ΅ поля ΠΏΠΎΡ‚Π΅Π½Ρ†ΠΈΠ°Π» большС?

Π’Π΅Ρ€Π½Ρ‹ΠΉ ΠΎΡ‚Π²Π΅Ρ‚ — Ρ‚ΠΎΡ‡ΠΊΠ° 1. Вспомним, Ρ‡Ρ‚ΠΎ Π»ΠΈΠ½ΠΈΠΈ напряТСнности Π½Π°Ρ‡ΠΈΠ½Π°ΡŽΡ‚ΡΡ Π½Π° ΠΏΠΎΠ»ΠΎΠΆΠΈΡ‚Π΅Π»ΡŒΠ½ΠΎΠΌ зарядС, Π° Π·Π½Π°Ρ‡ΠΈΡ‚ ΠΏΠΎΠ»ΠΎΠΆΠΈΡ‚Π΅Π»ΡŒΠ½Ρ‹ΠΉ заряд находится слСва, ΡΠ»Π΅Π΄ΠΎΠ²Π°Ρ‚Π΅Π»ΡŒΠ½ΠΎ ΠΌΠ°ΠΊΡΠΈΠΌΠ°Π»ΡŒΠ½Ρ‹ΠΌ ΠΏΠΎΡ‚Π΅Π½Ρ†ΠΈΠ°Π»ΠΎΠΌ ΠΎΠ±Π»Π°Π΄Π°Π΅Ρ‚ крайняя лСвая Ρ‚ΠΎΡ‡ΠΊΠ°.

Если происходит исслСдованиС поля, ΠΊΠΎΡ‚ΠΎΡ€ΠΎΠ΅ создаСтся ΠΎΡ‚Ρ€ΠΈΡ†Π°Ρ‚Π΅Π»ΡŒΠ½Ρ‹ΠΌ зарядом, Ρ‚ΠΎ ΠΏΠΎΡ‚Π΅Π½Ρ†ΠΈΠ°Π» поля Π²Π±Π»ΠΈΠ·ΠΈ заряда ΠΈΠΌΠ΅Π΅Ρ‚ ΠΎΡ‚Ρ€ΠΈΡ†Π°Ρ‚Π΅Π»ΡŒΠ½ΠΎΠ΅ Π·Π½Π°Ρ‡Π΅Π½ΠΈΠ΅, Π² этом Π»Π΅Π³ΠΊΠΎ ΡƒΠ±Π΅Π΄ΠΈΡ‚ΡŒΡΡ, Ссли Π² Ρ„ΠΎΡ€ΠΌΡƒΠ»Ρƒ ΠΏΠΎΠ΄ΡΡ‚Π°Π²ΠΈΡ‚ΡŒ заряд со Π·Π½Π°ΠΊΠΎΠΌ «ΠΌΠΈΠ½ΡƒΡ». Π§Π΅ΠΌ дальшС ΠΎΡ‚ ΠΎΡ‚Ρ€ΠΈΡ†Π°Ρ‚Π΅Π»ΡŒΠ½ΠΎΠ³ΠΎ заряда, Ρ‚Π΅ΠΌ ΠΏΠΎΡ‚Π΅Π½Ρ†ΠΈΠ°Π» поля большС.

Если происходит ΠΏΠ΅Ρ€Π΅ΠΌΠ΅Ρ‰Π΅Π½ΠΈΠ΅ ΠΏΠΎΠ»ΠΎΠΆΠΈΡ‚Π΅Π»ΡŒΠ½ΠΎΠ³ΠΎ заряда вдоль Π»ΠΈΠ½ΠΈΠΉ напряТСнности, Ρ‚ΠΎ Ρ€Π°Π·Π½ΠΎΡΡ‚ΡŒ ΠΏΠΎΡ‚Π΅Π½Ρ†ΠΈΠ°Π»ΠΎΠ² ΠΈ Ρ€Π°Π±ΠΎΡ‚Π° ΡΠ²Π»ΡΡŽΡ‚ΡΡ ΠΏΠΎΠ»ΠΎΠΆΠΈΡ‚Π΅Π»ΡŒΠ½Ρ‹ΠΌΠΈ. Если вдоль Π»ΠΈΠ½ΠΈΠΉ напряТСнности происходит ΠΏΠ΅Ρ€Π΅ΠΌΠ΅Ρ‰Π΅Π½ΠΈΠ΅ ΠΎΡ‚Ρ€ΠΈΡ†Π°Ρ‚Π΅Π»ΡŒΠ½ΠΎΠ³ΠΎ заряда, Ρ‚ΠΎ Ρ€Π°Π·Π½ΠΎΡΡ‚ΡŒ ΠΏΠΎΡ‚Π΅Π½Ρ†ΠΈΠ°Π»ΠΎΠ² ΠΈΠΌΠ΅Π΅Ρ‚ Π·Π½Π°ΠΊ «+», Ρ€Π°Π±ΠΎΡ‚Π° ΠΈΠΌΠ΅Π΅Ρ‚ Π·Π½Π°ΠΊ «-«.

ΠŸΠΎΡ€Π°ΡΡΡƒΠΆΠ΄Π°ΠΉΡ‚Π΅ ΡΠ°ΠΌΠΎΡΡ‚ΠΎΡΡ‚Π΅Π»ΡŒΠ½ΠΎ ΠΎΡ‚Ρ€ΠΈΡ†Π°Ρ‚Π΅Π»ΡŒΠ½Ρ‹Π΅ ΠΈΠ»ΠΈ ΠΏΠΎΠ»ΠΎΠΆΠΈΡ‚Π΅Π»ΡŒΠ½Ρ‹Π΅ значСния Π±ΡƒΠ΄ΡƒΡ‚ ΠΏΡ€ΠΈΠ½ΠΈΠΌΠ°Ρ‚ΡŒ Ρ€Π°Π±ΠΎΡ‚Π° ΠΈ Ρ€Π°Π·Π½ΠΎΡΡ‚ΡŒ ΠΏΠΎΡ‚Π΅Π½Ρ†ΠΈΠ°Π»ΠΎΠ², Ссли заряд ΠΏΠ΅Ρ€Π΅ΠΌΠ΅Ρ‰Π°Ρ‚ΡŒ Π² ΠΎΠ±Ρ€Π°Ρ‚Π½ΠΎΠΌ Π½Π°ΠΏΡ€Π°Π²Π»Π΅Π½ΠΈΠΈ ΠΎΡ‚Π½ΠΎΡΠΈΡ‚Π΅Π»ΡŒΠ½ΠΎ Π»ΠΈΠ½ΠΈΠΉ напряТСнности.

НапряТСниС Π² ΠΊΠ»Π΅Ρ‚ΠΊΠ°Ρ… сСтчатки Π³Π»Π°Π·Π° ΠΏΡ€ΠΈ попадания Π² Π½ΠΈΡ… свСта ΠΎΠΊΠΎΠ»ΠΎ 0,01 Π’.
НапряТСниС Π² Ρ‚Π΅Π»Π΅Ρ„ΠΎΠ½Π½Ρ‹Ρ… сСтях ΠΌΠΎΠΆΠ΅Ρ‚ Π΄ΠΎΡΡ‚ΠΈΠ³Π°Ρ‚ΡŒ 60 Π’.
ЭлСктричСский ΡƒΠ³ΠΎΡ€ΡŒ способСн ΡΠΎΠ·Π΄Π°Π²Π°Ρ‚ΡŒ напряТСниС Π΄ΠΎ 650 Π’.

Из опрСдСлСния ΠΏΠΎΡ‚Π΅Π½Ρ†ΠΈΠ°Π»Π° слСдуСт, Ρ‡Ρ‚ΠΎ ΠΏΠΎΡ‚Π΅Π½Ρ†ΠΈΠ°Π»ΡŒΠ½Π°Ρ энСргия элСктростатичСского взаимодСйствия Π΄Π²ΡƒΡ… зарядов q1 ΠΈ q2, находящихся Π½Π° расстоянии r Π΄Ρ€ΡƒΠ³ ΠΎΡ‚ Π΄Ρ€ΡƒΠ³Π°, числСнно Ρ€Π°Π²Π½Π° Ρ€Π°Π±ΠΎΡ‚Π΅, которая ΡΠΎΠ²Π΅Ρ€ΡˆΠ°Π΅Ρ‚ΡΡ ΠΏΡ€ΠΈ ΠΏΠ΅Ρ€Π΅ΠΌΠ΅Ρ‰Π΅Π½ΠΈΠΈ Ρ‚ΠΎΡ‡Π΅Ρ‡Π½ΠΎΠ³ΠΎ заряда q2 ΠΈΠ· бСсконСчности Π² Π΄Π°Π½Π½ΡƒΡŽ Ρ‚ΠΎΡ‡ΠΊΡƒ поля, созданного зарядом q1

Аналогично Π’ΠΎΠ³Π΄Π° энСргия взаимодСйствия Π΄Π²ΡƒΡ… Ρ‚ΠΎΡ‡Π΅Ρ‡Π½Ρ‹Ρ… зарядов

ЭнСргия взаимодСйствия n зарядов

ΠΏΠΎΡ‚Π΅Π½Ρ†ΠΈΠ°Π» элСктростатичСский — это… Π§Ρ‚ΠΎ Ρ‚Π°ΠΊΠΎΠ΅ ΠΏΠΎΡ‚Π΅Π½Ρ†ΠΈΠ°Π» элСктростатичСский?

энСргСтичСская характСристика элСктростатичСского поля; Ρ€Π°Π²Π΅Π½ ΠΎΡ‚Π½ΠΎΡˆΠ΅Π½ΠΈΡŽ ΠΏΠΎΡ‚Π΅Π½Ρ†ΠΈΠ°Π»ΡŒΠ½ΠΎΠΉ энСргии заряда Π² этом ΠΏΠΎΠ»Π΅ ΠΊ Π²Π΅Π»ΠΈΡ‡ΠΈΠ½Π΅ заряда. ЀизичСский смысл ΠΈΠΌΠ΅Π΅Ρ‚ Π½Π΅ сам ΠΏΠΎΡ‚Π΅Π½Ρ†ΠΈΠ°Π», опрСдСляСмый с Ρ‚ΠΎΡ‡Π½ΠΎΡΡ‚ΡŒΡŽ Π΄ΠΎ ΠΏΡ€ΠΎΠΈΠ·Π²ΠΎΠ»ΡŒΠ½ΠΎΠΉ постоянной, Π° Ρ€Π°Π·Π½ΠΎΡΡ‚ΡŒ ΠΏΠΎΡ‚Π΅Π½Ρ†ΠΈΠ°Π»ΠΎΠ².

ΠŸΠžΠ’Π•ΠΠ¦Π˜ΠΜΠ› Π­Π›Π•ΠšΠ’Π ΠžΠ‘Π’ΠΠ’Π˜ΜΠ§Π•Π‘ΠšΠ˜Π™, энСргСтичСская скалярная энСргСтичСская характСристика элСктростатичСского поля (см. Π­Π›Π•ΠšΠ’Π ΠžΠ‘Π’ΠΠ’Π˜Π§Π•Π‘ΠšΠžΠ• ΠŸΠžΠ›Π•). ΠŸΠΎΡ‚Π΅Π½Ρ†ΠΈΠ°Π» Ρ€Π°Π²Π΅Π½ ΠΎΡ‚Π½ΠΎΡˆΠ΅Π½ΠΈΡŽ ΠΏΠΎΡ‚Π΅Π½Ρ†ΠΈΠ°Π»ΡŒΠ½ΠΎΠΉ энСргии взаимодСйствия заряда с ΠΏΠΎΠ»Π΅ΠΌ ΠΊ Π²Π΅Π»ΠΈΡ‡ΠΈΠ½Π΅ этого заряда.
ΠŸΠΎΡ‚Π΅Π½Ρ†ΠΈΠ°Π» Π² ΠΊΠ°ΠΊΠΎΠΉ-Π»ΠΈΠ±ΠΎ Ρ‚ΠΎΡ‡ΠΊΠ΅ элСктростатичСского поля опрСдСляСт ΠΏΠΎΡ‚Π΅Π½Ρ†ΠΈΠ°Π»ΡŒΠ½ΡƒΡŽ ΡΠ½Π΅Ρ€Π³ΠΈΡŽ (U) Π΅Π΄ΠΈΠ½ΠΈΡ‡Π½ΠΎΠ³ΠΎ ΠΏΠΎΠ»ΠΎΠΆΠΈΡ‚Π΅Π»ΡŒΠ½ΠΎΠ³ΠΎ заряда, ΠΏΠΎΠΌΠ΅Ρ‰Π΅Π½Π½ΠΎΠ³ΠΎ Π² Π΄Π°Π½Π½ΡƒΡŽ Ρ‚ΠΎΡ‡ΠΊΡƒ.
j = U/Q.
ΠŸΠΎΡ‚Π΅Π½Ρ†ΠΈΠ°Π» j измСряСтся Ρ€Π°Π±ΠΎΡ‚ΠΎΠΉ, ΡΠΎΠ²Π΅Ρ€ΡˆΠ°Π΅ΠΌΠΎΠΉ силами поля ΠΏΡ€ΠΈ ΠΏΠ΅Ρ€Π΅ΠΌΠ΅Ρ‰Π΅Π½ΠΈΠΈ Π΅Π΄ΠΈΠ½ΠΈΡ‡Π½ΠΎΠ³ΠΎ ΠΏΠΎΠ»ΠΎΠΆΠΈΡ‚Π΅Π»ΡŒΠ½ΠΎΠ³ΠΎ заряда ΠΈΠ· Π΄Π°Π½Π½ΠΎΠΉ Ρ‚ΠΎΡ‡ΠΊΠΈ Π² Π½Π΅ΠΊΠΎΡ‚ΠΎΡ€ΡƒΡŽ Ρ‚ΠΎΡ‡ΠΊΡƒ, ΠΏΠΎΡ‚Π΅Π½Ρ†ΠΈΠ°Π» ΠΊΠΎΡ‚ΠΎΡ€ΠΎΠΉ принят Ρ€Π°Π²Π½Ρ‹ΠΌ Π½ΡƒΠ»ΡŽ.
ΠŸΠΎΡ‚Π΅Π½Ρ†ΠΈΠ°Π» поля j Ρ‚ΠΎΡ‡Π΅Ρ‡Π½ΠΎΠ³ΠΎ заряда Q, ΡΠΎΠ·Π΄Π°ΡŽΡ‰Π΅Π³ΠΎ ΠΏΠΎΠ»Π΅ Π½Π° расстоянии ΠΎΡ‚ Π΄Π°Π½Π½ΠΎΠΉ Ρ‚ΠΎΡ‡ΠΊΠΈ r, Ρ€Π°Π²Π΅Π½:
j = Q/4peΠΎr.
Π“Π΄Π΅ eΠΎ β€” элСктричСская постоянная, = 8,85.10-12Π€/ΠΌ.
ΠŸΠΎΠ²Π΅Ρ€Ρ…Π½ΠΎΡΡ‚ΠΈ, Π²ΠΎ всСх Ρ‚ΠΎΡ‡ΠΊΠ°Ρ… ΠΊΠΎΡ‚ΠΎΡ€ΠΎΠΉ ΠΏΠΎΡ‚Π΅Π½Ρ†ΠΈΠ°Π» элСктричСского поля ΠΈΠΌΠ΅Π΅Ρ‚ ΠΎΠ΄ΠΈΠ½Π°ΠΊΠΎΠ²ΠΎΠ΅ Π·Π½Π°Ρ‡Π΅Π½ΠΈΠ΅, Π½Π°Π·Ρ‹Π²Π°ΡŽΡ‚ΡΡ ΡΠΊΠ²ΠΈΠΏΠΎΡ‚Π΅Π½Ρ†ΠΈΠ°Π»ΡŒΠ½Ρ‹ΠΌΠΈ повСрхностями (см. Π­ΠšΠ’Π˜ΠŸΠžΠ’Π•ΠΠ¦Π˜ΠΠ›Π¬ΠΠΠ― ΠŸΠžΠ’Π•Π Π₯НОБВЬ).
ЀизичСский смысл ΠΈΠΌΠ΅Π΅Ρ‚ Π½Π΅ сам ΠΏΠΎΡ‚Π΅Π½Ρ†ΠΈΠ°Π», опрСдСляСмый, ΠΏΠΎΠ΄ΠΎΠ±Π½ΠΎ ΠΏΠΎΡ‚Π΅Π½Ρ†ΠΈΠ°Π»ΡŒΠ½ΠΎΠΉ энСргии, с Ρ‚ΠΎΡ‡Π½ΠΎΡΡ‚ΡŒΡŽ Π΄ΠΎ ΠΏΡ€ΠΎΠΈΠ·Π²ΠΎΠ»ΡŒΠ½ΠΎΠΉ постоянной, Π° Ρ€Π°Π·Π½ΠΎΡΡ‚ΡŒ ΠΏΠΎΡ‚Π΅Π½Ρ†ΠΈΠ°Π»ΠΎΠ². ΠŸΠΎΡ‚Π΅Π½Ρ†ΠΈΠ°Π» Ρ…Π°Ρ€Π°ΠΊΡ‚Π΅Ρ€ΠΈΠ·ΡƒΠ΅Ρ‚ Ρ€Π°Π±ΠΎΡ‚Ρƒ сил поля ΠΏΠΎ ΠΏΠ΅Ρ€Π΅ΠΌΠ΅Ρ‰Π΅Π½ΠΈΡŽ заряда, ΠΈ Π½Π΅ зависит ΠΎΡ‚ Ρ„ΠΎΡ€ΠΌΡ‹ Ρ‚Ρ€Π°Π΅ΠΊΡ‚ΠΎΡ€ΠΈΠΈ, Ρ‚. Π΅. ΠΏΠΎΡ‚Π΅Π½Ρ†ΠΈΠ°Π» опрСдСляСтся ΠΏΠΎΠ»ΠΎΠΆΠ΅Π½ΠΈΠ΅ΠΌ Π½Π°Ρ‡Π°Π»ΡŒΠ½ΠΎΠΉ ΠΈ ΠΊΠΎΠ½Π΅Ρ‡Π½ΠΎΠΉ Ρ‚ΠΎΡ‡Π΅ΠΊ.
ΠŸΠΎΡ‚Π΅Π½Ρ†ΠΈΠ°Π» элСктростатичСского поля, создаваСмого нСсколькими зарядами, Ρ€Π°Π²Π΅Π½ алгСбраичСской суммС ΠΏΠΎΡ‚Π΅Π½Ρ†ΠΈΠ°Π»ΠΎΠ², создаваСмых Π² этой Ρ‚ΠΎΡ‡ΠΊΠ΅ ΠΊΠ°ΠΆΠ΄Ρ‹ΠΌ ΠΈΠ· зарядов.
ΠΠ°ΠΏΡ€ΡΠΆΠ΅Π½Π½ΠΎΡΡ‚ΡŒ элСктростатичСского поля Π• ΠΈ ΠΏΠΎΡ‚Π΅Π½Ρ†ΠΈΠ°Π» j связаны ΡΠΎΠΎΡ‚Π½ΠΎΡˆΠ΅Π½ΠΈΠ΅ΠΌ:
Π•= -gradj.
Π•Π΄ΠΈΠ½ΠΈΡ†Π΅ΠΉ измСрСния ΠΏΠΎΡ‚Π΅Π½Ρ†ΠΈΠ°Π»Π° являСтся Π’ β€” Π²ΠΎΠ»ΡŒΡ‚ (см. Π’ΠžΠ›Π¬Π’). 1Π’ β€” ΠΏΠΎΡ‚Π΅Π½Ρ†ΠΈΠ°Π» Ρ‚Π°ΠΊΠΎΠΉ Ρ‚ΠΎΡ‡ΠΊΠΈ поля, Π² ΠΊΠΎΡ‚ΠΎΡ€ΠΎΠΉ заряд 1Кл ΠΎΠ±Π»Π°Π΄Π°Π΅Ρ‚ ΠΏΠΎΡ‚Π΅Π½Ρ†ΠΈΠ°Π»ΡŒΠ½ΠΎΠΉ энСргиСй 1Π”ΠΆ.
1Π’ = 1Π”ΠΆ/Кл.

ΠŸΠΎΡ‚Π΅Π½Ρ†ΠΈΠ°Π» элСктростатичСского поля. Π­ΠΊΠ²ΠΈΠΏΠΎΡ‚Π΅Π½Ρ†ΠΈΠ°Π»ΡŒΠ½Ρ‹Π΅ повСрхности

Π Π°Π±ΠΎΡ‚Π° ΠΏΠΎ ΠΏΠ΅Ρ€Π΅ΠΌΠ΅Ρ‰Π΅Π½ΠΈΡŽ заряда ΠΈΠ· Ρ‚ΠΎΡ‡ΠΊΠΈ А Π² Ρ‚ΠΎΡ‡ΠΊΡƒ Π’ зависит Ρ‚ΠΎΠ»ΡŒΠΊΠΎ ΠΎΡ‚ полоТСния Ρ‚ΠΎΡ‡Π΅ΠΊ А ΠΈ Π’ ΠΈ Π½Π΅ зависит ΠΎΡ‚ Ρ„ΠΎΡ€ΠΌΡ‹ ΠΏΡƒΡ‚ΠΈ, ΠΏΠΎ ΠΊΠΎΡ‚ΠΎΡ€ΠΎΠΌΡƒ двиТСтся ΠΏΡ€ΠΎΠ±Π½Ρ‹ΠΉ заряд. Π˜ΡΡ…ΠΎΠ΄Ρ ΠΈΠ· этого Ρ€Π°Π±ΠΎΡ‚Π°, ΠΏΠΎ ΠΏΠ΅Ρ€Π΅ΠΌΠ΅Ρ‰Π΅Π½ΠΈΡŽ заряда, Π±ΡƒΠ΄Π΅Ρ‚ Ρ€Π°Π²Π½Π° ΡƒΠ±Ρ‹Π»ΠΈ ΠΏΠΎΡ‚Π΅Π½Ρ†ΠΈΠ°Π»ΡŒΠ½ΠΎΠΉ энСргии W Π΄Π°Π½Π½ΠΎΠ³ΠΎ заряда:

Если Ρ€Π°Π±ΠΎΡ‚Π° зависит Ρ‚ΠΎΠ»ΡŒΠΊΠΎ ΠΎΡ‚ полоТСния Π½Π°Ρ‡Π°Π»Π° ΠΈΒ  ΠΊΠΎΠ½Ρ†Π° ΠΏΡƒΡ‚ΠΈ Π² элСктростатичСском ΠΏΠΎΠ»Π΅, Ρ‚ΠΎ ΠΎΠ½Π° ΠΌΠΎΠΆΠ΅Ρ‚ Π±Ρ‹Ρ‚ΡŒ Π²Ρ‹Ρ€Π°ΠΆΠ΅Π½Π° ΠΊΠ°ΠΊ Ρ€Π°Π·Π½ΠΈΡ†Π° Π΄Π²ΡƒΡ… чисСл.

Π’ΠΎΠ·ΡŒΠΌΠ΅ΠΌ ΠΏΡ€ΠΎΠΈΠ·Π²ΠΎΠ΄Π½ΡƒΡŽ Ρ‚ΠΎΡ‡ΠΊΡƒ М ΠΈ ΠΎΠ±ΠΎΠ·Π½Π°Ρ‡ΠΈΠΌ Ρ€Π°Π±ΠΎΡ‚Ρƒ ΠΏΠΎ ΠΏΠ΅Ρ€Π΅ΠΌΠ΅Ρ‰Π΅Π½ΠΈΡŽ ΠΏΡ€ΠΎΠ±Π½ΠΎΠ³ΠΎ заряда qΠΏΡ€ ΠΎΡ‚ М ΠΊ А Ρ‡Π΅Ρ€Π΅Π· Ο†(А), Π° ΠΎΡ‚ М ΠΊ Π’ Ρ‡Π΅Ρ€Π΅Π· Ο†(Π’). ПослС Ρ‡Π΅Π³ΠΎ Π±ΡƒΠ΄Π΅Ρ‚ осущСствлСно ΠΏΠ΅Ρ€Π΅ΠΌΠ΅Ρ‰Π΅Π½ΠΈΠ΅ Π΄Π°Π½Π½ΠΎΠ³ΠΎ заряда ΠΎΡ‚ А ΠΊ Π’ ΠΏΠΎ ΠΏΡƒΡ‚ΠΈ А- М – Π’.

Π’Π°ΠΊ ΠΊΠ°ΠΊ Ρ€Π°Π±ΠΎΡ‚Ρƒ ΠΏΠ΅Ρ€Π΅Ρ…ΠΎΠ΄Π° М – А ΠΌΡ‹ ΠΎΠ±ΠΎΠ·Π½Π°Ρ‡ΠΈΠ»ΠΈ ΠΊΠ°ΠΊ Ο†(А), Ρ‚ΠΎ ΠΎΠ±Ρ€Π°Ρ‚Π½Ρ‹ΠΉ ΠΏΠ΅Ρ€Π΅Ρ…ΠΎΠ΄ А – М Ρ‚Π°ΠΊΠΆΠ΅ Π±ΡƒΠ΄Π΅Ρ‚ Ο†(А), ΠΈΠ· Ρ‡Π΅Π³ΠΎ слСдуСт Ρ„ΠΎΡ€ΠΌΡƒΠ»Π°:

ПолоТСниС Ρ‚ΠΎΡ‡ΠΊΠΈ М ΠΏΠΎ сути Π±Π΅Π·Ρ€Π°Π·Π»ΠΈΡ‡Π½ΠΎ, Ρ‚Π°ΠΊ ΠΊΠ°ΠΊ Π² Π΄Π°Π½Π½ΠΎΠΌ случаС ΠΈΠ³Ρ€Π°Π΅Ρ‚ Ρ€ΠΎΠ»ΡŒ Ρ‚ΠΎΠ»ΡŒΠΊΠΎ Ρ€Π°Π·Π½ΠΎΡΡ‚ΡŒ Π·Π½Π°Ρ‡Π΅Π½ΠΈΠΉ Ρ„ΡƒΠ½ΠΊΡ†ΠΈΠΉ Ο†. Однако, Π·Π°Π΄Π°Π² ΠΊΠΎΠΎΡ€Π΄ΠΈΠ½Π°Ρ‚Ρ‹ Ρ‚ΠΎΡ‡ΠΊΠΈ М ΠΌΡ‹ ΠΎΠ΄Π½ΠΎΠ·Π½Π°Ρ‡Π½ΠΎ ΠΎΠΏΡ€Π΅Π΄Π΅Π»ΠΈΠΌ Π²Π΅Π»ΠΈΡ‡ΠΈΠ½Ρ‹ Ρ„ΡƒΠ½ΠΊΡ†ΠΈΠΉ Ο†(А) ΠΈ Ο†(Π’), хотя Π½Π° Π²Π΅Π»ΠΈΡ‡ΠΈΠ½Ρƒ разности Ο†(А) β€” Ο†(Π’) ΠΏΠΎΠ»ΠΎΠΆΠ΅Π½ΠΈΠ΅ Ρ‚ΠΎΡ‡ΠΊΠΈ М Π½ΠΈΠΊΠ°ΠΊ Π½Π΅ влияСт. Как Ρ‚ΠΎΠ»ΡŒΠΊΠΎ ΠΊΠΎΠΎΡ€Π΄ΠΈΠ½Π°Ρ‚Ρ‹ Ρ‚ΠΎΡ‡ΠΊΠΈ М Π²Ρ‹Π±Ρ€Π°Π½Ρ‹, число Ο† опрСдСляСтся Π² любой Ρ‚ΠΎΡ‡ΠΊΠ΅ пространства.

ΠžΡ‚ΡΡŽΠ΄Π° слСдуСт Π²Π°ΠΆΠ½Ρ‹ΠΉ Π²Ρ‹Π²ΠΎΠ΄ – Π²Π΅Π»ΠΈΡ‡ΠΈΠ½Π° Ο† являСтся Ρ„ΡƒΠ½ΠΊΡ†ΠΈΠ΅ΠΉ ΠΊΠΎΠΎΡ€Π΄ΠΈΠ½Π°Ρ‚ x, y, z ΠΈ скаляром элСктростатичСского поля. Данная скалярная функция Ο† называСтся ΠΏΠΎΡ‚Π΅Π½Ρ†ΠΈΠ°Π»ΠΎΠΌ элСктростатичСского поля. Π’ΠΎΡ‡ΠΊΠ° отсчСта М для удобства расчСтов помСщаСтся Π² Π±Π΅ΡΠΊΠΎΠ½Π΅Ρ‡Π½ΠΎΡΡ‚ΡŒ. ΠŸΠΎΡ‚Π΅Π½Ρ†ΠΈΠ°Π» бСсконСчно ΡƒΠ΄Π°Π»Π΅Π½Π½ΠΎΠΉ Ρ‚ΠΎΡ‡ΠΊΠΈ ΠΏΡ€ΠΈΠ½ΠΈΠΌΠ°ΡŽΡ‚ Ρ€Π°Π²Π½Ρ‹ΠΌ Π½ΡƒΠ»ΡŽ Ο† = 0.

ЀизичСская Π²Π΅Π»ΠΈΡ‡ΠΈΠ½Π°, которая Ρ€Π°Π²Π½Π° ΠΎΡ‚Π½ΠΎΡˆΠ΅Π½ΠΈΡŽ ΠΏΠΎΡ‚Π΅Π½Ρ†ΠΈΠ°Π»ΡŒΠ½ΠΎΠΉ энСргии, ΠΏΡ€ΠΈΠΎΠ±Ρ€Π΅Ρ‚Π°Π΅ΠΌΠΎΠΉ ΠΏΠΎΠ»ΠΎΠΆΠΈΡ‚Π΅Π»ΡŒΠ½Ρ‹ΠΌ зарядом qΠΏΡ€, ΠΏΡ€ΠΈ Π΅Π³ΠΎ пСрСносС ΠΈΠ· бСсконСчности Π² Π΄Π°Π½Π½ΡƒΡŽ Ρ‚ΠΎΡ‡ΠΊΡƒ пространства ΠΊ этому заряду, Ρ‚ΠΎ Π΅ΡΡ‚ΡŒ:

ΠŸΠΎΡ‚Π΅Π½Ρ†ΠΈΠ°Π» – это энСргСтичСская характСристика поля. ЧислСнно ΠΎΠ½ Ρ€Π°Π²Π΅Π½ Ρ€Π°Π±ΠΎΡ‚Π΅, ΠΊΠΎΡ‚ΠΎΡ€ΡƒΡŽ Π½ΡƒΠΆΠ½ΠΎ ΡΠΎΠ²Π΅Ρ€ΡˆΠΈΡ‚ΡŒ ΠΏΡ€ΠΈ пСрСнСсСнии Π΅Π΄ΠΈΠ½ΠΈΡ‡Π½ΠΎΠ³ΠΎ заряда ΠΈΠ· бСсконСчности, Π³Π΄Π΅ ΠΏΠΎΡ‚Π΅Π½Ρ†ΠΈΠ°Π»ΡŒΠ½Π°Ρ энСргия считаСтся Ρ€Π°Π²Π½ΠΎΠΉ Π½ΡƒΠ»ΡŽ, Π² Π΄Π°Π½Π½ΡƒΡŽ Ρ‚ΠΎΡ‡ΠΊΡƒ поля.

Из Ρ„ΠΎΡ€ΠΌΡƒΠ» (2) ΠΈ (Ρ„ΠΎΡ€ΠΌΡƒΠ»Ρ‹ 3 ΠΏΡ€ΠΈΠ²Π΅Π΄Π΅Π½Π½ΠΎΠΉ ΠΏΠΎ ΡΠ»Π΅Π΄ΡƒΡŽΡ‰Π΅ΠΉ ссылкС)Β ΠΏΠΎΠ»ΡƒΡ‡ΠΈΠΌ выраТСния ΠΏΠΎΡ‚Π΅Π½Ρ†ΠΈΠ°Π»Π° поля, ΠΊΠΎΡ‚ΠΎΡ€ΠΎΠ΅ создано Ρ‚ΠΎΡ‡Π΅Ρ‡Π½Ρ‹ΠΌ зарядом:

Когда ΠΏΠΎΠ»Π΅ образуСтся нСсколькими располоТСнными ΠΏΡ€ΠΎΠΈΠ·Π²ΠΎΠ»ΡŒΠ½ΠΎ зарядами q1, q2,… qn, Π΅Π³ΠΎ ΠΏΠΎΡ‚Π΅Π½Ρ†ΠΈΠ°Π» Ο† Π² Π΄Π°Π½Π½ΠΎΠΉ Ρ‚ΠΎΡ‡ΠΊΠ΅ Π±ΡƒΠ΄Π΅Ρ‚ Ρ€Π°Π²Π΅Π½ алгСбраичСской суммС ΠΏΠΎΡ‚Π΅Π½Ρ†ΠΈΠ°Π»ΠΎΠ² Ο†1, Ο†2, … Ο†n, ΠΊΠΎΡ‚ΠΎΡ€Ρ‹Π΅ создаСт ΠΊΠ°ΠΆΠ΄Ρ‹ΠΉ заряд Π² ΠΎΡ‚Π΄Π΅Π»ΡŒΠ½ΠΎΡΡ‚ΠΈ:

Если заряды q1, q2,… qn ΠΌΠΎΠΆΠ½ΠΎ ΡΡ‡ΠΈΡ‚Π°Ρ‚ΡŒ Ρ‚ΠΎΡ‡Π΅Ρ‡Π½Ρ‹ΠΌΠΈ, Ρ‚ΠΎ суммарный ΠΏΠΎΡ‚Π΅Π½Ρ†ΠΈΠ°Π» ΠΌΠΎΠΆΠ½ΠΎ ΠΏΠΎΡΡ‡ΠΈΡ‚Π°Ρ‚ΡŒ ΠΏΠΎ Ρ„ΠΎΡ€ΠΌΡƒΠ»Π΅:

Π“Π΄Π΅ r1, r2, … rn расстояниС ΠΎΡ‚ зарядов q1, q2, … qn Π΄ΠΎ Π΄Π°Π½Π½ΠΎΠΉ Ρ‚ΠΎΡ‡ΠΊΠΈ поля.

Π’ случаС Ссли ΠΏΠΎΠ»Π΅ ΠΎΠ±Ρ€Π°Π·ΠΎΠ²Π°Π½ΠΎ элСктричСским Π΄ΠΈΠΏΠΎΠ»Π΅ΠΌ, Ρ‚ΠΎ ΠΏΠΎΡ‚Π΅Π½Ρ†ΠΈΠ°Π» Π² ΠΊΠ°ΠΊΠΎΠΉ-Π»ΠΈΠ±ΠΎ Ρ‚ΠΎΡ‡ΠΊΠ΅ поля, находящСйся ΠΎΡ‚ Ρ†Π΅Π½Ρ‚Ρ€Π° диполя Π½Π° расстоянии r ΠΌΠΎΠΆΠ½ΠΎ ΠΎΠΏΡ€Π΅Π΄Π΅Π»ΠΈΡ‚ΡŒ ΠΏΠΎ Ρ„ΠΎΡ€ΠΌΡƒΠ»Π΅:

Π“Π΄Π΅ Ρ€ = qΒ·l – элСктричСский ΠΌΠΎΠΌΠ΅Π½Ρ‚ диполя (Π³Π΄Π΅ l – это ΠΏΠ»Π΅Ρ‡ΠΎ диполя), Π° Ξ± – ΡƒΠ³ΠΎΠ» ΠΌΠ΅ΠΆΠ΄Ρƒ ΠΏΠ»Π΅Ρ‡ΠΎΠΌ диполя l ΠΈ радиус Π²Π΅ΠΊΡ‚ΠΎΡ€ΠΎΠΌ r.

Π’ случаС, ΠΊΠΎΠ³Π΄Π° Ρ‚ΠΎΡ‡ΠΊΠ° Π»Π΅ΠΆΠΈΡ‚ Π½Π° оси диполя Ξ± = 0, ΠΏΠΎΡ‚Π΅Π½Ρ†ΠΈΠ°Π» Π² этой Ρ‚ΠΎΡ‡ΠΊΠ΅ Π±ΡƒΠ΄Π΅Ρ‚ Ρ€Π°Π²Π΅Π½:

Π›Π΅ΠΆΠ°Ρ‰ΠΈΠ΅ Π½Π° пСрпСндикулярС ΠΊ ΠΏΠ»Π΅Ρ‡Ρƒ диполя Ρ‚ΠΎΡ‡ΠΊΠΈ, восстановлСны с Π΅Π³ΠΎ сСрСдины, ΠΈΠΌΠ΅ΡŽΡ‚ Π½ΡƒΠ»Π΅Π²ΠΎΠΉ ΠΏΠΎΡ‚Π΅Π½Ρ†ΠΈΠ°Π» (Ο† = 0), Ρ‚Π°ΠΊ ΠΊΠ°ΠΊ Ξ± = 900.

Если ΠΈΠ· Ρ‚ΠΎΡ‡ΠΊΠΈ А Π² Ρ‚ΠΎΡ‡ΠΊΡƒ Π’ элСктростатичСского поля пСрСмСщаСтся заряд q/, Ρ‚ΠΎ ΠΏΡ€ΠΈ этом ΡΠΎΠ²Π΅Ρ€ΡˆΠ°Π΅Ρ‚ΡΡ Ρ€Π°Π±ΠΎΡ‚Π° ΠΏΡ€ΠΎΡ‚ΠΈΠ² элСктричСских сил:

Π“Π΄Π΅ Ο†1 ΠΈ Ο†2 ΠΏΠΎΡ‚Π΅Π½Ρ†ΠΈΠ°Π»Ρ‹ Π² Ρ‚ΠΎΡ‡ΠΊΠ°Ρ… А ΠΈ Π’ ΠΈΠ»ΠΈ

ΠžΡ‚ΡΡŽΠ΄Π° слСдуСт, Ρ‡Ρ‚ΠΎ ΡΠΎΠ²Π΅Ρ€ΡˆΠ°Π΅ΠΌΠ°Ρ ΠΏΠΎΠ»Π΅ΠΌ Ρ€Π°Π±ΠΎΡ‚Π° ΠΏΠΎ ΠΏΠ΅Ρ€Π΅ΠΌΠ΅Ρ‰Π΅Π½ΠΈΡŽ заряда измСряСтся ΠΏΡ€ΠΎΠΈΠ·Π²Π΅Π΄Π΅Π½ΠΈΠ΅ΠΌ заряда q/, пСрСносимого Π² элСктростатичСском ΠΏΠΎΠ»Π΅, Π½Π° Ρ€Π°Π·Π½ΠΎΡΡ‚ΡŒ ΠΏΠΎΡ‚Π΅Π½Ρ†ΠΈΠ°Π»ΠΎΠ² ΠΊΠΎΠ½Π΅Ρ‡Π½ΠΎΠΉ (Ο†2) ΠΈ Π½Π°Ρ‡Π°Π»ΡŒΠ½ΠΎΠΉ (Ο†1) Ρ‚ΠΎΡ‡Π΅ΠΊ ΠΏΡƒΡ‚ΠΈ ΠΈ Π½ΠΈΠΊΠ°ΠΊ Π½Π΅ зависит ΠΎΡ‚ Ρ„ΠΎΡ€ΠΌΡ‹ ΠΏΡƒΡ‚ΠΈ.

Π‘ΠΎΠ²ΠΎΠΊΡƒΠΏΠ½ΠΎΡΡ‚ΡŒ Ρ‚ΠΎΡ‡Π΅ΠΊ с ΠΎΠ΄ΠΈΠ½Π°ΠΊΠΎΠ²Ρ‹ΠΌ ΠΏΠΎΡ‚Π΅Π½Ρ†ΠΈΠ°Π»ΠΎΠΌ ΠΎΠ±Ρ€Π°Π·ΡƒΡŽΡ‚ ΡΠΊΠ²ΠΈΠΏΠΎΡ‚Π΅Π½Ρ†ΠΈΠ°Π»ΡŒΠ½ΡƒΡŽ ΠΏΠΎΠ²Π΅Ρ€Ρ…Π½ΠΎΡΡ‚ΡŒ ΠΈΠ»ΠΈ ΠΏΠΎΠ²Π΅Ρ€Ρ…Π½ΠΎΡΡ‚ΡŒ Ρ€Π°Π²Π½ΠΎΠ³ΠΎ ΠΏΠΎΡ‚Π΅Π½Ρ†ΠΈΠ°Π»Π° (Ο† = const). Π‘ ΠΏΠΎΠΌΠΎΡ‰ΡŒΡŽ Π΄Π°Π½Π½Ρ‹Ρ… Ρ‚ΠΎΡ‡Π΅ΠΊ ΡΠΊΠ²ΠΈΠΏΠΎΡ‚Π΅Π½Ρ†ΠΈΠ°Π»ΡŒΠ½ΡƒΡŽ ΠΏΠΎΠ²Π΅Ρ€Ρ…Π½ΠΎΡΡ‚ΡŒ ΠΌΠΎΠΆΠ½ΠΎ ΠΈΠ·ΠΎΠ±Ρ€Π°Π·ΠΈΡ‚ΡŒ графичСски.

На рисункС Π½ΠΈΠΆΠ΅ ΠΈΠ·ΠΎΠ±Ρ€Π°ΠΆΠ΅Π½ΠΎ элСктричСскоС ΠΏΠΎΠ»Π΅ Ρ€Π°Π²Π½ΠΎΠΌΠ΅Ρ€Π½ΠΎ заряТСнного диска, Π³Π΄Π΅ ΠΏΡƒΠ½ΠΊΡ‚ΠΈΡ€Π½Ρ‹Π΅ Π»ΠΈΠ½ΠΈΠΈ – ΡΠΊΠ²ΠΈΠΏΠΎΡ‚Π΅Π½Ρ†ΠΈΠ°Π»ΡŒΠ½Ρ‹Π΅ повСрхности, Π° ΡΠΏΠ»ΠΎΡˆΠ½Ρ‹Π΅ – Π»ΠΈΠ½ΠΈΠΈ напряТСнности.

Π”Π°Π½Π½Ρ‹ΠΉ рисунок ΠΈΠ»Π»ΡŽΡΡ‚Ρ€ΠΈΡ€ΡƒΠ΅Ρ‚ ΠΎΠ±Ρ‰Π΅Π΅ свойство ΡΠΊΠ²ΠΈΠΏΠΎΡ‚Π΅Π½Ρ†ΠΈΠ°Π»ΡŒΠ½Ρ‹Ρ… повСрхностСй ΠΈ силовых Π»ΠΈΠ½ΠΈΠΉ – ΡΠΊΠ²ΠΈΠΏΠΎΡ‚Π΅Π½Ρ†ΠΈΠ°Π»ΡŒΠ½Π°Ρ ΠΏΠΎΠ²Π΅Ρ€Ρ…Π½ΠΎΡΡ‚ΡŒ ΠΈ силовая линия, провСдСнная Ρ‡Π΅Ρ€Π΅Π· Π»ΡŽΠ±ΡƒΡŽ Ρ‚ΠΎΡ‡ΠΊΡƒ, Π² Π΄Π°Π½Π½ΠΎΠΉ Ρ‚ΠΎΡ‡ΠΊΠ΅ Π²Π·Π°ΠΈΠΌΠ½ΠΎ пСрпСндикулярны. ΠŸΠΎΡΠΊΠΎΠ»ΡŒΠΊΡƒ всС Ρ‚ΠΎΡ‡ΠΊΠΈ ΡΠΊΠ²ΠΈΠΏΠΎΡ‚Π΅Π½Ρ†ΠΈΠ°Π»ΡŒΠ½ΠΎΠΉ повСрхности ΠΈΠΌΠ΅ΡŽΡ‚ ΠΎΠ΄ΠΈΠ½Π°ΠΊΠΎΠ²Ρ‹ΠΉ ΠΏΠΎΡ‚Π΅Π½Ρ†ΠΈΠ°Π» (Ο†1 – Ο†2 = 0), Ρ‚ΠΎ Ρ€Π°Π±ΠΎΡ‚Π° Π½Π΅ ΡΠΎΠ²Π΅Ρ€ΡˆΠ°Π΅Ρ‚ΡΡ ΠΏΡ€ΠΈ ΠΏΠ΅Ρ€Π΅ΠΌΠ΅Ρ‰Π΅Π½ΠΈΠΈ заряда вдоль Π½Π΅Π΅. Из этого слСдуСт, Ρ‡Ρ‚ΠΎ Π΄Π΅ΠΉΡΡ‚Π²ΡƒΡŽΡ‰ΠΈΠΉ Π½Π° заряд Π²Π΅ΠΊΡ‚ΠΎΡ€ силы, Π° Π·Π½Π°Ρ‡ΠΈΡ‚ ΠΈ Π²Π΅ΠΊΡ‚ΠΎΡ€ напряТСнности всС врСмя пСрпСндикулярСн ΠΊ ΠΏΠ΅Ρ€Π΅ΠΌΠ΅Ρ‰Π΅Π½ΠΈΡŽ.

Если заряд пСрСмСщаСтся ΠΏΠΎ ΡΠΊΠ²ΠΈΠΏΠΎΡ‚Π΅Π½Ρ†ΠΈΠ°Π»ΡŒΠ½ΠΎΠΉ повСрхности (Ο† = const), Ρ‚ΠΎ Ρ€Π°Π±ΠΎΡ‚Π° поля Π±ΡƒΠ΄Π΅Ρ‚ Ρ€Π°Π²Π½Π° Π½ΡƒΠ»ΡŽ:

Π’ ΠΎΠ±Ρ‰Π΅ΠΌ случаС ΡΠΎΠ²Π΅Ρ€ΡˆΠ°Π΅ΠΌΠ°Ρ ΠΏΠΎΠ»Π΅ΠΌ Ρ€Π°Π±ΠΎΡ‚Π° ΠΏΠΎ ΠΏΠ΅Ρ€Π΅ΠΌΠ΅Ρ‰Π΅Π½ΠΈΡŽ заряда q/ Π±ΡƒΠ΄Π΅Ρ‚ Ρ€Π°Π²Π½Π°:

Π“Π΄Π΅ dS – элСмСнтарноС ΠΏΠ΅Ρ€Π΅ΠΌΠ΅Ρ‰Π΅Π½ΠΈΠ΅, Π° Π• – проСкция Π²Π΅ΠΊΡ‚ΠΎΡ€Π° напряТСнности Π• Π½Π° Π½Π°ΠΏΡ€Π°Π²Π»Π΅Π½ΠΈΠ΅ пСрСмСщСния.

Π’ Ρ€Π΅Π·ΡƒΠ»ΡŒΡ‚Π°Ρ‚Π΅ интСгрирования выраТСния (11) для ΠΎΠ΄Π½ΠΎΡ€ΠΎΠ΄Π½ΠΎΠ³ΠΎ поля ΠΏΠΎΠ»ΡƒΡ‡ΠΈΠΌ:

Π“Π΄Π΅ S – ΠΏΡƒΡ‚ΡŒ, Π° Ξ± – ΡƒΠ³ΠΎΠ» ΠΌΠ΅ΠΆΠ΄Ρƒ Π½Π°ΠΏΡ€Π°Π²Π»Π΅Π½ΠΈΠ΅ΠΌ Π²Π΅ΠΊΡ‚ΠΎΡ€Π° Π• ΠΈ пСрСмСщСния.

ΠŸΠΎΡ‚Π΅Π½Ρ†ΠΈΠ°Π» элСктричСского поля

ΠŸΠΎΡ‚Π΅Π½Ρ†ΠΈΠ°Π». Π­ΠΊΠ²ΠΈΠΏΠΎΡ‚Π΅Π½Ρ†ΠΈΠ°Π»ΡŒΠ½Ρ‹Π΅ повСрхности.

Π’ ΠΌΠ΅Ρ…Π°Π½ΠΈΠΊΠ΅ взаимодСйствиС Ρ‚Π΅Π» Ρ…Π°Ρ€Π°ΠΊΡ‚Π΅Ρ€ΠΈΠ·ΡƒΠ΅Ρ‚ силой ΠΈΠ»ΠΈ ΠΏΠΎΡ‚Π΅Π½Ρ†ΠΈΠ°Π»ΡŒΠ½ΠΎΠΉ энСргиСй. ЭлСктричСскоС ΠΏΠΎΠ»Π΅, ΠΊΠΎΡ‚ΠΎΡ€ΠΎΠ΅ обСспСчиваСт взаимодСйствиС ΠΌΠ΅ΠΆΠ΄Ρƒ элСктричСски заряТСнными Ρ‚Π΅Π»Π°ΠΌΠΈ, Ρ‚Π°ΠΊΠΆΠ΅ Ρ…Π°Ρ€Π°ΠΊΡ‚Π΅Ρ€ΠΈΠ·ΡƒΡŽΡ‚ двумя Π²Π΅Π»ΠΈΡ‡ΠΈΠ½Π°ΠΌΠΈ. ΠΠ°ΠΏΡ€ΡΠΆΠ΅Π½Π½ΠΎΡΡ‚ΡŒ элСктричСского поля β€” это силовая характСристика. Π’Π΅ΠΏΠ΅Ρ€ΡŒ Π²Π²Π΅Π΄Π΅ΠΌ ΡΠ½Π΅Ρ€Π³Π΅Ρ‚ΠΈΡ‡Π΅ΡΠΊΡƒΡŽ характСристику β€” ΠΏΠΎΡ‚Π΅Π½Ρ†ΠΈΠ°Π». Π‘ ΠΏΠΎΠΌΠΎΡ‰ΡŒΡŽ этой Π²Π΅Π»ΠΈΡ‡ΠΈΠ½Ρ‹ ΠΌΠΎΠΆΠ½ΠΎ Π±ΡƒΠ΄Π΅Ρ‚ ΡΡ€Π°Π²Π½ΠΈΠ²Π°Ρ‚ΡŒ ΠΌΠ΅ΠΆΠ΄Ρƒ собой Π»ΡŽΠ±Ρ‹Π΅ Ρ‚ΠΎΡ‡ΠΊΠΈ элСктричСского поля. Π’Π°ΠΊΠΈΠΌ ΠΎΠ±Ρ€Π°Π·ΠΎΠΌ, ΠΏΠΎΡ‚Π΅Π½Ρ†ΠΈΠ°Π» ΠΊΠ°ΠΊ характСристика поля Π΄ΠΎΠ»ΠΆΠ΅Π½ Π·Π°Π²ΠΈΡΠ΅Ρ‚ΡŒ ΠΎΡ‚ значСния заряда, содСрТащСгося Π² этих Ρ‚ΠΎΡ‡ΠΊΠ°Ρ…. ПодСлим ΠΎΠ±Π΅ части Ρ„ΠΎΡ€ΠΌΡƒΠ»Ρ‹ A = W1 β€” W2 Π½Π° заряд q, ΠΏΠΎΠ»ΡƒΡ‡ΠΈΠΌ

ΠžΡ‚Π½ΠΎΡˆΠ΅Π½ΠΈΠ΅ W/q Π½Π΅ зависит ΠΎΡ‚ значСния заряда ΠΈ принимаСтся Π·Π° ΡΠ½Π΅Ρ€Π³Π΅Ρ‚ΠΈΡ‡Π΅ΡΠΊΡƒΡŽ характСристику, ΠΊΠΎΡ‚ΠΎΡ€ΡƒΡŽ Π½Π°Π·Ρ‹Π²Π°ΡŽΡ‚ ΠΏΠΎΡ‚Π΅Π½Ρ†ΠΈΠ°Π»ΠΎΠΌ поля Π² Π΄Π°Π½Π½ΠΎΠΉ Ρ‚ΠΎΡ‡ΠΊΠ΅.Β ΠžΠ±ΠΎΠ·Π½Π°Ρ‡Π°ΡŽΡ‚ ΠΏΠΎΡ‚Π΅Π½Ρ†ΠΈΠ°Π» Π±ΡƒΠΊΠ²ΠΎΠΉ Ο†.

ΠŸΠΎΡ‚Π΅Π½Ρ†ΠΈΠ°Π» элСктричСского поля Ο† β€” скалярная энСргСтичСская характСристика поля, которая опрСдСляСтся ΠΎΡ‚Π½ΠΎΡˆΠ΅Π½ΠΈΠ΅ΠΌ ΠΏΠΎΡ‚Π΅Π½Ρ†ΠΈΠ°Π»ΡŒΠ½ΠΎΠΉ энСргии W ΠΏΠΎΠ»ΠΎΠΆΠΈΡ‚Π΅Π»ΡŒΠ½ΠΎΠ³ΠΎ заряда q Π² Π΄Π°Π½Π½ΠΎΠΉ Ρ‚ΠΎΡ‡ΠΊΠ΅ поля ΠΊ Π²Π΅Π»ΠΈΡ‡ΠΈΠ½Π΅ этого заряда:

Π•Π΄ΠΈΠ½ΠΈΡ†Π° ΠΏΠΎΡ‚Π΅Π½Ρ†ΠΈΠ°Π»Π° β€” Π²ΠΎΠ»ΡŒΡ‚:

Подобно ΠΏΠΎΡ‚Π΅Π½Ρ†ΠΈΠ°Π»ΡŒΠ½ΠΎΠΉ энСргии значСния ΠΏΠΎΡ‚Π΅Π½Ρ†ΠΈΠ°Π»Π° Π² Π΄Π°Π½Π½ΠΎΠΉ Ρ‚ΠΎΡ‡ΠΊΠ΅ зависит ΠΎΡ‚ Π²Ρ‹Π±ΠΎΡ€Π° Π½ΡƒΠ»Π΅Π²ΠΎΠ³ΠΎ уровня для отсчСта ΠΏΠΎΡ‚Π΅Π½Ρ†ΠΈΠ°Π»Π°.Β Π§Π°Ρ‰Π΅ всСго Π² элСктродинамикС Π·Π° Π½ΡƒΠ»Π΅Π²ΠΎΠΉ ΡƒΡ€ΠΎΠ²Π΅Π½ΡŒ Π±Π΅Ρ€ΡƒΡ‚ ΠΏΠΎΡ‚Π΅Π½Ρ†ΠΈΠ°Π» Ρ‚ΠΎΡ‡ΠΊΠΈ, Π»Π΅ΠΆΠ°Ρ‰Π΅ΠΉ Π² бСсконСчности, Π° Π² элСктротСхникС β€” Π½Π° повСрхности Π—Π΅ΠΌΠ»ΠΈ.

Π‘ Π²Π²Π΅Π΄Π΅Π½ΠΈΠ΅ΠΌ ΠΏΠΎΡ‚Π΅Π½Ρ†ΠΈΠ°Π»Π° Ρ„ΠΎΡ€ΠΌΡƒΠ»Ρƒ для опрСдСлСния Ρ€Π°Π±ΠΎΡ‚Ρ‹ ΠΏΠΎ ΠΏΠ΅Ρ€Π΅ΠΌΠ΅Ρ‰Π΅Π½ΠΈΡŽ заряда ΠΌΠ΅ΠΆΠ΄Ρƒ Ρ‚ΠΎΡ‡ΠΊΠ°ΠΌΠΈ 1 ΠΈ 2 ΠΌΠΎΠΆΠ½ΠΎ Π·Π°ΠΏΠΈΡΠ°Ρ‚ΡŒ Π² Π²ΠΈΠ΄Π΅

ΠŸΠΎΡΠΊΠΎΠ»ΡŒΠΊΡƒ ΠΏΡ€ΠΈ ΠΏΠ΅Ρ€Π΅ΠΌΠ΅Ρ‰Π΅Π½ΠΈΠΈ ΠΏΠΎΠ»ΠΎΠΆΠΈΡ‚Π΅Π»ΡŒΠ½ΠΎΠ³ΠΎ заряда Π² Π½Π°ΠΏΡ€Π°Π²Π»Π΅Π½ΠΈΠΈ Π²Π΅ΠΊΡ‚ΠΎΡ€Π° напряТСнности элСктричСскоС ΠΏΠΎΠ»Π΅ выполняСт ΠΏΠΎΠ»ΠΎΠΆΠΈΡ‚Π΅Π»ΡŒΠ½ΡƒΡŽ Ρ€Π°Π±ΠΎΡ‚Ρƒ A = q (Ο†1 β€” Ο†2 )> 0, Ρ‚ΠΎ ΠΏΠΎΡ‚Π΅Π½Ρ†ΠΈΠ°Π» Ο†1 большС Ρ‡Π΅ΠΌ ΠΏΠΎΡ‚Π΅Π½Ρ†ΠΈΠ°Π» Ο†2Β .Β Π’Π°ΠΊΠΈΠΌ ΠΎΠ±Ρ€Π°Π·ΠΎΠΌ, Π½Π°ΠΏΡ€ΡΠΆΠ΅Π½Π½ΠΎΡΡ‚ΡŒ элСктричСского поля Π½Π°ΠΏΡ€Π°Π²Π»Π΅Π½Π° Π² сторону ΡƒΠΌΠ΅Π½ΡŒΡˆΠ΅Π½ΠΈΡ ΠΏΠΎΡ‚Π΅Π½Ρ†ΠΈΠ°Π»Π°.

Если заряд ΠΏΠ΅Ρ€Π΅ΠΌΠ΅Ρ‰Π°Ρ‚ΡŒ с ΠΎΠΏΡ€Π΅Π΄Π΅Π»Π΅Π½Π½ΠΎΠΉ Ρ‚ΠΎΡ‡ΠΊΠΈ поля Π² Π±Π΅ΡΠΊΠΎΠ½Π΅Ρ‡Π½ΠΎΡΡ‚ΡŒ, Ρ‚ΠΎ Ρ€Π°Π±ΠΎΡ‚Π° A = q (Ο† β€” Ο†βˆžΒ ).Β ΠŸΠΎΡΠΊΠΎΠ»ΡŒΠΊΡƒ Ο†βˆžΒ = 0, Ρ‚ΠΎ A = qΟ†.Β Π’Π°ΠΊΠΈΠΌ ΠΎΠ±Ρ€Π°Π·ΠΎΠΌ, Π²Π΅Π»ΠΈΡ‡ΠΈΠ½Π° ΠΏΠΎΡ‚Π΅Π½Ρ†ΠΈΠ°Π»Π° Ο† ΠΎΠΏΡ€Π΅Π΄Π΅Π»Π΅Π½Π½ΠΎΠΉ Ρ‚ΠΎΡ‡ΠΊΠΈ поля опрСдСляСтся Ρ€Π°Π±ΠΎΡ‚ΠΎΠΉ, ΠΊΠΎΡ‚ΠΎΡ€ΡƒΡŽ выполняСт элСктричСскоС ΠΏΠΎΠ»Π΅, пСрСмСщая Π΅Π΄ΠΈΠ½ΠΈΡ‡Π½Ρ‹ΠΉ ΠΏΠΎΠ»ΠΎΠΆΠΈΡ‚Π΅Π»ΡŒΠ½Ρ‹ΠΉ заряд ΠΈΠ· этой Ρ‚ΠΎΡ‡ΠΊΠΈ Π² Π±Π΅ΡΠΊΠΎΠ½Π΅Ρ‡Π½ΠΎΡΡ‚ΡŒ,

Если элСктричСскоС ΠΏΠΎΠ»Π΅ создаСтся Ρ‚ΠΎΡ‡Π΅Ρ‡Π½Ρ‹ΠΌ зарядом q, Ρ‚ΠΎ Π² Ρ‚ΠΎΡ‡ΠΊΠ΅, Π»Π΅ΠΆΠ°Ρ‰Π΅ΠΉ Π½Π° расстоянии r ΠΎΡ‚ Π½Π΅Π³ΠΎ, ΠΏΠΎΡ‚Π΅Π½Ρ†ΠΈΠ°Π» Π²Ρ‹Ρ‡ΠΈΡΠ»ΡΡŽΡ‚ ΠΏΠΎ Ρ„ΠΎΡ€ΠΌΡƒΠ»Π΅

По этой Ρ„ΠΎΡ€ΠΌΡƒΠ»Π΅ Ρ€Π°ΡΡΡ‡ΠΈΡ‚Ρ‹Π²Π°ΡŽΡ‚ ΠΈ ΠΏΠΎΡ‚Π΅Π½Ρ†ΠΈΠ°Π» поля заряТСнного ΡˆΠ°Ρ€Π°.Β Π’ Ρ‚Π°ΠΊΠΎΠΌ случаС r β€” это расстояниС ΠΎΡ‚ Ρ†Π΅Π½Ρ‚Ρ€Π° ΡˆΠ°Ρ€Π° Π΄ΠΎ Π²Ρ‹Π±Ρ€Π°Π½Π½ΠΎΠΉ Ρ‚ΠΎΡ‡ΠΊΠΈ поля.Β Π‘ этой Ρ„ΠΎΡ€ΠΌΡƒΠ»Ρ‹ Π²ΠΈΠ΄Π½ΠΎ, Ρ‡Ρ‚ΠΎ Π½Π° ΠΎΠ΄ΠΈΠ½Π°ΠΊΠΎΠ²Ρ‹Ρ… расстояниях ΠΎΡ‚ Ρ‚ΠΎΡ‡Π΅Ρ‡Π½ΠΎΠ³ΠΎ заряда, ΠΊΠΎΡ‚ΠΎΡ€Ρ‹ΠΉ создаСт ΠΏΠΎΠ»Π΅, ΠΏΠΎΡ‚Π΅Π½Ρ†ΠΈΠ°Π» ΠΎΠ΄ΠΈΠ½Π°ΠΊΠΎΠ². ВсС эти Ρ‚ΠΎΡ‡ΠΊΠΈ Π»Π΅ΠΆΠ°Ρ‚ Π½Π° повСрхности сфСры, описанной радиусом r Π²ΠΎΠΊΡ€ΡƒΠ³ Ρ‚ΠΎΡ‡Π΅Ρ‡Π½ΠΎΠ³ΠΎ заряда.Β Π’Π°ΠΊΡƒΡŽ сфСру Π½Π°Π·Ρ‹Π²Π°ΡŽΡ‚ ΡΠΊΠ²ΠΈΠΏΠΎΡ‚Π΅Π½Ρ†ΠΈΠ°Π»ΡŒΠ½ΠΎΠΉ ΠΏΠΎΠ²Π΅Ρ€Ρ…Π½ΠΎΡΡ‚ΡŒΡŽ.

Π­ΠΊΠ²ΠΈΠΏΠΎΡ‚Π΅Π½Ρ†ΠΈΠ°Π»ΡŒΠ½Ρ‹Π΅ повСрхности β€” гСомСтричСскоС мСсто Ρ‚ΠΎΡ‡Π΅ΠΊ Π² элСктричСском ΠΏΠΎΠ»Π΅, ΠΊΠΎΡ‚ΠΎΡ€Ρ‹Π΅ ΠΈΠΌΠ΅ΡŽΡ‚ ΠΎΠ΄ΠΈΠ½Π°ΠΊΠΎΠ²Ρ‹ΠΉ ΠΏΠΎΡ‚Π΅Π½Ρ†ΠΈΠ°Π», β€” ΠΎΠ΄ΠΈΠ½ ΠΈΠ· ΠΌΠ΅Ρ‚ΠΎΠ΄ΠΎΠ² наглядного изобраТСния элСктричСских ΠΏΠΎΠ»Π΅ΠΉ.

Π­ΠΊΠ²ΠΈΠΏΠΎΡ‚Π΅Π½Ρ†ΠΈΠ°Π»ΡŒΠ½Ρ‹Π΅ повСрхности элСктричСских ΠΏΠΎΠ»Π΅ΠΉ, созданных Ρ‚ΠΎΡ‡Π΅Ρ‡Π½Ρ‹ΠΌΠΈ зарядами Ρ€Π°Π·Π½Ρ‹Ρ… Π·Π½Π°ΠΊΠΎΠ²

Π‘ΠΈΠ»ΠΎΠ²Ρ‹Π΅ Π»ΠΈΠ½ΠΈΠΈ всСгда пСрпСндикулярны ΡΠΊΠ²ΠΈΠΏΠΎΡ‚Π΅Π½Ρ†ΠΈΠ°Π»ΡŒΠ½Ρ‹Ρ… повСрхностСй.Β Π­Ρ‚ΠΎ ΠΎΠ·Π½Π°Ρ‡Π°Π΅Ρ‚, Ρ‡Ρ‚ΠΎ Ρ€Π°Π±ΠΎΡ‚Π° сил поля ΠΏΠΎ ΠΏΠ΅Ρ€Π΅ΠΌΠ΅Ρ‰Π΅Π½ΠΈΡŽ заряда ΠΏΠΎ ΡΠΊΠ²ΠΈΠΏΠΎΡ‚Π΅Π½Ρ†ΠΈΠ°Π»ΡŒΠ½ΠΎΠΉ повСрхности Ρ€Π°Π²Π½Π° Π½ΡƒΠ»ΡŽ.

Π’ случаС налоТСния элСктричСских ΠΏΠΎΠ»Π΅ΠΉ, созданных нСсколькими зарядами, ΠΏΠΎΡ‚Π΅Π½Ρ†ΠΈΠ°Π» элСктричСского поля Ρ€Π°Π²Π΅Π½ алгСбраичСской суммС ΠΏΠΎΡ‚Π΅Π½Ρ†ΠΈΠ°Π»ΠΎΠ² ΠΏΠΎΠ»Π΅ΠΉ, созданных ΠΎΡ‚Π΄Π΅Π»ΡŒΠ½Ρ‹ΠΌΠΈ зарядами, Ο† = Ο†1 + Ο†2 + Ο†3Β .Β Π­ΠΊΠ²ΠΈΠΏΠΎΡ‚Π΅Π½Ρ†ΠΈΠ°Π»ΡŒΠ½Ρ‹Π΅ повСрхности Ρ‚Π°ΠΊΠΈΡ… систСм ΠΈΠΌΠ΅ΡŽΡ‚ ΡΠ»ΠΎΠΆΠ½ΡƒΡŽ Ρ„ΠΎΡ€ΠΌΡƒ. НапримСр, для систСмы ΠΈΠ· Π΄Π²ΡƒΡ… ΠΎΠ΄ΠΈΠ½Π°ΠΊΠΎΠ²Ρ‹Ρ… ΠΏΠΎ Π·Π½Π°Ρ‡Π΅Π½ΠΈΡŽ ΠΎΠ΄Π½ΠΎΠΈΠΌΠ΅Π½Π½Ρ‹Ρ… зарядов ΡΠΊΠ²ΠΈΠΏΠΎΡ‚Π΅Π½Ρ†ΠΈΠ°Π»ΡŒΠ½Ρ‹Π΅ повСрхности ΠΈΠΌΠ΅ΡŽΡ‚ Π²ΠΈΠ΄, ΠΈΠ·ΠΎΠ±Ρ€Π°ΠΆΠ΅Π½Π½Ρ‹ΠΉ Π½Π° рисункС. Π­ΠΊΠ²ΠΈΠΏΠΎΡ‚Π΅Π½Ρ†ΠΈΠ°Π»ΡŒΠ½Ρ‹Π΅ повСрхности ΠΎΠ΄Π½ΠΎΡ€ΠΎΠ΄Π½ΠΎΠ³ΠΎ поля ΡΠ²Π»ΡŽΡ‚ΡΡ плоскостями.

Π­ΠΊΠ²ΠΈΠΏΠΎΡ‚Π΅Π½Ρ†ΠΈΠ°Π»ΡŒΠ½Ρ‹Π΅ повСрхности: Π° β€” поля Π΄Π²ΡƒΡ… ΠΎΠ΄ΠΈΠ½Π°ΠΊΠΎΠ²Ρ‹Ρ… зарядов Π± β€” ΠΎΠ΄Π½ΠΎΡ€ΠΎΠ΄Π½ΠΎΠ³ΠΎ поля

Π Π°Π·Π½ΠΎΡΡ‚ΡŒ ΠΏΠΎΡ‚Π΅Π½Ρ†ΠΈΠ°Π»ΠΎΠ²

ΠŸΡ€Π°ΠΊΡ‚ΠΈΡ‡Π΅ΡΠΊΠΎΠ΅ Π·Π½Π°Ρ‡Π΅Π½ΠΈΠ΅ ΠΈΠΌΠ΅Π΅Ρ‚ Π½Π΅ сам ΠΏΠΎΡ‚Π΅Π½Ρ†ΠΈΠ°Π» Π² Ρ‚ΠΎΡ‡ΠΊΠ΅, Π° ΠΈΠ·ΠΌΠ΅Π½Π΅Π½ΠΈΠ΅ (Ρ€Π°Π·Π½ΠΈΡ†Π°) ΠΏΠΎΡ‚Π΅Π½Ρ†ΠΈΠ°Π»Π° Ο†1 β€” Ο†2 , ΠΊΠΎΡ‚ΠΎΡ€ΠΎΠ΅ Π½Π΅ зависит ΠΎΡ‚ Π²Ρ‹Π±ΠΎΡ€Π° Π½ΡƒΠ»Π΅Π²ΠΎΠ³ΠΎ уровня отсчСта ΠΏΠΎΡ‚Π΅Π½Ρ†ΠΈΠ°Π»Π°. Π Π°Π·Π½ΠΎΡΡ‚ΡŒ ΠΏΠΎΡ‚Π΅Π½Ρ†ΠΈΠ°Π»ΠΎΠ² Ο†1 β€” Ο†2Β Π΅Ρ‰Π΅ Π½Π°Π·Ρ‹Π²Π°ΡŽΡ‚ напряТСниСм ΠΈ ΠΎΠ±ΠΎΠ·Π½Π°Ρ‡Π°ΡŽΡ‚ латинской Π±ΡƒΠΊΠ²ΠΎΠΉ U. Π’ΠΎΠ³Π΄Π° Ρ„ΠΎΡ€ΠΌΡƒΠ»Π° для Ρ€Π°Π±ΠΎΡ‚Ρ‹ ΠΏΠΎ ΠΏΠ΅Ρ€Π΅ΠΌΠ΅Ρ‰Π΅Π½ΠΈΡŽ заряда ΠΏΡ€ΠΈΠΎΠ±Ρ€Π΅Ρ‚Π°Π΅Ρ‚ Π²ΠΈΠ΄

НапряТСниС U β€” это физичСская Π²Π΅Π»ΠΈΡ‡ΠΈΠ½Π°, опрСдСляСмая Ρ€Π°Π±ΠΎΡ‚ΠΎΠΉ элСктричСского поля ΠΏΠΎ ΠΏΠ΅Ρ€Π΅ΠΌΠ΅Ρ‰Π΅Π½ΠΈΡŽ Π΅Π΄ΠΈΠ½ΠΈΡ‡Π½ΠΎΠ³ΠΎ ΠΏΠΎΠ»ΠΎΠΆΠΈΡ‚Π΅Π»ΡŒΠ½ΠΎΠ³ΠΎ заряда ΠΌΠ΅ΠΆΠ΄Ρƒ двумя Ρ‚ΠΎΡ‡ΠΊΠ°ΠΌΠΈ поля,

Π•Π΄ΠΈΠ½ΠΈΡ†Π° разности ΠΏΠΎΡ‚Π΅Π½Ρ†ΠΈΠ°Π»ΠΎΠ² (напряТСния), ΠΊΠ°ΠΊ ΠΈ ΠΏΠΎΡ‚Π΅Π½Ρ†ΠΈΠ°Π»Π°, β€” Π²ΠΎΠ»ΡŒΡ‚,

ΠŸΠΎΡΠΊΠΎΠ»ΡŒΠΊΡƒ Ρ€Π°Π±ΠΎΡ‚Π° сил поля ΠΏΠΎ ΠΏΠ΅Ρ€Π΅ΠΌΠ΅Ρ‰Π΅Π½ΠΈΡŽ заряда зависит Ρ‚ΠΎΠ»ΡŒΠΊΠΎ ΠΎΡ‚ разности ΠΏΠΎΡ‚Π΅Π½Ρ†ΠΈΠ°Π»ΠΎΠ², Ρ‚ΠΎ Π² случаС пСрСмСщСния заряда с ΠΏΠ΅Ρ€Π²ΠΎΠΉ ΡΠΊΠ²ΠΈΠΏΠΎΡ‚Π΅Π½Ρ†ΠΈΠ°Π»ΡŒΠ½ΠΎΠΉ повСрхности Π½Π° Π΄Ρ€ΡƒΠ³ΡƒΡŽ (ΠΏΠΎΡ‚Π΅Π½Ρ†ΠΈΠ°Π»Ρ‹ ΠΊΠΎΡ‚ΠΎΡ€Ρ‹Ρ… соотвСтствСнно Ο†1 ΠΈ Ο†2 ) выполнСнная ΠΏΠΎΠ»Π΅ΠΌ Ρ€Π°Π±ΠΎΡ‚Π° Π½Π΅ зависит ΠΎΡ‚ Ρ‚Ρ€Π°Π΅ΠΊΡ‚ΠΎΡ€ΠΈΠΈ этого двиТСния.

Бвязь напряТСнности элСктричСского поля с напряТСниСм

Из Ρ„ΠΎΡ€ΠΌΡƒΠ» A = Eqd ΠΈ A = qU ΠΌΠΎΠΆΠ½ΠΎ ΡƒΡΡ‚Π°Π½ΠΎΠ²ΠΈΡ‚ΡŒ связь ΠΌΠ΅ΠΆΠ΄Ρƒ Π½Π°ΠΏΡ€ΡΠΆΠ΅Π½Π½ΠΎΡΡ‚ΡŒΡŽ ΠΈ напряТСниСм элСктричСского поля: Ed = U. Π‘ этой Ρ„ΠΎΡ€ΠΌΡƒΠ»Ρ‹ слСдуСт:

  • Ρ‡Π΅ΠΌ мСньшС мСняСтся ΠΏΠΎΡ‚Π΅Π½Ρ†ΠΈΠ°Π» Π½Π° расстоянии d, Ρ‚Π΅ΠΌ мСньшС Π΅ΡΡ‚ΡŒ Π½Π°ΠΏΡ€ΡΠΆΠ΅Π½Π½ΠΎΡΡ‚ΡŒ элСктричСского поля;
  • Ссли ΠΏΠΎΡ‚Π΅Π½Ρ†ΠΈΠ°Π» Π½Π΅ мСняСтся, Ρ‚ΠΎ Π½Π°ΠΏΡ€ΡΠΆΠ΅Π½Π½ΠΎΡΡ‚ΡŒ Ρ€Π°Π²Π½Π° Π½ΡƒΠ»ΡŽ;
  • Π½Π°ΠΏΡ€ΡΠΆΠ΅Π½Π½ΠΎΡΡ‚ΡŒ элСктричСского поля Π½Π°ΠΏΡ€Π°Π²Π»Π΅Π½Π° ​​в сторону ΡƒΠΌΠ΅Π½ΡŒΡˆΠ΅Π½ΠΈΡ ΠΏΠΎΡ‚Π΅Π½Ρ†ΠΈΠ°Π»Π°.

ΠŸΠΎΡΠΊΠΎΠ»ΡŒΠΊΡƒ

Ρ‚ΠΎ ΠΈΠΌΠ΅Π½Π½ΠΎ ΠΈΠ· этой Ρ„ΠΎΡ€ΠΌΡƒΠ»Ρ‹ ΠΈ выводится Π΅Ρ‰Π΅ ΠΎΠ΄Π½Π° Π΅Π΄ΠΈΠ½ΠΈΡ†Π° напряТСнности β€” Π²ΠΎΠ»ΡŒΡ‚ Π½Π° ΠΌΠ΅Ρ‚Ρ€,

ΠŸΠΎΡ‚Π΅Π½Ρ†ΠΈΠ°Π» элСктростатичСского поля.

  1. ΠŸΠΎΡ‚Π΅Π½Ρ†ΠΈΠ°Π».

Наряду с описаниСм элСктричСского поля с ΠΏΠΎΠΌΠΎΡ‰ΡŒΡŽ Π΅Π³ΠΎ силовой характСристики – Π²Π΅ΠΊΡ‚ΠΎΡ€Π° , Π²ΠΎΠ·ΠΌΠΎΠΆΠ΅Π½ ΠΈ Π΄Ρ€ΡƒΠ³ΠΎΠΉ Π°Π΄Π΅ΠΊΠ²Π°Ρ‚Π½Ρ‹ΠΉ способ описания – энСргСтичСский.

Для ΠΏΠΎΡ‚Π΅Π½Ρ†ΠΈΠ°Π»ΡŒΠ½Ρ‹Ρ… ΠΏΠΎΠ»Π΅ΠΉ, создаваСмых Π½Π΅ΠΏΠΎΠ΄Π²ΠΈΠΆΠ½Ρ‹ΠΌΠΈ элСктричСскими зарядами, ΠΌΠΎΠΆΠ½ΠΎ ввСсти понятия ΠΏΠΎΡ‚Π΅Π½Ρ†ΠΈΠ°Π»ΡŒΠ½ΠΎΠΉ энСргии, ΠΏΠΎΡ‚Π΅Π½Ρ†ΠΈΠ°Π»Π° ΠΈ разности ΠΏΠΎΡ‚Π΅Π½Ρ†ΠΈΠ°Π»ΠΎΠ².

ΠŸΠΎΡ‚Π΅Π½Ρ†ΠΈΠ°Π» элСктростатичСского поля энСргСтичСская характСристика этого поля. ΠŸΠΎΡ‚Π΅Π½Ρ†ΠΈΠ°Π» ΠΎΠΏΡ€Π΅Π΄Π΅Π»Π΅Π½ с Ρ‚ΠΎΡ‡Π½ΠΎΡΡ‚ΡŒΡŽ Π΄ΠΎ Π°Π΄Π΄ΠΈΡ‚ΠΈΠ²Π½ΠΎΠΉ постоянной. Π—Π½Π°Ρ‡Π΅Π½ΠΈΠ΅ этой постоянной Π½Π΅ ΠΈΠ³Ρ€Π°Π΅Ρ‚ Ρ€ΠΎΠ»ΠΈ, ΠΏΠΎΡΠΊΠΎΠ»ΡŒΠΊΡƒ ΠΏΠΎΠ²Π΅Π΄Π΅Π½ΠΈΠ΅ частиц ΠΈΠ»ΠΈ Ρ‚Π΅Π» Π² элСктричСских полях опрСдСляСтся Π½Π΅ Π°Π±ΡΠΎΠ»ΡŽΡ‚Π½Ρ‹ΠΌΠΈ значСниями ΠΏΠΎΡ‚Π΅Π½Ρ†ΠΈΠ°Π»ΠΎΠ², Π° ΠΈΡ… разностями ΠΌΠ΅ΠΆΠ΄Ρƒ Ρ€Π°Π·Π»ΠΈΡ‡Π½Ρ‹ΠΌΠΈ Ρ‚ΠΎΡ‡ΠΊΠ°ΠΌΠΈ пространства. ΠŸΠΎΡΡ‚ΠΎΠΌΡƒ ΠΏΠΎΡ‚Π΅Π½Ρ†ΠΈΠ°Π»Ρƒ ΠΊΠ°ΠΊΠΎΠΉ-Π»ΠΈΠ±ΠΎ ΠΏΡ€ΠΎΠΈΠ·Π²ΠΎΠ»ΡŒΠ½ΠΎΠΉ Ρ‚ΠΎΡ‡ΠΊΠΈ поля ΠΌΠΎΠΆΠ½ΠΎ ΠΏΡ€ΠΈΠΏΠΈΡΠ°Ρ‚ΡŒ для опрСдСлСнности любоС Π·Π½Π°Ρ‡Π΅Π½ΠΈΠ΅ .

Π’ тСорСтичСской Ρ„ΠΈΠ·ΠΈΠΊΠ΅ Π·Π° Π½ΡƒΠ»Π΅Π²ΠΎΠΉ ΠΏΠΎΡ‚Π΅Π½Ρ†ΠΈΠ°Π» ΠΏΡ€ΠΈΠ½ΠΈΠΌΠ°ΡŽΡ‚ ΠΏΠΎΡ‚Π΅Π½Ρ†ΠΈΠ°Π» бСсконСчно ΡƒΠ΄Π°Π»Π΅Π½Π½ΠΎΠΉ Ρ‚ΠΎΡ‡ΠΊΠΈ пространства . На ΠΏΡ€Π°ΠΊΡ‚ΠΈΠΊΠ΅ Π·Π° Π½ΡƒΠ»Π΅Π²ΠΎΠΉ ΠΏΠΎΡ‚Π΅Π½Ρ†ΠΈΠ°Π» ΠΎΠ±Ρ‹Ρ‡Π½ΠΎ ΠΏΡ€ΠΈΠ½ΠΈΠΌΠ°ΡŽΡ‚ ΠΏΠΎΡ‚Π΅Π½Ρ†ΠΈΠ°Π» Π—Π΅ΠΌΠ»ΠΈ.

ΠŸΠΎΡ‚Π΅Π½Ρ†ΠΈΠ°Π» поля Ρ‚ΠΎΡ‡Π΅Ρ‡Π½ΠΎΠ³ΠΎ заряда:

, (6.1) Π³Π΄Π΅ — Π·Π½Π°Ρ‡Π΅Π½ΠΈΠ΅ ΠΏΠΎΡ‚Π΅Π½Ρ†ΠΈΠ°Π»Π°, ΠΎΡ‚ ΠΊΠΎΡ‚ΠΎΡ€ΠΎΠ³ΠΎ вСдСтся отсчСт, ΠΈΠ»ΠΈ, ΠΏΠΎΠ»ΠΎΠΆΠΈΠ² ,

. (6.2)

ΠŸΡ€ΠΈΠΌΠ΅Ρ‡Π°Π½ΠΈΠ΅. Π’ БИ ΠΏΠΎΡ‚Π΅Π½Ρ†ΠΈΠ°Π» Ρ‚ΠΎΡ‡Π΅Ρ‡Π½ΠΎΠ³ΠΎ заряда: .

ΠŸΠΎΡ‚Π΅Π½Ρ†ΠΈΠ°Π»ΡŒΠ½Π°Ρ энСргия заряда Π² элСктростатичСском ΠΏΠΎΠ»Π΅ опрСдСляСтся Ρ‡Π΅Ρ€Π΅Π· ΠΏΠΎΡ‚Π΅Π½Ρ†ΠΈΠ°Π» ΠΊΠ°ΠΊ

. (6.3)

Богласно (6.3) ΠΏΠΎΡ‚Π΅Π½Ρ†ΠΈΠ°Π» ΠΊΠ°ΠΊΠΎΠΉ-Π»ΠΈΠ±ΠΎ Ρ‚ΠΎΡ‡ΠΊΠΈ поля ΠΌΠΎΠΆΠ½ΠΎ ΠΎΠΏΡ€Π΅Π΄Π΅Π»ΠΈΡ‚ΡŒ ΠΊΠ°ΠΊ Π²Π΅Π»ΠΈΡ‡ΠΈΠ½Ρƒ, числСнно Ρ€Π°Π²Π½ΡƒΡŽ ΠΏΠΎΡ‚Π΅Π½Ρ†ΠΈΠ°Π»ΡŒΠ½ΠΎΠΉ энСргии Π΅Π΄ΠΈΠ½ΠΈΡ‡Π½ΠΎΠ³ΠΎ ΠΏΠΎΠ»ΠΎΠΆΠΈΡ‚Π΅Π»ΡŒΠ½ΠΎΠ³ΠΎ заряда Π² Π΄Π°Π½Π½ΠΎΠΉ Ρ‚ΠΎΡ‡ΠΊΠ΅ поля.

Π‘ ΡƒΡ‡Π΅Ρ‚ΠΎΠΌ (6.2) ΠΈ (6.3) Π²Ρ‹Ρ€Π°ΠΆΠ΅Π½ΠΈΠ΅ (5.2), ΠΏΠΎΠ»ΡƒΡ‡Π΅Π½Π½ΠΎΠ΅ Π² ΠΏΡ€Π΅Π΄Ρ‹Π΄ΡƒΡ‰Π΅ΠΌ ΠΏΠ°Ρ€Π°Π³Ρ€Π°Ρ„Π΅ для Ρ€Π°Π±ΠΎΡ‚Ρ‹ сил поля покоящСгося Ρ‚ΠΎΡ‡Π΅Ρ‡Π½ΠΎΠ³ΠΎ заряда, ΠΌΠΎΠΆΠ½ΠΎ Π·Π°ΠΏΠΈΡΠ°Ρ‚ΡŒ

, (6.4)

Π³Π΄Π΅ Ρ€Π°Π·Π½ΠΎΡΡ‚ΡŒ ΠΏΠΎΡ‚Π΅Π½Ρ†ΠΈΠ°Π»ΠΎΠ² ΠΌΠ΅ΠΆΠ΄Ρƒ Ρ‚ΠΎΡ‡ΠΊΠ°ΠΌΠΈ ΠΈ , ΡƒΠ±Ρ‹Π»ΡŒ ΠΏΠΎΡ‚Π΅Π½Ρ†ΠΈΠ°Π»ΡŒΠ½ΠΎΠΉ энСргии.

Π Π°Π·Π½ΠΎΡΡ‚ΡŒ ΠΏΠΎΡ‚Π΅Π½Ρ†ΠΈΠ°Π»ΠΎΠ² ΠΌΠ΅ΠΆΠ΄Ρƒ Ρ‚ΠΎΡ‡ΠΊΠ°ΠΌΠΈ ΠΈ ΠΎΠΏΡ€Π΅Π΄Π΅Π»ΡΡŽΡ‚ ΠΊΠ°ΠΊ Ρ€Π°Π±ΠΎΡ‚Ρƒ, ΡΠΎΠ²Π΅Ρ€ΡˆΠ°Π΅ΠΌΡƒΡŽ силами поля ΠΏΡ€ΠΈ ΠΏΠ΅Ρ€Π΅ΠΌΠ΅Ρ‰Π΅Π½ΠΈΠΈ Π΅Π΄ΠΈΠ½ΠΈΡ‡Π½ΠΎΠ³ΠΎ ΠΏΠΎΠ»ΠΎΠΆΠΈΡ‚Π΅Π»ΡŒΠ½ΠΎΠ³ΠΎ заряда ΠΏΠΎ ΠΏΡ€ΠΎΠΈΠ·Π²ΠΎΠ»ΡŒΠ½ΠΎΠΌΡƒ ΠΏΡƒΡ‚ΠΈ ΠΈΠ· Ρ‚ΠΎΡ‡ΠΊΠΈ Π² Ρ‚ΠΎΡ‡ΠΊΡƒ .

Π’Π°ΠΊΠΎΠ΅ ΠΎΠΏΡ€Π΅Π΄Π΅Π»Π΅Π½ΠΈΠ΅ ΠΈΠΌΠ΅Π΅Ρ‚ смысл ΠΈΠΌΠ΅Π½Π½ΠΎ ΠΏΠΎΡ‚ΠΎΠΌΡƒ, Ρ‡Ρ‚ΠΎ Ρ€Π°Π±ΠΎΡ‚Π° элСктростатичСского поля Π½Π΅ зависит ΠΎΡ‚ Ρ„ΠΎΡ€ΠΌΡ‹ ΠΏΡƒΡ‚ΠΈ, Π° опрСдСляСтся Ρ‚ΠΎΠ»ΡŒΠΊΠΎ полоТСниями Π΅Π³ΠΎ Π½Π°Ρ‡Π°Π»ΡŒΠ½ΠΎΠΉ ΠΈ ΠΊΠΎΠ½Π΅Ρ‡Π½ΠΎΠΉ Ρ‚ΠΎΡ‡Π΅ΠΊ.

Из (6.4) слСдуСт, Ρ‡Ρ‚ΠΎ Ρ€Π°Π·Π½ΠΎΡΡ‚ΡŒ ΠΏΠΎΡ‚Π΅Π½Ρ†ΠΈΠ°Π»ΠΎΠ² ΠΌΠ΅ΠΆΠ΄Ρƒ двумя ΠΏΡ€ΠΎΠΈΠ·Π²ΠΎΠ»ΡŒΠ½Ρ‹ΠΌΠΈ Ρ‚ΠΎΡ‡ΠΊΠ°ΠΌΠΈ элСктростатичСского поля ΠΌΠΎΠΆΠ½ΠΎ Π·Π°ΠΏΠΈΡΠ°Ρ‚ΡŒ ΠΊΠ°ΠΊ

, (6.5)

ΠΎΡ‚ΠΊΡƒΠ΄Π° ΠΌΠΎΠΆΠ½ΠΎ Π·Π°ΠΊΠ»ΡŽΡ‡ΠΈΡ‚ΡŒ, Ρ‡Ρ‚ΠΎ Π² элСктричСском ΠΏΠΎΠ»Π΅ сущСствуСт нСкоторая скалярная функция ΠΊΠΎΠΎΡ€Π΄ΠΈΠ½Π°Ρ‚ , ΡƒΠ±Ρ‹Π»ΡŒ ΠΊΠΎΡ‚ΠΎΡ€ΠΎΠΉ опрСдСляСтся Π²Ρ‹Ρ€Π°ΠΆΠ΅Π½ΠΈΠ΅ΠΌ (6.5).

БистСма элСктричСских зарядов.

Из ΠΏΡ€ΠΈΠ½Ρ†ΠΈΠΏΠ° супСрпозиции слСдуСт, Ρ‡Ρ‚ΠΎ свойство ΠΏΠΎΡ‚Π΅Π½Ρ†ΠΈΠ°Π»ΡŒΠ½ΠΎΡΡ‚ΠΈ справСдливо для элСктричСского поля любой систСмы ΠΈΠ»ΠΈ ΠΊΠΎΠ½Ρ„ΠΈΠ³ΡƒΡ€Π°Ρ†ΠΈΠΈ Π½Π΅ΠΏΠΎΠ΄Π²ΠΈΠΆΠ½Ρ‹Ρ… зарядов. ΠŸΠΎΡΡ‚ΠΎΠΌΡƒ ΠΏΠΎΡ‚Π΅Π½Ρ†ΠΈΠ°Π» поля, создаваСмого систСмой зарядов (Ρ‚ΠΎΡ‡Π΅Ρ‡Π½Ρ‹Ρ…), опрСдСляСтся Π²Ρ‹Ρ€Π°ΠΆΠ΅Π½ΠΈΠ΅ΠΌ:

. (6.6)

Π’ случаС Π½Π΅ΠΏΡ€Π΅Ρ€Ρ‹Π²Π½ΠΎ распрСдСлСнного заряда Π½Π° повСрхности ΠΈΠ»ΠΈ Π² Π½Π΅ΠΊΠΎΡ‚ΠΎΡ€ΠΎΠΉ области пространства ΠΏΠΎΡ‚Π΅Π½Ρ†ΠΈΠ°Π» опрСдСляСтся выраТСниями:

Ссли извСстна повСрхностная ΠΏΠ»ΠΎΡ‚Π½ΠΎΡΡ‚ΡŒ заряда ,

, (6.7)

Ссли Π·Π°Π΄Π°Π½Π° объСмная ΠΏΠ»ΠΎΡ‚Π½ΠΎΡΡ‚ΡŒ заряда ,

. (6.8)

Π•Π΄ΠΈΠ½ΠΈΡ†Ρ‹ ΠΏΠΎΡ‚Π΅Π½Ρ†ΠΈΠ°Π»Π°: Π² систСмС Π΅Π΄ΠΈΠ½ΠΈΡ†Π° ΠΏΠΎΡ‚Π΅Π½Ρ†ΠΈΠ°Π»Π° выбираСтся исходя ΠΈΠ· (6.2) — . Π’ систСмС БИ Π΅Π΄ΠΈΠ½ΠΈΡ†Π° ΠΏΠΎΡ‚Π΅Π½Ρ†ΠΈΠ°Π»Π° Π΅ΡΡ‚ΡŒ Π’ΠΎΠ»ΡŒΡ‚: 1 Π’ = 1 Π”ΠΆοƒ—ΠšΠ»-1.

Бвязь ΠΌΠ΅ΠΆΠ΄Ρƒ Π΅Π΄ΠΈΠ½ΠΈΡ†Π°ΠΌΠΈ:

.

  1. Бвязь напряТСнности ΠΈ ΠΏΠΎΡ‚Π΅Π½Ρ†ΠΈΠ°Π»Π°. Π£Ρ€Π°Π²Π½Π΅Π½ΠΈΠ΅ ΠŸΡƒΠ°ΡΡΠΎΠ½Π°.

Из курса ΠΌΠ΅Ρ…Π°Π½ΠΈΠΊΠΈ извСстно, Ρ‡Ρ‚ΠΎ сила, Π΄Π΅ΠΉΡΡ‚Π²ΡƒΡŽΡ‰Π°Ρ Π½Π° частицу Π² ΠΏΠΎΠ»Π΅, опрСдСляСтся ΠΊΠ°ΠΊ Π³Ρ€Π°Π΄ΠΈΠ΅Π½Ρ‚ ΠΏΠΎΡ‚Π΅Π½Ρ†ΠΈΠ°Π»ΡŒΠ½ΠΎΠΉ энСргии частицы Π² этом ΠΏΠΎΠ»Π΅

. (6.9)

ΠŸΠΎΠ»ΡŒΠ·ΡƒΡΡΡŒ опрСдСлСниями (2.5) ΠΈ (6.3), Π½Π°Ρ…ΠΎΠ΄ΠΈΠΌ связь ΠΌΠ΅ΠΆΠ΄Ρƒ ΠΏΠΎΡ‚Π΅Π½Ρ†ΠΈΠ°Π»ΠΎΠΌ ΠΈ Π½Π°ΠΏΡ€ΡΠΆΠ΅Π½Π½ΠΎΡΡ‚ΡŒΡŽ поля:

. (6.10)

Π’ΠΎΠ·ΡŒΠΌΠ΅ΠΌ Π΄ΠΈΠ²Π΅Ρ€Π³Π΅Π½Ρ†ΠΈΡŽ ΠΎΡ‚ ΠΎΠ±Π΅ΠΈΡ… частСй уравнСния (6.10)

. (6.11)

РаспишСм Ρ‚Π΅ΠΏΠ΅Ρ€ΡŒ скалярноС ΠΏΡ€ΠΎΠΈΠ·Π²Π΅Π΄Π΅Π½ΠΈΠ΅ Π²Π΅ΠΊΡ‚ΠΎΡ€Π½Ρ‹Ρ… ΠΎΠΏΠ΅Ρ€Π°Ρ‚ΠΎΡ€ΠΎΠ² Β«Π½Π°Π±Π»Π°Β»:

. (6.12)

Π’Ρ‹Ρ€Π°ΠΆΠ΅Π½ΠΈΠ΅ (6.12) опрСдСляСт ΠΎΠΏΠ΅Ρ€Π°Ρ‚ΠΎΡ€ Лапласа .

Π’ΠΎΡΠΏΠΎΠ»ΡŒΠ·ΡƒΠ΅ΠΌΡΡ Ρ‚Π΅ΠΏΠ΅Ρ€ΡŒ Ρ‚Π΅ΠΎΡ€Π΅ΠΌΠΎΠΉ Гаусса для Π²Π΅ΠΊΡ‚ΠΎΡ€Π° Π² Π΄ΠΈΡ„Ρ„Π΅Ρ€Π΅Π½Ρ†ΠΈΠ°Π»ΡŒΠ½ΠΎΠΉ Ρ„ΠΎΡ€ΠΌΠ΅ (4.13):

Бравнивая выраТСния (6.11) ΠΈ (4.13), ΠΏΠΎΠ»ΡƒΡ‡Π°Π΅ΠΌ ΡƒΡ€Π°Π²Π½Π΅Π½ΠΈΠ΅ ΠŸΡƒΠ°ΡΡΠΎΠ½Π°, ΠΎΠΏΡ€Π΅Π΄Π΅Π»ΡΡŽΡ‰Π΅Π΅ связь ΠΌΠ΅ΠΆΠ΄Ρƒ пространствСнным распрСдСлСниСм заряда (ΠΏΠ»ΠΎΡ‚Π½ΠΎΡΡ‚ΡŒΡŽ заряда ) ΠΈ ΠΏΠΎΡ‚Π΅Π½Ρ†ΠΈΠ°Π»ΠΎΠΌ :

, (6.13)

Или

. (6.14)

Π­Ρ‚ΠΎ ΠΎΠ΄Π½ΠΎ ΠΈΠ· основных ΡƒΡ€Π°Π²Π½Π΅Π½ΠΈΠΉ элСктростатики, ΠΊΠΎΡ‚ΠΎΡ€ΠΎΠ΅ позволяСт Π½Π°ΠΉΡ‚ΠΈ ΠΏΠΎΡ‚Π΅Π½Ρ†ΠΈΠ°Π» элСктричСского поля ΠΏΠΎ Π·Π°Π΄Π°Π½Π½ΠΎΠΌΡƒ Ρ€Π°ΡΠΏΡ€Π΅Π΄Π΅Π»Π΅Π½ΠΈΡŽ (плотности) заряда.

Π’Π΅ΠΎΡ€Π΅ΠΌΠ° СдинствСнности.

Π’ Ρ‚Π΅ΠΎΡ€ΠΈΠΈ доказываСтся, Ρ‡Ρ‚ΠΎ ΡƒΡ€Π°Π²Π½Π΅Π½ΠΈΠ΅ (6.13) ΠΈΠΌΠ΅Π΅Ρ‚ СдинствСнноС Ρ€Π΅ΡˆΠ΅Π½ΠΈΠ΅. Π­Ρ‚ΠΎ ΡƒΡ‚Π²Π΅Ρ€ΠΆΠ΄Π΅Π½ΠΈΠ΅ Π½Π°Π·Ρ‹Π²Π°ΡŽΡ‚ Ρ‚Π΅ΠΎΡ€Π΅ΠΌΠΎΠΉ СдинствСнности.

РСшСниС уравнСния (6.13) Π² ΠΎΠ±Ρ‰Π΅ΠΌ случаС – слоТная ΠΈ кропотливая Π·Π°Π΄Π°Ρ‡Π°. АналитичСскиС Ρ€Π΅ΡˆΠ΅Π½ΠΈΡ этого уравнСния ΠΏΠΎΠ»ΡƒΡ‡Π΅Π½Ρ‹ лишь для Π½Π΅ΠΌΠ½ΠΎΠ³ΠΈΡ… частных случаСв. ИспользованиС ΠΆΠ΅ Ρ‚Π΅ΠΎΡ€Π΅ΠΌΡ‹ СдинствСнности сущСствСнно ΠΎΠ±Π»Π΅Π³Ρ‡Π°Π΅Ρ‚ Ρ€Π΅ΡˆΠ΅Π½ΠΈΠ΅ Ρ†Π΅Π»ΠΎΠ³ΠΎ ряда элСктростатичСских Π·Π°Π΄Π°Ρ‡. Бмысл Ρ‚Π΅ΠΎΡ€Π΅ΠΌΡ‹ СдинствСнности ΠΌΠΎΠΆΠ½ΠΎ ΡΡ„ΠΎΡ€ΠΌΡƒΠ»ΠΈΡ€ΠΎΠ²Π°Ρ‚ΡŒ ΡΠ»Π΅Π΄ΡƒΡŽΡ‰ΠΈΠΌ ΠΎΠ±Ρ€Π°Π·ΠΎΠΌ: Ссли Ρ€Π΅ΡˆΠ΅Π½ΠΈΠ΅ Π·Π°Π΄Π°Ρ‡ΠΈ удовлСтворяСт ΡƒΡ€Π°Π²Π½Π΅Π½ΠΈΡŽ ΠŸΡƒΠ°ΡΡΠΎΠ½Π° ΠΈ Π³Ρ€Π°Π½ΠΈΡ‡Π½Ρ‹ΠΌ условиям (ΠΈΡ… ΠΌΡ‹ обсудим Π² ΡΠ»Π΅Π΄ΡƒΡŽΡ‰Π΅ΠΉ Π³Π»Π°Π²Π΅), Ρ‚ΠΎ ΠΌΠΎΠΆΠ½ΠΎ ΡƒΡ‚Π²Π΅Ρ€ΠΆΠ΄Π°Ρ‚ΡŒ, Ρ‡Ρ‚ΠΎ ΠΎΠ½ΠΎ являСтся ΠΏΡ€Π°Π²ΠΈΠ»ΡŒΠ½Ρ‹ΠΌ ΠΈ СдинствСнным, ΠΊΠ°ΠΊΠΈΠΌ Π±Ρ‹ способом (Π½Π°ΠΏΡ€ΠΈΠΌΠ΅Ρ€, ΠΏΡƒΡ‚Π΅ΠΌ Π΄ΠΎΠ³Π°Π΄ΠΊΠΈ) ΠΌΡ‹ Π½ΠΈ нашли Π΅Π³ΠΎ.

Π‘ физичСской Ρ‚ΠΎΡ‡ΠΊΠΈ зрСния содСрТаниС Ρ‚Π΅ΠΎΡ€Π΅ΠΌΡ‹ СдинствСнности довольно ΠΎΡ‡Π΅Π²ΠΈΠ΄Π½ΠΎ: Ссли ΠΏΡ€Π΅Π΄ΠΏΠΎΠ»ΠΎΠΆΠΈΡ‚ΡŒ, Ρ‡Ρ‚ΠΎ Π²ΠΎΠ·ΠΌΠΎΠΆΠ½ΠΎ Π½Π΅ ΠΎΠ΄Π½ΠΎ Ρ€Π΅ΡˆΠ΅Π½ΠΈΠ΅ Π·Π°Π΄Π°Ρ‡ΠΈ, Ρ‚ΠΎ сущСствуСт Π½Π΅ СдинствСнный ΠΏΠΎΡ‚Π΅Π½Ρ†ΠΈΠ°Π»ΡŒΠ½Ρ‹ΠΉ Β«Ρ€Π΅Π»ΡŒΠ΅Ρ„Β», ΡΠ»Π΅Π΄ΠΎΠ²Π°Ρ‚Π΅Π»ΡŒΠ½ΠΎ, Π² ΠΊΠ°ΠΆΠ΄ΠΎΠΉ Ρ‚ΠΎΡ‡ΠΊΠ΅ пространства ΠΏΠΎΠ»Π΅ ΠΎΠΏΡ€Π΅Π΄Π΅Π»Π΅Π½ΠΎ, Π²ΠΎΠΎΠ±Ρ‰Π΅ говоря, Π½Π΅ ΠΎΠ΄Π½ΠΎΠ·Π½Π°Ρ‡Π½ΠΎ, Ρ‚.Π΅. ΠΌΡ‹ ΠΏΡ€ΠΈΡ…ΠΎΠ΄ΠΈΠΌ ΠΊ физичСскому абсурду.

ΠŸΠΎΡ‚Π΅Π½Ρ†ΠΈΠ°Π» Π² элСктростатикС

ΠžΠΏΡ€Π΅Π΄Π΅Π»Π΅Π½ΠΈΠ΅ 1

ЭлСктростатичСский ΠΏΠΎΡ‚Π΅Π½Ρ†ΠΈΠ°Π» прСдставляСт ΡΠΊΠ°Π»ΡΡ€Π½ΡƒΡŽ ΡΠ½Π΅Ρ€Π³Π΅Ρ‚ΠΈΡ‡Π΅ΡΠΊΡƒΡŽ характСристику элСктростатичСского поля, Ρ…Π°Ρ€Π°ΠΊΡ‚Π΅Ρ€ΠΈΠ·ΡƒΡŽΡ‰ΡƒΡŽ ΠΏΠΎΡ‚Π΅Π½Ρ†ΠΈΠ°Π»ΡŒΠ½ΡƒΡŽ ΡΠ½Π΅Ρ€Π³ΠΈΡŽ, ΠΎΠ±Π»Π°Π΄Π°Ρ‚Π΅Π»Π΅ΠΌ ΠΊΠΎΡ‚ΠΎΡ€ΠΎΠΉ являСтся Π΅Π΄ΠΈΠ½ΠΈΡ‡Π½Ρ‹ΠΉ ΠΏΠΎΠ»ΠΎΠΆΠΈΡ‚Π΅Π»ΡŒΠ½Ρ‹ΠΉ ΠΏΡ€ΠΎΠ±Π½Ρ‹ΠΉ заряд, ΠΊΠΎΡ‚ΠΎΡ€Ρ‹ΠΉ помСстили Π² Π΄Π°Π½Π½ΡƒΡŽ Ρ‚ΠΎΡ‡ΠΊΡƒ поля. Π’ качСствС Π΅Π΄ΠΈΠ½ΠΈΡ†Ρ‹ измСрСния ΠΏΠΎΡ‚Π΅Π½Ρ†ΠΈΠ°Π»Π° Π² систСмС Π΅Π΄ΠΈΠ½ΠΈΡ† выступаСт Π²ΠΎΠ»ΡŒΡ‚.

ЭлСктростатичСскоС ΠΏΠΎΠ»Π΅

ЭлСктростатичСскоС ΠΏΠΎΠ»Π΅ прСдставляСт ΠΎΠ±Ρ€Π°Π·ΠΎΠ²Π°Π½Π½ΠΎΠ΅ Π½Π΅ΠΏΠΎΠ΄Π²ΠΈΠΆΠ½Ρ‹ΠΌΠΈ Π² пространствС ΠΈ Π½Π΅ΠΈΠ·ΠΌΠ΅Π½Π½Ρ‹ΠΌΠΈ Π²ΠΎ Π²Ρ€Π΅ΠΌΠ΅Π½ΠΈ элСктрозарядами ΠΏΠΎΠ»Π΅ (ΠΏΡ€ΠΈ условии отсутствия элСктричСских Ρ‚ΠΎΠΊΠΎΠ²). ЭлСктричСскоС ΠΏΠΎΠ»Π΅, Ρ‚Π°ΠΊΠΈΠΌ ΠΎΠ±Ρ€Π°Π·ΠΎΠΌ, считаСтся особым Π²ΠΈΠ΄ΠΎΠΌ ΠΌΠ°Ρ‚Π΅Ρ€ΠΈΠΈ, связанным с элСктричСскими зарядами ΠΈ ΠΏΠ΅Ρ€Π΅Π΄Π°ΡŽΡ‰ΠΈΠΌ воздСйствия зарядов Π΄Ρ€ΡƒΠ³ Π½Π° Π΄Ρ€ΡƒΠ³Π°.

Π’Π°ΠΊ, ΠΏΡ€ΠΈ присутствии Π² пространствС систСмы заряТСнных Ρ‚Π΅Π», Ρ‚ΠΎ Π² ΠΊΠ°ΠΆΠ΄ΠΎΠΉ Π΅Π³ΠΎ Ρ‚ΠΎΡ‡ΠΊΠ΅ Π±ΡƒΠ΄Π΅Ρ‚ Ρ„ΠΈΠΊΡΠΈΡ€ΠΎΠ²Π°Ρ‚ΡŒΡΡ сущСствованиС силового элСктричСского поля, опрСдСляСмого Ρ‡Π΅Ρ€Π΅Π· силу, Π²ΠΎΠ·Π΄Π΅ΠΉΡΡ‚Π²ΡƒΡŽΡ‰ΡƒΡŽ Π½Π° ΠΏΡ€ΠΎΠ±Π½Ρ‹ΠΉ Ρ‚ΠΎΡ‡Π΅Ρ‡Π½Ρ‹ΠΉ заряд, ΠΏΠΎΠΌΠ΅Ρ‰Π΅Π½Π½Ρ‹ΠΉ Π² Π΄Π°Π½Π½ΠΎΠ΅ ΠΏΠΎΠ»Π΅. ΠŸΡ€ΠΎΠ±Π½Ρ‹ΠΉ заряд Π΄ΠΎΠ»ΠΆΠ΅Π½ ΠΏΡ€ΠΈ этом Π±Ρ‹Ρ‚ΡŒ Π½ΠΈΡ‡Ρ‚ΠΎΠΆΠ½ΠΎ ΠΌΠ°Π»Ρ‹ΠΌ, Ρ‡Ρ‚ΠΎΠ±Ρ‹ Π½Π΅ ΠΎΠΊΠ°Π·Π°Ρ‚ΡŒ влияниС Π½Π° характСристику элСктростатичСского поля.

Рисунок 1. ЭлСктростатичСскоС ΠΏΠΎΠ»Π΅. Автор24 β€” ΠΈΠ½Ρ‚Π΅Ρ€Π½Π΅Ρ‚-Π±ΠΈΡ€ΠΆΠ° студСнчСских Ρ€Π°Π±ΠΎΡ‚

ЭлСктричСскоС ΠΏΠΎΠ»Π΅ Π½Π°Π·Ρ‹Π²Π°ΡŽΡ‚ ΠΎΠ΄Π½ΠΎΡ€ΠΎΠ΄Π½Ρ‹ΠΌ Π² ситуации, Ссли Π²Π΅ΠΊΡ‚ΠΎΡ€ Π΅Π³ΠΎ напряТСнности оказываСтся ΠΎΠ΄ΠΈΠ½Π°ΠΊΠΎΠ²Ρ‹ΠΌ Π²ΠΎ всСх Ρ‚ΠΎΡ‡ΠΊΠ°Ρ… поля. Π’ качСствС Π³Π»Π°Π²Π½Ρ‹Ρ… характСристик элСктростатичСского поля Π²Ρ‹Π΄Π΅Π»ΡΡŽΡ‚ ΡΠ»Π΅Π΄ΡƒΡŽΡ‰ΠΈΠ΅:

  • Π½Π°ΠΏΡ€ΡΠΆΠ΅Π½Π½ΠΎΡΡ‚ΡŒ;
  • ΠΏΠΎΡ‚Π΅Π½Ρ†ΠΈΠ°Π».

Π‘ΠΈΠ»ΠΎΠ²Ρ‹Π΅ Π»ΠΈΠ½ΠΈΠΈ Ρ‚Π°ΠΊΠΎΠ³ΠΎ поля ΠΎΠ±Π»Π°Π΄Π°ΡŽΡ‚ Ρ‚Π°ΠΊΠΈΠΌΠΈ свойствами:

  1. Они Π±ΡƒΠ΄ΡƒΡ‚ всСгда Π·Π°ΠΌΠΊΠ½ΡƒΡ‚Ρ‹ΠΌΠΈ, Ρ‚ΠΎ Π΅ΡΡ‚ΡŒ Π½Π°Ρ‡ΠΈΠ½Π°ΡŽΡ‰ΠΈΠΌΠΈΡΡ Π½Π° ΠΏΠΎΠ»ΠΎΠΆΠΈΡ‚Π΅Π»ΡŒΠ½Ρ‹Ρ… зарядах ΠΈ Π·Π°ΠΊΠ°Π½Ρ‡ΠΈΠ²Π°ΡŽΡ‰ΠΈΠΌΠΈΡΡ Π½Π° ΠΎΡ‚Ρ€ΠΈΡ†Π°Ρ‚Π΅Π»ΡŒΠ½Ρ‹Ρ…. Они Π½Π΅ ΠΏΠ΅Ρ€Π΅ΡΠ΅ΠΊΠ°ΡŽΡ‚ΡΡ ΠΌΠ΅ΠΆΠ΄Ρƒ собой ΠΈ Π½Π΅ ΠΊΠ°ΡΠ°ΡŽΡ‚ΡΡ Π΄Ρ€ΡƒΠ³ Π΄Ρ€ΡƒΠ³Π°.
  2. ΠŸΠ»ΠΎΡ‚Π½ΠΎΡΡ‚ΡŒ Π»ΠΈΠ½ΠΈΠΉ Ρ‚Π΅ΠΌ большС, Ρ‡Π΅ΠΌ большСй Π±ΡƒΠ΄Π΅Ρ‚ Π½Π°ΠΏΡ€ΡΠΆΡ‘Π½Π½ΠΎΡΡ‚ΡŒ. Π”Ρ€ΡƒΠ³ΠΈΠΌΠΈ словами, Π½Π°ΠΏΡ€ΡΠΆΡ‘Π½Π½ΠΎΡΡ‚ΡŒ поля являСтся прямо ΠΏΡ€ΠΎΠΏΠΎΡ€Ρ†ΠΈΠΎΠ½Π°Π»ΡŒΠ½ΠΎΠΉ количСству силовых Π»ΠΈΠ½ΠΈΠΉ, ΠΏΠ΅Ρ€Π΅ΡΠ΅ΠΊΠ°ΡŽΡ‰ΠΈΡ… ΠΏΠ»ΠΎΡ‰Π°Π΄ΠΊΡƒ Π΅Π΄ΠΈΠ½ΠΈΡ‡Π½ΠΎΠΉ ΠΏΠ»ΠΎΡ‰Π°Π΄ΠΈ, Ρ‡ΡŒΠ΅ располоТСниС Π±ΡƒΠ΄Π΅Ρ‚ пСрпСндикулярно линиям.

Π—Π°ΠΌΠ΅Ρ‡Π°Π½ΠΈΠ΅ 1

ЭлСктростатичСскоС ΠΏΠΎΠ»Π΅ ΠΎΠΊΠ°Π·Ρ‹Π²Π°Π΅Ρ‚ нСпосрСдствСнноС воздСйствиС Π½Π° любоС количСство зарядов, ΠΏΡ€ΠΈ этом Π²ΠΎΠ·Π½ΠΈΠΊΠ½Π΅Ρ‚ слоТная систСма взаимодСйствий. ΠΠ°ΠΏΡ€ΡΠΆΠ΅Π½Π½ΠΎΡΡ‚ΡŒ систСмы ΠΌΠΎΠΆΠ½ΠΎ Ρ€Π°ΡΡΠΌΠ°Ρ‚Ρ€ΠΈΠ²Π°Ρ‚ΡŒ с Ρ‚ΠΎΡ‡ΠΊΠΈ зрСния супСрпозиции, поэтому суммарноС влияниС числа зарядов являСтся Π²Π΅ΠΊΡ‚ΠΎΡ€Π½ΠΎΠΉ суммой всСх напряТСнностСй поля.

Π’ соотвСтствии с этим, Ρ‡Π΅ΠΌ большС Ρ‚Π°ΠΊΠΈΡ… Π»ΠΈΠ½ΠΈΠΉ, Ρ‚Π΅ΠΌ интСнсивнСС оказываСтся силовоС воздСйствиС. Π’ ΠΌΠ΅Ρ‚Π°Π»Π»Π°Ρ… (ΠΈ ΠΈΠ½Ρ‹Ρ… проводящих ΠΌΠ°Ρ‚Π΅Ρ€ΠΈΠ°Π»Π°Ρ…) Π½Π°ΠΏΡ€ΡΠΆΠ΅Π½Π½ΠΎΡΡ‚ΡŒ поля Π±ΡƒΠ΄Π΅Ρ‚ ΠΎΡ‚ΡΡƒΡ‚ΡΡ‚Π²ΠΎΠ²Π°Ρ‚ΡŒ (Π·Π° счСт встрСчно Π½Π°ΠΏΡ€Π°Π²Π»Π΅Π½Π½ΠΎΠ³ΠΎ дСйствия поля свободных носитСлСй заряда, ΠΏΡ€Π΅Π±Ρ‹Π²Π°ΡŽΡ‰ΠΈΡ… Π² структурС кристалличСской Ρ€Π΅ΡˆΠ΅Ρ‚ΠΊΠΈ).

ЀактичСски, силы ΠΎΠΏΠ΅Ρ€Π°Ρ‚ΠΈΠ²Π½ΠΎ ΡƒΡ€Π°Π²Π½ΠΈΠ²Π°ΡŽΡ‚ΡΡ, фиксируСтся отсутствиС Ρ‚ΠΎΠΊΠ°, Π° Π»ΠΈΠ½ΠΈΠΈ напряТСнности Π½Π΅ способны ΠΏΡ€ΠΎΠ½ΠΈΠΊΠ½ΡƒΡ‚ΡŒ Π² Ρ‚Π°ΠΊΠΎΠΉ ΠΏΡ€ΠΎΠ²ΠΎΠ΄Π½ΠΈΠΊ. Помимо Π²Π΅ΠΊΡ‚ΠΎΡ€Π½Ρ‹Ρ… Π²Π΅Π»ΠΈΡ‡ΠΈΠ½, ΠΏΠΎΠ»Π΅ ΠΌΠΎΠΆΠ΅Ρ‚ ΠΎΠΏΠΈΡΡ‹Π²Π°Ρ‚ΡŒΡΡ скалярными значСниями (ΠΈΠ΄Π΅Π°Π»ΡŒΠ½Ρ‹ΠΉ случай), взятыми Π² ΠΊΠ°ΠΆΠ΄ΠΎΠΉ Ρ‚ΠΎΡ‡ΠΊΠ΅. Π’Π°ΠΊΠΈΠ΅ значСния Π² элСктростатикС Ρ…Π°Ρ€Π°ΠΊΡ‚Π΅Ρ€ΠΈΠ·ΡƒΡŽΡ‚ ΠΏΠΎΡ‚Π΅Π½Ρ†ΠΈΠ°Π» поля.

ΠžΠΏΡ€Π΅Π΄Π΅Π»Π΅Π½ΠΈΠ΅ элСктростатичСского ΠΏΠΎΡ‚Π΅Π½Ρ†ΠΈΠ°Π»Π°

Π’Π΅Π»ΠΎ, ΠΏΡ€Π΅Π±Ρ‹Π²Π°ΡŽΡ‰Π΅Π΅ Π² ΠΏΠΎΡ‚Π΅Π½Ρ†ΠΈΠ°Π»ΡŒΠ½ΠΎΠΌ ΠΏΠΎΠ»Π΅ сил (Π° элСктростатичСскоС ΠΏΠΎΠ»Π΅ считаСтся ΠΏΠΎΡ‚Π΅Π½Ρ†ΠΈΠ°Π»ΡŒΠ½Ρ‹ΠΌ), ΠΈΠΌΠ΅Π΅Ρ‚ ΠΏΠΎΡ‚Π΅Π½Ρ†ΠΈΠ°Π»ΡŒΠ½ΡƒΡŽ ΡΠ½Π΅Ρ€Π³ΠΈΡŽ, посрСдством ΠΊΠΎΡ‚ΠΎΡ€ΠΎΠΉ силами поля Π±ΡƒΠ΄Π΅Ρ‚ ΡΠΎΠ²Π΅Ρ€ΡˆΠ°Ρ‚ΡŒΡΡ Ρ€Π°Π±ΠΎΡ‚Π°. Π Π°Π±ΠΎΡ‚Π° консСрвативных сил Π±ΡƒΠ΄Π΅Ρ‚ Π²Ρ‹ΠΏΠΎΠ»Π½ΡΡ‚ΡŒΡΡ Π·Π° счСт ΡƒΠ±Ρ‹Π»ΠΈ ΠΏΠΎΡ‚Π΅Π½Ρ†ΠΈΠ°Π»ΡŒΠ½ΠΎΠΉ энСргии

ЭлСктростатичСский ΠΏΠΎΡ‚Π΅Π½Ρ†ΠΈΠ°Π» являСтся ΡΠΏΠ΅Ρ†ΠΈΠ°Π»ΡŒΠ½Ρ‹ΠΌ Ρ‚Π΅Ρ€ΠΌΠΈΠ½ΠΎΠΌ Π² случаС Π²ΠΎΠ·ΠΌΠΎΠΆΠ½ΠΎΠΉ Π·Π°ΠΌΠ΅Π½Ρ‹ ΠΎΠ±Ρ‰Π΅Π³ΠΎ Ρ‚Π΅Ρ€ΠΌΠΈΠ½Π° Π² элСктродинамикС (скалярный ΠΏΠΎΡ‚Π΅Π½Ρ†ΠΈΠ°Π»). Π˜ΡΡ‚ΠΎΡ€ΠΈΡ‡Π΅ΡΠΊΠΈ Π² Ρ„ΠΈΠ·ΠΈΠΊΠ΅ ΠΏΠ΅Ρ€Π²Ρ‹ΠΌ Π½Π°Π±Π»ΡŽΠ΄Π°Π΅Ρ‚ΡΡ появлСниС Ρ‚Π΅Ρ€ΠΌΠΈΠ½Π° «элСктростатичСский ΠΏΠΎΡ‚Π΅Π½Ρ†ΠΈΠ°Π»Β», Π° ΡƒΠΆΠ΅ скалярный ΠΏΠΎΡ‚Π΅Π½Ρ†ΠΈΠ°Π» элСктродинамики стал Π΅Π³ΠΎ ΠΎΠ±ΠΎΠ±Ρ‰Π΅Π½ΠΈΠ΅ΠΌ.

Π’ связи с Ρ‚Π΅ΠΌ, Ρ‡Ρ‚ΠΎ ΠΏΠΎΡ‚Π΅Π½Ρ†ΠΈΠ°Π» (Ρ€Π°Π²Π½ΠΎ ΠΊΠ°ΠΊ ΠΈ ΠΏΠΎΡ‚Π΅Π½Ρ†ΠΈΠ°Π»ΡŒΠ½Π°Ρ энСргия) ΠΌΠΎΠΆΠ΅Ρ‚ ΠΎΠΏΡ€Π΅Π΄Π΅Π»ΡΡ‚ΡŒΡΡ с Ρ‚ΠΎΡ‡Π½ΠΎΡΡ‚ΡŒΡŽ Π΄ΠΎ ΠΏΡ€ΠΎΠΈΠ·Π²ΠΎΠ»ΡŒΠ½ΠΎΠΉ постоянной (ΠΈ всС Π²Π΅Π»ΠΈΡ‡ΠΈΠ½Ρ‹, ΠΊΠΎΡ‚ΠΎΡ€Ρ‹Π΅ Π²ΠΎΠ·ΠΌΠΎΠΆΠ½ΠΎ ΠΈΠ·ΠΌΠ΅Ρ€ΠΈΡ‚ΡŒ: напряТСнности поля, силы, ΠΎΡΡ‚Π°ΡŽΡ‚ΡΡ Π½Π΅ΠΈΠ·ΠΌΠ΅Π½Π½Ρ‹ΠΌΠΈ Π² нСзависимости ΠΎΡ‚ Π²Ρ‹Π±ΠΎΡ€Π° способа постоянной Π²Π΅Π»ΠΈΡ‡ΠΈΠ½Ρ‹) нСпосрСдствСнным физичСским смыслом (Ссли Π½Π΅ ΠΈΠΌΠ΅ΡŽΡ‚ΡΡ Π² Π²ΠΈΠ΄Ρƒ ΠΊΠ²Π°Π½Ρ‚ΠΎΠ²Ρ‹Π΅ эффСкты) ΠΎΠ±Π»Π°Π΄Π°Π΅Ρ‚ Π½Π΅ сам ΠΏΠΎΡ‚Π΅Π½Ρ†ΠΈΠ°Π», Π° Ρ€Π°Π·Π½ΠΎΡΡ‚ΡŒ ΠΏΠΎΡ‚Π΅Π½Ρ†ΠΈΠ°Π»ΠΎΠ².

ΠŸΡ€ΠΈ этом принято ΡΡ‡ΠΈΡ‚Π°Ρ‚ΡŒ, Ρ‡Ρ‚ΠΎ ΠΏΡ€ΠΎΡ‡ΠΈΠ΅ заряды ΠΏΡ€ΠΈ ΠΏΠΎΠ΄ΠΎΠ±Π½ΠΎΠΉ ΠΎΠΏΠ΅Ρ€Π°Ρ†ΠΈΠΈ Β«Π·Π°ΠΌΠΎΡ€ΠΎΠΆΠ΅Π½Ρ‹Β» (Π½Π΅ΠΏΠΎΠ΄Π²ΠΈΠΆΠ½Ρ‹ Π² ΠΌΠΎΠΌΠ΅Π½Ρ‚ Ρ‚Π°ΠΊΠΎΠ³ΠΎ пСрСмСщСния (подразумСваСтся Π²ΠΎΠΎΠ±Ρ€Π°ΠΆΠ°Π΅ΠΌΠΎΠ΅, Π° Π½Π΅ Ρ€Π΅Π°Π»ΡŒΠ½ΠΎΠ΅ ΠΏΠ΅Ρ€Π΅ΠΌΠ΅Ρ‰Π΅Π½ΠΈΠ΅). ΠŸΡ€ΠΈ этом, Π² Ρ€Π΅Π΄ΠΊΠΈΡ… случаях, с Ρ†Π΅Π»ΡŒΡŽ снятия нСоднозначности, ΠΈΡΠΏΠΎΠ»ΡŒΠ·ΡƒΡŽΡ‚ ΠΎΠΏΡ€Π΅Π΄Π΅Π»Π΅Π½Π½Ρ‹Π΅ «СстСствСнныС» условия.

Рисунок 2. ΠŸΠΎΡ‚Π΅Π½Ρ†ΠΈΠ°Π» элСктростатичСского поля. Автор24 β€” ΠΈΠ½Ρ‚Π΅Ρ€Π½Π΅Ρ‚-Π±ΠΈΡ€ΠΆΠ° студСнчСских Ρ€Π°Π±ΠΎΡ‚

Π’Π°ΠΊ, Π½Π°ΠΏΡ€ΠΈΠΌΠ΅Ρ€, часто ΠΏΠΎΡ‚Π΅Π½Ρ†ΠΈΠ°Π» опрСдСляСтся Ρ‚Π°ΠΊΠΈΠΌ ΠΎΠ±Ρ€Π°Π·ΠΎΠΌ, Ρ‡Ρ‚ΠΎΠ±Ρ‹ Π΅Π³ΠΎ Π·Π½Π°Ρ‡Π΅Π½ΠΈΠ΅ ΠΎΠΊΠ°Π·Ρ‹Π²Π°Π»ΠΎΡΡŒ Ρ€Π°Π²Π½ΠΎΠ·Π½Π°Ρ‡Π½ΠΎ Π½ΡƒΠ»Π΅Π²ΠΎΠΌΡƒ Π½Π° бСсконСчности для ΠΊΠ°ΠΊΠΎΠ³ΠΎ-Π»ΠΈΠ±ΠΎ Ρ‚ΠΎΡ‡Π΅Ρ‡Π½ΠΎΠ³ΠΎ заряда. Π’ Ρ‚Π°ΠΊΠΎΠΉ ситуации для любой ΠΊΠΎΠ½Π΅Ρ‡Π½ΠΎΠΉ систСмы зарядов Π±ΡƒΠ΄Π΅Ρ‚ Π²Ρ‹ΠΏΠΎΠ»Π½ΠΈΠΌΠΎ Π½Π° бСсконСчности Π°Π½Π°Π»ΠΎΠ³ΠΈΡ‡Π½ΠΎΠ΅ условиС, Π° Π½Π°Π΄ ΠΏΡ€ΠΎΠΈΠ·Π²ΠΎΠ»ΠΎΠΌ Π²Ρ‹Π±ΠΎΡ€Π° константы ΠΏΡ€ΠΈ этом ΠΌΠΎΠΆΠ½ΠΎ Π±ΡƒΠ΄Π΅Ρ‚ Π½Π΅ Π·Π°Π΄ΡƒΠΌΡ‹Π²Π°Ρ‚ΡŒΡΡ.

ΠžΡΠΎΠ±Π΅Π½Π½ΠΎΡΡ‚ΠΈ кулоновского ΠΏΠΎΡ‚Π΅Π½Ρ†ΠΈΠ°Π»Π°

Иногда Ρ‚Π°ΠΊΠΎΠΉ Ρ‚Π΅Ρ€ΠΌΠΈΠ½, ΠΊΠ°ΠΊ «кулоновский ΠΏΠΎΡ‚Π΅Π½Ρ†ΠΈΠ°Π»Β» примСняСтся ΠΏΡ€ΠΈ ΠΎΠ±ΠΎΠ·Π½Π°Ρ‡Π΅Π½ΠΈΠΈ элСктростатичСского ΠΏΠΎΡ‚Π΅Π½Ρ†ΠΈΠ°Π»Π° (Π² Ρ„ΠΎΡ€ΠΌΠ°Ρ‚Π΅ ΠΏΠΎΠ»Π½ΠΎΠ³ΠΎ синонима). ΠŸΡ€ΠΈ этом ΠΎΠ½ΠΈ нСсколько Ρ€Π°Π·Π»ΠΈΡ‡Π½Ρ‹ ΠΊΠ°ΡΠ°Ρ‚Π΅Π»ΡŒΠ½ΠΎ области примСнСния.

Π—Π°ΠΌΠ΅Ρ‡Π°Π½ΠΈΠ΅ 2

Π—Π°Ρ‡Π°ΡΡ‚ΡƒΡŽ, ΠΏΠΎΠ΄ «кулоновским ΠΏΠΎΡ‚Π΅Π½Ρ†ΠΈΠ°Π»ΠΎΠΌΒ» ΠΏΠΎΠ½ΠΈΠΌΠ°ΡŽΡ‚ элСктростатичСский ΠΏΠΎΡ‚Π΅Π½Ρ†ΠΈΠ°Π» ΠΎΠ΄Π½ΠΎΠ³ΠΎ (ΠΈΠ»ΠΈ, Π²ΠΎΠ·ΠΌΠΎΠΆΠ½ΠΎ, Π½Π΅ΡΠΊΠΎΠ»ΡŒΠΊΠΈΡ…) Ρ‚ΠΎΡ‡Π΅Ρ‡Π½ΠΎΠ³ΠΎ заряда, ΠΊΠΎΡ‚ΠΎΡ€Ρ‹ΠΉ ΠΏΠΎΠ»ΡƒΡ‡Π΅Π½ посрСдством слоТСния кулоновского ΠΏΠΎΡ‚Π΅Π½Ρ†ΠΈΠ°Π»Π° ΠΊΠ°ΠΆΠ΄ΠΎΠ³ΠΎ ΠΈΠ· Π½ΠΈΡ….

Π—Π°Ρ‡Π°ΡΡ‚ΡƒΡŽ Π΄Π°ΠΆΠ΅ Π² ситуации с ΠΏΠΎΡ‚Π΅Π½Ρ†ΠΈΠ°Π»ΠΎΠΌ, созданным нСпосрСдствСнно Π½Π΅ΠΏΡ€Π΅Ρ€Ρ‹Π²Π½ΠΎ распрСдСлСнными зарядами, Ссли Π΅Π³ΠΎ ΠΈ Π½Π°Π·Ρ‹Π²Π°ΡŽΡ‚ «кулоновским», Ρ‚ΠΎ это ΠΌΠΎΠΆΠ΅Ρ‚ ΠΎΠ·Π½Π°Ρ‡Π°Ρ‚ΡŒ Π΅Π³ΠΎ Π²Ρ‹Ρ€Π°ΠΆΠ΅Π½ΠΈΠ΅ Π² Π²ΠΈΠ΄Π΅ суммы числа элСмСнтов (ΠΏΡƒΡΡ‚ΡŒ ΠΈ бСсконСчного), Π½Π° ΠΊΠΎΡ‚ΠΎΡ€Ρ‹Π΅ разбиваСтся заряТСнный объСм, ΠΎΠ΄Π½Π°ΠΊΠΎ ΠΏΡ€ΠΈ этом ΠΏΠΎΡ‚Π΅Π½Ρ†ΠΈΠ°Π» ΠΊΠ°ΠΆΠ΄ΠΎΠ³ΠΎ рассчитываСтся Π² Π²ΠΈΠ΄Π΅ ΠΏΠΎΡ‚Π΅Π½Ρ†ΠΈΠ°Π»Π° Ρ‚ΠΎΡ‡Π΅Ρ‡Π½ΠΎΠ³ΠΎ заряда.

ΠŸΡ€ΠΈ этом, Π² связи с Ρ‚Π΅ΠΌ, Ρ‡Ρ‚ΠΎ элСктростатичСский ΠΏΠΎΡ‚Π΅Π½Ρ†ΠΈΠ°Π» ΠΌΠΎΠΆΠ΅Ρ‚ Π±Ρ‹Ρ‚ΡŒ, Π² ΠΏΡ€ΠΈΠ½Ρ†ΠΈΠΏΠ΅, Π²Ρ‹Ρ€Π°ΠΆΠ°Ρ‚ΡŒΡΡ ΠΏΠΎΠ΄ΠΎΠ±Π½Ρ‹ΠΌ ΠΎΠ±Ρ€Π°Π·ΠΎΠΌ практичСски всСгда, Ρ€Π°Π·Π³Ρ€Π°Π½ΠΈΡ‡Π΅Π½ΠΈΠ΅ Ρ‚Π΅Ρ€ΠΌΠΈΠ½ΠΎΠ² Π² Ρ‚Π°ΠΊΠΎΠΌ случаС становится довольно Ρ€Π°Π·ΠΌΡ‹Ρ‚Ρ‹ΠΌ.

Рисунок 3. ΠšΡƒΠ»ΠΎΠ½ΠΎΠ²ΡΠΊΠΈΠ΅ силы. Автор24 β€” ΠΈΠ½Ρ‚Π΅Ρ€Π½Π΅Ρ‚-Π±ΠΈΡ€ΠΆΠ° студСнчСских Ρ€Π°Π±ΠΎΡ‚

Под «кулоновским» Ρ‚Π°ΠΊΠΆΠ΅ понимаСтся ΠΏΠΎΡ‚Π΅Π½Ρ†ΠΈΠ°Π» любой ΠΏΡ€ΠΈΡ€ΠΎΠ΄Ρ‹ (ΠΈΠ½Ρ‹ΠΌΠΈ словами, ΠΎΠ½ Π½Π΅ ΠΎΠ±ΡΠ·Π°Ρ‚Π΅Π»ΡŒΠ½ΠΎ Π΄ΠΎΠ»ΠΆΠ΅Π½ Π±Ρ‹Ρ‚ΡŒ элСктричСским), ΠΊΠΎΡ‚ΠΎΡ€Ρ‹ΠΉ ΠΏΡ€ΠΈ Π½Π°Π»ΠΈΡ‡ΠΈΠΈ Ρ‚ΠΎΡ‡Π΅Ρ‡Π½ΠΎΠ³ΠΎ ΠΈΠ»ΠΈ сфСричСски симмСтричного источника Π±ΡƒΠ΄Π΅Ρ‚ зависимым ΠΎΡ‚ расстояния Π½Π° $\frac {1}{Π³}$ (Π³Ρ€Π°Π²ΠΈΡ‚Π°Ρ†ΠΈΠΎΠ½Π½Ρ‹ΠΉ ΠΏΠΎΡ‚Π΅Π½Ρ†ΠΈΠ°Π» Π² Ρ‚Π΅ΠΎΡ€ΠΈΠΈ тяготСния ΠΡŒΡŽΡ‚ΠΎΠ½Π°, Π½Π°ΠΏΡ€ΠΈΠΌΠ΅Ρ€, Ρ…ΠΎΡ‚ΡŒ Π΅Π³ΠΎ часто Π½Π°Π·Ρ‹Π²Π°ΡŽΡ‚ Β«Π½ΡŒΡŽΡ‚ΠΎΠ½ΠΎΠ²ΡΠΊΠΈΠΌΒ», ΠΏΠΎΡΠΊΠΎΠ»ΡŒΠΊΡƒ ΠΎΠ½ Π±Ρ‹Π» исслСдован Ρ€Π°Π½ΡŒΡˆΠ΅)). ОсобСнно это происходит Π² случаС нСобходимости обозначСния всСго класса ΠΏΠΎΡ‚Π΅Π½Ρ†ΠΈΠ°Π»ΠΎΠ² (Π² ΠΎΡ‚Π»ΠΈΡ‡ΠΈΠ΅ ΠΎΡ‚ ΠΏΠΎΡ‚Π΅Π½Ρ†ΠΈΠ°Π»ΠΎΠ² с Π½Π΅ΠΊΠΎΡ‚ΠΎΡ€Ρ‹ΠΌΠΈ Π΄Ρ€ΡƒΠ³ΠΈΠΌΠΈ «зависимостями» ΠΎΡ‚ расстояния.

Π­Π›Π•ΠšΠ’Π ΠžΠ‘Π’ΠΠ’Π˜Π§Π•Π‘ΠšΠ˜Π™ ΠŸΠžΠ’Π•ΠΠ¦Π˜ΠΠ›

Π­Π›Π•ΠšΠ’Π ΠžΠ‘Π’ΠΠ’Π˜Π§Π•Π‘ΠšΠ˜Π™ ΠŸΠžΠ’Π•ΠΠ¦Π˜ΠΠ›

ЭлСктростатичСская сила — это консСрвативная сила. Π­Ρ‚ΠΎ ΠΎΠ·Π½Π°Ρ‡Π°Π΅Ρ‚, Ρ‡Ρ‚ΠΎ Ρ€Π°Π±ΠΎΡ‚Π° ΠΎΡ‚ частицы зависит Ρ‚ΠΎΠ»ΡŒΠΊΠΎ ΠΎΡ‚ Π½Π°Ρ‡Π°Π»ΡŒΠ½ΠΎΠ³ΠΎ ΠΈ ΠΊΠΎΠ½Π΅Ρ‡Π½ΠΎΠ³ΠΎ полоТСния частица, Π° Π½Π΅ Π½Π° ΠΏΡ€ΠΎΠΉΠ΄Π΅Π½Π½ΠΎΠΌ ΠΏΡƒΡ‚ΠΈ. Π‘ ΠΊΠ°ΠΆΠ΄ΠΎΠΉ консСрвативной силой ΠΏΠΎΡ‚Π΅Π½Ρ†ΠΈΠ°Π»ΡŒΠ½Π°Ρ энСргия ΠΌΠΎΠΆΠ΅Ρ‚ Π±Ρ‹Ρ‚ΡŒ связана. Π’Π²Π΅Π΄Π΅Π½ΠΈΠ΅ ΠΏΠΎΡ‚Π΅Π½Ρ†ΠΈΠ°Π»ΡŒΠ½ΠΎΠΉ энСргии ΠΏΠΎΠ»Π΅Π·Π΅Π½, Ρ‚Π°ΠΊ ΠΊΠ°ΠΊ позволяСт Π½Π°ΠΌ ΠΏΡ€ΠΈΠΌΠ΅Π½ΡΡ‚ΡŒ сохранСниС мСханичСской энСргии, которая ΡƒΠΏΡ€ΠΎΡ‰Π°Π΅Ρ‚ Ρ€Π΅ΡˆΠ΅Π½ΠΈΠ΅ большого количСства Π·Π°Π΄Π°Ρ‡.

ΠŸΠΎΡ‚Π΅Π½Ρ†ΠΈΠ°Π»ΡŒΠ½Π°Ρ энСргия U, связанная с консСрвативной силой F, опрСдСляСтся Π² ΡΠ»Π΅Π΄ΡƒΡŽΡ‰ΠΈΠΌ ΠΎΠ±Ρ€Π°Π·ΠΎΠΌ

(25,1)

Π³Π΄Π΅ U (P 0 ) — ΠΏΠΎΡ‚Π΅Π½Ρ†ΠΈΠ°Π»ΡŒΠ½Π°Ρ энСргия Π² исходной ΠΏΠΎΠ·ΠΈΡ†ΠΈΠΈ. P 0 (ΠΎΠ±Ρ‹Ρ‡Π½ΠΎ U (P 0 ) = 0), Π° ΠΈΠ½Ρ‚Π΅Π³Ρ€Π°Π» ΠΏΠΎ траСкториям бСрСтся ΠΏΠΎ Π»ΡŽΠ±ΠΎΠΌΡƒ ΡƒΠ΄ΠΎΠ±Π½Ρ‹ΠΉ ΠΏΡƒΡ‚ΡŒ, ΡΠΎΠ΅Π΄ΠΈΠ½ΡΡŽΡ‰ΠΈΠΉ P 0 ΠΈ P 1 . ΠŸΠΎΡΠΊΠΎΠ»ΡŒΠΊΡƒ сила F являСтся консСрвативным, ΠΈΠ½Ρ‚Π΅Π³Ρ€Π°Π» Π² ΡƒΡ€Π°Π²Π½Π΅Π½ΠΈΠΈ (25.1) Π½Π΅ Π±ΡƒΠ΄Π΅Ρ‚ Π·Π°Π²ΠΈΡΠ΅Ρ‚ΡŒ ΠΎΡ‚ Π²Ρ‹Π±Ρ€Π°Π½Π½ΠΎΠ³ΠΎ ΠΏΡƒΡ‚ΠΈ.Если Ρ€Π°Π±ΠΎΡ‚Π° W ΠΏΠΎΠ»ΠΎΠΆΠΈΡ‚Π΅Π»ΡŒΠ½Π° (сила ΠΈ ΠΏΠ΅Ρ€Π΅ΠΌΠ΅Ρ‰Π΅Π½ΠΈΠ΅ ΡƒΠΊΠ°Π·Ρ‹Π²Π°ΡŽΡ‚ Π½Π° ΠΎΠ΄Π½ΠΎ ΠΈ Ρ‚ΠΎ ΠΆΠ΅ Π½Π°ΠΏΡ€Π°Π²Π»Π΅Π½ΠΈΠΈ) ΠΏΠΎΡ‚Π΅Π½Ρ†ΠΈΠ°Π»ΡŒΠ½Π°Ρ энСргия ΠΏΡ€ΠΈ P 1 Π±ΡƒΠ΄Π΅Ρ‚ мСньшС, Ρ‡Π΅ΠΌ ΠΏΠΎΡ‚Π΅Π½Ρ†ΠΈΠ°Π»ΡŒΠ½Π°Ρ энСргия ΠΏΡ€ΠΈ P 0 . Если энСргия сохраняСтся, ΡƒΠΌΠ΅Π½ΡŒΡˆΠ΅Π½ΠΈΠ΅ ΠΏΠΎΡ‚Π΅Π½Ρ†ΠΈΠ°Π»ΡŒΠ½Π°Ρ энСргия ΠΏΡ€ΠΈΠ²Π΅Π΄Π΅Ρ‚ ΠΊ ΡƒΠ²Π΅Π»ΠΈΡ‡Π΅Π½ΠΈΡŽ кинСтичСской энСргии. Если Ρ€Π°Π±ΠΎΡ‚Π° W ΠΎΡ‚Ρ€ΠΈΡ†Π°Ρ‚Π΅Π»Π΅Π½ (сила ΠΈ смСщСниС Π½Π°ΠΏΡ€Π°Π²Π»Π΅Π½Ρ‹ Π² ΠΏΡ€ΠΎΡ‚ΠΈΠ²ΠΎΠΏΠΎΠ»ΠΎΠΆΠ½Ρ‹Π΅ стороны), ΠΏΠΎΡ‚Π΅Π½Ρ†ΠΈΠ°Π»ΡŒΠ½Π°Ρ энСргия ΠΏΡ€ΠΈ P 1 Π±ΡƒΠ΄Π΅Ρ‚ большС, Ρ‡Π΅ΠΌ ΠΏΠΎΡ‚Π΅Π½Ρ†ΠΈΠ°Π»ΡŒΠ½Π°Ρ энСргия ΠΏΡ€ΠΈ П 0 .Если энСргия сохраняСтся, ΡƒΠ²Π΅Π»ΠΈΡ‡Π΅Π½ΠΈΠ΅ ΠΏΠΎΡ‚Π΅Π½Ρ†ΠΈΠ°Π»ΡŒΠ½ΠΎΠΉ энСргии ΠΏΡ€ΠΈΠ²Π΅Π΄Π΅Ρ‚ ΠΊ ΡƒΠΌΠ΅Π½ΡŒΡˆΠ΅Π½ΠΈΡŽ кинСтичСской энСргии. Если Π² элСктростатичСских ΠΏΡ€ΠΎΠ±Π»Π΅ΠΌΠ°Ρ… рСпСрная Ρ‚ΠΎΡ‡ΠΊΠ° P 0 ΠΎΠ±Ρ‹Ρ‡Π½ΠΎ выбираСтся ΡΠΎΠΎΡ‚Π²Π΅Ρ‚ΡΡ‚Π²ΡƒΡŽΡ‰Π΅ΠΉ бСсконСчноС расстояниС, ΠΈ ΠΏΠΎΡ‚Π΅Π½Ρ†ΠΈΠ°Π»ΡŒΠ½Π°Ρ энСргия Π² этой Ρ‚ΠΎΡ‡ΠΊΠ΅ отсчСта принимаСтся Ρ€Π°Π²Π½ΠΎΠΉ Π±Ρ‹Ρ‚ΡŒ Ρ€Π°Π²Π½Ρ‹ΠΌ Π½ΡƒΠ»ΡŽ. Π’ΠΎΠ³Π΄Π° ΡƒΡ€Π°Π²Π½Π΅Π½ΠΈΠ΅ (25.1) ΠΌΠΎΠΆΠ½ΠΎ ΠΏΠ΅Ρ€Π΅ΠΏΠΈΡΠ°Ρ‚ΡŒ ΠΊΠ°ΠΊ:

(25,2)

Π§Ρ‚ΠΎΠ±Ρ‹ ΠΎΠΏΠΈΡΠ°Ρ‚ΡŒ ΠΏΠΎΡ‚Π΅Π½Ρ†ΠΈΠ°Π»ΡŒΠ½ΡƒΡŽ ΡΠ½Π΅Ρ€Π³ΠΈΡŽ, ΡΠ²ΡΠ·Π°Π½Π½ΡƒΡŽ с распрСдСлСниСм заряда, прСдставлСна ​​концСпция элСктростатичСского ΠΏΠΎΡ‚Π΅Π½Ρ†ΠΈΠ°Π»Π° V .Π’ элСктростатичСский ΠΏΠΎΡ‚Π΅Π½Ρ†ΠΈΠ°Π» V Π² Π΄Π°Π½Π½ΠΎΠΌ ΠΏΠΎΠ»ΠΎΠΆΠ΅Π½ΠΈΠΈ опрСдСляСтся ΠΊΠ°ΠΊ ΠΏΠΎΡ‚Π΅Π½Ρ†ΠΈΠ°Π» энСргия ΠΏΡ€ΠΎΠ±Π½ΠΎΠΉ частицы, дСлСнная Π½Π° заряд q этого ΠΎΠ±ΡŠΠ΅ΠΊΡ‚Π°:

(25,3)

На послСднСм шагС уравнСния (25.3) ΠΌΡ‹ ΠΏΡ€Π΅Π΄ΠΏΠΎΠ»ΠΎΠΆΠΈΠ»ΠΈ, Ρ‡Ρ‚ΠΎ рСпСрная Ρ‚ΠΎΡ‡ΠΊΠ° P 0 бСрСтся Π½Π° бСсконСчности, ΠΈ Ρ‡Ρ‚ΠΎ элСктростатичСский ΠΏΠΎΡ‚Π΅Π½Ρ†ΠΈΠ°Π» Π½Π° эта Ρ‚ΠΎΡ‡ΠΊΠ° Ρ€Π°Π²Π½Π° 0. ΠŸΠΎΡΠΊΠΎΠ»ΡŒΠΊΡƒ сила, приходящаяся Π½Π° Π΅Π΄ΠΈΠ½ΠΈΡ†Ρƒ заряда, являСтся элСктричСским ΠΏΠΎΠ»Π΅ (см. Π³Π»Π°Π²Ρƒ 23), ΡƒΡ€. (25.3) ΠΌΠΎΠΆΠ½ΠΎ ΠΏΠ΅Ρ€Π΅ΠΏΠΈΡΠ°Ρ‚ΡŒ ΠΊΠ°ΠΊ

(25.4)

Π•Π΄ΠΈΠ½ΠΈΡ†Π° элСктростатичСского ΠΏΠΎΡ‚Π΅Π½Ρ†ΠΈΠ°Π»Π° — Π²ΠΎΠ»ΡŒΡ‚ (Π’), Π° 1 Π’ = 1 Π”ΠΆ / К = 1 Нм / Кл. Π£Ρ€Π°Π²Π½Π΅Π½ΠΈΠ΅ (25.4) ΠΏΠΎΠΊΠ°Π·Ρ‹Π²Π°Π΅Ρ‚, Ρ‡Ρ‚ΠΎ Π² качСствС Π΅Π΄ΠΈΠ½ΠΈΡ†Ρ‹ элСктричСского поля ΠΌΡ‹ Ρ‚Π°ΠΊΠΆΠ΅ ΠΌΠΎΠΆΠ΅ΠΌ ΠΈΡΠΏΠΎΠ»ΡŒΠ·ΠΎΠ²Π°Ρ‚ΡŒ Π’ / ΠΌ.

ΠžΠ±Ρ‹Ρ‡Π½ΠΎ ΠΈΡΠΏΠΎΠ»ΡŒΠ·ΡƒΠ΅ΠΌΠ°Ρ Π΅Π΄ΠΈΠ½ΠΈΡ†Π° измСрСния энСргии частицы — элСктрон-Π²ΠΎΠ»ΡŒΡ‚ (эВ). ΠΊΠΎΡ‚ΠΎΡ€Ρ‹ΠΉ опрСдСляСтся ΠΊΠ°ΠΊ ΠΈΠ·ΠΌΠ΅Π½Π΅Π½ΠΈΠ΅ кинСтичСской энСргии элСктрона, ΠΊΠΎΡ‚ΠΎΡ€Ρ‹ΠΉ двиТСтся Π½Π°Π΄ Ρ€Π°Π·Π½ΠΎΡΡ‚ΡŒΡŽ ΠΏΠΎΡ‚Π΅Π½Ρ†ΠΈΠ°Π»ΠΎΠ² 1 Π’. Π­Π»Π΅ΠΊΡ‚Ρ€ΠΎΠ½-Π²ΠΎΠ»ΡŒΡ‚ ΠΌΠΎΠΆΠ½ΠΎ ΡΠ²ΡΠ·Π°Ρ‚ΡŒ с ДТоуля Ρ‡Π΅Ρ€Π΅Π· ΡƒΡ€Π°Π²Π½Π΅Π½ΠΈΠ΅ (25.3). Π£Ρ€Π°Π²Π½Π΅Π½ΠΈΠ΅ (25.3) ΠΏΠΎΠΊΠ°Π·Ρ‹Π²Π°Π΅Ρ‚, Ρ‡Ρ‚ΠΎ ΠΈΠ·ΠΌΠ΅Π½Π΅Π½ΠΈΠ΅ энСргии элСктрон, ΠΊΠΎΠ³Π΄Π° ΠΎΠ½ пСрСсСкаСт Ρ€Π°Π·Π½ΠΎΡΡ‚ΡŒ ΠΏΠΎΡ‚Π΅Π½Ρ†ΠΈΠ°Π»ΠΎΠ² 1 Π’, Ρ€Π°Π²Π½Π° 1,6 . 10 -19 Π”ΠΆ ΠΈ, Ρ‚Π°ΠΊΠΈΠΌ ΠΎΠ±Ρ€Π°Π·ΠΎΠΌ, Π·Π°ΠΊΠ»ΡŽΡ‡Π°Π΅ΠΌ, Ρ‡Ρ‚ΠΎ 1 эВ = 1,6 . 10 -19 Π”ΠΆ

Заряд q пСрСмСщаСтся ΠΈΠ· P 0 Π² P 1 Π² нСпосрСдствСнной близости ΠΎΡ‚ заряд q ‘(см. рисунок 25.1). ЭлСктростатичСский ΠΏΠΎΡ‚Π΅Π½Ρ†ΠΈΠ°Π» ΠΏΡ€ΠΈ P 1 ΠΌΠΎΠΆΠ΅Ρ‚ ΠΎΠΏΡ€Π΅Π΄Π΅Π»ΡΡ‚ΡŒΡΡ с использованиСм ΡƒΡ€. (25.4) ΠΈ вычисляя ΠΈΠ½Ρ‚Π΅Π³Ρ€Π°Π» ΠΏΠΎ ΠΏΠΎΠΊΠ°Π·Π°Π½Π½ΠΎΠΉ Ρ‚Ρ€Π°Π΅ΠΊΡ‚ΠΎΡ€ΠΈΠΈ Π½Π° рисункС 25.1. Π’Π΄ΠΎΠ»ΡŒ ΠΊΡ€ΡƒΠ³ΠΎΠ²ΠΎΠΉ части ΠΏΡƒΡ‚ΠΈ элСктричСскоС ΠΏΠΎΠ»Π΅ ΠΈ смСщСния пСрпСндикулярны, Π° ΠΈΠ·ΠΌΠ΅Π½Π΅Π½ΠΈΠ΅ элСктростатичСского ΠΏΠΎΡ‚Π΅Π½Ρ†ΠΈΠ°Π»Π° Π±ΡƒΠ΄Π΅Ρ‚ ноль. Π‘Π»Π΅Π΄ΠΎΠ²Π°Ρ‚Π΅Π»ΡŒΠ½ΠΎ, ΡƒΡ€Π°Π²Π½Π΅Π½ΠΈΠ΅ (25.4) ΠΌΠΎΠΆΠ½ΠΎ ΠΏΠ΅Ρ€Π΅ΠΏΠΈΡΠ°Ρ‚ΡŒ ΠΊΠ°ΠΊ

(25,5)

Если заряд q ‘ΠΏΠΎΠ»ΠΎΠΆΠΈΡ‚Π΅Π»ΡŒΠ½Ρ‹ΠΉ, ΠΏΠΎΡ‚Π΅Π½Ρ†ΠΈΠ°Π» увСличиваСтся с ΡƒΠΌΠ΅Π½ΡŒΡˆΠ΅Π½ΠΈΠ΅ΠΌ расстояниС r. ЭлСктричСскоС ΠΏΠΎΠ»Π΅ Π½Π°ΠΏΡ€Π°Π²Π»Π΅Π½ΠΎ Π² сторону ΠΎΡ‚ ΠΏΠΎΠ»ΠΎΠΆΠΈΡ‚Π΅Π»ΡŒΠ½ΠΎΠ³ΠΎ заряда, ΠΈ ΠΌΡ‹ ΠΏΡ€ΠΈΡˆΠ»ΠΈ ΠΊ Π²Ρ‹Π²ΠΎΠ΄Ρƒ, Ρ‡Ρ‚ΠΎ элСктричСскоС ΠΏΠΎΠ»Π΅ Π½Π°ΠΏΡ€Π°Π²Π»Π΅Π½ΠΎ ΠΈΠ· областСй с высоким элСктростатичСским ΠΏΠΎΡ‚Π΅Π½Ρ†ΠΈΠ°Π» Π² сторону областСй с Π½ΠΈΠ·ΠΊΠΈΠΌ элСктростатичСским ΠΏΠΎΡ‚Π΅Π½Ρ†ΠΈΠ°Π»ΠΎΠΌ.

Рисунок 25.1. ΠŸΡƒΡ‚ΡŒ, ΠΏΠΎ ΠΊΠΎΡ‚ΠΎΡ€ΠΎΠΌΡƒ слСдуСт заряд q ΠΌΠ΅ΠΆΠ΄Ρƒ P 0 ΠΈ П 1 . Из опрСдСлСния элСктростатичСского ΠΏΠΎΡ‚Π΅Π½Ρ†ΠΈΠ°Π»Π° Π² Ρ‚Π΅Ρ€ΠΌΠΈΠ½Π°Ρ… ΠΏΠΎΡ‚Π΅Π½Ρ†ΠΈΠ°Π»ΡŒΠ½ΠΎΠΉ энСргии (ΡƒΡ€Π°Π²Π½Π΅Π½ΠΈΠ΅ (25.3)) ясно, Ρ‡Ρ‚ΠΎ ΠΏΠΎΡ‚Π΅Π½Ρ†ΠΈΠ°Π»ΡŒΠ½Π°Ρ энСргия заряд q ΠΏΠΎΠ΄ дСйствиСм элСктричСского поля, создаваСмого зарядом q ‘, Ρ€Π°Π²Π΅Π½ прСдоставлСно

(25,6)

ΠŸΡ€ΠΈΠΌΠ΅Ρ€: Π·Π°Π΄Π°Ρ‡Π° 25.21

ΠŸΠΎΠ»Π½Ρ‹ΠΉ заряд Q Ρ€Π°Π²Π½ΠΎΠΌΠ΅Ρ€Π½ΠΎ распрСдСлСн ΠΏΠΎ прямому ΡΡ‚Π΅Ρ€ΠΆΠ½ΡŽ Π΄Π»ΠΈΠ½Π° L.НайдитС ΠΏΠΎΡ‚Π΅Π½Ρ†ΠΈΠ°Π» Π² Ρ‚ΠΎΡ‡ΠΊΠ΅ P Π½Π° расстоянии h ΠΎΡ‚ сСрСдины ΡΡ‚Π΅Ρ€ΠΆΠ΅Π½ΡŒ (см. рисунок 25.2).

ΠŸΠΎΡ‚Π΅Π½Ρ†ΠΈΠ°Π» Π² Ρ‚ΠΎΡ‡ΠΊΠ΅ P ΠΈΠ·-Π·Π° нСбольшого сСгмСнта стСрТня Π΄Π»ΠΈΠ½ΠΎΠΉ dx ΠΈ заряд dQ, располоТСнный Π² ΠΏΠΎΠ·ΠΈΡ†ΠΈΠΈ, ΡƒΠΊΠ°Π·Π°Π½Π½ΠΎΠΉ Π½Π° рисункС 25.3, Ρ€Π°Π²Π΅Π½

(25,7)

Заряд сСгмСнта dQ связан с ΠΎΠ±Ρ‰ΠΈΠΌ зарядом Q ΠΈ Π΄Π»ΠΈΠ½ΠΎΠΉ L

. (25,8)

ΠšΠΎΠΌΠ±ΠΈΠ½ΠΈΡ€ΡƒΡ уравнСния (25.7) ΠΈ (25.8) ΠΏΠΎΠ»ΡƒΡ‡Π°Π΅ΠΌ ΡΠ»Π΅Π΄ΡƒΡŽΡ‰Π΅Π΅ Π²Ρ‹Ρ€Π°ΠΆΠ΅Π½ΠΈΠ΅ для dV:

(25.9)

Рисунок 25.2. ΠŸΡ€ΠΎΠ±Π»Π΅ΠΌΠ° 25.21. Рисунок 25.3. РСшСниС ΠΏΡ€ΠΎΠ±Π»Π΅ΠΌΡ‹ 25.21. ΠŸΠΎΠ»Π½Ρ‹ΠΉ ΠΏΠΎΡ‚Π΅Π½Ρ†ΠΈΠ°Π» Π² Ρ‚ΠΎΡ‡ΠΊΠ΅ P ΠΌΠΎΠΆΠ΅Ρ‚ Π±Ρ‹Ρ‚ΡŒ ΠΏΠΎΠ»ΡƒΡ‡Π΅Π½ суммированиСм ΠΏΠΎ всСм нСбольшиС сСгмСнты. Π­Ρ‚ΠΎ эквивалСнтно ΠΈΠ½Ρ‚Π΅Π³Ρ€ΠΈΡ€ΠΎΠ²Π°Π½ΠΈΡŽ уравнСния (25.9) ΠΌΠ΅ΠΆΠ΄Ρƒ x = — L / 2 ΠΈ Ρ… = L / 2.

(25.10)

ΠŸΡ€ΠΈΠΌΠ΅Ρ€: Π·Π°Π΄Π°Ρ‡Π° 25.15

ΠΠ»ΡŒΡ„Π°-частица с кинСтичСской энСргиСй 1.7 x 10 -12 Π”ΠΆ составляСт выстрСлил прямо Π² ядро ​​платины с ΠΎΡ‡Π΅Π½ΡŒ большого расстояния. Π§Ρ‚ΠΎ Π±ΡƒΠ΄Π΅Ρ‚ Π±Ρ‹Ρ‚ΡŒ расстояниСм блиТайшСго приблиТСния? ЭлСктричСский заряд Π°Π»ΡŒΡ„Ρ‹ частица — 2e, Π° ядра ΠΏΠ»Π°Ρ‚ΠΈΠ½Ρ‹ — 78e. Π›Π΅Ρ‡ΠΈΡ‚ΡŒ Π°Π»ΡŒΡ„Ρƒ частицы ΠΈ ядра ΠΊΠ°ΠΊ сфСричСскиС зарядовыС распрСдСлСния ΠΈ ΠΏΡ€Π΅Π½Π΅Π±Ρ€Π΅Ρ‡ΡŒ Π΄Π²ΠΈΠΆΠ΅Π½ΠΈΠ΅ ядра.

ΠΠ°Ρ‡Π°Π»ΡŒΠ½Π°Ρ мСханичСская энСргия Ρ€Π°Π²Π½Π° кинСтичСской энСргии Π°Π»ΡŒΡ„Π° частица

(25.11)

Из-Π·Π° элСктричСского отталкивания ΠΌΠ΅ΠΆΠ΄Ρƒ Π°Π»ΡŒΡ„Π°-частицСй ΠΈ ΠΏΠ»Π°Ρ‚ΠΈΠ½ΠΎΠΉ ядра, Π°Π»ΡŒΡ„Π°-частица замСдлится. На расстоянии блиТайшСго ΠΏΡ€ΠΈΠ±Π»ΠΈΠΆΠ°ΡŽΡ‚ΡΡ ΠΊ Π½ΡƒΠ»ΡŽ ΡΠΊΠΎΡ€ΠΎΡΡ‚ΡŒ Π°Π»ΡŒΡ„Π°-частицы ΠΈ, ΡΠ»Π΅Π΄ΠΎΠ²Π°Ρ‚Π΅Π»ΡŒΠ½ΠΎ, Π΅Π΅ кинСтичСская энСргия Ρ€Π°Π²Π½Π° Π½ΡƒΠ»ΡŽ. Полная мСханичСская энСргия Π² этой Ρ‚ΠΎΡ‡ΠΊΠ΅ Ρ€Π°Π²Π½Π° ΠΏΠΎΡ‚Π΅Π½Ρ†ΠΈΠ°Π»ΡŒΠ½Π°Ρ энСргия систСмы

(25.12)

Π³Π΄Π΅ q 1 — заряд Π°Π»ΡŒΡ„Π°-частицы, q 2 — заряд ядра ΠΏΠ»Π°Ρ‚ΠΈΠ½Ρ‹, d — расстояниС наибольшСго сблиТСния.ΠŸΡ€ΠΈΠΌΠ΅Π½ΡΡ Π·Π°ΠΊΠΎΠ½ сохранСния мСханичСской энСргии, ΠΏΠΎΠ»ΡƒΡ‡Π°Π΅ΠΌ

(25,13)

РасстояниС блиТайшСго сблиТСния ΠΌΠΎΠΆΠ΅Ρ‚ Π±Ρ‹Ρ‚ΡŒ ΠΏΠΎΠ»ΡƒΡ‡Π΅Π½ΠΎ ΠΈΠ· уравнСния (25.13)

(25,14)

ЭлСктричСскоС ΠΏΠΎΠ»Π΅ являСтся консСрвативным ΠΏΠΎΠ»Π΅ΠΌ, ΠΏΠΎΡΠΊΠΎΠ»ΡŒΠΊΡƒ элСктричСская сила — это консСрвативная сила. ΠžΡ‚ΡΡŽΠ΄Π° слСдуСт, Ρ‡Ρ‚ΠΎ ΠΈΠ½Ρ‚Π΅Π³Ρ€Π°Π» ΠΏΠΎ путям

(25,15)

ΠΌΠ΅ΠΆΠ΄Ρƒ Ρ‚ΠΎΡ‡ΠΊΠΎΠΉ P 0 ΠΈ Ρ‚ΠΎΡ‡ΠΊΠΎΠΉ P 1 Π½Π΅ зависит ΠΎΡ‚ ΠΏΡƒΡ‚ΠΈ ΠΌΠ΅ΠΆΠ΄Ρƒ этими двумя Ρ‚ΠΎΡ‡ΠΊΠ°ΠΌΠΈ.Π’ этом случаС ΠΈΠ½Ρ‚Π΅Π³Ρ€Π°Π» ΠΏΠΎ путям для любого Π·Π°ΠΌΠΊΠ½ΡƒΡ‚ΠΎΠ³ΠΎ ΠΏΡƒΡ‚ΠΈ Π±ΡƒΠ΄Π΅Ρ‚ Π½ΡƒΠ»Π΅ΠΌ:

(25,16)

Π£Ρ€Π°Π²Π½Π΅Π½ΠΈΠ΅ (25.16) ΠΌΠΎΠΆΠ½ΠΎ ΠΈΡΠΏΠΎΠ»ΡŒΠ·ΠΎΠ²Π°Ρ‚ΡŒ для Π΄ΠΎΠΊΠ°Π·Π°Ρ‚Π΅Π»ΡŒΡΡ‚Π²Π° интСрСсной Ρ‚Π΅ΠΎΡ€Π΅ΠΌΡ‹:

«Π²Π½ΡƒΡ‚Ρ€ΠΈ Π·Π°ΠΊΡ€Ρ‹Ρ‚ΠΎΠΉ пустой полости Π²Π½ΡƒΡ‚Ρ€ΠΈ ΠΎΠ΄Π½ΠΎΡ€ΠΎΠ΄Π½ΠΎΠ³ΠΎ ΠΏΡ€ΠΎΠ²ΠΎΠ΄Π½ΠΈΠΊΠ° элСктричСская ΠΏΠΎΠ»Π΅ Ρ‚ΠΎΡ‡Π½ΠΎ Ρ€Π°Π²Π½ΠΎ Π½ΡƒΠ»ΡŽ Β».

Рисунок 25.4. ΠŸΠΎΠΏΠ΅Ρ€Π΅Ρ‡Π½ΠΎΠ΅ сСчСниС полости Π²Π½ΡƒΡ‚Ρ€ΠΈ сфСричСской Ρ„ΠΎΡ€ΠΌΡ‹. Π΄ΠΈΡ€ΠΈΠΆΠ΅Ρ€. На рисункС 25.4 ΠΏΠΎΠΊΠ°Π·Π°Π½ΠΎ ΠΏΠΎΠΏΠ΅Ρ€Π΅Ρ‡Π½ΠΎΠ΅ сСчСниС Π²ΠΎΠ·ΠΌΠΎΠΆΠ½ΠΎΠΉ полости Π²Π½ΡƒΡ‚Ρ€ΠΈ. сфСричСский ΠΏΡ€ΠΎΠ²ΠΎΠ΄Π½ΠΈΠΊ. ΠŸΡ€Π΅Π΄ΠΏΠΎΠ»ΠΎΠΆΠΈΠΌ, Ρ‡Ρ‚ΠΎ Π²Π½ΡƒΡ‚Ρ€ΠΈ ΠΏΡ€ΠΎΠ²ΠΎΠ΄Π½ΠΈΠΊΠ° Π΅ΡΡ‚ΡŒ ΠΏΠΎΠ»Π΅ ΠΈ ΠΎΠ΄ΠΈΠ½ Π»ΠΈΠ½ΠΈΠΉ поля ΠΏΠΎΠΊΠ°Π·Π°Π½ΠΎ Π½Π° рисункС 25.4. Рассмотрим ΠΈΠ½Ρ‚Π΅Π³Ρ€Π°Π» ΠΏΠΎ путям ΠΎΡ‚ ΡƒΡ€Π°Π²Π½Π΅Π½ΠΈΠ΅ (25.16) ΠΏΠΎ ΠΏΡƒΡ‚ΠΈ, ΡƒΠΊΠ°Π·Π°Π½Π½ΠΎΠΌΡƒ Π½Π° рисункС 25.4. Π’ Π³Π»Π°Π²Π΅ 24 Π±Ρ‹Π»ΠΎ ΠΏΠΎΠΊΠ°Π·Π°Π½ΠΎ Ρ‡Ρ‚ΠΎ элСктричСскоС ΠΏΠΎΠ»Π΅ Π²Π½ΡƒΡ‚Ρ€ΠΈ ΠΏΡ€ΠΎΠ²ΠΎΠ΄Π½ΠΈΠΊΠ° Ρ€Π°Π²Π½ΠΎ Π½ΡƒΠ»ΡŽ. Π’Π°ΠΊΠΈΠΌ ΠΎΠ±Ρ€Π°Π·ΠΎΠΌ, Π²ΠΊΠ»Π°Π΄ ΠΏΡƒΡ‚ΡŒ Π²Π½ΡƒΡ‚Ρ€ΠΈ ΠΏΡ€ΠΎΠ²ΠΎΠ΄Π½ΠΈΠΊΠ° Π΄ΠΎ ΠΈΠ½Ρ‚Π΅Π³Ρ€Π°Π»Π° ΠΏΠΎ путям Ρ€Π°Π²Π΅Π½ Π½ΡƒΠ»ΡŽ. ΠŸΠΎΡΠΊΠΎΠ»ΡŒΠΊΡƒ ΠΎΡΡ‚Π°Π²ΡˆΠ°ΡΡΡ Ρ‡Π°ΡΡ‚ΡŒ ΠΏΡƒΡ‚ΠΈ выбираСтся ΠΏΠΎ силовой Π»ΠΈΠ½ΠΈΠΈ, Π½Π°ΠΏΡ€Π°Π²Π»Π΅Π½ΠΈΠ΅ двиТСния ΠΏΠΎΠ»Π΅ ΠΏΠ°Ρ€Π°Π»Π»Π΅Π»ΡŒΠ½ΠΎ Π½Π°ΠΏΡ€Π°Π²Π»Π΅Π½ΠΈΡŽ ΠΏΡƒΡ‚ΠΈ, поэтому ΠΈΠ½Ρ‚Π΅Π³Ρ€Π°Π» ΠΏΠΎ ΠΏΡƒΡ‚ΠΈ Π±ΡƒΠ΄Π΅Ρ‚ Π½Π΅Π½ΡƒΠ»Π΅Π²Ρ‹ΠΌ. Π­Ρ‚ΠΎ, ΠΎΡ‡Π΅Π²ΠΈΠ΄Π½ΠΎ, Π½Π°Ρ€ΡƒΡˆΠ°Π΅Ρ‚ ΡƒΡ€Π°Π²Π½Π΅Π½ΠΈΠ΅ (25.16), ΠΈ ΠΌΡ‹ Π΄ΠΎΠ»ΠΆΠ½Ρ‹ Π·Π°ΠΊΠ»ΡŽΡ‡ΠΈΡ‚ΡŒ, Ρ‡Ρ‚ΠΎ ΠΏΠΎΠ»Π΅ Π²Π½ΡƒΡ‚Ρ€ΠΈ полости Ρ€Π°Π²Π½ΠΎ Π½ΡƒΠ»ΡŽ (Π² этом случаС ΠΈΠ½Ρ‚Π΅Π³Ρ€Π°Π» ΠΏΠΎ путям Ρ€Π°Π²Π΅Π½ ΠΊΠΎΠ½Π΅Ρ‡Π½ΠΎ Ρ€Π°Π²Π½ΠΎ Π½ΡƒΠ»ΡŽ).

ЭлСктростатичСский ΠΏΠΎΡ‚Π΅Π½Ρ†ΠΈΠ°Π» V связан с элСктростатичСским ΠΏΠΎΠ»Π΅ΠΌ E. Если элСктричСскоС ΠΏΠΎΠ»Π΅ E извСстно, элСктростатичСский ΠΏΠΎΡ‚Π΅Π½Ρ†ΠΈΠ°Π» V ΠΌΠΎΠΆΠ½ΠΎ ΠΏΠΎΠ»ΡƒΡ‡ΠΈΡ‚ΡŒ, ΠΈΡΠΏΠΎΠ»ΡŒΠ·ΡƒΡ ΡƒΡ€Π°Π²Π½Π΅Π½ΠΈΠ΅ (25.4), ΠΈ Π½Π°ΠΎΠ±ΠΎΡ€ΠΎΡ‚. Π’ этом Ρ€Π°Π·Π΄Π΅Π»Π΅ ΠΌΡ‹ обсудим, ΠΊΠ°ΠΊ элСктричСскиС ΠΏΠΎΠ»Π΅ E ΠΌΠΎΠΆΠ½ΠΎ ΠΏΠΎΠ»ΡƒΡ‡ΠΈΡ‚ΡŒ, Ссли извСстСн элСктростатичСский ΠΏΠΎΡ‚Π΅Π½Ρ†ΠΈΠ°Π».

Рисунок 25.5. РасчСт элСктричСского поля E. Рассмотрим Π΄Π²Π΅ Ρ‚ΠΎΡ‡ΠΊΠΈ, ΠΏΠΎΠΊΠ°Π·Π°Π½Π½Ρ‹Π΅ Π½Π° рисункС 25.5. Π­Ρ‚ΠΈ Π΄Π²ΠΎΠ΅ ΠΏΠΎΡ‡Ρ‚ΠΈ ΠΈΠ΄Π΅Π½Ρ‚ΠΈΡ‡Π½Ρ‹Π΅ ΠΏΠΎΠ·ΠΈΡ†ΠΈΠΈ Ρ€Π°Π·Π΄Π΅Π»Π΅Π½Ρ‹ бСсконСчно ΠΌΠ°Π»Ρ‹ΠΌ расстояниСм dL.Π’ ΠΈΠ·ΠΌΠ΅Π½Π΅Π½ΠΈΠ΅ элСктростатичСского ΠΏΠΎΡ‚Π΅Π½Ρ†ΠΈΠ°Π»Π° ΠΌΠ΅ΠΆΠ΄Ρƒ P 1 ΠΈ P 2 даСтся

(25,17)

Π³Π΄Π΅ ΡƒΠ³ΠΎΠ» [Ρ‚Π΅Ρ‚Π°] — это ΡƒΠ³ΠΎΠ» ΠΌΠ΅ΠΆΠ΄Ρƒ Π½Π°ΠΏΡ€Π°Π²Π»Π΅Π½ΠΈΠ΅ΠΌ элСктричСского ΠΏΠΎΠ»Π΅ ΠΈ Π½Π°ΠΏΡ€Π°Π²Π»Π΅Π½ΠΈΠ΅ смСщСния (см. рисунок 25.5). Π£Ρ€Π°Π²Π½Π΅Π½ΠΈΠ΅ (25.17) ΠΌΠΎΠΆΠ½ΠΎ ΠΏΠ΅Ρ€Π΅ΠΏΠΈΡΠ°Ρ‚ΡŒ ΠΊΠ°ΠΊ

(25.18)

Π³Π΄Π΅ E L ΠΎΠ±ΠΎΠ·Π½Π°Ρ‡Π°Π΅Ρ‚ ΡΠΎΡΡ‚Π°Π²Π»ΡΡŽΡ‰ΡƒΡŽ элСктричСского поля вдоль Ось L.Если Π²Ρ‹Π±Ρ€Π°Ρ‚ΡŒ Π½Π°ΠΏΡ€Π°Π²Π»Π΅Π½ΠΈΠ΅ смСщСния, ΡΠΎΠ²ΠΏΠ°Π΄Π°ΡŽΡ‰Π΅Π΅ с По оси абсцисс ΡƒΡ€Π°Π²Π½Π΅Π½ΠΈΠ΅ (25.18) ΠΏΡ€ΠΈΠ½ΠΈΠΌΠ°Π΅Ρ‚ Π²ΠΈΠ΄

(25,19)

Для смСщСний ΠΏΠΎ оси y ΠΈ оси z ΠΏΠΎΠ»ΡƒΡ‡Π°Π΅ΠΌ

(25.20)

(25.21)

ПолноС элСктричСскоС ΠΏΠΎΠ»Π΅ E ΠΌΠΎΠΆΠ½ΠΎ ΠΏΠΎΠ»ΡƒΡ‡ΠΈΡ‚ΡŒ ΠΈΠ· элСктростатичСского ΠΏΠΎΡ‚Π΅Π½Ρ†ΠΈΠ°Π»Π° V объСдиняя уравнСния (25.19), (25.20) ΠΈ (25.21):

(25.22)

Π£Ρ€Π°Π²Π½Π΅Π½ΠΈΠ΅ (25.22) ΠΎΠ±Ρ‹Ρ‡Π½ΠΎ записываСтся Π² ΡΠ»Π΅Π΄ΡƒΡŽΡ‰Π΅ΠΌ Π²ΠΈΠ΄Π΅

(25,23)

Π³Π΄Π΅ —V — Π³Ρ€Π°Π΄ΠΈΠ΅Π½Ρ‚ ΠΏΠΎΡ‚Π΅Π½Ρ†ΠΈΠ°Π»Π° V.

Π’ΠΎ ΠΌΠ½ΠΎΠ³ΠΈΡ… элСктростатичСских Π·Π°Π΄Π°Ρ‡Π°Ρ… элСктричСскоС ΠΏΠΎΠ»Π΅ ΠΎΠΏΡ€Π΅Π΄Π΅Π»Π΅Π½Π½ΠΎΠ³ΠΎ заряда распространСниС Π΄ΠΎΠ»ΠΆΠ½ΠΎ Π±Ρ‹Ρ‚ΡŒ ΠΎΡ†Π΅Π½Π΅Π½ΠΎ. РасчСт элСктричСского поля ΠΌΠΎΠΆΠ΅Ρ‚ Π±Ρ‹Ρ‚ΡŒ осущСствляСтся двумя Ρ€Π°Π·Π½Ρ‹ΠΌΠΈ ΠΌΠ΅Ρ‚ΠΎΠ΄Π°ΠΌΠΈ:

1. элСктричСскоС ΠΏΠΎΠ»Π΅ ΠΌΠΎΠΆΠ½ΠΎ Ρ€Π°ΡΡΡ‡ΠΈΡ‚Π°Ρ‚ΡŒ, примСняя Π·Π°ΠΊΠΎΠ½ ΠšΡƒΠ»ΠΎΠ½Π° ΠΈ Π²Π΅ΠΊΡ‚ΠΎΡ€ слоТСниС Π²ΠΊΠ»Π°Π΄ΠΎΠ² ΠΎΡ‚ всСх начислСний распрСдСлСния заряда.

2. ΠΏΠΎΠ»Π½Ρ‹ΠΉ элСктростатичСский ΠΏΠΎΡ‚Π΅Π½Ρ†ΠΈΠ°Π» V ΠΌΠΎΠΆΠ΅Ρ‚ Π±Ρ‹Ρ‚ΡŒ ΠΏΠΎΠ»ΡƒΡ‡Π΅Π½ ΠΈΠ· алгСбраичСской суммы ΠΏΠΎΡ‚Π΅Π½Ρ†ΠΈΠ°Π»Π° ΠΈΠ·-Π·Π° всСх зарядов, ΡΠΎΡΡ‚Π°Π²Π»ΡΡŽΡ‰ΠΈΡ… распрСдСлСниС зарядов, ΠΈ Π·Π°Ρ‚Π΅ΠΌ ΠΈΡΠΏΠΎΠ»ΡŒΠ·ΡƒΡ ΡƒΡ€Π°Π²Π½Π΅Π½ΠΈΠ΅ (25.23) для расчСта элСктричСского поля E.

Π’ΠΎ ΠΌΠ½ΠΎΠ³ΠΈΡ… случаях ΠΌΠ΅Ρ‚ΠΎΠ΄ 2 ΠΏΡ€ΠΎΡ‰Π΅, ΠΏΠΎΡΠΊΠΎΠ»ΡŒΠΊΡƒ расчСт элСктростатичСской ΠΏΠΎΡ‚Π΅Π½Ρ†ΠΈΠ°Π» Π²ΠΊΠ»ΡŽΡ‡Π°Π΅Ρ‚ Π°Π»Π³Π΅Π±Ρ€Π°ΠΈΡ‡Π΅ΡΠΊΡƒΡŽ сумму, Π² Ρ‚ΠΎ врСмя ΠΊΠ°ΠΊ ΠΌΠ΅Ρ‚ΠΎΠ΄ 1 полагаСтся Π½Π° Π²Π΅ΠΊΡ‚ΠΎΡ€Π½ΡƒΡŽ сумму.

ΠŸΡ€ΠΈΠΌΠ΅Ρ€: Π·Π°Π΄Π°Ρ‡Π° 25.32

Π’ Π½Π΅ΠΊΠΎΡ‚ΠΎΡ€ΠΎΠΉ области пространства элСктростатичСский ΠΏΠΎΡ‚Π΅Π½Ρ†ΠΈΠ°Π» ΠΈΠΌΠ΅Π΅Ρ‚ ΡΠ»Π΅Π΄ΡƒΡŽΡ‰ΠΈΠΉ Π²ΠΈΠ΄: функция x, y ΠΈ z:

(25.24)

Π³Π΄Π΅ ΠΏΠΎΡ‚Π΅Π½Ρ†ΠΈΠ°Π» измСряСтся Π² Π²ΠΎΠ»ΡŒΡ‚Π°Ρ…, Π° расстояниС — Π² ΠΌΠ΅Ρ‚Ρ€Π°Ρ…. Найди элСктричСскоС ΠΏΠΎΠ»Π΅ Π² Ρ‚ΠΎΡ‡ΠΊΠ°Ρ… x = 2 ΠΌ, y = 2 ΠΌ.

ΠšΠΎΠΌΠΏΠΎΠ½Π΅Π½Ρ‚Ρ‹ x, y ΠΈ z элСктричСского поля E ΠΌΠΎΠ³ΡƒΡ‚ Π±Ρ‹Ρ‚ΡŒ ΠΏΠΎΠ»ΡƒΡ‡Π΅Π½Ρ‹ ΠΈΠ· Π³Ρ€Π°Π΄ΠΈΠ΅Π½Ρ‚ ΠΏΠΎΡ‚Π΅Π½Ρ†ΠΈΠ°Π»Π° V (ΡƒΡ€Π°Π²Π½Π΅Π½ΠΈΠ΅ (25.23)):

(25,25)

(25,26)

(25,27)

ΠžΡ†Π΅Π½ΠΊΠ° ΡƒΡ€Π°Π²Π½Π΅Π½ΠΈΠΉ (25.25), (25.26) ΠΈ (25.27) ΠΏΡ€ΠΈ x = 2 ΠΌ ΠΈ y = 2 ΠΌ Π΄Π°Π΅Ρ‚

(25.28)

(25.29)

(25.30)

Π’Π°ΠΊΠΈΠΌ ΠΎΠ±Ρ€Π°Π·ΠΎΠΌ

(25.31)

ΠŸΡ€ΠΈΠΌΠ΅Ρ€: Π·Π°Π΄Π°Ρ‡Π° 25.36

ΠšΠΎΠ»ΡŒΡ†ΠΎ (диск с отвСрстиСм) ΠΈΠ· Π±ΡƒΠΌΠ°Π³ΠΈ ΠΈΠΌΠ΅Π΅Ρ‚ внСшний радиус R ΠΈ Π²Π½ΡƒΡ‚Ρ€Π΅Π½Π½ΠΈΠΉ радиус R / 2 (см. рисунок 25.6). Π’Π΅Π»ΠΈΡ‡ΠΈΠ½Π° элСктричСского заряда Q Ρ€Π°Π²Π½Π° Ρ€Π°Π²Π½ΠΎΠΌΠ΅Ρ€Π½ΠΎ распрСдСляСтся ΠΏΠΎ Π±ΡƒΠΌΠ°Π³Π΅.

Π°) НайдитС ΠΏΠΎΡ‚Π΅Π½Ρ†ΠΈΠ°Π» ΠΊΠ°ΠΊ Ρ„ΡƒΠ½ΠΊΡ†ΠΈΡŽ расстояния Π½Π° оси ΠΊΠΎΠ»ΡŒΡ†Π΅Π²ΠΎΠ΅ пространство.

Π±) НайдитС элСктричСскоС ΠΏΠΎΠ»Π΅ Π½Π° оси ΠΊΠΎΠ»ΡŒΡ†Π΅Π²ΠΎΠ³ΠΎ пространства.

ΠžΠΏΡ€Π΅Π΄Π΅Π»ΠΈΠΌ ось x Ρ‚Π°ΠΊ, Ρ‡Ρ‚ΠΎΠ±Ρ‹ ΠΎΠ½Π° совпадала с осью ΠΊΠΎΠ»ΡŒΡ†Π΅Π²ΠΎΠ³ΠΎ пространства (см. Рис. 25,7). ΠŸΠ΅Ρ€Π²Ρ‹ΠΉ шаг Π² вычислСнии ΠΏΠΎΠ»Π½ΠΎΠ³ΠΎ элСктростатичСского ΠΏΠΎΡ‚Π΅Π½Ρ†ΠΈΠ°Π»Π° Π² Ρ‚ΠΎΡ‡ΠΊΠ΅ P ΠΈΠ·-Π·Π° ΠΊΠΎΠ»ΡŒΡ†Π΅Π²ΠΎΠ³ΠΎ пространства Π½Π΅ΠΎΠ±Ρ…ΠΎΠ΄ΠΈΠΌΠΎ Ρ€Π°ΡΡΡ‡ΠΈΡ‚Π°Ρ‚ΡŒ элСктростатичСский ΠΏΠΎΡ‚Π΅Π½Ρ†ΠΈΠ°Π» Π² Ρ‚ΠΎΡ‡ΠΊΠ΅ P ΠΈΠ·-Π·Π° нСбольшого участка Π·Π°Ρ‚Ρ€ΡƒΠ±Π½ΠΎΠ³ΠΎ пространства. Рассмотрим ΠΊΠΎΠ»ΡŒΡ†ΠΎ радиуса r ΠΈ ΡˆΠΈΡ€ΠΈΠ½Ρ‹ dr, ΠΊΠ°ΠΊ ΠΏΠΎΠΊΠ°Π·Π°Π½ΠΎ Π½Π° рисункС 25.7. ЭлСктростатичСский ΠΏΠΎΡ‚Π΅Π½Ρ†ΠΈΠ°Π» dV ΠΏΡ€ΠΈ P, создаваСмый это ΠΊΠΎΠ»ΡŒΡ†ΠΎ даСтся ΠΏΠΎ Π½ΠΎΠΌΠ΅Ρ€Ρƒ

(25.32)

Π³Π΄Π΅ dQ — заряд ΠΊΠΎΠ»ΡŒΡ†Π°. ΠŸΠ»ΠΎΡ‚Π½ΠΎΡΡ‚ΡŒ заряда [rho] ΠΊΠΎΠ»ΡŒΡ†Π΅Π²ΠΎΠ³ΠΎ пространства Ρ€Π°Π²Π½Π° Ρ€Π°Π²Π½ΠΎ

(25.33)

Рисунок 25.6. ΠŸΡ€ΠΎΠ±Π»Π΅ΠΌΠ° 25.36. (25.33) заряд ΠΊΠΎΠ»ΡŒΡ†Π° dQ ΠΌΠΎΠΆΠ½ΠΎ Π²Ρ‹Ρ‡ΠΈΡΠ»ΠΈΡ‚ΡŒ

(25,34)

ΠŸΠΎΠ΄ΡΡ‚Π°Π²Π»ΡΡ ΡƒΡ€Π°Π²Π½Π΅Π½ΠΈΠ΅ (25.34) Π² ΡƒΡ€Π°Π²Π½Π΅Π½ΠΈΠ΅ (25.32), ΠΏΠΎΠ»ΡƒΡ‡Π°Π΅ΠΌ

(25,35)

ΠŸΠΎΠ»Π½Ρ‹ΠΉ элСктростатичСский ΠΏΠΎΡ‚Π΅Π½Ρ†ΠΈΠ°Π» ΠΌΠΎΠΆΠ΅Ρ‚ Π±Ρ‹Ρ‚ΡŒ ΠΏΠΎΠ»ΡƒΡ‡Π΅Π½ ΠΏΡƒΡ‚Π΅ΠΌ интСгрирования уравнСния (25.35) ΠΏΠΎ всСму Π·Π°Ρ‚Ρ€ΡƒΠ±Π½ΠΎΠΌΡƒ пространству:

(25.36)

Рисунок 25.7. РасчСт элСктростатичСского ΠΏΠΎΡ‚Π΅Π½Ρ†ΠΈΠ°Π»Π° Π² Π—Π°Π΄Π°Ρ‡Π΅ 25,36. Из-Π·Π° симмСтрии Π·Π°Π΄Π°Ρ‡ΠΈ элСктричСскоС ΠΏΠΎΠ»Π΅ Π±ΡƒΠ΄Π΅Ρ‚ Π½Π°ΠΏΡ€Π°Π²Π»Π΅Π½ ΠΏΠΎ оси абсцисс. ΠΠ°ΠΏΡ€ΡΠΆΠ΅Π½Π½ΠΎΡΡ‚ΡŒ поля ΠΌΠΎΠΆΠ½ΠΎ ΠΏΠΎΠ»ΡƒΡ‡ΠΈΡ‚ΡŒ, ΠΏΡ€ΠΈΠΌΠ΅Π½ΠΈΠ² ΠΎΡ‚ уравнСния (25.23) Π΄ΠΎ уравнСния (25.36):

(25,37)

ΠŸΠΎΡΠΊΠΎΠ»ΡŒΠΊΡƒ элСктростатичСскоС ΠΏΠΎΠ»Π΅ ΠΈ элСктростатичСский ΠΏΠΎΡ‚Π΅Π½Ρ†ΠΈΠ°Π» связаны, ΠΌΡ‹ ΠΌΠΎΠΆΠ΅Ρ‚ Π·Π°ΠΌΠ΅Π½ΠΈΡ‚ΡŒ силовыС Π»ΠΈΠ½ΠΈΠΈ Ρ‚Π°ΠΊ Π½Π°Π·Ρ‹Π²Π°Π΅ΠΌΡ‹ΠΌΠΈ ΡΠΊΠ²ΠΈΠΏΠΎΡ‚Π΅Π½Ρ†ΠΈΠ°Π»ΡŒΠ½Ρ‹ΠΌΠΈ повСрхностями . Π­ΠΊΠ²ΠΈΠΏΠΎΡ‚Π΅Π½Ρ†ΠΈΠ°Π»ΡŒΠ½Ρ‹Π΅ повСрхности ΠΎΠΏΡ€Π΅Π΄Π΅Π»ΡΡŽΡ‚ΡΡ ΠΊΠ°ΠΊ повСрхности, Π½Π° ΠΊΠΎΡ‚ΠΎΡ€Ρ‹Ρ… каТдая Ρ‚ΠΎΡ‡ΠΊΠ° ΠΈΠΌΠ΅Π΅Ρ‚ ΠΎΠ΄ΠΈΠ½Π°ΠΊΠΎΠ²Ρ‹Π΅ элСктростатичСский ΠΏΠΎΡ‚Π΅Π½Ρ†ΠΈΠ°Π».ΠšΠΎΠΌΠΏΠΎΠ½Π΅Π½Ρ‚Π° элСктричСского поля, ΠΏΠ°Ρ€Π°Π»Π»Π΅Π»ΡŒΠ½Π°Ρ этому ΠΏΠΎΠ²Π΅Ρ€Ρ…Π½ΠΎΡΡ‚ΡŒ Π΄ΠΎΠ»ΠΆΠ½Π° Π±Ρ‹Ρ‚ΡŒ Ρ€Π°Π²Π½Π° Π½ΡƒΠ»ΡŽ, Ρ‚Π°ΠΊ ΠΊΠ°ΠΊ ΠΈΠ·ΠΌΠ΅Π½Π΅Π½ΠΈΠ΅ ΠΏΠΎΡ‚Π΅Π½Ρ†ΠΈΠ°Π»Π° ΠΌΠ΅ΠΆΠ΄Ρƒ всСми Ρ‚ΠΎΡ‡ΠΊΠ°ΠΌΠΈ Π½Π° эта ΠΏΠΎΠ²Π΅Ρ€Ρ…Π½ΠΎΡΡ‚ΡŒ Ρ€Π°Π²Π½Π° Π½ΡƒΠ»ΡŽ. Π­Ρ‚ΠΎ ΠΎΠ·Π½Π°Ρ‡Π°Π΅Ρ‚, Ρ‡Ρ‚ΠΎ Π½Π°ΠΏΡ€Π°Π²Π»Π΅Π½ΠΈΠ΅ элСктричСского ΠΏΠΎΠ»Π΅ пСрпСндикулярно ΡΠΊΠ²ΠΈΠΏΠΎΡ‚Π΅Π½Ρ†ΠΈΠ°Π»ΡŒΠ½Ρ‹ΠΌ повСрхностям.

На рисункС 25.8 ΠΏΠΎΠΊΠ°Π·Π°Π½ элСктричСский диполь, располоТСнный вдоль оси z. Π­Ρ‚ΠΎ состоит ΠΈΠ· Π΄Π²Π° заряда + Q ΠΈ — Q, Ρ€Π°Π·Π΄Π΅Π»Π΅Π½Π½Ρ‹Π΅ расстояниСм L. ΠΏΠΎΡ‚Π΅Π½Ρ†ΠΈΠ°Π» Π² Ρ‚ΠΎΡ‡ΠΊΠ΅ P ΠΌΠΎΠΆΠ½ΠΎ Π½Π°ΠΉΡ‚ΠΈ, суммируя ΠΏΠΎΡ‚Π΅Π½Ρ†ΠΈΠ°Π»Ρ‹, Π³Π΅Π½Π΅Ρ€ΠΈΡ€ΡƒΠ΅ΠΌΡ‹Π΅ ΠΊΠ°ΠΆΠ΄Ρ‹ΠΌ ΠΈΠ· Π΄Π²ΡƒΡ… зарядов:

(25.38)

Рисунок 25.8. ЭлСктричСский диполь. Если Ρ‚ΠΎΡ‡ΠΊΠ° P находится Π΄Π°Π»Π΅ΠΊΠΎ ΠΎΡ‚ диполя (r >> L), ΠΌΡ‹ ΠΌΠΎΠΆΠ½ΠΎ ΡΠ΄Π΅Π»Π°Ρ‚ΡŒ ΠΏΡ€ΠΈΠ±Π»ΠΈΠΆΠ΅Π½ΠΈΠ΅, Ρ‡Ρ‚ΠΎ r 1 ΠΈ r 2 ΠΏΠ°Ρ€Π°Π»Π»Π΅Π»ΡŒΠ½Ρ‹. Π’ Π΄Π°Π½Π½ΠΎΠΌ случаС

(25,39)

ΠΈ

(25,40)

Π’Π΅ΠΏΠ΅Ρ€ΡŒ элСктростатичСский ΠΏΠΎΡ‚Π΅Π½Ρ†ΠΈΠ°Π» Π² Ρ‚ΠΎΡ‡ΠΊΠ΅ P ΠΌΠΎΠΆΠ½ΠΎ ΠΏΠ΅Ρ€Π΅ΠΏΠΈΡΠ°Ρ‚ΡŒ ΠΊΠ°ΠΊ

. (25,41)

Π³Π΄Π΅ p — Π΄ΠΈΠΏΠΎΠ»ΡŒΠ½Ρ‹ΠΉ ΠΌΠΎΠΌΠ΅Π½Ρ‚ распрСдСлСния заряда.ЭлСктричСскоС ΠΏΠΎΠ»Π΅ диполь ΠΌΠΎΠΆΠ½ΠΎ ΠΏΠΎΠ»ΡƒΡ‡ΠΈΡ‚ΡŒ ΠΈΠ· уравнСния (25.41), взяв Π³Ρ€Π°Π΄ΠΈΠ΅Π½Ρ‚ (см. ΡƒΡ€Π°Π²Π½Π΅Π½ΠΈΠ΅ (25.23)).


ΠžΡ‚ΠΏΡ€Π°Π²Π»ΡΠΉΡ‚Π΅ ΠΊΠΎΠΌΠΌΠ΅Π½Ρ‚Π°Ρ€ΠΈΠΈ, вопросы ΠΈ / ΠΈΠ»ΠΈ прСдлоТСния ΠΏΠΎ элСктронной ΠΏΠΎΡ‡Ρ‚Π΅ Π½Π° адрСс [email protected] ΠΈ / ΠΈΠ»ΠΈ посСтитС домашнюю страницу Ѐрэнка Π’ΠΎΠ»ΡŒΡ„ΡΠ°.

Π£Ρ‡Π΅Π±Π½ΠΎΠ΅ пособиС ΠΏΠΎ Ρ„ΠΈΠ·ΠΈΠΊΠ΅: ЭлСктричСский ΠΏΠΎΡ‚Π΅Π½Ρ†ΠΈΠ°Π»

Π’ ΠΏΡ€Π΅Π΄Ρ‹Π΄ΡƒΡ‰Π΅ΠΌ Ρ€Π°Π·Π΄Π΅Π»Π΅ Π£Ρ€ΠΎΠΊΠ° 1 Π±Ρ‹Π»ΠΎ рассмотрСно, Ρ‡Ρ‚ΠΎ Π΄Π²ΠΈΠΆΠ΅Π½ΠΈΠ΅ ΠΏΠΎΠ»ΠΎΠΆΠΈΡ‚Π΅Π»ΡŒΠ½ΠΎΠ³ΠΎ тСстового заряда Π² элСктричСском ΠΏΠΎΠ»Π΅ сопровоТдаСтся измСнСниями ΠΏΠΎΡ‚Π΅Π½Ρ†ΠΈΠ°Π»ΡŒΠ½ΠΎΠΉ энСргии.Гравитационная аналогия использовалась для объяснСния ΠΏΡ€ΠΈΡ‡ΠΈΠ½ взаимосвязи ΠΌΠ΅ΠΆΠ΄Ρƒ мСстополоТСниСм ΠΈ ΠΏΠΎΡ‚Π΅Π½Ρ†ΠΈΠ°Π»ΡŒΠ½ΠΎΠΉ энСргиСй. ΠŸΠ΅Ρ€Π΅ΠΌΠ΅Ρ‰Π΅Π½ΠΈΠ΅ ΠΏΠΎΠ»ΠΎΠΆΠΈΡ‚Π΅Π»ΡŒΠ½ΠΎΠ³ΠΎ ΠΏΡ€ΠΎΠ±Π½ΠΎΠ³ΠΎ заряда ΠΏΡ€ΠΎΡ‚ΠΈΠ² направлСния элСктричСского поля ΠΏΠΎΡ…ΠΎΠΆΠ΅ Π½Π° ΠΏΠ΅Ρ€Π΅ΠΌΠ΅Ρ‰Π΅Π½ΠΈΠ΅ массы Π²Π²Π΅Ρ€Ρ… Π² ΠΏΡ€Π΅Π΄Π΅Π»Π°Ρ… Π³Ρ€Π°Π²ΠΈΡ‚Π°Ρ†ΠΈΠΎΠ½Π½ΠΎΠ³ΠΎ поля Π—Π΅ΠΌΠ»ΠΈ. Оба двиТСния Π±Ρ‹Π»ΠΈ Π±Ρ‹ ΠΏΠΎΠ΄ΠΎΠ±Π½Ρ‹ , ΠΈΠ΄ΡƒΡ‰Π΅ΠΌΡƒ ΠΏΡ€ΠΎΡ‚ΠΈΠ² ΠΏΡ€ΠΈΡ€ΠΎΠ΄Ρ‹ , ΠΈ ΠΏΠΎΡ‚Ρ€Π΅Π±ΠΎΠ²Π°Π»ΠΈ Π±Ρ‹ Ρ€Π°Π±ΠΎΡ‚Ρ‹ внСшнСй силы. Π­Ρ‚Π° Ρ€Π°Π±ΠΎΡ‚Π°, Π² свою ΠΎΡ‡Π΅Ρ€Π΅Π΄ΡŒ, ΡƒΠ²Π΅Π»ΠΈΡ‡ΠΈΡ‚ ΠΏΠΎΡ‚Π΅Π½Ρ†ΠΈΠ°Π»ΡŒΠ½ΡƒΡŽ ΡΠ½Π΅Ρ€Π³ΠΈΡŽ ΠΎΠ±ΡŠΠ΅ΠΊΡ‚Π°. Π‘ Π΄Ρ€ΡƒΠ³ΠΎΠΉ стороны, Π΄Π²ΠΈΠΆΠ΅Π½ΠΈΠ΅ ΠΏΠΎΠ»ΠΎΠΆΠΈΡ‚Π΅Π»ΡŒΠ½ΠΎΠ³ΠΎ ΠΏΡ€ΠΎΠ±Π½ΠΎΠ³ΠΎ заряда Π² Π½Π°ΠΏΡ€Π°Π²Π»Π΅Π½ΠΈΠΈ элСктричСского поля Π±ΡƒΠ΄Π΅Ρ‚ ΠΏΠΎΡ…ΠΎΠΆΠ΅ Π½Π° ΠΏΠ°Π΄Π΅Π½ΠΈΠ΅ массы Π² Π³Ρ€Π°Π²ΠΈΡ‚Π°Ρ†ΠΈΠΎΠ½Π½ΠΎΠΌ ΠΏΠΎΠ»Π΅ Π—Π΅ΠΌΠ»ΠΈ.Оба двиТСния Π±Ρ‹Π»ΠΈ Π±Ρ‹ ΠΏΠΎΡ…ΠΎΠΆΠΈ Π½Π° , ΠΈΠ΄ΡƒΡ‰ΠΈΠ΅ ΠΏΠΎ ΠΏΡ€ΠΈΡ€ΠΎΠ΄Π΅ , ΠΈ происходили Π±Ρ‹ Π±Π΅Π· нСобходимости Ρ€Π°Π±ΠΎΡ‚Ρ‹ внСшнСй силы. Π­Ρ‚ΠΎ Π΄Π²ΠΈΠΆΠ΅Π½ΠΈΠ΅ ΠΏΡ€ΠΈΠ²Π΅Π΄Π΅Ρ‚ ΠΊ ΠΏΠΎΡ‚Π΅Ρ€Π΅ ΠΏΠΎΡ‚Π΅Π½Ρ†ΠΈΠ°Π»ΡŒΠ½ΠΎΠΉ энСргии. ΠŸΠΎΡ‚Π΅Π½Ρ†ΠΈΠ°Π»ΡŒΠ½Π°Ρ энСргия — это запасСнная энСргия полоТСния ΠΎΠ±ΡŠΠ΅ΠΊΡ‚Π°, ΠΈ ΠΎΠ½Π° связана с располоТСниСм ΠΎΠ±ΡŠΠ΅ΠΊΡ‚Π° Π² ΠΏΠΎΠ»Π΅. Π’ этом Ρ€Π°Π·Π΄Π΅Π»Π΅ Π£Ρ€ΠΎΠΊΠ° 1 ΠΌΡ‹ прСдставим ΠΊΠΎΠ½Ρ†Π΅ΠΏΡ†ΠΈΡŽ элСктричСского ΠΏΠΎΡ‚Π΅Π½Ρ†ΠΈΠ°Π»Π° ΠΈ свяТСм это понятиС с ΠΏΠΎΡ‚Π΅Π½Ρ†ΠΈΠ°Π»ΡŒΠ½ΠΎΠΉ энСргиСй ΠΏΠΎΠ»ΠΎΠΆΠΈΡ‚Π΅Π»ΡŒΠ½ΠΎΠ³ΠΎ тСстового заряда Π² Ρ€Π°Π·Π»ΠΈΡ‡Π½Ρ‹Ρ… мСстах Π² ΠΏΡ€Π΅Π΄Π΅Π»Π°Ρ… элСктричСского поля.

Π’ΠΎΠ·Π²Ρ€Π°Ρ‰Π΅Π½ΠΈΠ΅ ΠΊ Π³Ρ€Π°Π²ΠΈΡ‚Π°Ρ†ΠΈΠΎΠ½Π½ΠΎΠΉ Π°Π½Π°Π»ΠΎΠ³ΠΈΠΈ

Π’ΠΎΠΊΡ€ΡƒΠ³ Π—Π΅ΠΌΠ»ΠΈ сущСствуСт Π³Ρ€Π°Π²ΠΈΡ‚Π°Ρ†ΠΈΠΎΠ½Π½ΠΎΠ΅ ΠΏΠΎΠ»Π΅, ΠΊΠΎΡ‚ΠΎΡ€ΠΎΠ΅ ΠΎΠΊΠ°Π·Ρ‹Π²Π°Π΅Ρ‚ Π³Ρ€Π°Π²ΠΈΡ‚Π°Ρ†ΠΈΠΎΠ½Π½ΠΎΠ΅ влияниС Π½Π° всС массы, находящиСся Π² ΠΎΠΊΡ€ΡƒΠΆΠ°ΡŽΡ‰Π΅ΠΌ Π΅Π΅ пространствС. ΠŸΠ΅Ρ€Π΅ΠΌΠ΅Ρ‰Π΅Π½ΠΈΠ΅ ΠΎΠ±ΡŠΠ΅ΠΊΡ‚Π° Π²Π²Π΅Ρ€Ρ… ΠΏΡ€ΠΎΡ‚ΠΈΠ² Π³Ρ€Π°Π²ΠΈΡ‚Π°Ρ†ΠΈΠΎΠ½Π½ΠΎΠ³ΠΎ поля ΡƒΠ²Π΅Π»ΠΈΡ‡ΠΈΠ²Π°Π΅Ρ‚ Π΅Π³ΠΎ Π³Ρ€Π°Π²ΠΈΡ‚Π°Ρ†ΠΈΠΎΠ½Π½ΡƒΡŽ ΠΏΠΎΡ‚Π΅Π½Ρ†ΠΈΠ°Π»ΡŒΠ½ΡƒΡŽ ΡΠ½Π΅Ρ€Π³ΠΈΡŽ. ΠžΠ±ΡŠΠ΅ΠΊΡ‚, двиТущийся Π²Π½ΠΈΠ· Π² ΠΏΡ€Π΅Π΄Π΅Π»Π°Ρ… Π³Ρ€Π°Π²ΠΈΡ‚Π°Ρ†ΠΈΠΎΠ½Π½ΠΎΠ³ΠΎ поля, потСрял Π±Ρ‹ ΠΏΠΎΡ‚Π΅Π½Ρ†ΠΈΠ°Π»ΡŒΠ½ΡƒΡŽ Π³Ρ€Π°Π²ΠΈΡ‚Π°Ρ†ΠΈΠΎΠ½Π½ΡƒΡŽ ΡΠ½Π΅Ρ€Π³ΠΈΡŽ. Когда гравитационная ΠΏΠΎΡ‚Π΅Π½Ρ†ΠΈΠ°Π»ΡŒΠ½Π°Ρ энСргия Π±Ρ‹Π»Π° Π²Π²Π΅Π΄Π΅Π½Π° Π² Π‘Π»ΠΎΠΊΠ΅ 5 ЀизичСского Класса, ΠΎΠ½Π° Π±Ρ‹Π»Π° ΠΎΠΏΡ€Π΅Π΄Π΅Π»Π΅Π½Π° ΠΊΠ°ΠΊ энСргия, запасСнная Π² ΠΎΠ±ΡŠΠ΅ΠΊΡ‚Π΅ ΠΈΠ·-Π·Π° Π΅Π³ΠΎ Π²Π΅Ρ€Ρ‚ΠΈΠΊΠ°Π»ΡŒΠ½ΠΎΠ³ΠΎ полоТСния Π½Π°Π΄ Π—Π΅ΠΌΠ»Π΅ΠΉ.ΠšΠΎΠ»ΠΈΡ‡Π΅ΡΡ‚Π²ΠΎ Π³Ρ€Π°Π²ΠΈΡ‚Π°Ρ†ΠΈΠΎΠ½Π½ΠΎΠΉ ΠΏΠΎΡ‚Π΅Π½Ρ†ΠΈΠ°Π»ΡŒΠ½ΠΎΠΉ энСргии, хранящСйся Π² ΠΎΠ±ΡŠΠ΅ΠΊΡ‚Π΅, зависСло ΠΎΡ‚ массы, ΠΊΠΎΡ‚ΠΎΡ€ΠΎΠΉ ΠΎΠ±Π»Π°Π΄Π°Π» ΠΎΠ±ΡŠΠ΅ΠΊΡ‚, ΠΈ ΠΎΡ‚ высоты, Π½Π° ΠΊΠΎΡ‚ΠΎΡ€ΡƒΡŽ ΠΎΠ½ Π±Ρ‹Π» поднят. Гравитационная ΠΏΠΎΡ‚Π΅Π½Ρ†ΠΈΠ°Π»ΡŒΠ½Π°Ρ энСргия зависСла ΠΎΡ‚ массы ΠΎΠ±ΡŠΠ΅ΠΊΡ‚Π° ΠΈ Π΅Π³ΠΎ высоты. Π£ ΠΎΠ±ΡŠΠ΅ΠΊΡ‚Π° с ΡƒΠ΄Π²ΠΎΠ΅Π½Π½ΠΎΠΉ массой Π±ΡƒΠ΄Π΅Ρ‚ Π²Π΄Π²ΠΎΠ΅ большС ΠΏΠΎΡ‚Π΅Π½Ρ†ΠΈΠ°Π»ΡŒΠ½ΠΎΠΉ энСргии, Π° Ρƒ ΠΎΠ±ΡŠΠ΅ΠΊΡ‚Π° с ΡƒΠ΄Π²ΠΎΠ΅Π½Π½ΠΎΠΉ высотой Π±ΡƒΠ΄Π΅Ρ‚ Π²Π΄Π²ΠΎΠ΅ большС ΠΏΠΎΡ‚Π΅Π½Ρ†ΠΈΠ°Π»ΡŒΠ½ΠΎΠΉ энСргии. ΠžΠ±Ρ‹Ρ‡Π½ΠΎ высокиС ΠΏΠΎΠ·ΠΈΡ†ΠΈΠΈ Π½Π°Π·Ρ‹Π²Π°ΡŽΡ‚ локациями с высоким ΠΏΠΎΡ‚Π΅Π½Ρ†ΠΈΠ°Π»ΠΎΠΌ энСргии. Взгляд Π½Π° Π΄ΠΈΠ°Π³Ρ€Π°ΠΌΠΌΡƒ справа ΠΏΠΎΠΊΠ°Π·Ρ‹Π²Π°Π΅Ρ‚ ΠΎΡˆΠΈΠ±ΠΎΡ‡Π½ΠΎΡΡ‚ΡŒ Ρ‚Π°ΠΊΠΎΠ³ΠΎ утвСрТдСния.ΠžΠ±Ρ€Π°Ρ‚ΠΈΡ‚Π΅ Π²Π½ΠΈΠΌΠ°Π½ΠΈΠ΅ Π½Π° Ρ‚ΠΎ, Ρ‡Ρ‚ΠΎ Π³Ρ€ΡƒΠ· вСсом 1 ΠΊΠ³, ΡƒΠ΄Π΅Ρ€ΠΆΠΈΠ²Π°Π΅ΠΌΡ‹ΠΉ Π½Π° высотС 2 ΠΌΠ΅Ρ‚Ρ€Π°, ΠΈΠΌΠ΅Π΅Ρ‚ Ρ‚Π°ΠΊΡƒΡŽ ​​ТС ΠΏΠΎΡ‚Π΅Π½Ρ†ΠΈΠ°Π»ΡŒΠ½ΡƒΡŽ ΡΠ½Π΅Ρ€Π³ΠΈΡŽ, ΠΊΠ°ΠΊ Π³Ρ€ΡƒΠ· вСсом 2 ΠΊΠ³, ΡƒΠ΄Π΅Ρ€ΠΆΠΈΠ²Π°Π΅ΠΌΡ‹ΠΉ Π½Π° высотС 1 ΠΌΠ΅Ρ‚Ρ€. ΠŸΠΎΡ‚Π΅Π½Ρ†ΠΈΠ°Π»ΡŒΠ½Π°Ρ энСргия зависит Π½Π΅ Ρ‚ΠΎΠ»ΡŒΠΊΠΎ ΠΎΡ‚ мСстополоТСния; это Ρ‚Π°ΠΊΠΆΠ΅ зависит ΠΎΡ‚ массы. Π’ этом смыслС ΠΏΠΎΡ‚Π΅Π½Ρ†ΠΈΠ°Π»ΡŒΠ½Π°Ρ гравитационная энСргия зависит ΠΊΠ°ΠΊ ΠΌΠΈΠ½ΠΈΠΌΡƒΠΌ ΠΎΡ‚ Π΄Π²ΡƒΡ… Ρ‚ΠΈΠΏΠΎΠ² Π²Π΅Π»ΠΈΡ‡ΠΈΠ½:

1) Масса — свойство ΠΎΠ±ΡŠΠ΅ΠΊΡ‚Π°, ΠΈΡΠΏΡ‹Ρ‚Ρ‹Π²Π°ΡŽΡ‰Π΅Π³ΠΎ дСйствиС Π³Ρ€Π°Π²ΠΈΡ‚Π°Ρ†ΠΈΠΎΠ½Π½ΠΎΠ³ΠΎ поля, ΠΈ

2) Высота — ΠΏΠΎΠ»ΠΎΠΆΠ΅Π½ΠΈΠ΅ Π² Π³Ρ€Π°Π²ΠΈΡ‚Π°Ρ†ΠΈΠΎΠ½Π½ΠΎΠΌ ΠΏΠΎΠ»Π΅

Π’Π°ΠΊΠΈΠΌ ΠΎΠ±Ρ€Π°Π·ΠΎΠΌ, Π½Π΅ΠΏΡ€Π°Π²ΠΈΠ»ΡŒΠ½ΠΎ Π½Π°Π·Ρ‹Π²Π°Ρ‚ΡŒ высокиС ΠΏΠΎΠ·ΠΈΡ†ΠΈΠΈ Π² Π³Ρ€Π°Π²ΠΈΡ‚Π°Ρ†ΠΈΠΎΠ½Π½ΠΎΠΌ ΠΏΠΎΠ»Π΅ Π—Π΅ΠΌΠ»ΠΈ позициями с высокой ΠΏΠΎΡ‚Π΅Π½Ρ†ΠΈΠ°Π»ΡŒΠ½ΠΎΠΉ энСргиСй.Но Π΅ΡΡ‚ΡŒ Π»ΠΈ ΠΊΠ°ΠΊΠΎΠ΅-Ρ‚ΠΎ количСство, ΠΊΠΎΡ‚ΠΎΡ€ΠΎΠ΅ ΠΌΠΎΠΆΠ½ΠΎ Π±Ρ‹Π»ΠΎ Π±Ρ‹ ΠΈΡΠΏΠΎΠ»ΡŒΠ·ΠΎΠ²Π°Ρ‚ΡŒ для ΠΎΡ†Π΅Π½ΠΊΠΈ Ρ‚Π°ΠΊΠΈΡ… высот ΠΊΠ°ΠΊ ΠΈΠΌΠ΅ΡŽΡ‰ΠΈΡ… большой ΠΏΠΎΡ‚Π΅Π½Ρ†ΠΈΠ°Π» обСспСчСния Π±ΠΎΠ»ΡŒΡˆΠΈΡ… количСств ΠΏΠΎΡ‚Π΅Π½Ρ†ΠΈΠ°Π»ΡŒΠ½ΠΎΠΉ энСргии массам, ΠΊΠΎΡ‚ΠΎΡ€Ρ‹Π΅ Ρ‚Π°ΠΌ находятся? Π”Π°! Π₯отя это Π½Π΅ обсуТдаСтся Π²ΠΎ врСмя Π±Π»ΠΎΠΊΠ° ΠΏΠΎ Π³Ρ€Π°Π²ΠΈΡ‚Π°Ρ†ΠΈΠΎΠ½Π½ΠΎΠΉ ΠΏΠΎΡ‚Π΅Π½Ρ†ΠΈΠ°Π»ΡŒΠ½ΠΎΠΉ энСргии, ΠΌΠΎΠΆΠ½ΠΎ Π±Ρ‹Π»ΠΎ Π±Ρ‹ ввСсти Π²Π΅Π»ΠΈΡ‡ΠΈΠ½Ρƒ, ΠΈΠ·Π²Π΅ΡΡ‚Π½ΡƒΡŽ ΠΊΠ°ΠΊ Π³Ρ€Π°Π²ΠΈΡ‚Π°Ρ†ΠΈΠΎΠ½Π½Ρ‹ΠΉ ΠΏΠΎΡ‚Π΅Π½Ρ†ΠΈΠ°Π» — ΠΏΠΎΡ‚Π΅Π½Ρ†ΠΈΠ°Π»ΡŒΠ½Π°Ρ энСргия Π½Π° ΠΊΠΈΠ»ΠΎΠ³Ρ€Π°ΠΌΠΌ. Π“Ρ€Π°Π²ΠΈΡ‚Π°Ρ†ΠΈΠΎΠ½Π½Ρ‹ΠΉ ΠΏΠΎΡ‚Π΅Π½Ρ†ΠΈΠ°Π» — это Π²Π΅Π»ΠΈΡ‡ΠΈΠ½Π°, ΠΊΠΎΡ‚ΠΎΡ€ΡƒΡŽ ΠΌΠΎΠΆΠ½ΠΎ ΠΈΡΠΏΠΎΠ»ΡŒΠ·ΠΎΠ²Π°Ρ‚ΡŒ для ΠΎΡ†Π΅Π½ΠΊΠΈ Ρ€Π°Π·Π»ΠΈΡ‡Π½Ρ‹Ρ… мСст Π½Π° повСрхности Π—Π΅ΠΌΠ»ΠΈ с Ρ‚ΠΎΡ‡ΠΊΠΈ зрСния Ρ‚ΠΎΠ³ΠΎ, ΠΊΠ°ΠΊΠΎΠΉ ΠΏΠΎΡ‚Π΅Π½Ρ†ΠΈΠ°Π»ΡŒΠ½ΠΎΠΉ энСргиСй Π±ΡƒΠ΄Π΅Ρ‚ ΠΎΠ±Π»Π°Π΄Π°Ρ‚ΡŒ ΠΊΠ°ΠΆΠ΄Ρ‹ΠΉ ΠΊΠΈΠ»ΠΎΠ³Ρ€Π°ΠΌΠΌ массы, ΠΊΠΎΠ³Π΄Π° ΠΎΠ½ Π±ΡƒΠ΄Π΅Ρ‚ ΠΏΠΎΠΌΠ΅Ρ‰Π΅Π½ Ρ‚ΡƒΠ΄Π°.Π’Π΅Π»ΠΈΡ‡ΠΈΠ½Π° Π³Ρ€Π°Π²ΠΈΡ‚Π°Ρ†ΠΈΠΎΠ½Π½ΠΎΠ³ΠΎ ΠΏΠΎΡ‚Π΅Π½Ρ†ΠΈΠ°Π»Π° опрСдСляСтся ΠΊΠ°ΠΊ ПЭ / масса. ΠŸΠΎΡΠΊΠΎΠ»ΡŒΠΊΡƒ Ρ‡ΠΈΡΠ»ΠΈΡ‚Π΅Π»ΡŒ ΠΈ Π·Π½Π°ΠΌΠ΅Π½Π°Ρ‚Π΅Π»ΡŒ PE / масса ΠΏΡ€ΠΎΠΏΠΎΡ€Ρ†ΠΈΠΎΠ½Π°Π»ΡŒΠ½Ρ‹ массС ΠΎΠ±ΡŠΠ΅ΠΊΡ‚Π°, Π²Ρ‹Ρ€Π°ΠΆΠ΅Π½ΠΈΠ΅ становится нСзависимым ΠΎΡ‚ массы. Π“Ρ€Π°Π²ΠΈΡ‚Π°Ρ†ΠΈΠΎΠ½Π½Ρ‹ΠΉ ΠΏΠΎΡ‚Π΅Π½Ρ†ΠΈΠ°Π» — это Π²Π΅Π»ΠΈΡ‡ΠΈΠ½Π°, зависящая ΠΎΡ‚ мСстополоТСния, которая Π½Π΅ зависит ΠΎΡ‚ массы ΠΎΠ±ΡŠΠ΅ΠΊΡ‚Π°, ΠΈΡΠΏΡ‹Ρ‚Ρ‹Π²Π°ΡŽΡ‰Π΅Π³ΠΎ ΠΏΠΎΠ»Π΅. Π“Ρ€Π°Π²ΠΈΡ‚Π°Ρ†ΠΈΠΎΠ½Π½Ρ‹ΠΉ ΠΏΠΎΡ‚Π΅Π½Ρ†ΠΈΠ°Π» описываСт эффСкты Π³Ρ€Π°Π²ΠΈΡ‚Π°Ρ†ΠΈΠΎΠ½Π½ΠΎΠ³ΠΎ поля Π½Π° ΠΎΠ±ΡŠΠ΅ΠΊΡ‚Ρ‹, ΠΊΠΎΡ‚ΠΎΡ€Ρ‹Π΅ находятся Π² Ρ€Π°Π·Π»ΠΈΡ‡Π½Ρ‹Ρ… мСстах Π²Π½ΡƒΡ‚Ρ€ΠΈ Π½Π΅Π³ΠΎ.


Если Π³Ρ€Π°Π²ΠΈΡ‚Π°Ρ†ΠΈΠΎΠ½Π½Ρ‹ΠΉ ΠΏΠΎΡ‚Π΅Π½Ρ†ΠΈΠ°Π» — это срСдство ΠΎΡ†Π΅Π½ΠΊΠΈ Ρ€Π°Π·Π»ΠΈΡ‡Π½Ρ‹Ρ… мСст Π²Π½ΡƒΡ‚Ρ€ΠΈ Π³Ρ€Π°Π²ΠΈΡ‚Π°Ρ†ΠΈΠΎΠ½Π½ΠΎΠ³ΠΎ поля с Ρ‚ΠΎΡ‡ΠΊΠΈ зрСния количСства ΠΏΠΎΡ‚Π΅Π½Ρ†ΠΈΠ°Π»ΡŒΠ½ΠΎΠΉ энСргии Π½Π° Π΅Π΄ΠΈΠ½ΠΈΡ†Ρƒ массы, Ρ‚ΠΎΠ³Π΄Π° концСпция элСктричСского ΠΏΠΎΡ‚Π΅Π½Ρ†ΠΈΠ°Π»Π° Π΄ΠΎΠ»ΠΆΠ½Π° ΠΈΠΌΠ΅Ρ‚ΡŒ Π°Π½Π°Π»ΠΎΠ³ΠΈΡ‡Π½ΠΎΠ΅ Π·Π½Π°Ρ‡Π΅Π½ΠΈΠ΅.Рассмотрим элСктричСскоС ΠΏΠΎΠ»Π΅, создаваСмоС ΠΏΠΎΠ»ΠΎΠΆΠΈΡ‚Π΅Π»ΡŒΠ½ΠΎ заряТСнным Π³Π΅Π½Π΅Ρ€Π°Ρ‚ΠΎΡ€ΠΎΠΌ Π’Π°Π½ Π΄Π΅ Π“Ρ€Π°Π°Ρ„Π°. НаправлСниС элСктричСского поля находится Π² Ρ‚ΠΎΠΌ Π½Π°ΠΏΡ€Π°Π²Π»Π΅Π½ΠΈΠΈ, Π² ΠΊΠΎΡ‚ΠΎΡ€ΠΎΠΌ Π±ΡƒΠ΄Π΅Ρ‚ ΠΏΡ€ΠΎΡ‚Π°Π»ΠΊΠΈΠ²Π°Ρ‚ΡŒΡΡ ΠΏΠΎΠ»ΠΎΠΆΠΈΡ‚Π΅Π»ΡŒΠ½Ρ‹ΠΉ ΠΈΡΠΏΡ‹Ρ‚Π°Ρ‚Π΅Π»ΡŒΠ½Ρ‹ΠΉ заряд; Π² этом случаС Π½Π°ΠΏΡ€Π°Π²Π»Π΅Π½ΠΈΠ΅ Π½Π°ΠΏΡ€Π°Π²Π»Π΅Π½ΠΎ Π½Π°Ρ€ΡƒΠΆΡƒ ΠΎΡ‚ сфСры Π’Π°Π½ Π΄Π΅ Π“Ρ€Π°Π°Ρ„Π°. ΠŸΠΎΡ‚Ρ€Π΅Π±ΡƒΠ΅Ρ‚ΡΡ Ρ€Π°Π±ΠΎΡ‚Π°, Ρ‡Ρ‚ΠΎΠ±Ρ‹ ΠΏΠ΅Ρ€Π΅ΠΌΠ΅ΡΡ‚ΠΈΡ‚ΡŒ ΠΏΠΎΠ»ΠΎΠΆΠΈΡ‚Π΅Π»ΡŒΠ½Ρ‹ΠΉ ΠΈΡΠΏΡ‹Ρ‚Π°Ρ‚Π΅Π»ΡŒΠ½Ρ‹ΠΉ заряд ΠΊ сфСрС ΠΏΡ€ΠΎΡ‚ΠΈΠ² элСктричСского поля. ΠšΠΎΠ»ΠΈΡ‡Π΅ΡΡ‚Π²ΠΎ силы, задСйствованной ΠΏΡ€ΠΈ Π²Ρ‹ΠΏΠΎΠ»Π½Π΅Π½ΠΈΠΈ Ρ€Π°Π±ΠΎΡ‚Ρ‹, зависит ΠΎΡ‚ количСства ΠΏΠ΅Ρ€Π΅ΠΌΠ΅Ρ‰Π°Π΅ΠΌΠΎΠ³ΠΎ заряда (согласно Π·Π°ΠΊΠΎΠ½Ρƒ элСктричСской силы ΠšΡƒΠ»ΠΎΠ½Π°).Π§Π΅ΠΌ большС заряд Π½Π° ΠΈΡΠΏΡ‹Ρ‚Π°Ρ‚Π΅Π»ΡŒΠ½ΠΎΠΌ зарядС, Ρ‚Π΅ΠΌ большС сила отталкивания ΠΈ Ρ‚Π΅ΠΌ большС Ρ€Π°Π±ΠΎΡ‚Ρ‹, которая Π΄ΠΎΠ»ΠΆΠ½Π° Π±Ρ‹Ρ‚ΡŒ ΠΏΡ€ΠΎΠ΄Π΅Π»Π°Π½Π° с Π½ΠΈΠΌ, Ρ‡Ρ‚ΠΎΠ±Ρ‹ ΠΏΠ΅Ρ€Π΅ΠΌΠ΅ΡΡ‚ΠΈΡ‚ΡŒ Π΅Π³ΠΎ Π½Π° Ρ‚Π°ΠΊΠΎΠ΅ ΠΆΠ΅ расстояниС. Если Π΄Π²Π° ΠΎΠ±ΡŠΠ΅ΠΊΡ‚Π° с Ρ€Π°Π·Π½Ρ‹ΠΌ зарядом — ΠΎΠ΄ΠΈΠ½ ΠΈΠ· ΠΊΠΎΡ‚ΠΎΡ€Ρ‹Ρ… Π² Π΄Π²Π° Ρ€Π°Π·Π° большС заряда Π΄Ρ€ΡƒΠ³ΠΎΠ³ΠΎ — ΠΏΠ΅Ρ€Π΅ΠΌΠ΅Ρ‰Π°ΡŽΡ‚ΡΡ Π½Π° ΠΎΠ΄ΠΈΠ½Π°ΠΊΠΎΠ²ΠΎΠ΅ расстояниС Π² элСктричСскоС ΠΏΠΎΠ»Π΅, Ρ‚ΠΎ ΠΎΠ±ΡŠΠ΅ΠΊΡ‚ с Π΄Π²ΠΎΠΉΠ½Ρ‹ΠΌ зарядом ΠΏΠΎΡ‚Ρ€Π΅Π±ΡƒΠ΅Ρ‚ Π²Π΄Π²ΠΎΠ΅ большСй силы ΠΈ, ΡΠ»Π΅Π΄ΠΎΠ²Π°Ρ‚Π΅Π»ΡŒΠ½ΠΎ, Π² Π΄Π²Π° Ρ€Π°Π·Π° большС Ρ€Π°Π±ΠΎΡ‚Ρ‹. Π­Ρ‚Π° Ρ€Π°Π±ΠΎΡ‚Π° ΠΈΠ·ΠΌΠ΅Π½ΠΈΡ‚ ΠΏΠΎΡ‚Π΅Π½Ρ†ΠΈΠ°Π»ΡŒΠ½ΡƒΡŽ ΡΠ½Π΅Ρ€Π³ΠΈΡŽ Π½Π° Π²Π΅Π»ΠΈΡ‡ΠΈΠ½Ρƒ, Ρ€Π°Π²Π½ΡƒΡŽ количСству ΠΏΡ€ΠΎΠ΄Π΅Π»Π°Π½Π½ΠΎΠΉ Ρ€Π°Π±ΠΎΡ‚Ρ‹. Π’Π°ΠΊΠΈΠΌ ΠΎΠ±Ρ€Π°Π·ΠΎΠΌ, элСктричСская ΠΏΠΎΡ‚Π΅Π½Ρ†ΠΈΠ°Π»ΡŒΠ½Π°Ρ энСргия зависит ΠΎΡ‚ количСства заряда Π½Π° ΠΎΠ±ΡŠΠ΅ΠΊΡ‚Π΅, ΠΈΡΠΏΡ‹Ρ‚Ρ‹Π²Π°ΡŽΡ‰Π΅ΠΌ ΠΏΠΎΠ»Π΅, ΠΈ ΠΎΡ‚ мСстополоТСния Π²Π½ΡƒΡ‚Ρ€ΠΈ поля.Π’Π°ΠΊ ΠΆΠ΅, ΠΊΠ°ΠΊ ΠΈ ΠΏΠΎΡ‚Π΅Π½Ρ†ΠΈΠ°Π»ΡŒΠ½Π°Ρ гравитационная энСргия, элСктричСская ΠΏΠΎΡ‚Π΅Π½Ρ†ΠΈΠ°Π»ΡŒΠ½Π°Ρ энСргия зависит ΠΊΠ°ΠΊ ΠΌΠΈΠ½ΠΈΠΌΡƒΠΌ ΠΎΡ‚ Π΄Π²ΡƒΡ… Ρ‚ΠΈΠΏΠΎΠ² Π²Π΅Π»ΠΈΡ‡ΠΈΠ½:

1) ЭлСктричСский заряд — свойство ΠΎΠ±ΡŠΠ΅ΠΊΡ‚Π°, ΠΈΡΠΏΡ‹Ρ‚Ρ‹Π²Π°ΡŽΡ‰Π΅Π³ΠΎ элСктричСскоС ΠΏΠΎΠ»Π΅, ΠΈ

2) РасстояниС ΠΎΡ‚ источника — мСстополоТСниС Π² ΠΏΡ€Π΅Π΄Π΅Π»Π°Ρ… элСктричСского поля

Π’ Ρ‚ΠΎ врСмя ΠΊΠ°ΠΊ элСктричСская ΠΏΠΎΡ‚Π΅Π½Ρ†ΠΈΠ°Π»ΡŒΠ½Π°Ρ энСргия зависит ΠΎΡ‚ заряда ΠΎΠ±ΡŠΠ΅ΠΊΡ‚Π°, ΠΈΡΠΏΡ‹Ρ‚Ρ‹Π²Π°ΡŽΡ‰Π΅Π³ΠΎ элСктричСскоС ΠΏΠΎΠ»Π΅, элСктричСский ΠΏΠΎΡ‚Π΅Π½Ρ†ΠΈΠ°Π» зависит ΠΈΡΠΊΠ»ΡŽΡ‡ΠΈΡ‚Π΅Π»ΡŒΠ½ΠΎ ΠΎΡ‚ мСстополоТСния.ЭлСктричСский ΠΏΠΎΡ‚Π΅Π½Ρ†ΠΈΠ°Π» — это ΠΏΠΎΡ‚Π΅Π½Ρ†ΠΈΠ°Π»ΡŒΠ½Π°Ρ энСргия Π½Π° заряд.

ΠŸΠΎΠ½ΡΡ‚ΠΈΠ΅ элСктричСского ΠΏΠΎΡ‚Π΅Π½Ρ†ΠΈΠ°Π»Π° ΠΈΡΠΏΠΎΠ»ΡŒΠ·ΡƒΠ΅Ρ‚ΡΡ для выраТСния эффСкта элСктричСского поля источника с Ρ‚ΠΎΡ‡ΠΊΠΈ зрСния мСстополоТСния Π²Π½ΡƒΡ‚Ρ€ΠΈ элСктричСского поля. ΠŸΡ€ΠΎΠ±Π½Ρ‹ΠΉ заряд с ΡƒΠ΄Π²ΠΎΠ΅Π½Π½Ρ‹ΠΌ количСством заряда Π±ΡƒΠ΄Π΅Ρ‚ ΠΎΠ±Π»Π°Π΄Π°Ρ‚ΡŒ ΡƒΠ΄Π²ΠΎΠ΅Π½Π½ΠΎΠΉ ΠΏΠΎΡ‚Π΅Π½Ρ†ΠΈΠ°Π»ΡŒΠ½ΠΎΠΉ энСргиСй Π² Π΄Π°Π½Π½ΠΎΠΌ мСстС; ΠΎΠ΄Π½Π°ΠΊΠΎ Π΅Π³ΠΎ элСктричСский ΠΏΠΎΡ‚Π΅Π½Ρ†ΠΈΠ°Π» Π² этом мСстС Π±ΡƒΠ΄Π΅Ρ‚ Ρ‚Π°ΠΊΠΈΠΌ ΠΆΠ΅, ΠΊΠ°ΠΊ ΠΈ Ρƒ любого Π΄Ρ€ΡƒΠ³ΠΎΠ³ΠΎ ΠΈΡΠΏΡ‹Ρ‚Π°Ρ‚Π΅Π»ΡŒΠ½ΠΎΠ³ΠΎ заряда. ΠŸΠΎΠ»ΠΎΠΆΠΈΡ‚Π΅Π»ΡŒΠ½Ρ‹ΠΉ тСстовый заряд Π±ΡƒΠ΄Π΅Ρ‚ ΠΈΠΌΠ΅Ρ‚ΡŒ высокий элСктричСский ΠΏΠΎΡ‚Π΅Π½Ρ†ΠΈΠ°Π», ΠΊΠΎΠ³Π΄Π° ΠΎΠ½ удСрТиваСтся рядом с ΠΏΠΎΠ»ΠΎΠΆΠΈΡ‚Π΅Π»ΡŒΠ½Ρ‹ΠΌ зарядом источника, ΠΈ с Π±ΠΎΠ»Π΅Π΅ Π½ΠΈΠ·ΠΊΠΈΠΌ элСктричСским ΠΏΠΎΡ‚Π΅Π½Ρ†ΠΈΠ°Π»ΠΎΠΌ, ΠΊΠΎΠ³Π΄Π° ΠΎΠ½ находится дальшС.Π’ этом смыслС элСктричСский ΠΏΠΎΡ‚Π΅Π½Ρ†ΠΈΠ°Π» становится просто свойством мСстополоТСния Π²Π½ΡƒΡ‚Ρ€ΠΈ элСктричСского поля. ΠŸΡ€Π΅Π΄ΠΏΠΎΠ»ΠΎΠΆΠΈΠΌ, Ρ‡Ρ‚ΠΎ элСктричСский ΠΏΠΎΡ‚Π΅Π½Ρ†ΠΈΠ°Π» Π² Π΄Π°Π½Π½ΠΎΠΌ мСстС составляСт 12 Π΄ΠΆΠΎΡƒΠ»Π΅ΠΉ Π½Π° ΠΊΡƒΠ»ΠΎΠ½, Ρ‚ΠΎΠ³Π΄Π° это элСктричСский ΠΏΠΎΡ‚Π΅Π½Ρ†ΠΈΠ°Π» заряТСнного ΠΎΠ±ΡŠΠ΅ΠΊΡ‚Π° Π² 1 ΠΈΠ»ΠΈ 2 ΠΊΡƒΠ»ΠΎΠ½Π°. Π£Ρ‚Π²Π΅Ρ€ΠΆΠ΄Π΅Π½ΠΈΠ΅, Ρ‡Ρ‚ΠΎ элСктричСский ΠΏΠΎΡ‚Π΅Π½Ρ†ΠΈΠ°Π» Π² Π΄Π°Π½Π½ΠΎΠΌ мСстС составляСт 12 Π΄ΠΆΠΎΡƒΠ»Π΅ΠΉ Π½Π° ΠΊΡƒΠ»ΠΎΠ½, ΠΎΠ·Π½Π°Ρ‡Π°Π»ΠΎ Π±Ρ‹, Ρ‡Ρ‚ΠΎ 2-кулоновский ΠΎΠ±ΡŠΠ΅ΠΊΡ‚ Π±ΡƒΠ΄Π΅Ρ‚ ΠΎΠ±Π»Π°Π΄Π°Ρ‚ΡŒ 24 дТоулями ΠΏΠΎΡ‚Π΅Π½Ρ†ΠΈΠ°Π»ΡŒΠ½ΠΎΠΉ энСргии Π² этом мСстС, Π° ΠΎΠ±ΡŠΠ΅ΠΊΡ‚ 0,5 ΠΊΡƒΠ»ΠΎΠ½ΠΎΠ² Π±ΡƒΠ΄Π΅Ρ‚ ΠΈΡΠΏΡ‹Ρ‚Ρ‹Π²Π°Ρ‚ΡŒ 6 Π΄ΠΆΠΎΡƒΠ»Π΅ΠΉ ΠΏΠΎΡ‚Π΅Π½Ρ†ΠΈΠ°Π»ΡŒΠ½ΠΎΠΉ энСргии Π² этом мСстС.

ЭлСктричСский ΠΏΠΎΡ‚Π΅Π½Ρ†ΠΈΠ°Π» Π² цСпях

Когда ΠΌΡ‹ Π½Π°Ρ‡Π½Π΅ΠΌ ΠΎΠ±ΡΡƒΠΆΠ΄Π°Ρ‚ΡŒ элСктричСскиС схСмы, ΠΌΡ‹ Π·Π°ΠΌΠ΅Ρ‚ΠΈΠΌ, Ρ‡Ρ‚ΠΎ элСктричСская Ρ†Π΅ΠΏΡŒ с Π±Π°Ρ‚Π°Ρ€Π΅ΠΉΠ½Ρ‹ΠΌ ΠΏΠΈΡ‚Π°Π½ΠΈΠ΅ΠΌ ΠΈΠΌΠ΅Π΅Ρ‚ мСста с высоким ΠΈ Π½ΠΈΠ·ΠΊΠΈΠΌ ΠΏΠΎΡ‚Π΅Π½Ρ†ΠΈΠ°Π»ΠΎΠΌ. Заряд, двиТущийся ΠΏΠΎ ΠΏΡ€ΠΎΠ²ΠΎΠ΄Π°ΠΌ Ρ†Π΅ΠΏΠΈ, Π±ΡƒΠ΄Π΅Ρ‚ ΡΡ‚Π°Π»ΠΊΠΈΠ²Π°Ρ‚ΡŒΡΡ с измСнСниями элСктричСского ΠΏΠΎΡ‚Π΅Π½Ρ†ΠΈΠ°Π»Π° ΠΏΡ€ΠΈ ΠΏΡ€ΠΎΡ…ΠΎΠΆΠ΄Π΅Π½ΠΈΠΈ ΠΏΠΎ Ρ†Π΅ΠΏΠΈ. Π’ элСктрохимичСских элСмСнтах Π±Π°Ρ‚Π°Ρ€Π΅ΠΈ ΠΌΠ΅ΠΆΠ΄Ρƒ двумя Π²Ρ‹Π²ΠΎΠ΄Π°ΠΌΠΈ создаСтся элСктричСскоС ΠΏΠΎΠ»Π΅, Π½Π°ΠΏΡ€Π°Π²Π»Π΅Π½Π½ΠΎΠ΅ ΠΎΡ‚ ΠΏΠΎΠ»ΠΎΠΆΠΈΡ‚Π΅Π»ΡŒΠ½ΠΎΠ³ΠΎ Π²Ρ‹Π²ΠΎΠ΄Π° ΠΊ ΠΎΡ‚Ρ€ΠΈΡ†Π°Ρ‚Π΅Π»ΡŒΠ½ΠΎΠΌΡƒ.Π’Π°ΠΊΠΈΠΌ ΠΎΠ±Ρ€Π°Π·ΠΎΠΌ, ΠΏΠ΅Ρ€Π΅ΠΌΠ΅Ρ‰Π΅Π½ΠΈΠ΅ ΠΏΠΎΠ»ΠΎΠΆΠΈΡ‚Π΅Π»ΡŒΠ½ΠΎΠ³ΠΎ тСстового заряда Ρ‡Π΅Ρ€Π΅Π· ячСйки ΠΎΡ‚ ΠΎΡ‚Ρ€ΠΈΡ†Π°Ρ‚Π΅Π»ΡŒΠ½ΠΎΠ³ΠΎ Π²Ρ‹Π²ΠΎΠ΄Π° ΠΊ ΠΏΠΎΠ»ΠΎΠΆΠΈΡ‚Π΅Π»ΡŒΠ½ΠΎΠΌΡƒ ΠΏΠΎΡ‚Ρ€Π΅Π±ΡƒΠ΅Ρ‚ Ρ€Π°Π±ΠΎΡ‚Ρ‹, Ρ‚Π°ΠΊΠΈΠΌ ΠΎΠ±Ρ€Π°Π·ΠΎΠΌ увСличивая ΠΏΠΎΡ‚Π΅Π½Ρ†ΠΈΠ°Π»ΡŒΠ½ΡƒΡŽ ΡΠ½Π΅Ρ€Π³ΠΈΡŽ ΠΊΠ°ΠΆΠ΄ΠΎΠ³ΠΎ кулоновского заряда, ΠΊΠΎΡ‚ΠΎΡ€Ρ‹ΠΉ двиТСтся ΠΏΠΎ этому ΠΏΡƒΡ‚ΠΈ. Π­Ρ‚ΠΎ соотвСтствуСт двиТСнию ΠΏΠΎΠ»ΠΎΠΆΠΈΡ‚Π΅Π»ΡŒΠ½ΠΎΠ³ΠΎ заряда ΠΏΡ€ΠΎΡ‚ΠΈΠ² элСктричСского поля. По этой ΠΏΡ€ΠΈΡ‡ΠΈΠ½Π΅ ΠΏΠΎΠ»ΠΎΠΆΠΈΡ‚Π΅Π»ΡŒΠ½Ρ‹ΠΉ Π²Ρ‹Π²ΠΎΠ΄ описываСтся ΠΊΠ°ΠΊ Π²Ρ‹Π²ΠΎΠ΄ с высоким ΠΏΠΎΡ‚Π΅Π½Ρ†ΠΈΠ°Π»ΠΎΠΌ. ΠŸΠΎΠ΄ΠΎΠ±Π½Ρ‹Π΅ рассуТдСния ΠΏΡ€ΠΈΠ²Π΅Π»ΠΈ Π±Ρ‹ ΠΊ Π·Π°ΠΊΠ»ΡŽΡ‡Π΅Π½ΠΈΡŽ, Ρ‡Ρ‚ΠΎ Π΄Π²ΠΈΠΆΠ΅Π½ΠΈΠ΅ ΠΏΠΎΠ»ΠΎΠΆΠΈΡ‚Π΅Π»ΡŒΠ½ΠΎΠ³ΠΎ заряда ΠΏΠΎ ΠΏΡ€ΠΎΠ²ΠΎΠ΄Π°ΠΌ ΠΎΡ‚ ΠΏΠΎΠ»ΠΎΠΆΠΈΡ‚Π΅Π»ΡŒΠ½ΠΎΠ³ΠΎ Π²Ρ‹Π²ΠΎΠ΄Π° ΠΊ ΠΎΡ‚Ρ€ΠΈΡ†Π°Ρ‚Π΅Π»ΡŒΠ½ΠΎΠΌΡƒ Π΄ΠΎΠ»ΠΆΠ½ΠΎ ΠΏΡ€ΠΎΠΈΡΡ…ΠΎΠ΄ΠΈΡ‚ΡŒ СстСствСнным ΠΎΠ±Ρ€Π°Π·ΠΎΠΌ.Π’Π°ΠΊΠΎΠ΅ Π΄Π²ΠΈΠΆΠ΅Π½ΠΈΠ΅ ΠΏΠΎΠ»ΠΎΠΆΠΈΡ‚Π΅Π»ΡŒΠ½ΠΎΠ³ΠΎ ΠΈΡΠΏΡ‹Ρ‚Π°Ρ‚Π΅Π»ΡŒΠ½ΠΎΠ³ΠΎ заряда Π΄ΠΎΠ»ΠΆΠ½ΠΎ ΠΏΡ€ΠΎΠΈΡΡ…ΠΎΠ΄ΠΈΡ‚ΡŒ Π² Π½Π°ΠΏΡ€Π°Π²Π»Π΅Π½ΠΈΠΈ элСктричСского поля ΠΈ Π½Π΅ Ρ‚Ρ€Π΅Π±ΡƒΠ΅Ρ‚ Ρ€Π°Π±ΠΎΡ‚Ρ‹. Заряд Π±ΡƒΠ΄Π΅Ρ‚ Ρ‚Π΅Ρ€ΡΡ‚ΡŒ ΠΏΠΎΡ‚Π΅Π½Ρ†ΠΈΠ°Π»ΡŒΠ½ΡƒΡŽ ΡΠ½Π΅Ρ€Π³ΠΈΡŽ ΠΏΡ€ΠΈ Π΄Π²ΠΈΠΆΠ΅Π½ΠΈΠΈ ΠΏΠΎ внСшнСй Ρ†Π΅ΠΏΠΈ ΠΎΡ‚ ΠΏΠΎΠ»ΠΎΠΆΠΈΡ‚Π΅Π»ΡŒΠ½ΠΎΠ³ΠΎ Π²Ρ‹Π²ΠΎΠ΄Π° ΠΊ ΠΎΡ‚Ρ€ΠΈΡ†Π°Ρ‚Π΅Π»ΡŒΠ½ΠΎΠΌΡƒ. ΠžΡ‚Ρ€ΠΈΡ†Π°Ρ‚Π΅Π»ΡŒΠ½Π°Ρ ΠΊΠ»Π΅ΠΌΠΌΠ° описываСтся ΠΊΠ°ΠΊ ΠΊΠ»Π΅ΠΌΠΌΠ° с Π½ΠΈΠ·ΠΊΠΈΠΌ ΠΏΠΎΡ‚Π΅Π½Ρ†ΠΈΠ°Π»ΠΎΠΌ. Π­Ρ‚ΠΎ Π½Π°Π·Π½Π°Ρ‡Π΅Π½ΠΈΠ΅ высокого ΠΈ Π½ΠΈΠ·ΠΊΠΎΠ³ΠΎ ΠΏΠΎΡ‚Π΅Π½Ρ†ΠΈΠ°Π»Π° ΠΊΠ»Π΅ΠΌΠΌΠ°ΠΌ элСктрохимичСской ячСйки ΠΏΡ€Π΅Π΄ΠΏΠΎΠ»Π°Π³Π°Π΅Ρ‚ Ρ‚Ρ€Π°Π΄ΠΈΡ†ΠΈΠΎΠ½Π½ΠΎΠ΅ соглашСниС ΠΎ Ρ‚ΠΎΠΌ, Ρ‡Ρ‚ΠΎ элСктричСскиС поля основаны Π½Π° Π½Π°ΠΏΡ€Π°Π²Π»Π΅Π½ΠΈΠΈ двиТСния ΠΏΠΎΠ»ΠΎΠΆΠΈΡ‚Π΅Π»ΡŒΠ½Ρ‹Ρ… тСстовых зарядов.

Π’ ΠΎΠΏΡ€Π΅Π΄Π΅Π»Π΅Π½Π½ΠΎΠΌ смыслС элСктричСская Ρ†Π΅ΠΏΡŒ — это Π½Π΅ Ρ‡Ρ‚ΠΎ ΠΈΠ½ΠΎΠ΅, ΠΊΠ°ΠΊ систСма прСобразования энСргии. Π’ элСктрохимичСских элСмСнтах элСктричСской Ρ†Π΅ΠΏΠΈ с Π±Π°Ρ‚Π°Ρ€Π΅ΠΉΠ½Ρ‹ΠΌ ΠΏΠΈΡ‚Π°Π½ΠΈΠ΅ΠΌ химичСская энСргия ΠΈΡΠΏΠΎΠ»ΡŒΠ·ΡƒΠ΅Ρ‚ΡΡ для Ρ€Π°Π±ΠΎΡ‚Ρ‹ с ΠΏΠΎΠ»ΠΎΠΆΠΈΡ‚Π΅Π»ΡŒΠ½Ρ‹ΠΌ ΠΈΡΠΏΡ‹Ρ‚Π°Ρ‚Π΅Π»ΡŒΠ½Ρ‹ΠΌ зарядом, Ρ‡Ρ‚ΠΎΠ±Ρ‹ ΠΏΠ΅Ρ€Π΅ΠΌΠ΅ΡΡ‚ΠΈΡ‚ΡŒ Π΅Π³ΠΎ с Π²Ρ‹Π²ΠΎΠ΄Π° с Π½ΠΈΠ·ΠΊΠΈΠΌ ΠΏΠΎΡ‚Π΅Π½Ρ†ΠΈΠ°Π»ΠΎΠΌ Π½Π° Π²Ρ‹Π²ΠΎΠ΄ с высоким ΠΏΠΎΡ‚Π΅Π½Ρ†ΠΈΠ°Π»ΠΎΠΌ. Π₯имичСская энСргия прСобразуСтся Π² ΡΠ»Π΅ΠΊΡ‚Ρ€ΠΈΡ‡Π΅ΡΠΊΡƒΡŽ ΠΏΠΎΡ‚Π΅Π½Ρ†ΠΈΠ°Π»ΡŒΠ½ΡƒΡŽ ΡΠ½Π΅Ρ€Π³ΠΈΡŽ Π²ΠΎ Π²Π½ΡƒΡ‚Ρ€Π΅Π½Π½Π΅ΠΉ Ρ†Π΅ΠΏΠΈ (Ρ‚ΠΎ Π΅ΡΡ‚ΡŒ Π² Π±Π°Ρ‚Π°Ρ€Π΅Π΅). Попав Π½Π° ΠΊΠ»Π΅ΠΌΠΌΡƒ с высоким ΠΏΠΎΡ‚Π΅Π½Ρ†ΠΈΠ°Π»ΠΎΠΌ, ΠΏΠΎΠ»ΠΎΠΆΠΈΡ‚Π΅Π»ΡŒΠ½Ρ‹ΠΉ тСстовый заряд Π±ΡƒΠ΄Π΅Ρ‚ ΠΏΠ΅Ρ€Π΅ΠΌΠ΅Ρ‰Π°Ρ‚ΡŒΡΡ ΠΏΠΎ внСшнСй Ρ†Π΅ΠΏΠΈ ΠΈ Π²ΠΎΠ·Π΄Π΅ΠΉΡΡ‚Π²ΠΎΠ²Π°Ρ‚ΡŒ Π½Π° Π»Π°ΠΌΠΏΠΎΡ‡ΠΊΡƒ, Π΄Π²ΠΈΠ³Π°Ρ‚Π΅Π»ΡŒ ΠΈΠ»ΠΈ ΠΊΠ°Ρ‚ΡƒΡˆΠΊΠΈ нагрСватСля, прСобразовывая свою ΡΠ»Π΅ΠΊΡ‚Ρ€ΠΈΡ‡Π΅ΡΠΊΡƒΡŽ ΠΏΠΎΡ‚Π΅Π½Ρ†ΠΈΠ°Π»ΡŒΠ½ΡƒΡŽ ΡΠ½Π΅Ρ€Π³ΠΈΡŽ Π² ΠΏΠΎΠ»Π΅Π·Π½Ρ‹Π΅ Ρ„ΠΎΡ€ΠΌΡ‹, для ΠΊΠΎΡ‚ΠΎΡ€Ρ‹Ρ… схСма Π±Ρ‹Π»Π° Ρ€Π°Π·Ρ€Π°Π±ΠΎΡ‚Π°Π½Π°.ΠŸΠΎΠ»ΠΎΠΆΠΈΡ‚Π΅Π»ΡŒΠ½Ρ‹ΠΉ тСстовый заряд возвращаСтся ΠΊ ΠΎΡ‚Ρ€ΠΈΡ†Π°Ρ‚Π΅Π»ΡŒΠ½ΠΎΠΉ ΠΊΠ»Π΅ΠΌΠΌΠ΅ с Π½ΠΈΠ·ΠΊΠΎΠΉ энСргиСй ΠΈ Π½ΠΈΠ·ΠΊΠΈΠΌ ΠΏΠΎΡ‚Π΅Π½Ρ†ΠΈΠ°Π»ΠΎΠΌ, Π³ΠΎΡ‚ΠΎΠ²Ρ‹ΠΉ ΠΊ ΠΏΠΎΠ²Ρ‚ΠΎΡ€Π΅Π½ΠΈΡŽ Ρ†ΠΈΠΊΠ»Π° (ΠΈΠ»ΠΈ, Π»ΡƒΡ‡ΡˆΠ΅ ΡΠΊΠ°Π·Π°Ρ‚ΡŒ, Ρ†Π΅ΠΏΠΈ , ) снова ΠΈ снова.

ΠŸΡ€ΠΎΠ²Π΅Ρ€ΡŒΡ‚Π΅ своС ΠΏΠΎΠ½ΠΈΠΌΠ°Π½ΠΈΠ΅

1. Π’Π΅Π»ΠΈΡ‡ΠΈΠ½Π° элСктричСского ΠΏΠΎΡ‚Π΅Π½Ρ†ΠΈΠ°Π»Π° опрСдСляСтся ΠΊΠ°ΠΊ Π²Π΅Π»ΠΈΡ‡ΠΈΠ½Π° _____.

Π°. элСктричСская ΠΏΠΎΡ‚Π΅Π½Ρ†ΠΈΠ°Π»ΡŒΠ½Π°Ρ энСргия

Π³. сила, Π΄Π΅ΠΉΡΡ‚Π²ΡƒΡŽΡ‰Π°Ρ Π½Π° заряд

Π³.ΠΏΠΎΡ‚Π΅Π½Ρ†ΠΈΠ°Π»ΡŒΠ½Π°Ρ энСргия Π½Π° заряд

Π³. усилиС Π½Π° заряд

2. Π—Π°ΠΏΠΎΠ»Π½ΠΈΡ‚Π΅ ΡΠ»Π΅Π΄ΡƒΡŽΡ‰ΡƒΡŽ запись:

Когда Π½Π°Π΄ ΠΏΠΎΠ»ΠΎΠΆΠΈΡ‚Π΅Π»ΡŒΠ½Ρ‹ΠΌ ΠΈΡΠΏΡ‹Ρ‚Π°Ρ‚Π΅Π»ΡŒΠ½Ρ‹ΠΌ зарядом выполняСтся Ρ€Π°Π±ΠΎΡ‚Π° с ΠΏΠΎΠΌΠΎΡ‰ΡŒΡŽ внСшнСй силы ΠΏΠΎ ΠΏΠ΅Ρ€Π΅ΠΌΠ΅Ρ‰Π΅Π½ΠΈΡŽ Π΅Π³ΠΎ ΠΈΠ· ΠΎΠ΄Π½ΠΎΠ³ΠΎ мСста Π² Π΄Ρ€ΡƒΠ³ΠΎΠ΅, ΠΏΠΎΡ‚Π΅Π½Ρ†ΠΈΠ°Π»ΡŒΠ½Π°Ρ энСргия _________ (увСличиваСтся, ΡƒΠΌΠ΅Π½ΡŒΡˆΠ°Π΅Ρ‚ΡΡ) ΠΈ элСктричСский ΠΏΠΎΡ‚Π΅Π½Ρ†ΠΈΠ°Π» _________ (увСличиваСтся, ΡƒΠΌΠ΅Π½ΡŒΡˆΠ°Π΅Ρ‚ΡΡ).

3.На ΡΠ»Π΅Π΄ΡƒΡŽΡ‰ΠΈΡ… Π΄ΠΈΠ°Π³Ρ€Π°ΠΌΠΌΠ°Ρ… ΠΏΠΎΠΊΠ°Π·Π°Π½ΠΎ элСктричСскоС ΠΏΠΎΠ»Π΅ (прСдставлСнноС стрСлками) ΠΈ Π΄Π²Π΅ Ρ‚ΠΎΡ‡ΠΊΠΈ, ΠΎΠ±ΠΎΠ·Π½Π°Ρ‡Π΅Π½Π½Ρ‹Π΅ A ΠΈ B, располоТСнныС Π²Π½ΡƒΡ‚Ρ€ΠΈ элСктричСского поля. ΠŸΠΎΠ»ΠΎΠΆΠΈΡ‚Π΅Π»ΡŒΠ½Ρ‹ΠΉ тСстовый заряд ΠΏΠΎΠΊΠ°Π·Π°Π½ Π² Ρ‚ΠΎΡ‡ΠΊΠ΅ A. Для ΠΊΠ°ΠΆΠ΄ΠΎΠΉ Π΄ΠΈΠ°Π³Ρ€Π°ΠΌΠΌΡ‹ ΡƒΠΊΠ°ΠΆΠΈΡ‚Π΅, Π½ΡƒΠΆΠ½ΠΎ Π»ΠΈ ΠΏΡ€ΠΎΠ΄Π΅Π»Π°Ρ‚ΡŒ Ρ€Π°Π±ΠΎΡ‚Ρƒ Π½Π°Π΄ зарядом, Ρ‡Ρ‚ΠΎΠ±Ρ‹ ΠΏΠ΅Ρ€Π΅ΠΌΠ΅ΡΡ‚ΠΈΡ‚ΡŒ Π΅Π³ΠΎ ΠΈΠ· Ρ‚ΠΎΡ‡ΠΊΠΈ A Π² Ρ‚ΠΎΡ‡ΠΊΡƒ B. НаконСц, ΡƒΠΊΠ°ΠΆΠΈΡ‚Π΅ Ρ‚ΠΎΡ‡ΠΊΡƒ (A ΠΈΠ»ΠΈ B) с наибольшСй элСктричСской ΠΏΠΎΡ‚Π΅Π½Ρ†ΠΈΠ°Π»ΡŒΠ½ΠΎΠΉ энСргиСй ΠΈ наибольший элСктричСский ΠΏΠΎΡ‚Π΅Π½Ρ†ΠΈΠ°Π».

2.2: ЭлСктростатичСский ΠΏΠΎΡ‚Π΅Π½Ρ†ΠΈΠ°Π» — Physics LibreTexts

Π˜ΡΠΏΡ‹Ρ‚Π°Ρ‚Π΅Π»ΡŒΠ½Ρ‹Π΅ заряды

ΠΠ»ΡŒΡ‚Π΅Ρ€Π½Π°Ρ‚ΠΈΠ²Π½Ρ‹ΠΉ способ Π²Π·Π³Π»ΡΠ½ΡƒΡ‚ΡŒ Π½Π° элСктричСскиС поля с Ρ‚ΠΎΡ‡ΠΊΠΈ зрСния Ρ‚ΠΎΠ³ΠΎ, Ρ‡Ρ‚ΠΎ ΠΌΡ‹ Π΄Π΅Π»Π°Π»ΠΈ Π² Ρ€Π°Π·Π΄Π΅Π»Π΅ 1.2, — с Ρ‚ΠΎΡ‡ΠΊΠΈ зрСния ΠΈΡΠΏΡ‹Ρ‚Π°Ρ‚Π΅Π»ΡŒΠ½ΠΎΠ³ΠΎ заряда . ИдСя состоит Π² Ρ‚ΠΎΠΌ, Ρ‡Ρ‚ΠΎΠ±Ρ‹ ΠΈΡΠΏΠΎΠ»ΡŒΠ·ΠΎΠ²Π°Ρ‚ΡŒ Π·Π°Ρ€ΡΠΆΠ΅Π½Π½ΡƒΡŽ Ρ‚ΠΎΡ‡Π΅Ρ‡Π½ΡƒΡŽ частицу ΠΊΠ°ΠΊ срСдство измСрСния Π²Π΅ΠΊΡ‚ΠΎΡ€ΠΎΠ² элСктричСской силы Π² Ρ€Π°Π·Π»ΠΈΡ‡Π½Ρ‹Ρ… Ρ‚ΠΎΡ‡ΠΊΠ°Ρ… пространства. Когда всС Π²Π΅ΠΊΡ‚ΠΎΡ€Ρ‹ силы нанСсСны Π½Π° ΠΊΠ°Ρ€Ρ‚Ρƒ, ΠΌΡ‹ Π·Π°Ρ‚Π΅ΠΌ Π΄Π΅Π»ΠΈΠΌ ΠΈΡ… Π½Π° заряд Ρ‚ΠΎΡ‡Π΅Ρ‡Π½ΠΎΠΉ частицы, ΠΈ Π½ΠΎΠ²Ρ‹Π΅ Π²Π΅ΠΊΡ‚ΠΎΡ€Ρ‹ становятся Π²Π΅ΠΊΡ‚ΠΎΡ€Π°ΠΌΠΈ элСктричСского поля. РаспространСнный (Π½ΠΎ нСсколько странный) способ Π·Π°ΠΏΠΈΡΠ°Ρ‚ΡŒ это матСматичСски:

\ [\ overrightarrow E \ left (\ overrightarrow r \ right) = \ lim \ limits_ {q_ {test} \ rightarrow 0} \ dfrac {\ overrightarrow F_ {on \; q_ {test}}} {q_ {test} }, \; \; \; \; \; \; \ text {Π³Π΄Π΅} \ overrightarrow r \ text {- позиция} q_ {test} \]

ΠžΠ±Π»Π°ΡΡ‚ΡŒ Π²ΠΎΠΊΡ€ΡƒΠ³ скоплСния заряда Π°Π½Π°Π»ΠΎΠ³ΠΈΡ‡Π½Ρ‹ΠΌ ΠΎΠ±Ρ€Π°Π·ΠΎΠΌ ΠΌΠΎΠΆΠ΅Ρ‚ Π±Ρ‹Ρ‚ΡŒ протСстирована с заряТСнной Ρ‚ΠΎΡ‡Π΅Ρ‡Π½ΠΎΠΉ частицСй.Π’ ΠΊΠ°ΠΆΠ΄ΠΎΠΉ Ρ‚ΠΎΡ‡ΠΊΠ΅ пространства ΠΌΠΎΠΆΠ½ΠΎ ΠΈΠ·ΠΌΠ΅Ρ€ΠΈΡ‚ΡŒ ΠΏΠΎΡ‚Π΅Π½Ρ†ΠΈΠ°Π»ΡŒΠ½ΡƒΡŽ ΡΠ½Π΅Ρ€Π³ΠΈΡŽ, которая сущСствуСт, ΠΊΠΎΠ³Π΄Π° ΠΈΡΠΏΡ‹Ρ‚Π°Ρ‚Π΅Π»ΡŒΠ½Ρ‹ΠΉ заряд пСрСносится ΠΈΠ· бСсконСчности Π² Π·Π°Π΄Π°Π½Π½ΠΎΠ΅ ΠΏΠΎΠ»ΠΎΠΆΠ΅Π½ΠΈΠ΅, Π° Π·Π°Ρ‚Π΅ΠΌ количСство ΠΈΡΠΏΡ‹Ρ‚Π°Ρ‚Π΅Π»ΡŒΠ½ΠΎΠ³ΠΎ заряда ΠΌΠΎΠΆΠ΅Ρ‚ Π±Ρ‹Ρ‚ΡŒ Ρ€Π°Π·Π΄Π΅Π»Π΅Π½ΠΎ, Ρ‚Π°ΠΊ Ρ‡Ρ‚ΠΎ всС, Ρ‡Ρ‚ΠΎ остаСтся, являСтся Ρ„ΡƒΠ½ΠΊΡ†ΠΈΠ΅ΠΉ источника. обвинСния. Π—Π°ΠΏΠΈΡˆΠ΅ΠΌ это Ρ‚Π°ΠΊ:

\ [V \ left (\ overrightarrow r \ right) = \ lim \ limits_ {q_ {test} \ rightarrow 0} \ dfrac {\ Delta U \ left (q_ {test}: \ infty \ rightarrow \ overrightarrow r \ right )} {q_ {test}}, \; \; \; \; \; \; \ text {Π³Π΄Π΅} \ overrightarrow r \ text {- Π²Π΅ΠΊΡ‚ΠΎΡ€ полоТСния} q_ {test} \]

Π­Ρ‚ΠΎΡ‚ процСсс ΠΎΡ‚ΠΎΠ±Ρ€Π°ΠΆΠ°Π΅Ρ‚ скалярноС ΠΏΠΎΠ»Π΅ , ΠΏΠΎΡΠΊΠΎΠ»ΡŒΠΊΡƒ с ΠΊΠ°ΠΆΠ΄ΠΎΠΉ Ρ‚ΠΎΡ‡ΠΊΠΎΠΉ Π² ​​пространствС связано число (Π° Π½Π΅ Π²Π΅ΠΊΡ‚ΠΎΡ€, ΠΊΠ°ΠΊ Π² случаС элСктричСского поля), ΠΈ всС эти числа относятся ΠΊ ΠΏΡ€ΠΎΠΈΠ·Π²ΠΎΠ»ΡŒΠ½ΠΎ Π²Ρ‹Π±Ρ€Π°Π½Π½ΠΎΠΌΡƒ Π·Π½Π°Ρ‡Π΅Π½ΠΈΡŽ нуля Π² Ρ‚ΠΎΡ‡ΠΊΠ΅ Π±Π΅ΡΠΊΠΎΠ½Π΅Ρ‡Π½ΠΎΡΡ‚ΡŒ.Π’Π°ΠΊ ΠΆΠ΅, ΠΊΠ°ΠΊ Π²Π΅ΠΊΡ‚ΠΎΡ€Ρ‹ элСктричСского поля Π½Π΅ ΡΠΎΠ²ΠΏΠ°Π΄Π°ΡŽΡ‚ с Π²Π΅ΠΊΡ‚ΠΎΡ€Π°ΠΌΠΈ силы, значСния Π² этом скалярном ΠΏΠΎΠ»Π΅ Π½Π΅ ΡΠ²Π»ΡΡŽΡ‚ΡΡ ΠΏΠΎΡ‚Π΅Π½Ρ†ΠΈΠ°Π»ΡŒΠ½Ρ‹ΠΌΠΈ энСргиями — Π±ΠΎΠ»Π΅Π΅ Ρ‚ΠΎΠ³ΠΎ, это ΠΌΠΎΠΆΠ½ΠΎ ΡƒΠ²ΠΈΠ΄Π΅Ρ‚ΡŒ Π΄Π°ΠΆΠ΅ Π² Π΅Π΄ΠΈΠ½ΠΈΡ†Π°Ρ… этих чисСл, ΠΊΠΎΡ‚ΠΎΡ€Ρ‹Π΅ ΠΏΡ€Π΅Π΄ΡΡ‚Π°Π²Π»ΡΡŽΡ‚ собой Π΄ΠΆΠΎΡƒΠ»ΠΈ, Ρ€Π°Π·Π΄Π΅Π»Π΅Π½Π½Ρ‹Π΅ Π½Π° ΠΊΡƒΠ»ΠΎΠ½Ρ‹. ΠžΡ‚Π½ΠΎΡˆΠ΅Π½ΠΈΠ΅ Π΄ΠΆΠΎΡƒΠ»Π΅ΠΉ Π½Π° ΠΊΡƒΠ»ΠΎΠ½ ΠΏΠΎΠ»ΡƒΡ‡ΠΈΠ»ΠΎ собствСнноС Π½Π°Π·Π²Π°Π½ΠΈΠ΅: Π²ΠΎΠ»ΡŒΡ‚ . Π‘ΠΎΠ·Π΄Π°Π½Π½ΠΎΠ΅ Π½Π°ΠΌΠΈ скалярноС ΠΏΠΎΠ»Π΅ называСтся элСктростатичСским ΠΏΠΎΡ‚Π΅Π½Ρ†ΠΈΠ°Π»ΠΎΠΌ . Подобно Π²Π΅ΠΊΡ‚ΠΎΡ€Ρƒ элСктричСского поля, это Π²Π΅Π»ΠΈΡ‡ΠΈΠ½Π°, которая опрСдСляСтся Π² ΠΊΠ°ΠΆΠ΄ΠΎΠΉ Ρ‚ΠΎΡ‡ΠΊΠ΅ пространства Π²Π±Π»ΠΈΠ·ΠΈ Π½Π΅ΠΊΠΎΡ‚ΠΎΡ€ΠΎΠ³ΠΎ элСктричСского заряда.Π’ ΠΎΡ‚Π»ΠΈΡ‡ΠΈΠ΅ ΠΎΡ‚ Π²Π΅ΠΊΡ‚ΠΎΡ€ΠΎΠ² элСктричСского поля, эти Π²Π΅Π»ΠΈΡ‡ΠΈΠ½Ρ‹ ΡΠ²Π»ΡΡŽΡ‚ΡΡ скалярами — Ρƒ Π½ΠΈΡ… Π½Π΅Ρ‚ направлСния.

ΠŸΡ€Π΅Π΄ΡƒΠΏΡ€Π΅ΠΆΠ΄Π΅Π½ΠΈΠ΅

Π’ΠΎΠ·ΠΌΠΎΠΆΠ½ΠΎ, самая большая ΠΏΡ€ΠΎΠ±Π»Π΅ΠΌΠ° для студСнтов, ΠΏΠ»ΠΎΡ…ΠΎ Π·Π½Π°ΠΊΠΎΠΌΡ‹Ρ… с элСктростатикой, — это использованиС слова Β«ΠΏΠΎΡ‚Π΅Π½Ρ†ΠΈΠ°Π»Β» Π² словС «элСктростатичСский ΠΏΠΎΡ‚Π΅Π½Ρ†ΠΈΠ°Π»Β». Π­Ρ‚ΠΎ Π½Π°Π·Π²Π°Π½ΠΈΠ΅ происходит ΠΎΡ‚ Ρ‚ΠΎΠ³ΠΎ Ρ„Π°ΠΊΡ‚Π°, Ρ‡Ρ‚ΠΎ ΠΎΠ½ΠΎ связано с элСктричСской ΠΏΠΎΡ‚Π΅Π½Ρ†ΠΈΠ°Π»ΡŒΠ½ΠΎΠΉ энСргиСй, Π½ΠΎ эти Π²Π΅Π»ΠΈΡ‡ΠΈΠ½Ρ‹ ΠΎΡ‡Π΅Π½ΡŒ Ρ€Π°Π·Π½Ρ‹Π΅, ΠΈ Ρ‡ΠΈΡ‚Π°Ρ‚Π΅Π»ΡŽ рСкомСндуСтся ΠΏΠΎΠΌΠ½ΠΈΡ‚ΡŒ ΠΎΠ± этом.

БупСрпозиция

Когда имССтся Π±ΠΎΠ»Π΅Π΅ ΠΎΠ΄Π½ΠΎΠ³ΠΎ источника элСктричСского поля Π² нСпосрСдствСнной близости ΠΎΡ‚ Ρ‚ΠΎΡ‡ΠΊΠΈ Π² пространствС, Π²ΠΊΠ»Π°Π΄Ρ‹ этих источников Π² ΠΏΠΎΠ»Π΅ Π² этой Ρ‚ΠΎΡ‡ΠΊΠ΅ ΠΌΠΎΠΆΠ½ΠΎ ΡΠ»ΠΎΠΆΠΈΡ‚ΡŒ.Π­Ρ‚ΠΎ ΠΌΠΎΠΆΠ½ΠΎ ΡƒΠ²ΠΈΠ΄Π΅Ρ‚ΡŒ просто ΠΈΠ· ΠΏΠΎΠ΄Ρ…ΠΎΠ΄Π° ΠΊ ΠΈΡΠΏΡ‹Ρ‚Π°Ρ‚Π΅Π»ΡŒΠ½ΠΎΠΌΡƒ заряду — ΠΎΡ‡Π΅Π²ΠΈΠ΄Π½ΠΎ, Ρ‡Ρ‚ΠΎ силы, Π΄Π΅ΠΉΡΡ‚Π²ΡƒΡŽΡ‰ΠΈΠ΅ Π½Π° ΠΈΡΠΏΡ‹Ρ‚Π°Ρ‚Π΅Π»ΡŒΠ½Ρ‹ΠΉ заряд, ΠΌΠΎΠΆΠ½ΠΎ ΡΠ»ΠΎΠΆΠΈΡ‚ΡŒ, ΠΈ ΠΊΠΎΠ³Π΄Π° ΠΈΡΠΏΡ‹Ρ‚Π°Ρ‚Π΅Π»ΡŒΠ½Ρ‹ΠΉ заряд Ρ€Π°Π·Π΄Π΅Π»Π΅Π½, сумма Π²Π΅ΠΊΡ‚ΠΎΡ€ΠΎΠ² элСктричСского поля остаСтся. {th} \) заряда Π΄ΠΎ полоТСния Π² пространствС, ΠΎΠ±ΠΎΠ·Π½Π°Ρ‡Π΅Π½Π½ΠΎΠ³ΠΎ Π²Π΅ΠΊΡ‚ΠΎΡ€ΠΎΠΌ полоТСния \ (\ overrightarrow r \).Π‘Π»Π΅Π΄ΡƒΠ΅Ρ‚ ΠΏΠΎΠ΄Ρ‡Π΅Ρ€ΠΊΠ½ΡƒΡ‚ΡŒ, Ρ‡Ρ‚ΠΎ \ (U \ left (q_ {test} \ right) \) , Π° Π½Π΅ прСдставляСт ΠΏΠΎΠ»Π½ΡƒΡŽ ΠΏΠΎΡ‚Π΅Π½Ρ†ΠΈΠ°Π»ΡŒΠ½ΡƒΡŽ ΡΠ½Π΅Ρ€Π³ΠΈΡŽ ΠΏΠΎΠ»Π½ΠΎΠΉ сборки заряда (Π½Π΅Ρ‚ Ρ‚Π΅Ρ€ΠΌΠΈΠ½ΠΎΠ², Π²ΠΊΠ»ΡŽΡ‡Π°ΡŽΡ‰ΠΈΡ… Ρ‚Π°ΠΊΠΈΠ΅ Ρ„Π°ΠΊΡ‚ΠΎΡ€Ρ‹, ΠΊΠ°ΠΊ \ (q_1q_2 \), Π½Π°ΠΏΡ€ΠΈΠΌΠ΅Ρ€) — ΠΎΠ½ прСдставляСт Ρ‚ΠΎΠ»ΡŒΠΊΠΎ Ρ‡Π°ΡΡ‚ΡŒ ΠΏΠΎΡ‚Π΅Π½Ρ†ΠΈΠ°Π»ΡŒΠ½ΠΎΠΉ энСргии, ΠΏΠΎΠ»ΡƒΡ‡Π΅Π½Π½ΠΎΠΉ систСмой ΠΈΠ·-Π·Π° ввСдСния ΠΈΡΠΏΡ‹Ρ‚Π°Ρ‚Π΅Π»ΡŒΠ½ΠΎΠ³ΠΎ заряда, пСрСносимого ΠΈΠ· бСсконСчности. Π’Π°ΠΊΠΈΠΌ ΠΎΠ±Ρ€Π°Π·ΠΎΠΌ, элСктростатичСский ΠΏΠΎΡ‚Π΅Π½Ρ†ΠΈΠ°Π» рассматриваСт всС заряды, ΠΊΠΎΡ‚ΠΎΡ€Ρ‹Π΅ Π½Π΅ ΡΠ²Π»ΡΡŽΡ‚ΡΡ ΠΏΡ€ΠΎΠ±Π½Ρ‹ΠΌ зарядом, ΠΊΠ°ΠΊ ΠΊΠΎΠ»Π»Π΅ΠΊΡ‚ΠΈΠ²Π½Ρ‹ΠΉ источник скалярного поля.ΠžΠ±Ρ€Π°Ρ‚ΠΈΡ‚Π΅ Π²Π½ΠΈΠΌΠ°Π½ΠΈΠ΅, Ρ‡Ρ‚ΠΎ, приняв соглашСниС \ (U \ left (\ infty \ right) = 0 \), ΠΌΡ‹ Ρ‚Π°ΠΊΠΆΠ΅ сдСлали это для элСктростатичСского ΠΏΠΎΡ‚Π΅Π½Ρ†ΠΈΠ°Π»Π°. И, ΠΊΠ°ΠΊ ΠΈ Π² случаС с ΠΏΠΎΡ‚Π΅Π½Ρ†ΠΈΠ°Π»ΡŒΠ½ΠΎΠΉ энСргиСй, ΠΏΠΎΠ»ΠΎΠΆΠ΅Π½ΠΈΠ΅, ΠΊΠΎΡ‚ΠΎΡ€ΠΎΠ΅ ΠΌΡ‹ Π²Ρ‹Π±ΠΈΡ€Π°Π΅ΠΌ для обозначСния Π½ΡƒΠ»Π΅Π²ΠΎΠ³ΠΎ элСктричСского ΠΏΠΎΡ‚Π΅Π½Ρ†ΠΈΠ°Π»Π°, ΠΏΡ€ΠΎΠΈΠ·Π²ΠΎΠ»ΡŒΠ½ΠΎ.

ВсС Π²Π΅Ρ‰ΠΈ, ΠΊΠΎΡ‚ΠΎΡ€Ρ‹Π΅ ΠΌΡ‹ Ρ€Π°Π·Ρ€Π°Π±ΠΎΡ‚Π°Π»ΠΈ для элСктричСских ΠΏΠΎΠ»Π΅ΠΉ, Ρ‚Π°ΠΊΠΆΠ΅ ΠΏΡ€ΠΈΠΌΠ΅Π½ΠΈΠΌΡ‹ ΠΊ ΠΏΠΎΡ‚Π΅Π½Ρ†ΠΈΠ°Π»Π°ΠΌ, с Ρ‚ΠΎΠΉ лишь Ρ€Π°Π·Π½ΠΈΡ†Π΅ΠΉ, Ρ‡Ρ‚ΠΎ ΠΏΠΎΡ‚Π΅Π½Ρ†ΠΈΠ°Π»Ρ‹ Π½Π°ΠΊΠ»Π°Π΄Ρ‹Π²Π°ΡŽΡ‚ΡΡ Π΄Ρ€ΡƒΠ³ Π½Π° Π΄Ρ€ΡƒΠ³Π° ΠΊΠ°ΠΊ скаляры, Π° Π½Π΅ ΠΊΠ°ΠΊ Π²Π΅ΠΊΡ‚ΠΎΡ€Ρ‹ (Ρ‡Ρ‚ΠΎ Π½Π° самом Π΄Π΅Π»Π΅ Π²ΠΎ ΠΌΠ½ΠΎΠ³ΠΈΡ… случаях ΡƒΠΏΡ€ΠΎΡ‰Π°Π΅Ρ‚ Ρ€Π°Π±ΠΎΡ‚Ρƒ с Π½ΠΈΠΌΠΈ). Π‘ΡƒΡ‚ΡŒ Π² Ρ‚ΠΎΠΌ, Ρ‡Ρ‚ΠΎ ΠΊΠΎΠ³Π΄Π° Ρƒ нас Π΅ΡΡ‚ΡŒ Π½Π°Π±ΠΎΡ€ исходных зарядов, Π²ΠΊΠ»ΡŽΡ‡Π°Ρ Π½Π΅ΠΏΡ€Π΅Ρ€Ρ‹Π²Π½ΠΎΠ΅ распрСдСлСниС, ΠΌΡ‹ ΠΌΠΎΠΆΠ΅ΠΌ ΠΎΠΏΡ€Π΅Π΄Π΅Π»ΠΈΡ‚ΡŒ ΠΏΠΎΡ‚Π΅Π½Ρ†ΠΈΠ°Π» Π² ΠΊΠ°ΠΆΠ΄ΠΎΠΉ Ρ‚ΠΎΡ‡ΠΊΠ΅ пространства, ΠΈ Ссли ΠΌΡ‹ помСстим Ρ‚ΡƒΠ΄Π° Ρ‚ΠΎΡ‡Π΅Ρ‡Π½Ρ‹ΠΉ заряд, ΠΌΡ‹ ΠΌΠΎΠΆΠ΅ΠΌ ΠΎΠΏΡ€Π΅Π΄Π΅Π»ΠΈΡ‚ΡŒ Π΅Π³ΠΎ ΠΏΠΎΡ‚Π΅Π½Ρ†ΠΈΠ°Π»ΡŒΠ½ΡƒΡŽ ΡΠ½Π΅Ρ€Π³ΠΈΡŽ, ΡƒΠΌΠ½ΠΎΠΆΠΈΠ² заряд Π½Π° элСктричСский ΠΏΠΎΡ‚Π΅Π½Ρ†ΠΈΠ°Π»:

\ [U = qV \ left (\ overrightarrow r \ right), \; \; \; \; \; \; \ text {where} \ overrightarrow r = \ text {Π²Π΅ΠΊΡ‚ΠΎΡ€ полоТСния заряда} q \]

Бходство с ΡƒΡ€Π°Π²Π½Π΅Π½ΠΈΠ΅ΠΌ 1.2.2 ΠΎΡ‡Π΅Π²ΠΈΠ΄Π΅Π½ — ΠΌΡ‹ просто Π·Π°ΠΌΠ΅Π½ΠΈΠ»ΠΈ силу ΠΈ ΠΏΠΎΠ»Π΅ энСргиСй ΠΈ ΠΏΠΎΡ‚Π΅Π½Ρ†ΠΈΠ°Π»ΠΎΠΌ.

ΠŸΡ€Π΅Π΄ΡƒΠΏΡ€Π΅ΠΆΠ΄Π΅Π½ΠΈΠ΅

ΠœΡ‹ часто Π±ΡƒΠ΄Π΅ΠΌ ΠΈΡΠΏΠΎΠ»ΡŒΠ·ΠΎΠ²Π°Ρ‚ΡŒ Ρ‚Π°ΠΊΠΎΠΉ язык, ΠΊΠ°ΠΊ Β«ΠΏΠΎΡ‚Π΅Π½Ρ†ΠΈΠ°Π»ΡŒΠ½Π°Ρ энСргия Ρ‚ΠΎΡ‡Π΅Ρ‡Π½ΠΎΠ³ΠΎ заряда», Π½ΠΎ, ΠΊΠ°ΠΊ ΠΈ Π² случаС со всСй ΠΏΠΎΡ‚Π΅Π½Ρ†ΠΈΠ°Π»ΡŒΠ½ΠΎΠΉ энСргиСй, ΠΌΡ‹ Π½Π° самом Π΄Π΅Π»Π΅ ΠΈΠΌΠ΅Π΅ΠΌ Π² Π²ΠΈΠ΄Ρƒ Β«ΠΏΠΎΡ‚Π΅Π½Ρ†ΠΈΠ°Π»ΡŒΠ½ΡƒΡŽ ΡΠ½Π΅Ρ€Π³ΠΈΡŽ, Π΄ΠΎΠ±Π°Π²Π»Π΅Π½Π½ΡƒΡŽ ΠΊ систСмС благодаря Π½Π°Π»ΠΈΡ‡ΠΈΡŽ Ρ‚ΠΎΡ‡Π΅Ρ‡Π½ΠΎΠ³ΠΎ заряда». Π§Ρ‚ΠΎΠ±Ρ‹ ввСсти ΠΏΠΎΡ‚Π΅Π½Ρ†ΠΈΠ°Π»ΡŒΠ½ΡƒΡŽ ΡΠ½Π΅Ρ€Π³ΠΈΡŽ, трСбуСтся взаимодСйствиС Ρ‡Π΅Ρ€Π΅Π· ΠΊΠΎΠ½ΡΠ΅Ρ€Π²Π°Ρ‚ΠΈΠ²Π½ΡƒΡŽ силу, Π° для взаимодСйствия Ρ‚Ρ€Π΅Π±ΡƒΡŽΡ‚ΡΡ Π΄Π²Π΅ сущности. НапримСр, ΠΎΠ±ΡŠΠ΅ΠΊΡ‚ Π½Π΅ ΠΌΠΎΠΆΠ΅Ρ‚ ΠΈΠΌΠ΅Ρ‚ΡŒ свою ΡΠΎΠ±ΡΡ‚Π²Π΅Π½Π½ΡƒΡŽ Π³Ρ€Π°Π²ΠΈΡ‚Π°Ρ†ΠΈΠΎΠ½Π½ΡƒΡŽ ΠΏΠΎΡ‚Π΅Π½Ρ†ΠΈΠ°Π»ΡŒΠ½ΡƒΡŽ ΡΠ½Π΅Ρ€Π³ΠΈΡŽ (хотя ΠΌΡ‹ часто относимся ΠΊ Π½Π΅ΠΌΡƒ ΠΈΠΌΠ΅Π½Π½ΠΎ Ρ‚Π°ΠΊ) — ΠΎΠ½ Π΄ΠΎΠ»ΠΆΠ΅Π½ Π²Π·Π°ΠΈΠΌΠΎΠ΄Π΅ΠΉΡΡ‚Π²ΠΎΠ²Π°Ρ‚ΡŒ с Π—Π΅ΠΌΠ»Π΅ΠΉ. {- 1} \).]

ΠžΠ±Ρ€Π°Ρ‚ΠΈΡ‚Π΅ Π²Π½ΠΈΠΌΠ°Π½ΠΈΠ΅, Ρ‡Ρ‚ΠΎ Π·Π½Π°ΠΊΠΈ ΠΏΠ΅Ρ€Π΅Π²Π΅Ρ€Π½ΡƒΡ‚Ρ‹ с ΠΎΠ±Π΅ΠΈΡ… сторон уравнСния. Π’Π΅Π»ΠΈΡ‡ΠΈΠ½Π° слСва ΠΎΠ±Ρ‹Ρ‡Π½ΠΎ обозначаСтся ΠΊΠ°ΠΊ ΠΏΠ°Π΄Π΅Π½ΠΈΠ΅ ΠΏΠΎΡ‚Π΅Π½Ρ†ΠΈΠ°Π»Π° ΠΎΡ‚ A Π΄ΠΎ B . ΠšΠΎΠ½Π΅Ρ‡Π½ΠΎ, ΠΏΠΎΡ‚Π΅Π½Ρ†ΠΈΠ°Π» Π½Π΅ ΠΈΠΌΠ΅Π΅Ρ‚ , поэтому, Π²ΠΎΠ·ΠΌΠΎΠΆΠ½ΠΎ, ΠΏΠΎΡ‚Π΅Π½Ρ†ΠΈΠ°Π»ΡŒΠ½ΠΎΠ΅ ΠΈΠ·ΠΌΠ΅Π½Π΅Π½ΠΈΠ΅ — Π»ΡƒΡ‡ΡˆΠΈΠΉ язык. ΠŸΡ€ΠΈΡ‡ΠΈΠ½Π° этой Ρ„ΠΎΡ€ΠΌΡƒΠ»ΠΈΡ€ΠΎΠ²ΠΊΠΈ, вСроятно, ΡƒΡ…ΠΎΠ΄ΠΈΡ‚ корнями Π² ΠΊΠΎΠ½ΠΊΡ€Π΅Ρ‚Π½Ρ‹ΠΉ случай выполнСния ΠΈΠ½Ρ‚Π΅Π³Ρ€Π°Π»Π° ΠΏΠΎ ΠΏΡƒΡ‚ΠΈ, ΠΊΠΎΡ‚ΠΎΡ€Ρ‹ΠΉ слСдуСт Π½Π°ΠΏΡ€Π°Π²Π»Π΅Π½ΠΈΡŽ элСктричСского поля. ΠžΠ±Ρ€Π°Ρ‚ΠΈΡ‚Π΅ Π²Π½ΠΈΠΌΠ°Π½ΠΈΠ΅, Ρ‡Ρ‚ΠΎ Π² этом случаС \ (\ overrightarrow E \) всСгда Π² Ρ‚ΠΎΠΌ ΠΆΠ΅ Π½Π°ΠΏΡ€Π°Π²Π»Π΅Π½ΠΈΠΈ, Ρ‡Ρ‚ΠΎ ΠΈ \ (\ overrightarrow {dl} \), Ρ‡Ρ‚ΠΎ Π΄Π°Π΅Ρ‚ ΠΏΠΎΠ»ΠΎΠΆΠΈΡ‚Π΅Π»ΡŒΠ½Ρ‹ΠΉ Π»ΠΈΠ½Π΅ΠΉΠ½Ρ‹ΠΉ ΠΈΠ½Ρ‚Π΅Π³Ρ€Π°Π».Если Π»ΠΈΠ½Π΅ΠΉΠ½Ρ‹ΠΉ ΠΈΠ½Ρ‚Π΅Π³Ρ€Π°Π» ΠΏΠΎΠ»ΠΎΠΆΠΈΡ‚Π΅Π»ΡŒΠ½Ρ‹ΠΉ, Ρ‚ΠΎ \ (U_A> U_B \), Ρ‡Ρ‚ΠΎ ΠΎΠ·Π½Π°Ρ‡Π°Π΅Ρ‚, Ρ‡Ρ‚ΠΎ ΠΏΠΎΡ‚Π΅Π½Ρ†ΠΈΠ°Π» ΠΏΠ°Π΄Π°Π΅Ρ‚ с \ (A \) Π΄ΠΎ \ (B \). Π­Ρ‚ΠΎ Π΄Π°Π΅Ρ‚ Π½Π°ΠΌ ΠΏΠΎΠ»Π΅Π·Π½ΠΎΠ΅ практичСскоС ΠΏΡ€Π°Π²ΠΈΠ»ΠΎ:

ЭлСктричСскиС поля ΡƒΠΊΠ°Π·Ρ‹Π²Π°ΡŽΡ‚ Π² Π½Π°ΠΏΡ€Π°Π²Π»Π΅Π½ΠΈΠΈ ΡƒΠΌΠ΅Π½ΡŒΡˆΠ΅Π½ΠΈΡ элСктричСского ΠΏΠΎΡ‚Π΅Π½Ρ†ΠΈΠ°Π»Π°.

Всякий Ρ€Π°Π·, ΠΊΠΎΠ³Π΄Π° Ρƒ нас Π΅ΡΡ‚ΡŒ Ρ‚Π°ΠΊΠΎΠ΅ ΠΈΠ½Ρ‚Π΅Π³Ρ€Π°Π»ΡŒΠ½ΠΎΠ΅ ΠΎΡ‚Π½ΠΎΡˆΠ΅Π½ΠΈΠ΅, Ρ‚ΠΎΠ³Π΄Π°, ΠΊΠ°ΠΊ ΠΌΡ‹ Π²ΠΈΠ΄Π΅Π»ΠΈ для Π·Π°ΠΊΠΎΠ½Π° Гаусса, Ρ‚Π°ΠΊΠΆΠ΅ доступно Π΄ΠΈΡ„Ρ„Π΅Ρ€Π΅Π½Ρ†ΠΈΠ°Π»ΡŒΠ½ΠΎΠ΅ (локальноС) ΠΎΡ‚Π½ΠΎΡˆΠ΅Π½ΠΈΠ΅. ΠœΡ‹ Π΄Π΅ΠΉΡΡ‚Π²ΠΈΡ‚Π΅Π»ΡŒΠ½ΠΎ Π²ΠΈΠ΄Π΅Π»ΠΈ это Π΅Ρ‰Π΅ Π² нашСм исслСдовании ΠΌΠ΅Ρ…Π°Π½ΠΈΠΊΠΈ, ΠΈ это Ρ‚Π°ΠΊΠΆΠ΅ проявляСтся здСсь:

\ [\ overrightarrow F = — \ overrightarrow \ nabla U \; \; \; \ΠŸΡ€Π°Π²Π°Ρ стрСлка \;\;\; \ overrightarrow E = — \ overrightarrow \ nabla V \]

Π₯отя это интСрСсно, читатСля ΠΌΠΎΠΆΠ½ΠΎ ΠΏΡ€ΠΎΡΡ‚ΠΈΡ‚ΡŒ, Ссли ΠΎΠ½ спросит, для Ρ‡Π΅Π³ΠΎ это Π½ΡƒΠΆΠ½ΠΎ.ПослСднСС ΡΠΎΠΎΡ‚Π½ΠΎΡˆΠ΅Π½ΠΈΠ΅ особСнно дСйствСнно ΠΏΠΎ ΡΠ»Π΅Π΄ΡƒΡŽΡ‰Π΅ΠΉ ΠΏΡ€ΠΈΡ‡ΠΈΠ½Π΅. ΠŸΡ€Π΅Π΄ΠΏΠΎΠ»ΠΎΠΆΠΈΠΌ, ΠΌΡ‹ Ρ…ΠΎΡ‚ΠΈΠΌ Π²Ρ‹Ρ‡ΠΈΡΠ»ΠΈΡ‚ΡŒ элСктричСскоС ΠΏΠΎΠ»Π΅ распрСдСлСния заряда. ΠŸΡ€Π΅Π΄ΠΏΠΎΠ»Π°Π³Π°Ρ, Ρ‡Ρ‚ΠΎ Ρƒ нас Π½Π΅Ρ‚ ΡƒΠΌΠ½ΠΎΠ³ΠΎ способа ΠΈΡΠΏΠΎΠ»ΡŒΠ·ΠΎΠ²Π°Ρ‚ΡŒ для этого Π·Π°ΠΊΠΎΠ½ Гаусса, ΠΌΡ‹ Π΄ΠΎΠ»ΠΆΠ½Ρ‹ Π²Ρ‹ΠΏΠΎΠ»Π½ΠΈΡ‚ΡŒ вычислСниС, ΠΊΠ°ΠΊ ΠΌΡ‹ Π΄Π΅Π»Π°Π»ΠΈ Π² Ρ€Π°Π·Π΄Π΅Π»Π΅ 1.3. Π§Π°ΡΡ‚ΡŒ Ρ‚ΠΎΠ³ΠΎ, Ρ‡Ρ‚ΠΎ Π΄Π΅Π»Π°Π΅Ρ‚ эти вычислСния слоТными, — это отслСТиваниС Ρ‚Ρ€Π΅Ρ… Ρ€Π°Π·Π»ΠΈΡ‡Π½Ρ‹Ρ… ΠΊΠΎΠΌΠΏΠΎΠ½Π΅Π½Ρ‚ΠΎΠ² Π²Π΅ΠΊΡ‚ΠΎΡ€Π° элСктричСского поля (Ρ‚ΠΎ Π΅ΡΡ‚ΡŒ Ρ‚Ρ€Π΅Ρ… ΠΎΡ‚Π΄Π΅Π»ΡŒΠ½Ρ‹Ρ… ΠΈΠ½Ρ‚Π΅Π³Ρ€Π°Π»ΠΎΠ²). Если вмСсто этого Π²Ρ‹Ρ‡ΠΈΡΠ»ΠΈΡ‚ΡŒ ΠΏΠΎΡ‚Π΅Π½Ρ†ΠΈΠ°Π»ΡŒΠ½ΠΎΠ΅ ΠΏΠΎΠ»Π΅ (ΠΎΠ΄ΠΈΠ½ ΠΈΠ½Ρ‚Π΅Π³Ρ€Π°Π» Π±Π΅Π· Π²Π΅ΠΊΡ‚ΠΎΡ€ΠΎΠ²), ΠΌΡ‹ ΠΌΠΎΠΆΠ΅ΠΌ Π²Π·ΡΡ‚ΡŒ ΠΏΡ€ΠΎΠΈΠ·Π²ΠΎΠ΄Π½Ρ‹Ρ… (Π³Ρ€Π°Π΄ΠΈΠ΅Π½Ρ‚), Ρ‡Ρ‚ΠΎΠ±Ρ‹ ΠΏΠΎΠ»ΡƒΡ‡ΠΈΡ‚ΡŒ элСктричСскоС ΠΏΠΎΠ»Π΅.Π’ ΡΠ»Π΅Π΄ΡƒΡŽΡ‰Π΅ΠΌ Ρ€Π°Π·Π΄Π΅Π»Π΅ ΠΌΡ‹ ΡƒΠ²ΠΈΠ΄ΠΈΠΌ, ΠΊΠ°ΠΊ Π²Ρ‹Ρ‡ΠΈΡΠ»ΠΈΡ‚ΡŒ ΠΏΠΎΡ‚Π΅Π½Ρ†ΠΈΠ°Π»ΡŒΠ½ΠΎΠ΅ ΠΏΠΎΠ»Π΅ ΠΏΠΎ Ρ€Π°ΡΠΏΡ€Π΅Π΄Π΅Π»Π΅Π½ΠΈΡŽ заряда.

ΠŸΡ€Π΅Π΄ΡƒΠΏΡ€Π΅ΠΆΠ΄Π΅Π½ΠΈΠ΅

Бвязь ΠΌΠ΅ΠΆΠ΄Ρƒ ΠΏΠΎΠ»Π΅ΠΌ ΠΈ ΠΏΠΎΡ‚Π΅Π½Ρ†ΠΈΠ°Π»ΠΎΠΌ часто понимаСтся Π½Π΅ΠΏΡ€Π°Π²ΠΈΠ»ΡŒΠ½ΠΎ, Π² Π΅Ρ‰Π΅ ΠΎΠ΄Π½ΠΎΠΌ Π²ΠΎΠΏΠ»ΠΎΡ‰Π΅Π½ΠΈΠΈ ΠΏΡƒΡ‚Π°Π½ΠΈΡ†Ρ‹ Π²Π΅Π»ΠΈΡ‡ΠΈΠ½Ρ‹ с ΠΈΠ·ΠΌΠ΅Π½Π΅Π½ΠΈΠ΅ΠΌ этой Π²Π΅Π»ΠΈΡ‡ΠΈΠ½Ρ‹ (Π½Π°ΠΏΡ€ΠΈΠΌΠ΅Ρ€, ΠΎΡˆΠΈΠ±ΠΎΡ‡Π½ΠΎΠ΅ принятиС ускорСния со ΡΠΊΠΎΡ€ΠΎΡΡ‚ΡŒΡŽ. Π’Π°ΠΊ ΠΆΠ΅, ΠΊΠ°ΠΊ нулСвая мгновСнная ΡΠΊΠΎΡ€ΠΎΡΡ‚ΡŒ Π½Π΅ ΠΎΠ·Π½Π°Ρ‡Π°Π΅Ρ‚, Ρ‡Ρ‚ΠΎ ускорСниС Ρ€Π°Π²Π½ΠΎ Π½ΡƒΠ»ΡŽ, Π½ΡƒΠ»Π΅Π²ΠΎΠΉ ΠΏΠΎΡ‚Π΅Π½Ρ†ΠΈΠ°Π» ΠΏΡ€ΠΈ Ρ‚ΠΎΡ‡ΠΊΠ° Π² пространствС Π½Π΅ ΠΎΠ·Π½Π°Ρ‡Π°Π΅Ρ‚, Ρ‡Ρ‚ΠΎ ΠΏΠΎΠ»Π΅ Ρ‚Π°ΠΌ Ρ€Π°Π²Π½ΠΎ Π½ΡƒΠ»ΡŽ. Π’ самом Π΄Π΅Π»Π΅, ΠΌΡ‹ ΠΌΠΎΠΆΠ΅ΠΌ ΠΎΠΏΡ€Π΅Π΄Π΅Π»ΠΈΡ‚ΡŒ ΠΏΠΎΡ‚Π΅Π½Ρ†ΠΈΠ°Π» Ρ€Π°Π²Π½Ρ‹ΠΌ Π½ΡƒΠ»ΡŽ Π³Π΄Π΅ ΡƒΠ³ΠΎΠ΄Π½ΠΎ, нСзависимо ΠΎΡ‚ Ρ‚ΠΎΠ³ΠΎ, Ρ‡Ρ‚ΠΎ это Π·Π° ΠΏΠΎΠ»Π΅! Π­Ρ‚ΠΎ ΡΠΊΠΎΡ€ΠΎΡΡ‚ΡŒ измСнСния ΠΏΠΎΡ‚Π΅Π½Ρ†ΠΈΠ°Π»Π°, которая опрСдСляСт ΠΏΠΎΠ»Π΅, Π° Π½Π΅ Π·Π½Π°Ρ‡Π΅Π½ΠΈΠ΅ ΠΏΠΎΡ‚Π΅Π½Ρ†ΠΈΠ°Π»Π°.

Π€ΠΎΡ€ΠΌΡƒΠ»Ρ‹ Π³Ρ€Π°Π΄ΠΈΠ΅Π½Ρ‚Π°

Π•Ρ‰Π΅ Π² Ρ€Π°Π·Π΄Π΅Π»Π΅ 1.6 ΠΌΡ‹ Π²ΠΏΠ΅Ρ€Π²Ρ‹Π΅ ΡΡ‚ΠΎΠ»ΠΊΠ½ΡƒΠ»ΠΈΡΡŒ с использованиСм Π²Π΅ΠΊΡ‚ΠΎΡ€Π½ΠΎΠ³ΠΎ исчислСния, ΠΊΠΎΠ³Π΄Π° ΡƒΠ·Π½Π°Π»ΠΈ, Ρ‡Ρ‚ΠΎ Π½Π°ΠΌ придСтся ΡƒΡ‡ΠΈΡ‚Ρ‹Π²Π°Ρ‚ΡŒ расходимости элСктричСских ΠΏΠΎΠ»Π΅ΠΉ, Ρ‡Ρ‚ΠΎΠ±Ρ‹ ΠΏΡ€ΠΈΠΌΠ΅Π½ΠΈΡ‚ΡŒ Π·Π°ΠΊΠΎΠ½ Гаусса Π² ΠΎΠΏΡ€Π΅Π΄Π΅Π»Π΅Π½Π½Ρ‹Ρ… прилоТСниях. Π’Π΅ΠΏΠ΅Ρ€ΡŒ ΠΌΡ‹ сталкиваСмся с ΠΎΠ΄Π½ΠΈΠΌ ΠΈΠ· родствСнников ΠΎΠΏΠ΅Ρ€Π°Ρ†ΠΈΠΈ Π΄ΠΈΠ²Π΅Ρ€Π³Π΅Π½Ρ†ΠΈΠΈ — Π³Ρ€Π°Π΄ΠΈΠ΅Π½Ρ‚ΠΎΠΌ. Как ΠΈ Π² случаС с расхоТдСниСм, ΠΏΠΎΠ»Π΅Π·Π½ΠΎ ΠΏΡ€ΠΎΡΠΌΠΎΡ‚Ρ€Π΅Ρ‚ΡŒ Π½Π΅ΠΊΠΎΡ‚ΠΎΡ€Ρ‹Π΅ Ρ„ΠΎΡ€ΠΌΡƒΠ»Ρ‹ для Π³Ρ€Π°Π΄ΠΈΠ΅Π½Ρ‚ΠΎΠ² Π² ΠΎΠΏΡ€Π΅Π΄Π΅Π»Π΅Π½Π½Ρ‹Ρ… особых ΠΎΠ±ΡΡ‚ΠΎΡΡ‚Π΅Π»ΡŒΡΡ‚Π²Π°Ρ…. Как ΠΈ Π² случаС с Π΄ΠΈΠ²Π΅Ρ€Π³Π΅Π½Ρ†ΠΈΠ΅ΠΉ, Ρ„ΠΎΡ€ΠΌΡƒΠ»Π° Π³Ρ€Π°Π΄ΠΈΠ΅Π½Ρ‚Π° Π² Π΄Π΅ΠΊΠ°Ρ€Ρ‚ΠΎΠ²Ρ‹Ρ… ΠΊΠΎΠΎΡ€Π΄ΠΈΠ½Π°Ρ‚Π°Ρ… Ρ€Π°Π±ΠΎΡ‚Π°Π΅Ρ‚ Π²ΠΎ всСх случаях, Ρ‚ΠΎΠ³Π΄Π° ΠΊΠ°ΠΊ Π³Ρ€Π°Π΄ΠΈΠ΅Π½Ρ‚ Π² цилиндричСских ΠΈ сфСричСских ΠΊΠΎΠΎΡ€Π΄ΠΈΠ½Π°Ρ‚Π°Ρ… упрощаСтся Ρ‚ΠΎΠ»ΡŒΠΊΠΎ Ρ‚ΠΎΠ³Π΄Π°, ΠΊΠΎΠ³Π΄Π° скалярная функция зависит Ρ‚ΠΎΠ»ΡŒΠΊΠΎ ΠΎΡ‚ \ (r \) (ΠΊΠ°ΠΊ ΠΈ Ρ€Π°Π½ΡŒΡˆΠ΅, Π² цилиндричСских ΠΊΠΎΠΎΡ€Π΄ΠΈΠ½Π°Ρ‚Π°Ρ… это — расстояниС Π΄ΠΎ оси, Π° Π² сфСричСских ΠΊΠΎΠΎΡ€Π΄ΠΈΠ½Π°Ρ‚Π°Ρ… — расстояниС Π΄ΠΎ Ρ‚ΠΎΡ‡ΠΊΠΈ):

Π”Π΅ΠΊΠ°Ρ€Ρ‚ΠΎΠ²Ρ‹ ΠΊΠΎΠΎΡ€Π΄ΠΈΠ½Π°Ρ‚Ρ‹

\ [\ overrightarrow \ nabla V \ left (x, \; y, \; z \ right) = \ dfrac {\ partial V} {\ partial x} \ widehat i + \ dfrac {\ partial V} {\ partial y } \ widehat j + \ dfrac {\ partial V} {\ partial z} \ widehat k \]

ЦилиндричСскиС ΠΊΠΎΠΎΡ€Π΄ΠΈΠ½Π°Ρ‚Ρ‹

\ [\ overrightarrow \ nabla V \ left (r, \ cancel {\ phi}, \ cancel {z} \ right) = \ dfrac {\ partial V} {\ partial r} \ widehat r \]

БфСричСскиС ΠΊΠΎΠΎΡ€Π΄ΠΈΠ½Π°Ρ‚Ρ‹

\ [\ overrightarrow \ nabla V \ left (r, \ cancel {\ theta}, \ cancel {\ phi} \ right) = \ dfrac {\ partial V} {\ partial r} \ widehat r \]

ΠŸΡ€ΠΈΠΌΠ΅Ρ€ \ (\ PageIndex {1} \)

Π’ ΠΎΠΏΡ€Π΅Π΄Π΅Π»Π΅Π½Π½ΠΎΠΉ области пространства Π²ΠΎΠΊΡ€ΡƒΠ³ Π½Π°Ρ‡Π°Π»Π° ΠΊΠΎΠΎΡ€Π΄ΠΈΠ½Π°Ρ‚ элСктростатичСскоС ΠΏΠΎΡ‚Π΅Π½Ρ†ΠΈΠ°Π»ΡŒΠ½ΠΎΠ΅ ΠΏΠΎΠ»Π΅ удовлСтворяСт:

\ [V \ left (x, y, z \ right) = \ alpha \; Ρ… + \ Π±Π΅Ρ‚Π° \; Ρƒ ^ 2 + \ Π³Π°ΠΌΠΌΠ° \; z ^ 3 \ nonumber \]

НайдитС ΠΏΠ»ΠΎΡ‚Π½ΠΎΡΡ‚ΡŒ заряда Π² Π½Π°Ρ‡Π°Π»Π΅ ΠΊΠΎΠΎΡ€Π΄ΠΈΠ½Π°Ρ‚ Π² Ρ‚Π΅Ρ€ΠΌΠΈΠ½Π°Ρ… \ (\ alpha \), \ (\ beta \) ΠΈ \ (\ gamma \).2 \; \ widehat k \ nonumber \]

Π’Π΅ΠΏΠ΅Ρ€ΡŒ дивСргСнция поля Π΄Π°Π΅Ρ‚ Π½Π°ΠΌ ΠΏΠ»ΠΎΡ‚Π½ΠΎΡΡ‚ΡŒ заряда (Π·Π°ΠΊΠΎΠ½ Гаусса Π² локальной Ρ„ΠΎΡ€ΠΌΠ΅):

\ [\ dfrac {\ rho \ left (x, y, z \ right)} {\ epsilon_o} = \ nabla \ cdot \ overrightarrow E = \ dfrac {\ partial E_x} {\ partial x} + \ dfrac { \ partial E_y} {\ partial y} + \ dfrac {\ partial E_z} {\ partial z} = 0-2 \ beta-6 \ gamma \; z \; \; \; \ΠŸΡ€Π°Π²Π°Ρ стрСлка \;\;\; \ Π² ΠΊΠΎΡ€ΠΎΠ±ΠΊΠ΅ {\ rho \ left (0,0,0 \ right) = — 2 \ beta \ epsilon_o} \ nonumber \]

ΠžΠ±Ρ€Π°Ρ‚ΠΈΡ‚Π΅ Π²Π½ΠΈΠΌΠ°Π½ΠΈΠ΅, Ρ‡Ρ‚ΠΎ Π² Π½Π°Ρ‡Π°Π»Π΅ ΠΊΠΎΠΎΡ€Π΄ΠΈΠ½Π°Ρ‚ ΠΏΠΎΡ‚Π΅Π½Ρ†ΠΈΠ°Π» Ρ€Π°Π²Π΅Π½ Π½ΡƒΠ»ΡŽ, Π½ΠΎ элСктричСскоС ΠΏΠΎΠ»Π΅ Π½Π΅Ρ‚, Ρ€Π°Π²Π½ΠΎ ΠΊΠ°ΠΊ ΠΈ ΠΏΠ»ΠΎΡ‚Π½ΠΎΡΡ‚ΡŒ заряда.

Π˜Π΄Π΅Π½Ρ‚ΠΈΡ‡Π½ΠΎΡΡ‚ΡŒ ΠΈΠ· Π²Π΅ΠΊΡ‚ΠΎΡ€Π½ΠΎΠ³ΠΎ исчислСния

ΠœΡ‹ Π²Ρ‹Π²Π΅Π»ΠΈ Π·Π°Π²ΠΈΡΠΈΠΌΠΎΡΡ‚ΡŒ ΠΎΡ‚Ρ€ΠΈΡ†Π°Ρ‚Π΅Π»ΡŒΠ½ΠΎΠ³ΠΎ Π³Ρ€Π°Π΄ΠΈΠ΅Π½Ρ‚Π° ΠΌΠ΅ΠΆΠ΄Ρƒ ΠΏΠΎΡ‚Π΅Π½Ρ†ΠΈΠ°Π»ΠΎΠΌ ΠΈ элСктричСским ΠΏΠΎΠ»Π΅ΠΌ ΠΈΠ· Ρ‚ΠΎΠΉ ΠΆΠ΅ связи ΠΌΠ΅ΠΆΠ΄Ρƒ консСрвативной силой ΠΈ связанной с Π½Π΅ΠΉ ΠΏΠΎΡ‚Π΅Π½Ρ†ΠΈΠ°Π»ΡŒΠ½ΠΎΠΉ энСргиСй (ΡƒΡ€Π°Π²Π½Π΅Π½ΠΈΠ΅ 2.2.7). Π§Ρ‚ΠΎΠ±Ρ‹ сила ΠΈΠΌΠ΅Π»Π° ΡΠ²ΡΠ·Π°Π½Π½ΡƒΡŽ ΠΏΠΎΡ‚Π΅Π½Ρ†ΠΈΠ°Π»ΡŒΠ½ΡƒΡŽ ΡΠ½Π΅Ρ€Π³ΠΈΡŽ, Π½Π΅ΠΎΠ±Ρ…ΠΎΠ΄ΠΈΠΌΠΎ, Ρ‡Ρ‚ΠΎΠ±Ρ‹ ΠΎΠ½Π° Π±Ρ‹Π»Π° консСрвативной. ΠœΡ‹ всС врСмя ΠΏΡ€Π΅Π΄ΠΏΠΎΠ»Π°Π³Π°Π»ΠΈ, Ρ‡Ρ‚ΠΎ элСктричСская сила консСрвативна. Как ΠΌΡ‹ ΡƒΠ²ΠΈΠ΄ΠΈΠΌ ΠΏΠΎΠ·ΠΆΠ΅, Π½Π° самом Π΄Π΅Π»Π΅ это Π½Π΅ всСгда Ρ‚Π°ΠΊ.ΠžΠΊΠ°Π·Ρ‹Π²Π°Π΅Ρ‚ΡΡ, Ρ‡Ρ‚ΠΎ элСктромагнитноС ΠΏΠΎΠ»Π΅ являСтся консСрвативным, Π½ΠΎ ΠΌΠ°Π³Π½ΠΈΡ‚Π½ΠΎΠ΅ ΠΏΠΎΠ»Π΅ ΠΌΠΎΠΆΠ΅Ρ‚ ΠΏΠ΅Ρ€Π΅Π΄Π°Π²Π°Ρ‚ΡŒ ΡΠ½Π΅Ρ€Π³ΠΈΡŽ Π² / ΠΈΠ· элСктричСского поля, Ρ‡Ρ‚ΠΎ само ΠΏΠΎ сСбС Π΄Π΅Π»Π°Π΅Ρ‚ элСктричСскоС ΠΏΠΎΠ»Π΅ нСконсСрвативным. Для этого Π½Π΅ΠΎΠ±Ρ…ΠΎΠ΄ΠΈΠΌΠΎ, Ρ‡Ρ‚ΠΎΠ±Ρ‹ заряды источника двигались, ΠΈ, ΠΏΠΎΡΠΊΠΎΠ»ΡŒΠΊΡƒ ΠΌΡ‹ всС Π΅Ρ‰Π΅ обсуТдаСм Ρ‚ΠΎΠ»ΡŒΠΊΠΎ элСктро статику , ΠΌΡ‹ ΠΌΠΎΠΆΠ΅ΠΌ бСзопасно ΠΏΡ€ΠΎΠ΄ΠΎΠ»ΠΆΠ°Ρ‚ΡŒ ΠΈΡΠΏΠΎΠ»ΡŒΠ·ΠΎΠ²Π°Ρ‚ΡŒ элСктростатичСский ΠΏΠΎΡ‚Π΅Π½Ρ†ΠΈΠ°Π» ΠΈ ΡΠΎΠΎΡ‚Π½ΠΎΡˆΠ΅Π½ΠΈΠ΅ ΠΎΡ‚Ρ€ΠΈΡ†Π°Ρ‚Π΅Π»ΡŒΠ½ΠΎΠ³ΠΎ Π³Ρ€Π°Π΄ΠΈΠ΅Π½Ρ‚Π°.

БущСствованиС Ρ„ΡƒΠ½ΠΊΡ†ΠΈΠΈ ΠΏΠΎΡ‚Π΅Π½Ρ†ΠΈΠ°Π»ΡŒΠ½ΠΎΠΉ энСргии достаточно, Ρ‡Ρ‚ΠΎΠ±Ρ‹ Π΄ΠΎΠΊΠ°Π·Π°Ρ‚ΡŒ, Ρ‡Ρ‚ΠΎ сила консСрвативна, хотя Π΄ΠΎΠΊΠ°Π·Π°Ρ‚ΡŒ это ΠΌΠΎΠΆΠ΅Ρ‚ Π±Ρ‹Ρ‚ΡŒ ΠΏΡ€ΠΎΠ±Π»Π΅ΠΌΠ°Ρ‚ΠΈΡ‡Π½ΠΎ Π±Π΅Π· инструмСнтов, прСдоставляСмых Π²Π΅ΠΊΡ‚ΠΎΡ€Π½Ρ‹ΠΌ исчислСниСм.ΠŸΡ€ΠΎΡΡ‚ΠΎΠΉ тСст Π½Π° ΠΊΠΎΠ½ΡΠ΅Ρ€Π²Π°Ρ‚ΠΈΠ²Π½ΠΎΡΡ‚ΡŒ силы — исчСзновСниС Π΅Π΅ ΠΈΠ·Π³ΠΈΠ±Π°:

\ [\ overrightarrow \ nabla \ times \ overrightarrow F = 0 \; \; \; \ leftrightarrow \; \; \; F \; являСтся \; консСрвативным \; \; \; \ leftrightarrow \; \; \; \ overrightarrow F = — \ overrightarrow \ nabla U \]

ΠŸΡ€ΠΈΡ‡ΠΈΠ½Π°, ΠΏΠΎ ΠΊΠΎΡ‚ΠΎΡ€ΠΎΠΉ это Ρ€Π°Π±ΠΎΡ‚Π°Π΅Ρ‚ ΠΊΠ°ΠΊ тСст, Π·Π°ΠΊΠ»ΡŽΡ‡Π°Π΅Ρ‚ΡΡ Π² Ρ‚ΠΎΠΌ, Ρ‡Ρ‚ΠΎ гСомСтрия Π·Π°Π²ΠΈΡ‚ΠΊΠ° ΠΈ Π³Ρ€Π°Π΄ΠΈΠ΅Π½Ρ‚Π° Ρ‚Π°ΠΊΠΎΠ²Π°, Ρ‡Ρ‚ΠΎ Π·Π°Π²ΠΈΡ‚ΠΎΠΊ Π²Π΅ΠΊΡ‚ΠΎΡ€Π½ΠΎΠ³ΠΎ поля, Π²ΠΎΠ·Π½ΠΈΠΊΠ°ΡŽΡ‰ΠΈΠΉ ΠΈΠ· Π³Ρ€Π°Π΄ΠΈΠ΅Π½Ρ‚Π° скалярного поля, всСгда ΠΎΠ΄ΠΈΠ½Π°ΠΊΠΎΠ²ΠΎ Ρ€Π°Π²Π΅Π½ Π½ΡƒΠ»ΡŽ:

\ [\ overrightarrow \ nabla \ times \ left [\ overrightarrow \ nabla \ left (Ρ‡Ρ‚ΠΎ ΡƒΠ³ΠΎΠ΄Π½ΠΎ \ Π²ΠΏΡ€Π°Π²ΠΎ) \ Π²ΠΏΡ€Π°Π²ΠΎ] \ эквив 0 \]

Π˜Ρ‚Π°ΠΊ, Ссли силу ΠΌΠΎΠΆΠ½ΠΎ Π·Π°ΠΏΠΈΡΠ°Ρ‚ΡŒ ΠΊΠ°ΠΊ ΠΎΡ‚Ρ€ΠΈΡ†Π°Ρ‚Π΅Π»ΡŒΠ½Ρ‹ΠΉ Π³Ρ€Π°Π΄ΠΈΠ΅Π½Ρ‚ Ρ„ΡƒΠ½ΠΊΡ†ΠΈΠΈ ΠΏΠΎΡ‚Π΅Π½Ρ†ΠΈΠ°Π»ΡŒΠ½ΠΎΠΉ энСргии, Ρ‚ΠΎ Π΅Π΅ ΠΈΠ·Π³ΠΈΠ± Π΄ΠΎΠ»ΠΆΠ΅Π½ ΠΈΡΡ‡Π΅Π·Π½ΡƒΡ‚ΡŒ, ΠΈ это соотвСтствуСт консСрвативной силС.Распространяя это Π½Π° элСктростатику, ΠΌΡ‹ Π²ΠΈΠ΄ΠΈΠΌ, Ρ‡Ρ‚ΠΎ Ссли элСктричСскоС ΠΏΠΎΠ»Π΅ ΠΌΠΎΠΆΠ½ΠΎ Π²Ρ‹Ρ€Π°Π·ΠΈΡ‚ΡŒ ΠΊΠ°ΠΊ ΠΎΡ‚Ρ€ΠΈΡ†Π°Ρ‚Π΅Π»ΡŒΠ½Ρ‹ΠΉ Π³Ρ€Π°Π΄ΠΈΠ΅Π½Ρ‚ ΠΏΠΎΡ‚Π΅Π½Ρ†ΠΈΠ°Π»Π°, Ρ‚ΠΎ Π΅Π³ΠΎ Ρ€ΠΎΡ‚ΠΎΡ€ исчСзаСт. Π˜Ρ‚Π°ΠΊ, ΠΈΠΌΠ΅Π΅ΠΌ:

\ [\ overrightarrow \ nabla \ times \ overrightarrow E = 0 \; \; \; \ leftrightarrow \; \; \; элСктростатика \; \; \; \ leftrightarrow \; \; \; \ overrightarrow E = — \ overrightarrow \ nabla V \]

Π­ΠΊΠ²ΠΈΠΏΠΎΡ‚Π΅Π½Ρ†ΠΈΠ°Π»ΡŒΠ½Ρ‹Π΅ повСрхности

БлСдствиСм Π³Ρ€Π°Π΄ΠΈΠ΅Π½Ρ‚Π½ΠΎΠ³ΠΎ ΠΎΡ‚Π½ΠΎΡˆΠ΅Π½ΠΈΡ являСтся Ρ‚ΠΎ, Ρ‡Ρ‚ΠΎ ΠΈΡ… взаимосвязь ΠΈΠΌΠ΅Π΅Ρ‚ Π³Π΅ΠΎΠΌΠ΅Ρ‚Ρ€ΠΈΡ‡Π΅ΡΠΊΡƒΡŽ ΠΏΡ€ΠΈΡ€ΠΎΠ΄Ρƒ. ΠŸΠ΅Ρ€Π²ΠΎΠ΅ проявлСниС этого состоит Π² Ρ‚ΠΎΠΌ, Ρ‡Ρ‚ΠΎ Π³Ρ€Π°Π΄ΠΈΠ΅Π½Ρ‚ скалярного поля ΡƒΠΊΠ°Π·Ρ‹Π²Π°Π΅Ρ‚ Π² Ρ‚ΠΎΠΌ Π½Π°ΠΏΡ€Π°Π²Π»Π΅Π½ΠΈΠΈ, Π³Π΄Π΅ скалярныС значСния растут быстрСС всСго.Π’Π°ΠΊΠΈΠΌ ΠΎΠ±Ρ€Π°Π·ΠΎΠΌ, Π½Π°Π»ΠΈΡ‡ΠΈΠ΅ ΠΎΡ‚Ρ€ΠΈΡ†Π°Ρ‚Π΅Π»ΡŒΠ½ΠΎΠ³ΠΎ Π·Π½Π°ΠΊΠ° ΠΎΠ·Π½Π°Ρ‡Π°Π΅Ρ‚, Ρ‡Ρ‚ΠΎ элСктричСскоС ΠΏΠΎΠ»Π΅ Π½Π°ΠΏΡ€Π°Π²Π»Π΅Π½ΠΎ Π² сторону самого быстрого спуска элСктричСского ΠΏΠΎΡ‚Π΅Π½Ρ†ΠΈΠ°Π»Π°. Π­Ρ‚ΠΎ ΠΏΠΎΠ΄Ρ‚Π²Π΅Ρ€ΠΆΠ΄Π°Π΅Ρ‚ установлСнноС Π²Ρ‹ΡˆΠ΅ эмпиричСскоС ΠΏΡ€Π°Π²ΠΈΠ»ΠΎ.

ΠœΡ‹ ΠΌΠΎΠΆΠ΅ΠΌ ΠΏΡ€ΠΎΠ΄Π΅ΠΌΠΎΠ½ΡΡ‚Ρ€ΠΈΡ€ΠΎΠ²Π°Ρ‚ΡŒ эту Π³Π΅ΠΎΠΌΠ΅Ρ‚Ρ€ΠΈΡ‡Π΅ΡΠΊΡƒΡŽ взаимосвязь с ΠΏΠΎΠΌΠΎΡ‰ΡŒΡŽ Π΄ΠΈΠ°Π³Ρ€Π°ΠΌΠΌΡ‹. Π”Π°Π²Π°ΠΉΡ‚Π΅ прСдставим, Ρ‡Ρ‚ΠΎ ΠΌΡ‹ Π½Π°Ρ‡ΠΈΠ½Π°Π΅ΠΌ с ΠΎΠΏΡ€Π΅Π΄Π΅Π»Π΅Π½Π½ΠΎΠΉ Ρ‚ΠΎΡ‡ΠΊΠΈ Π² пространствС ΠΈ измСряСм Ρ‚Π°ΠΌ ΠΏΠΎΡ‚Π΅Π½Ρ†ΠΈΠ°Π» (послС обозначСния Π½ΡƒΠ»Π΅Π²ΠΎΠΉ Ρ‚ΠΎΡ‡ΠΊΠΈ). Π—Π°Ρ‚Π΅ΠΌ ΠΌΡ‹ Π²Ρ‹Π±ΠΈΡ€Π°Π΅ΠΌ Π±Π»ΠΈΠ·Π»Π΅ΠΆΠ°Ρ‰ΠΈΠ΅ Ρ‚ΠΎΡ‡ΠΊΠΈ ΠΈ Π½Π°Ρ…ΠΎΠ΄ΠΈΠΌ Π½Π°ΠΏΡ€Π°Π²Π»Π΅Π½ΠΈΠ΅, Π² ΠΊΠΎΡ‚ΠΎΡ€ΠΎΠΌ ΠΌΡ‹ ΠΌΠΎΠΆΠ΅ΠΌ ΠΏΠ΅Ρ€Π΅ΠΌΠ΅ΡΡ‚ΠΈΡ‚ΡŒ наш Π΄Π΅Ρ‚Π΅ΠΊΡ‚ΠΎΡ€, Ρ‡Ρ‚ΠΎΠ±Ρ‹ ΠΏΠΎΡ‚Π΅Π½Ρ†ΠΈΠ°Π» Π½Π΅ измСнился.Если ΠΌΡ‹ ΠΏΡ€ΠΎΠ΄ΠΎΠ»ΠΆΠΈΠΌ эту ΠΏΡ€ΠΎΡ†Π΅Π΄ΡƒΡ€Ρƒ ΠΈ нанСсСм Π½Π° ΠΊΠ°Ρ€Ρ‚Ρƒ всС пространство, Π³Π΄Π΅ ΠΏΠΎΡ‚Π΅Π½Ρ†ΠΈΠ°Π» Π½Π΅ мСняСтся, ΠΌΡ‹ ΠΎΠ±Π½Π°Ρ€ΡƒΠΆΠΈΠΌ, Ρ‡Ρ‚ΠΎ это ΠΏΠΎΠ²Π΅Ρ€Ρ…Π½ΠΎΡΡ‚ΡŒ. ΠŸΠΎΡΠΊΠΎΠ»ΡŒΠΊΡƒ эта вообраТаСмая ΠΏΠΎΠ²Π΅Ρ€Ρ…Π½ΠΎΡΡ‚ΡŒ сущСствуСт с ΠΎΠ΄Π½ΠΈΠΌ Ρ€Π°Π²Π½Ρ‹ΠΌ ΠΏΠΎΡ‚Π΅Π½Ρ†ΠΈΠ°Π»ΠΎΠΌ, ΠΎΠ½Π° называСтся ΡΠΊΠ²ΠΈΠΏΠΎΡ‚Π΅Π½Ρ†ΠΈΠ°Π»ΡŒΠ½ΠΎΠΉ ΠΏΠΎΠ²Π΅Ρ€Ρ…Π½ΠΎΡΡ‚ΡŒΡŽ . Π’ΠΎΡ‚ Π΄Π²ΡƒΡ…ΠΌΠ΅Ρ€Π½ΠΎΠ΅ ΠΈΠ·ΠΎΠ±Ρ€Π°ΠΆΠ΅Π½ΠΈΠ΅ Π½Π°Π±ΠΎΡ€Π° Ρ‚Π°ΠΊΠΈΡ… повСрхностСй:

Рисунок 2.2.1 — Π­ΠΊΠ²ΠΈΠΏΠΎΡ‚Π΅Π½Ρ†ΠΈΠ°Π»ΡŒΠ½Ρ‹Π΅ повСрхности

ΠŸΡ€ΠΈ ΠΏΠΎΠ»ΠΎΠΆΠΈΡ‚Π΅Π»ΡŒΠ½ΠΎΠΌ зарядС источника силовыС Π»ΠΈΠ½ΠΈΠΈ Π½Π°ΠΏΡ€Π°Π²Π»Π΅Π½Ρ‹ Π½Π°Ρ€ΡƒΠΆΡƒ, Ρ‡Ρ‚ΠΎ Π΄Π΅ΠΉΡΡ‚Π²ΠΈΡ‚Π΅Π»ΡŒΠ½ΠΎ ΡƒΠΊΠ°Π·Ρ‹Π²Π°Π΅Ρ‚ ΠΎΡ‚ Π±ΠΎΠ»Π΅Π΅ высокого ΠΏΠΎΡ‚Π΅Π½Ρ†ΠΈΠ°Π»Π° ΠΊ Π±ΠΎΠ»Π΅Π΅ Π½ΠΈΠ·ΠΊΠΎΠΌΡƒ, Π½ΠΎ Π΅ΡΡ‚ΡŒ ΠΊΠΎΠ΅-Ρ‡Ρ‚ΠΎ Π±ΠΎΠ»Π΅Π΅ ΠΊΠΎΠ½ΠΊΡ€Π΅Ρ‚Π½ΠΎΠ΅, ΠΈΠ· Ρ‡Π΅Π³ΠΎ ΠΌΡ‹ ΠΌΠΎΠΆΠ΅ΠΌ ΡΠ΄Π΅Π»Π°Ρ‚ΡŒ Π²Ρ‹Π²ΠΎΠ΄ ΠΎ гСомСтричСской взаимосвязи поля ΠΈ ΠΏΠΎΡ‚Π΅Π½Ρ†ΠΈΠ°Π»Π°.Π‘ \ overrightarrow E \ cdot \ overrightarrow {dl} \]

ΠšΠΎΠ½Π΅Ρ‡Π½ΠΎ, элСктричСскоС ΠΏΠΎΠ»Π΅ Π½Π΅ Ρ€Π°Π²Π½ΠΎ Π½ΡƒΠ»ΡŽ Π²Π΅Π·Π΄Π΅, Π³Π΄Π΅ ΠΌΡ‹ ΠΈΠ΄Π΅ΠΌ, ΠΈ расстояниС, ΠΊΠΎΡ‚ΠΎΡ€ΠΎΠ΅ ΠΌΡ‹ ΠΏΡ€ΠΎΡ…ΠΎΠ΄ΠΈΠΌ, Π½Π΅ Ρ€Π°Π²Π½ΠΎ Π½ΡƒΠ»ΡŽ, Ρ‚Π°ΠΊ ΠΊΠ°ΠΊ ΠΆΠ΅ этот ΠΈΠ½Ρ‚Π΅Π³Ρ€Π°Π» ΠΌΠΎΠΆΠ΅Ρ‚ ΠΎΠΊΠ°Π·Π°Ρ‚ΡŒΡΡ Ρ€Π°Π²Π½Ρ‹ΠΌ Π½ΡƒΠ»ΡŽ? ΠœΠΎΠΆΠ΅Ρ‚ Π±Ρ‹Ρ‚ΡŒ, Ρ‡Π°ΡΡ‚ΡŒ этого отмСняСт Π΄Ρ€ΡƒΠ³ΠΈΠ΅ части? НСт, ΠΏΠΎΡ‚ΠΎΠΌΡƒ Ρ‡Ρ‚ΠΎ это происходит Π½Π° ΠΊΠ°ΠΆΠ΄ΠΎΠΌ ΠΏΡƒΡ‚ΠΈ, ΠΊΠΎΡ‚ΠΎΡ€Ρ‹ΠΉ ΠΌΡ‹ Π²Ρ‹Π±ΠΈΡ€Π°Π΅ΠΌ, ΠΌΠ΅ΠΆΠ΄Ρƒ Π»ΡŽΠ±Ρ‹ΠΌΠΈ двумя Ρ‚ΠΎΡ‡ΠΊΠ°ΠΌΠΈ, ΠΏΠΎΠΊΠ° этот ΠΏΡƒΡ‚ΡŒ остаСтся Π½Π° ΡΠΊΠ²ΠΈΠΏΠΎΡ‚Π΅Π½Ρ†ΠΈΠ°Π»ΡŒΠ½ΠΎΠΌ ΡƒΡ€ΠΎΠ²Π½Π΅. ΠžΡ‚Π²Π΅Ρ‚ Π·Π°ΠΊΠ»ΡŽΡ‡Π°Π΅Ρ‚ΡΡ Π² Ρ‚ΠΎΠΌ, Ρ‡Ρ‚ΠΎ этот ΠΈΠ½Ρ‚Π΅Π³Ρ€Π°Π» ΠΌΠΎΠΆΠ΅Ρ‚ Π±Ρ‹Ρ‚ΡŒ Ρ€Π°Π²Π΅Π½ Π½ΡƒΠ»ΡŽ Ρ‚ΠΎΠ»ΡŒΠΊΠΎ Ρ‚ΠΎΠ³Π΄Π°, ΠΊΠΎΠ³Π΄Π° Π² ΠΊΠ°ΠΆΠ΄ΠΎΠΉ Ρ‚ΠΎΡ‡ΠΊΠ΅ эквипотСнциала элСктричСскоС ΠΏΠΎΠ»Π΅ пСрпСндикулярно \ (\ overrightarrow {dl} \).Π”Ρ€ΡƒΠ³ΠΈΠΌΠΈ словами:

ЭлСктричСскоС ΠΏΠΎΠ»Π΅ Π²Π΅Π·Π΄Π΅ пСрпСндикулярно ΡΠΊΠ²ΠΈΠΏΠΎΡ‚Π΅Π½Ρ†ΠΈΠ°Π»ΡŒΠ½Ρ‹ΠΌ повСрхностям.

Π’ΠΎ, Ρ‡Ρ‚ΠΎ элСктричСскиС поля пСрпСндикулярны ΡΠΊΠ²ΠΈΠΏΠΎΡ‚Π΅Π½Ρ†ΠΈΠ°Π»ΡŒΠ½Ρ‹ΠΌ повСрхностям, Π·Π²ΡƒΡ‡ΠΈΡ‚ ΠΎΡ‡Π΅Π½ΡŒ Π·Π½Π°ΠΊΠΎΠΌΠΎ. Π’ΠΎ ΠΆΠ΅ самоС ΠΌΡ‹ сказали ΠΎ проводящих повСрхностях для элСктростатики. Π”Π΅ΠΉΡΡ‚Π²ΠΈΡ‚Π΅Π»ΡŒΠ½ΠΎ, сразу Π΄Π΅Π»Π°Π΅ΠΌ Π²Ρ‹Π²ΠΎΠ΄, Ρ‡Ρ‚ΠΎ для элСктростатики:

ΠŸΡ€ΠΎΠ²ΠΎΠ΄Π½ΠΈΠΊΠΈ ΡΠΊΠ²ΠΈΠΏΠΎΡ‚Π΅Π½Ρ†ΠΈΠ°Π»ΡŒΠ½Ρ‹Π΅.

ΠžΠ±Ρ€Π°Ρ‚ΠΈΡ‚Π΅ Π²Π½ΠΈΠΌΠ°Π½ΠΈΠ΅, Ρ‡Ρ‚ΠΎ это ΡƒΡ‚Π²Π΅Ρ€ΠΆΠ΄Π΅Π½ΠΈΠ΅ Π²Ρ‹Ρ…ΠΎΠ΄ΠΈΡ‚ Π·Π° Ρ€Π°ΠΌΠΊΠΈ Ρ‚ΠΎΠ»ΡŒΠΊΠΎ повСрхности ΠΏΡ€ΠΎΠ²ΠΎΠ΄Π½ΠΈΠΊΠ°. ΠœΡ‹ Π·Π½Π°Π΅ΠΌ, Ρ‡Ρ‚ΠΎ Π²Π½ΡƒΡ‚Ρ€ΠΈ ΠΌΠ΅Ρ‚Π°Π»Π»Π° ΠΏΡ€ΠΎΠ²ΠΎΠ΄Π½ΠΈΠΊΠ° Π½Π΅Ρ‚ элСктричСского поля, поэтому, ΠΊΠΎΠ³Π΄Π° ΠΌΡ‹ ΠΈΠ΄Π΅ΠΌ ΠΎΡ‚ повСрхности ΠΏΡ€ΠΎΠ²ΠΎΠ΄Π½ΠΈΠΊΠ° ΠΊ ΠΌΠ΅Ρ‚Π°Π»Π»Ρƒ, элСктричСский ΠΏΠΎΡ‚Π΅Π½Ρ†ΠΈΠ°Π» Π½Π΅ ΠΌΠΎΠΆΠ΅Ρ‚ ΠΈΠ·ΠΌΠ΅Π½ΡΡ‚ΡŒΡΡ (элСктричСскиС поля Π²ΠΎΠ·Π½ΠΈΠΊΠ°ΡŽΡ‚ ΠΈΠ·-Π·Π° ΠΈΠ·ΠΌΠ΅Π½Π΅Π½ΠΈΠΉ элСктричСского ΠΏΠΎΡ‚Π΅Π½Ρ†ΠΈΠ°Π»Π°), поэтому элСктричСский ΠΏΠΎΡ‚Π΅Π½Ρ†ΠΈΠ°Π» ΠΎΠ΄ΠΈΠ½Π°ΠΊΠΎΠ² Π²ΠΎ всСм ΠΏΡ€ΠΎΠ²ΠΎΠ΄Π½ΠΈΠΊΠ΅.

Когда Π½Π°ΠΌ ΠΏΡ€Π΅Π΄ΠΎΡΡ‚Π°Π²Π»ΡΡŽΡ‚ нСсколько ΡΠΊΠ²ΠΈΠΏΠΎΡ‚Π΅Π½Ρ†ΠΈΠ°Π»ΡŒΠ½Ρ‹Ρ… повСрхностСй, ΠΊΠ°ΠΊ ΠΌΡ‹ здСсь, ΠΌΡ‹ ΠΌΠΎΠΆΠ΅ΠΌ ΡΠ΄Π΅Π»Π°Ρ‚ΡŒ Π²Ρ‹Π²ΠΎΠ΄ ΠΎΠ± элСктричСском ΠΏΠΎΠ»Π΅ большС, Ρ‡Π΅ΠΌ просто ΠΎ Π΅Π³ΠΎ Π½Π°ΠΏΡ€Π°Π²Π»Π΅Π½ΠΈΠΈ. ΠžΠΏΠ΅Ρ€Π°Ρ†ΠΈΡ Π³Ρ€Π°Π΄ΠΈΠ΅Π½Ρ‚Π° измСряСт Π½Π°ΠΏΡ€Π°Π²Π»Π΅Π½Π½ΡƒΡŽ ΡΠΊΠΎΡ€ΠΎΡΡ‚ΡŒ измСнСния . Π­Ρ‚ΠΎ ΠΎΠ·Π½Π°Ρ‡Π°Π΅Ρ‚, Ρ‡Ρ‚ΠΎ Ссли каТдая показанная ΡΠΊΠ²ΠΈΠΏΠΎΡ‚Π΅Π½Ρ†ΠΈΠ°Π»ΡŒΠ½Π°Ρ ΠΏΠΎΠ²Π΅Ρ€Ρ…Π½ΠΎΡΡ‚ΡŒ Ρ€Π°Π·Π΄Π΅Π»Π΅Π½Π° ΠΎΠ΄Π½ΠΈΠΌ ΠΈ Ρ‚Π΅ΠΌ ΠΆΠ΅ числом Π²ΠΎΠ»ΡŒΡ‚ (ΠΊΠ°ΠΊ Π½Π° Π΄ΠΈΠ°Π³Ρ€Π°ΠΌΠΌΠ΅ Π²Ρ‹ΡˆΠ΅), Ρ‚ΠΎ области, Π³Π΄Π΅ эти повСрхности находятся Π±Π»ΠΈΠΆΠ΅ всСго Π΄Ρ€ΡƒΠ³ ΠΊ Π΄Ρ€ΡƒΠ³Ρƒ, ΠΈΠ·ΠΌΠ΅Π½ΡΡŽΡ‚ΡΡ быстрСС всСго, Ρ‡Ρ‚ΠΎ ΠΎΠ·Π½Π°Ρ‡Π°Π΅Ρ‚, Ρ‡Ρ‚ΠΎ Π²Π΅Π»ΠΈΡ‡ΠΈΠ½Π° элСктричСского поля Ρ‚Π°ΠΌ большС всСго.

ΠŸΡ€ΠΈΠΌΠ΅Ρ€ \ (\ PageIndex {2} \)

ЗаряТСнная частица двиТСтся Π² элСктричСском ΠΏΠΎΠ»Π΅, ΡΠΊΠ²ΠΈΠΏΠΎΡ‚Π΅Π½Ρ†ΠΈΠ°Π»ΡŒΠ½Ρ‹Π΅ повСрхности ΠΊΠΎΡ‚ΠΎΡ€ΠΎΠ³ΠΎ ΠΏΠΎΠΊΠ°Π·Π°Π½Ρ‹ Π½Π° схСмС. ЕдинствСнная сила, испытываСмая зарядом, Π²ΠΎΠ·Π½ΠΈΠΊΠ°Π΅Ρ‚ ΠΈΠ·-Π·Π° этого поля. Заряд двиТСтся ΠΌΠ΅Π΄Π»Π΅Π½Π½Π΅Π΅ Π² Ρ‚ΠΎΡ‡ΠΊΠ΅ \ (A \), Ρ‡Π΅ΠΌ Π² Ρ‚ΠΎΡ‡ΠΊΠ΅ \ (B \).

  1. Заряд частицы: ΠΏΠΎΠ»ΠΎΠΆΠΈΡ‚Π΅Π»ΡŒΠ½Ρ‹ΠΉ / ΠΎΡ‚Ρ€ΠΈΡ†Π°Ρ‚Π΅Π»ΡŒΠ½Ρ‹ΠΉ / Π½Π΅ ΠΌΠΎΠ³Ρƒ ΠΎΠΏΡ€Π΅Π΄Π΅Π»ΠΈΡ‚ΡŒ
  2. Π’Π΅Π»ΠΈΡ‡ΠΈΠ½Π° ускорСния заряда большС Π²: Ρ‚ΠΎΡ‡ΠΊΠ° A / Ρ‚ΠΎΡ‡ΠΊΠ° B / Π½Π΅ ΠΌΠΎΠ³Ρƒ ΡΠΊΠ°Π·Π°Ρ‚ΡŒ.
  3. Каково Π½Π°ΠΏΡ€Π°Π²Π»Π΅Π½ΠΈΠ΅ ускорСния заряда Π² ΠΊΠ°ΠΆΠ΄ΠΎΠΉ Ρ‚ΠΎΡ‡ΠΊΠ΅?
РСшСниС

Π°. ΠšΠΈΠ½Π΅Ρ‚ΠΈΡ‡Π΅ΡΠΊΠ°Ρ энСргия частицы ΡƒΠ²Π΅Π»ΠΈΡ‡ΠΈΠ»Π°ΡΡŒ ΠΎΡ‚ Ρ‚ΠΎΡ‡ΠΊΠΈ A ΠΊ Ρ‚ΠΎΡ‡ΠΊΠ΅ B, Ρ‡Ρ‚ΠΎ ΠΎΠ·Π½Π°Ρ‡Π°Π΅Ρ‚, Ρ‡Ρ‚ΠΎ Π΅Π΅ ΠΏΠΎΡ‚Π΅Π½Ρ†ΠΈΠ°Π»ΡŒΠ½Π°Ρ энСргия снизилась. Но Π΅Π³ΠΎ элСктростатичСский ΠΏΠΎΡ‚Π΅Π½Ρ†ΠΈΠ°Π» вырос, поэтому, ΠΏΠΎΡΠΊΠΎΠ»ΡŒΠΊΡƒ \ (\ Delta U = q \ Delta V \), Ρ‚ΠΎ \ (\ Delta U <0 \) ΠΈ \ (\ Delta V> 0 \) ΠΎΠ·Π½Π°Ρ‡Π°Π΅Ρ‚, Ρ‡Ρ‚ΠΎ \ (q <0 \ ).

Π³. ВсС эквипотСнциалы ΠΎΡ‚Π»ΠΈΡ‡Π°ΡŽΡ‚ΡΡ ΠΎΠ΄ΠΈΠ½Π°ΠΊΠΎΠ²Ρ‹ΠΌ напряТСниСм, поэтому Ρ‚Π΅, ΠΊΠΎΡ‚ΠΎΡ€Ρ‹Π΅ располоТСны Π±Π»ΠΈΠΆΠ΅ Π΄Ρ€ΡƒΠ³ ΠΊ Π΄Ρ€ΡƒΠ³Ρƒ, ΡƒΠΊΠ°Π·Ρ‹Π²Π°ΡŽΡ‚ Π½Π° ΠΎΠ±Π»Π°ΡΡ‚ΡŒ, Π³Π΄Π΅ элСктричСскоС ΠΏΠΎΠ»Π΅ сильнСС.Π‘Π»Π΅Π΄ΠΎΠ²Π°Ρ‚Π΅Π»ΡŒΠ½ΠΎ, ΠΏΠΎΠ»Π΅ сильнСС Π² Ρ‚ΠΎΡ‡ΠΊΠ΅ A, Π° это Π·Π½Π°Ρ‡ΠΈΡ‚, Ρ‡Ρ‚ΠΎ Π½Π° Π½Π΅Π³ΠΎ дСйствуСт большая Ρ€Π΅Π·ΡƒΠ»ΡŒΡ‚ΠΈΡ€ΡƒΡŽΡ‰Π°Ρ сила, Ρ‡Π΅ΠΌ Π² Ρ‚ΠΎΡ‡ΠΊΠ΅ B.

Π³. Π‘ΠΈΠ»Π°, создаваСмая элСктричСским ΠΏΠΎΠ»Π΅ΠΌ, Π΄ΠΎΠ»ΠΆΠ½Π° Π±Ρ‹Ρ‚ΡŒ ΠΏΠ°Ρ€Π°Π»Π»Π΅Π»ΡŒΠ½Π° элСктричСскому полю, ΠΊΠΎΡ‚ΠΎΡ€ΠΎΠ΅ Π΄ΠΎΠ»ΠΆΠ½ΠΎ Π±Ρ‹Ρ‚ΡŒ пСрпСндикулярно ΡΠΊΠ²ΠΈΠΏΠΎΡ‚Π΅Π½Ρ†ΠΈΠ°Π»ΡŒΠ½ΠΎΠΉ повСрхности. Π’Π°ΠΊΠΈΠΌ ΠΎΠ±Ρ€Π°Π·ΠΎΠΌ, силы Π² Ρ‚ΠΎΡ‡ΠΊΠ°Ρ… A ΠΈ B Π΄ΠΎΠ»ΠΆΠ½Ρ‹ Π±Ρ‹Ρ‚ΡŒ Π»ΠΈΠ±ΠΎ слСва, Π»ΠΈΠ±ΠΎ справа, Π½ΠΎ ΠΌΠΎΠΆΠ΅ΠΌ Π»ΠΈ ΠΌΡ‹ ΡΠΊΠ°Π·Π°Ρ‚ΡŒ, Π² ΠΊΠ°ΠΊΡƒΡŽ сторону? ПолС ΡƒΠΊΠ°Π·Ρ‹Π²Π°Π΅Ρ‚ ΠΎΡ‚ Π±ΠΎΠ»Π΅Π΅ высокого ΠΏΠΎΡ‚Π΅Π½Ρ†ΠΈΠ°Π»Π° ΠΊ Π±ΠΎΠ»Π΅Π΅ Π½ΠΈΠ·ΠΊΠΎΠΌΡƒ, поэтому Π² Ρ‚ΠΎΡ‡ΠΊΠ΅ A ΠΎΠ½ΠΎ ΡƒΠΊΠ°Π·Ρ‹Π²Π°Π΅Ρ‚ Π²Π»Π΅Π²ΠΎ, Π° Π² Ρ‚ΠΎΡ‡ΠΊΠ΅ B — Π²ΠΏΡ€Π°Π²ΠΎ.Заряд ΠΎΡ‚Ρ€ΠΈΡ†Π°Ρ‚Π΅Π»ΡŒΠ½Ρ‹ΠΉ, поэтому силы ΠΏΡ€ΠΎΡ‚ΠΈΠ²ΠΎΠΏΠΎΠ»ΠΎΠΆΠ½Ρ‹ Π½Π°ΠΏΡ€Π°Π²Π»Π΅Π½ΠΈΡŽ элСктричСского поля. Частица ускоряСтся Π²ΠΏΡ€Π°Π²ΠΎ Π² Ρ‚ΠΎΡ‡ΠΊΠ΅ A ΠΈ Π²Π»Π΅Π²ΠΎ Π² Ρ‚ΠΎΡ‡ΠΊΠ΅ B.

Π”Π°, Π²Ρ‹ ΠΌΠΎΠΆΠ΅Ρ‚Π΅ нанСсти Π½Π° ΠΊΠ°Ρ€Ρ‚Ρƒ элСктричСскоС ΠΏΠΎΠ»Π΅ Π΄ΠΎΠΌΠ°

Но ΠΊΠ°ΠΊ насчСт числСнного значСния элСктричСского поля ΠΌΠ΅ΠΆΠ΄Ρƒ этими проводящими пластинами? Если я просто ΠΏΡ€ΠΎΠΉΠ΄Ρƒ прямо посСрСдинС ΠΎΡ‚ ΠΎΠ΄Π½ΠΎΠΉ пластины ΠΊ Π΄Ρ€ΡƒΠ³ΠΎΠΉ, я ΠΌΠΎΠ³Ρƒ ΠΏΠΎΠ»ΡƒΡ‡ΠΈΡ‚ΡŒ значСния элСктричСского ΠΏΠΎΡ‚Π΅Π½Ρ†ΠΈΠ°Π»Π° для Ρ€Π°Π·Π½Ρ‹Ρ… Π·Π½Π°Ρ‡Π΅Π½ΠΈΠΉ y. Π’ΠΎΡ‚ ΠΊΠ°ΠΊ это выглядит:

ВспомнитС взаимосвязь ΠΌΠ΅ΠΆΠ΄Ρƒ элСктричСским ΠΏΠΎΠ»Π΅ΠΌ ΠΈ ΠΏΠΎΡ‚Π΅Π½Ρ†ΠΈΠ°Π»ΠΎΠΌ.ЭлСктричСскоС ΠΏΠΎΠ»Π΅ — это ΠΎΡ‚Ρ€ΠΈΡ†Π°Ρ‚Π΅Π»ΡŒΠ½Π°Ρ Π²Π΅Π»ΠΈΡ‡ΠΈΠ½Π° измСнСния ΠΏΠΎΡ‚Π΅Π½Ρ†ΠΈΠ°Π»Π°, дСлСнная Π½Π° ΠΈΠ·ΠΌΠ΅Π½Π΅Π½ΠΈΠ΅ полоТСния. Если Π²Ρ‹ построитС Π³Ρ€Π°Ρ„ΠΈΠΊ зависимости ΠΏΠΎΡ‚Π΅Π½Ρ†ΠΈΠ°Π»Π° ΠΎΡ‚ полоТСния, это Ρ‚ΠΎ ΠΆΠ΅ самоС, Ρ‡Ρ‚ΠΎ ΠΈ Π½Π°ΠΊΠ»ΠΎΠ½. ΠžΠ±Ρ€Π°Ρ‚ΠΈΡ‚Π΅ Π²Π½ΠΈΠΌΠ°Π½ΠΈΠ΅, Ρ‡Ρ‚ΠΎ Π³Ρ€Π°Ρ„ΠΈΠΊ Π²Ρ‹ΡˆΠ΅ являСтся Π»ΠΈΠ½Π΅ΠΉΠ½ΠΎΠΉ Ρ„ΡƒΠ½ΠΊΡ†ΠΈΠ΅ΠΉ. Π­Ρ‚ΠΎ ΠΎΠ·Π½Π°Ρ‡Π°Π΅Ρ‚, Ρ‡Ρ‚ΠΎ Π½Π°ΠΊΠ»ΠΎΠ½ ΠΈ, ΡΠ»Π΅Π΄ΠΎΠ²Π°Ρ‚Π΅Π»ΡŒΠ½ΠΎ, элСктричСскоС ΠΏΠΎΠ»Π΅ постоянны. По Π½Π°ΠΊΠ»ΠΎΠ½Ρƒ я ΠΏΠΎΠ»ΡƒΡ‡Π°ΡŽ постоянноС элСктричСскоС ΠΏΠΎΠ»Π΅ 0,713 Π²ΠΎΠ»ΡŒΡ‚ Π½Π° см (0,00713 Π’ / ΠΌ). О, 1 Π’ / ΠΌ — это Π½ΡŒΡŽΡ‚ΠΎΠ½ Π½Π° ΠΊΡƒΠ»ΠΎΠ½. Оба ΡΠ²Π»ΡΡŽΡ‚ΡΡ эквивалСнтными Π΅Π΄ΠΈΠ½ΠΈΡ†Π°ΠΌΠΈ измСрСния элСктричСского поля.

Но ΠΏΠΎΠ΄ΠΎΠΆΠ΄ΠΈΡ‚Π΅! ЭлСктричСскоС ΠΏΠΎΠ»Π΅ связано с элСктричСской силой, Π° это Π·Π½Π°Ρ‡ΠΈΡ‚, Ρ‡Ρ‚ΠΎ ΠΎΠ½ΠΎ Π΄ΠΎΠ»ΠΆΠ½ΠΎ Π±Ρ‹Ρ‚ΡŒ Π²Π΅ΠΊΡ‚ΠΎΡ€ΠΎΠΌ.ВычислСнноС Π²Ρ‹ΡˆΠ΅ Π·Π½Π°Ρ‡Π΅Π½ΠΈΠ΅ основано Π½Π° Π½Π°ΠΊΠ»ΠΎΠ½Π΅, поэтому это просто скалярноС Π·Π½Π°Ρ‡Π΅Π½ΠΈΠ΅. Π§Ρ‚ΠΎ ΠΆ, это Π»Π΅Π³ΠΊΠΎ ΠΈΡΠΏΡ€Π°Π²ΠΈΡ‚ΡŒ. ΠŸΠΎΡΠΊΠΎΠ»ΡŒΠΊΡƒ я построил ΠΏΠΎΡ‚Π΅Π½Ρ†ΠΈΠ°Π» ΠΎΡ‚Π½ΠΎΡΠΈΡ‚Π΅Π»ΡŒΠ½ΠΎ полоТСния y , это Π΄Π°Π΅Ρ‚ ΠΌΠ½Π΅ y ΠΊΠΎΠΌΠΏΠΎΠ½Π΅Π½Ρ‚ элСктричСского поля. Π§Ρ‚ΠΎΠ±Ρ‹ Π½Π°ΠΉΡ‚ΠΈ ΠΊΠΎΠΌΠΏΠΎΠ½Π΅Π½Ρ‚ x , ΠΌΠ½Π΅ Ρ‚Π°ΠΊΠΆΠ΅ Π½ΡƒΠΆΠ½ΠΎ ΠΏΠΎΡΡ‚Ρ€ΠΎΠΈΡ‚ΡŒ элСктричСский ΠΏΠΎΡ‚Π΅Π½Ρ†ΠΈΠ°Π» Π² этом Π½Π°ΠΏΡ€Π°Π²Π»Π΅Π½ΠΈΠΈ.

Но Π² этом случаС ΠΏΠΎΡ‚Π΅Π½Ρ†ΠΈΠ°Π» Π΄Π΅ΠΉΡΡ‚Π²ΠΈΡ‚Π΅Π»ΡŒΠ½ΠΎ Π½Π΅ сильно мСняСтся Π² Π½Π°ΠΏΡ€Π°Π²Π»Π΅Π½ΠΈΠΈ x . Π­Ρ‚ΠΎ ΠΎΠ·Π½Π°Ρ‡Π°Π΅Ρ‚, Ρ‡Ρ‚ΠΎ x-ΠΊΠΎΠΌΠΏΠΎΠ½Π΅Π½Ρ‚Π° элСктричСского поля Π±ΡƒΠ΄Π΅Ρ‚ Ρ€Π°Π²Π½Π° Π½ΡƒΠ»ΡŽ Π’ / ΠΌ.ЧСстно говоря, это Ρ…ΠΎΡ€ΠΎΡˆΠ°Ρ ΠΎΡΠΎΠ±Π΅Π½Π½ΠΎΡΡ‚ΡŒ этих ΠΏΠ°Ρ€Π°Π»Π»Π΅Π»ΡŒΠ½Ρ‹Ρ… проводящих пластин — ΠΎΠ½ΠΈ ΡΠΎΠ·Π΄Π°ΡŽΡ‚ постоянноС элСктричСскоС ΠΏΠΎΠ»Π΅ Π² ΠΎΠ΄Π½ΠΎΠΌ Π½Π°ΠΏΡ€Π°Π²Π»Π΅Π½ΠΈΠΈ.

Π—Π°Ρ‡Π΅ΠΌ Π½Π°ΠΌ Π±ΡƒΠΌΠ°Π³Π°?

Π˜Ρ‚Π°ΠΊ, это ΠΊΡ€Π°Ρ‚ΠΊΠΎΠ΅ Π²Π²Π΅Π΄Π΅Π½ΠΈΠ΅ Π² элСктричСскиС поля ΠΈ Ρ€Π°Π·Π½ΠΎΡΡ‚ΡŒ элСктричСских ΠΏΠΎΡ‚Π΅Π½Ρ†ΠΈΠ°Π»ΠΎΠ². Π’Π΅ΠΏΠ΅Ρ€ΡŒ ΠΎΡ‚Π²Π΅Ρ‚ Π½Π° Π²Π°ΠΆΠ½Ρ‹ΠΉ вопрос, ΠΊΠΎΡ‚ΠΎΡ€Ρ‹ΠΉ Π²Ρ‹ Π½Π΅ Π·Π°Π΄Π°Π²Π°Π»ΠΈ:

ΠŸΡ€Π΅Π΄ΠΏΠΎΠ»ΠΎΠΆΠΈΠΌ, я Π²ΠΎΠ·ΡŒΠΌΡƒ 9-Π²ΠΎΠ»ΡŒΡ‚ΠΎΠ²ΡƒΡŽ Π±Π°Ρ‚Π°Ρ€Π΅ΡŽ ΠΈ ΠΈΡΠΏΠΎΠ»ΡŒΠ·ΡƒΡŽ нСсколько ΠΏΡ€ΠΎΠ²ΠΎΠ΄ΠΎΠ² для ΠΏΠΎΠ΄ΠΊΠ»ΡŽΡ‡Π΅Π½ΠΈΡ ΠΊΠ»Π΅ΠΌΠΌ ΠΊ Π΄Π²ΡƒΠΌ ΠΏΠ°Ρ€Π°Π»Π»Π΅Π»ΡŒΠ½Ρ‹ΠΌ полосам ΠΈΠ· алюминиСвой Ρ„ΠΎΠ»ΡŒΠ³ΠΈ, Ρ€Π°Π·Π΄Π΅Π»Π΅Π½Π½Ρ‹ΠΌ расстояниСм 10 см — Π±Π΅Π· ΠΊΠ°ΠΊΠΈΡ…-Π»ΠΈΠ±ΠΎ Π±ΡƒΠΌΠ°Π³Π°.ΠœΠΎΠ³Ρƒ Π»ΠΈ я ΠΏΠΎΠ²Ρ‚ΠΎΡ€ΠΈΡ‚ΡŒ этот экспСримСнт, Ρ‡Ρ‚ΠΎΠ±Ρ‹ Ρ€Π°ΡΡΡ‡ΠΈΡ‚Π°Ρ‚ΡŒ элСктричСскоС ΠΏΠΎΠ»Π΅ ΠΌΠ΅ΠΆΠ΄Ρƒ этими пластинами?

ΠžΡ‚Π²Π΅Ρ‚ — Π½Π΅Ρ‚. Π― имСю Π²Π²ΠΈΠ΄Ρƒ, это Π΄ΠΎΠ»ΠΆΠ½ΠΎ Ρ€Π°Π±ΠΎΡ‚Π°Ρ‚ΡŒ . ВСория состоит Π² Ρ‚ΠΎΠΌ, Ρ‡Ρ‚ΠΎ Ρƒ вас Π΅ΡΡ‚ΡŒ ΠΈΠ·ΠΌΠ΅Π½Π΅Π½ΠΈΠ΅ ΠΏΠΎΡ‚Π΅Π½Ρ†ΠΈΠ°Π»Π° Π½Π° Π΄Π²ΡƒΡ… кусках алюминия ΠΈ ΠΈΠ·ΠΌΠ΅Π½Π΅Π½ΠΈΠ΅ расстояния. ΠŸΠΎΡΠΊΠΎΠ»ΡŒΠΊΡƒ Ρƒ вас Π΄Π²Π΅ ΠΏΠ°Ρ€Π°Π»Π»Π΅Π»ΡŒΠ½Ρ‹Π΅ пластины, элСктричСскоС ΠΏΠΎΠ»Π΅ Π΄ΠΎΠ»ΠΆΠ½ΠΎ Π±Ρ‹Ρ‚ΡŒ довольно постоянным. Но это Π½Π΅ сработаСт. Если Π²Ρ‹ Π²ΠΎΠ·ΡŒΠΌΠ΅Ρ‚Π΅ Π²ΠΎΠ»ΡŒΡ‚ΠΌΠ΅Ρ‚Ρ€ ΠΈ ΠΏΠΎΠ΄ΠΊΠ»ΡŽΡ‡ΠΈΡ‚Π΅ ΠΎΠ΄ΠΈΠ½ Ρ‰ΡƒΠΏ ΠΊ ΠΎΡ‚Ρ€ΠΈΡ†Π°Ρ‚Π΅Π»ΡŒΠ½ΠΎΠΉ полосС, Π° Π²Ρ‚ΠΎΡ€ΠΎΠΉ помСститС прямо посСрСдинС, ΠΎΠ½ Π΄ΠΎΠ»ΠΆΠ΅Π½ ΠΏΠΎΠΊΠ°Π·Π°Ρ‚ΡŒ 4.5 Π²ΠΎΠ»ΡŒΡ‚. ВмСсто этого ΠΎΠ½ Π±ΡƒΠ΄Π΅Ρ‚ ΠΏΠΎΠΊΠ°Π·Ρ‹Π²Π°Ρ‚ΡŒ ноль Π²ΠΎΠ»ΡŒΡ‚.

ΠŸΡ€ΠΎΠΈΠ·ΠΎΡˆΠ»Π° ошибка ΠΏΡ€ΠΈ настройкС ΠΏΠΎΠ»ΡŒΠ·ΠΎΠ²Π°Ρ‚Π΅Π»ΡŒΡΠΊΠΎΠ³ΠΎ Ρ„Π°ΠΉΠ»Π° cookie

Π­Ρ‚ΠΎΡ‚ сайт ΠΈΡΠΏΠΎΠ»ΡŒΠ·ΡƒΠ΅Ρ‚ Ρ„Π°ΠΉΠ»Ρ‹ cookie для ΠΏΠΎΠ²Ρ‹ΡˆΠ΅Π½ΠΈΡ ΠΏΡ€ΠΎΠΈΠ·Π²ΠΎΠ΄ΠΈΡ‚Π΅Π»ΡŒΠ½ΠΎΡΡ‚ΠΈ. Если ваш Π±Ρ€Π°ΡƒΠ·Π΅Ρ€ Π½Π΅ ΠΏΡ€ΠΈΠ½ΠΈΠΌΠ°Π΅Ρ‚ Ρ„Π°ΠΉΠ»Ρ‹ cookie, Π²Ρ‹ Π½Π΅ ΠΌΠΎΠΆΠ΅Ρ‚Π΅ ΠΏΡ€ΠΎΡΠΌΠ°Ρ‚Ρ€ΠΈΠ²Π°Ρ‚ΡŒ этот сайт.


Настройка вашСго Π±Ρ€Π°ΡƒΠ·Π΅Ρ€Π° для ΠΏΡ€ΠΈΠ΅ΠΌΠ° Ρ„Π°ΠΉΠ»ΠΎΠ² cookie

БущСствуСт мноТСство ΠΏΡ€ΠΈΡ‡ΠΈΠ½, ΠΏΠΎ ΠΊΠΎΡ‚ΠΎΡ€Ρ‹ΠΌ cookie Π½Π΅ ΠΌΠΎΠΆΠ΅Ρ‚ Π±Ρ‹Ρ‚ΡŒ установлСн ΠΏΡ€Π°Π²ΠΈΠ»ΡŒΠ½ΠΎ. НиТС ΠΏΡ€ΠΈΠ²Π΅Π΄Π΅Π½Ρ‹ Π½Π°ΠΈΠ±ΠΎΠ»Π΅Π΅ частыС ΠΏΡ€ΠΈΡ‡ΠΈΠ½Ρ‹:

  • Π’ вашСм Π±Ρ€Π°ΡƒΠ·Π΅Ρ€Π΅ ΠΎΡ‚ΠΊΠ»ΡŽΡ‡Π΅Π½Ρ‹ Ρ„Π°ΠΉΠ»Ρ‹ cookie. Π’Π°ΠΌ Π½Π΅ΠΎΠ±Ρ…ΠΎΠ΄ΠΈΠΌΠΎ ΡΠ±Ρ€ΠΎΡΠΈΡ‚ΡŒ настройки своСго Π±Ρ€Π°ΡƒΠ·Π΅Ρ€Π°, Ρ‡Ρ‚ΠΎΠ±Ρ‹ ΠΎΠ½ ΠΏΡ€ΠΈΠ½ΠΈΠΌΠ°Π» Ρ„Π°ΠΉΠ»Ρ‹ cookie, ΠΈΠ»ΠΈ Ρ‡Ρ‚ΠΎΠ±Ρ‹ ΡΠΏΡ€ΠΎΡΠΈΡ‚ΡŒ вас, Ρ…ΠΎΡ‚ΠΈΡ‚Π΅ Π»ΠΈ Π²Ρ‹ ΠΏΡ€ΠΈΠ½ΠΈΠΌΠ°Ρ‚ΡŒ Ρ„Π°ΠΉΠ»Ρ‹ cookie.
  • Π’Π°Ρˆ Π±Ρ€Π°ΡƒΠ·Π΅Ρ€ ΡΠΏΡ€Π°ΡˆΠΈΠ²Π°Π΅Ρ‚ вас, Ρ…ΠΎΡ‚ΠΈΡ‚Π΅ Π»ΠΈ Π²Ρ‹ ΠΏΡ€ΠΈΠ½ΠΈΠΌΠ°Ρ‚ΡŒ Ρ„Π°ΠΉΠ»Ρ‹ cookie, ΠΈ Π²Ρ‹ ΠΎΡ‚ΠΊΠ°Π·Π°Π»ΠΈΡΡŒ. Π§Ρ‚ΠΎΠ±Ρ‹ ΠΏΡ€ΠΈΠ½ΡΡ‚ΡŒ Ρ„Π°ΠΉΠ»Ρ‹ cookie с этого сайта, Π½Π°ΠΆΠΌΠΈΡ‚Π΅ ΠΊΠ½ΠΎΠΏΠΊΡƒ «Назад» ΠΈ ΠΏΡ€ΠΈΠΌΠΈΡ‚Π΅ Ρ„Π°ΠΉΠ»Ρ‹ cookie.
  • Π’Π°Ρˆ Π±Ρ€Π°ΡƒΠ·Π΅Ρ€ Π½Π΅ ΠΏΠΎΠ΄Π΄Π΅Ρ€ΠΆΠΈΠ²Π°Π΅Ρ‚ Ρ„Π°ΠΉΠ»Ρ‹ cookie. Если Π²Ρ‹ ΠΏΠΎΠ΄ΠΎΠ·Ρ€Π΅Π²Π°Π΅Ρ‚Π΅ это, ΠΏΠΎΠΏΡ€ΠΎΠ±ΡƒΠΉΡ‚Π΅ Π΄Ρ€ΡƒΠ³ΠΎΠΉ Π±Ρ€Π°ΡƒΠ·Π΅Ρ€.
  • Π”Π°Ρ‚Π° Π½Π° вашСм ΠΊΠΎΠΌΠΏΡŒΡŽΡ‚Π΅Ρ€Π΅ Π² ΠΏΡ€ΠΎΡˆΠ»ΠΎΠΌ. Если часы вашСго ΠΊΠΎΠΌΠΏΡŒΡŽΡ‚Π΅Ρ€Π° ΠΏΠΎΠΊΠ°Π·Ρ‹Π²Π°ΡŽΡ‚ Π΄Π°Ρ‚Ρƒ Π΄ΠΎ 1 января 1970 Π³., Π±Ρ€Π°ΡƒΠ·Π΅Ρ€ автоматичСски Π·Π°Π±ΡƒΠ΄Π΅Ρ‚ Ρ„Π°ΠΉΠ» cookie. Π§Ρ‚ΠΎΠ±Ρ‹ ΠΈΡΠΏΡ€Π°Π²ΠΈΡ‚ΡŒ это, установитС ΠΏΡ€Π°Π²ΠΈΠ»ΡŒΠ½ΠΎΠ΅ врСмя ΠΈ Π΄Π°Ρ‚Ρƒ Π½Π° своСм ΠΊΠΎΠΌΠΏΡŒΡŽΡ‚Π΅Ρ€Π΅.
  • Π’Ρ‹ установили ΠΏΡ€ΠΈΠ»ΠΎΠΆΠ΅Π½ΠΈΠ΅, ΠΊΠΎΡ‚ΠΎΡ€ΠΎΠ΅ отслСТиваСт ΠΈΠ»ΠΈ Π±Π»ΠΎΠΊΠΈΡ€ΡƒΠ΅Ρ‚ установку Ρ„Π°ΠΉΠ»ΠΎΠ² cookie. Π’Ρ‹ Π΄ΠΎΠ»ΠΆΠ½Ρ‹ ΠΎΡ‚ΠΊΠ»ΡŽΡ‡ΠΈΡ‚ΡŒ ΠΏΡ€ΠΈΠ»ΠΎΠΆΠ΅Π½ΠΈΠ΅ ΠΏΡ€ΠΈ Π²Ρ…ΠΎΠ΄Π΅ Π² систСму ΠΈΠ»ΠΈ ΠΏΡ€ΠΎΠΊΠΎΠ½ΡΡƒΠ»ΡŒΡ‚ΠΈΡ€ΠΎΠ²Π°Ρ‚ΡŒΡΡ с систСмным администратором.

ΠŸΠΎΡ‡Π΅ΠΌΡƒ этому сайту Ρ‚Ρ€Π΅Π±ΡƒΡŽΡ‚ΡΡ Ρ„Π°ΠΉΠ»Ρ‹ cookie?

Π­Ρ‚ΠΎΡ‚ сайт ΠΈΡΠΏΠΎΠ»ΡŒΠ·ΡƒΠ΅Ρ‚ Ρ„Π°ΠΉΠ»Ρ‹ cookie для ΠΏΠΎΠ²Ρ‹ΡˆΠ΅Π½ΠΈΡ ΠΏΡ€ΠΎΠΈΠ·Π²ΠΎΠ΄ΠΈΡ‚Π΅Π»ΡŒΠ½ΠΎΡΡ‚ΠΈ, запоминая, Ρ‡Ρ‚ΠΎ Π²Ρ‹ вошли Π² систСму, ΠΊΠΎΠ³Π΄Π° ΠΏΠ΅Ρ€Π΅Ρ…ΠΎΠ΄ΠΈΡ‚Π΅ со страницы Π½Π° страницу. Π§Ρ‚ΠΎΠ±Ρ‹ ΠΏΡ€Π΅Π΄ΠΎΡΡ‚Π°Π²ΠΈΡ‚ΡŒ доступ Π±Π΅Π· Ρ„Π°ΠΉΠ»ΠΎΠ² cookie ΠΏΠΎΡ‚Ρ€Π΅Π±ΡƒΠ΅Ρ‚, Ρ‡Ρ‚ΠΎΠ±Ρ‹ сайт создавал Π½ΠΎΠ²Ρ‹ΠΉ сСанс для ΠΊΠ°ΠΆΠ΄ΠΎΠΉ посСщаСмой страницы, Ρ‡Ρ‚ΠΎ замСдляСт Ρ€Π°Π±ΠΎΡ‚Ρƒ систСмы Π΄ΠΎ Π½Π΅ΠΏΡ€ΠΈΠ΅ΠΌΠ»Π΅ΠΌΠΎΠ³ΠΎ уровня.


Π§Ρ‚ΠΎ сохраняСтся Π² Ρ„Π°ΠΉΠ»Π΅ cookie?

Π­Ρ‚ΠΎΡ‚ сайт Π½Π΅ Ρ…Ρ€Π°Π½ΠΈΡ‚ Π½ΠΈΡ‡Π΅Π³ΠΎ, ΠΊΡ€ΠΎΠΌΠ΅ автоматичСски сгСнСрированного ΠΈΠ΄Π΅Π½Ρ‚ΠΈΡ„ΠΈΠΊΠ°Ρ‚ΠΎΡ€Π° сСанса Π² cookie; никакая другая информация Π½Π΅ фиксируСтся.

Как ΠΏΡ€Π°Π²ΠΈΠ»ΠΎ, Π² Ρ„Π°ΠΉΠ»Π΅ cookie ΠΌΠΎΠΆΠ΅Ρ‚ Ρ…Ρ€Π°Π½ΠΈΡ‚ΡŒΡΡ Ρ‚ΠΎΠ»ΡŒΠΊΠΎ информация, ΠΊΠΎΡ‚ΠΎΡ€ΡƒΡŽ Π²Ρ‹ прСдоставляСтС, ΠΈΠ»ΠΈ Π²Ρ‹Π±ΠΎΡ€, ΠΊΠΎΡ‚ΠΎΡ€Ρ‹ΠΉ Π²Ρ‹ Π΄Π΅Π»Π°Π΅Ρ‚Π΅ ΠΏΡ€ΠΈ посСщСнии Π²Π΅Π±-сайта. НапримСр, сайт Π½Π΅ ΠΌΠΎΠΆΠ΅Ρ‚ ΠΎΠΏΡ€Π΅Π΄Π΅Π»ΠΈΡ‚ΡŒ вашС имя элСктронной ΠΏΠΎΡ‡Ρ‚Ρ‹, ΠΏΠΎΠΊΠ° Π²Ρ‹ Π½Π΅ Π²Π²Π΅Π΄Π΅Ρ‚Π΅ Π΅Π³ΠΎ. Π Π°Π·Ρ€Π΅ΡˆΠ΅Π½ΠΈΠ΅ Π²Π΅Π±-сайту ΡΠΎΠ·Π΄Π°Π²Π°Ρ‚ΡŒ Ρ„Π°ΠΉΠ»Ρ‹ cookie Π½Π΅ Π΄Π°Π΅Ρ‚ этому ΠΈΠ»ΠΈ Π»ΡŽΠ±ΠΎΠΌΡƒ Π΄Ρ€ΡƒΠ³ΠΎΠΌΡƒ сайту доступа ΠΊ ΠΎΡΡ‚Π°Π»ΡŒΠ½ΠΎΠΉ части вашСго ΠΊΠΎΠΌΠΏΡŒΡŽΡ‚Π΅Ρ€Π°, ΠΈ Ρ‚ΠΎΠ»ΡŒΠΊΠΎ сайт, ΠΊΠΎΡ‚ΠΎΡ€Ρ‹ΠΉ создал Ρ„Π°ΠΉΠ» cookie, ΠΌΠΎΠΆΠ΅Ρ‚ Π΅Π³ΠΎ ΠΏΡ€ΠΎΡ‡ΠΈΡ‚Π°Ρ‚ΡŒ.

ЭлСктричСская ΠΏΠΎΡ‚Π΅Π½Ρ†ΠΈΠ°Π»ΡŒΠ½Π°Ρ энСргия (U) ΠΈ ЭлСктричСский ΠΏΠΎΡ‚Π΅Π½Ρ†ΠΈΠ°Π» (Π’): (ΠŸΡ€ΠΈΠΌΠ΅Ρ‡Π°Π½ΠΈΡ ΠΈΠ· C

ЭлСктричСская ΠΏΠΎΡ‚Π΅Π½Ρ†ΠΈΠ°Π»ΡŒΠ½Π°Ρ энСргия (U) ΠΈ элСктричСский ΠΏΠΎΡ‚Π΅Π½Ρ†ΠΈΠ°Π» (V): ( Записи Π»Π΅ΠΊΡ†ΠΈΠΉ К. Эркалса PHYS 221 )

Рассмотрим кондСнсатор с ΠΏΠ°Ρ€Π°Π»Π»Π΅Π»ΡŒΠ½Ρ‹ΠΌΠΈ пластинами, ΠΊΠΎΡ‚ΠΎΡ€Ρ‹ΠΉ ΠΏΡ€ΠΎΠΈΠ·Π²ΠΎΠ΄ΠΈΡ‚ ΠΎΠ΄Π½ΠΎΡ€ΠΎΠ΄Π½ΡƒΡŽ элСктричСскоС ΠΏΠΎΠ»Π΅ ΠΌΠ΅ΠΆΠ΄Ρƒ Π΅Π³ΠΎ большими пластинами. Π­Ρ‚ΠΎ достигаСтся ΠΏΡƒΡ‚Π΅ΠΌ ΠΏΠΎΠ΄ΠΊΠ»ΡŽΡ‡Π΅Π½ΠΈΡ ΠΊΠ°ΠΆΠ΄ΠΎΠΉ пластины ΠΊ ΠΎΠ΄Π½ΠΎΠΌΡƒ ΠΈΠ· Π²Ρ‹Π²ΠΎΠ΄ΠΎΠ² источник питания (Π½Π°ΠΏΡ€ΠΈΠΌΠ΅Ρ€, аккумулятор).

Рисунок 1: ЭлСктричСскоС ΠΏΠΎΠ»Π΅ создаСтся заряТСнными пластинами. Ρ€Π°Π·Π΄Π΅Π»Π΅Π½Π½Ρ‹Π΅ расстояниСм l. ОбвинСния Π½Π° пластинах стоят + Q ΠΈ Q.

Рисунок 2: ЭлСктричСский заряд q пСрСмСщаСтся ΠΈΠ· Ρ‚ΠΎΡ‡ΠΊΠΈ A Π² сторону Ρ‚ΠΎΡ‡ΠΊΠ° B с внСшнСй силой T ΠΏΡ€ΠΎΡ‚ΠΈΠ² элСктричСской силы qE.

Рис. 3, 4: Когда ΠΎΠ½ пСрСмСщаСтся Π½Π° расстояниС d, Π΅Π³ΠΎ ΠΏΠΎΡ‚Π΅Π½Ρ†ΠΈΠ°Π»ΡŒΠ½Π°Ρ энСргия Π² Ρ‚ΠΎΡ‡ΠΊΠ΅ B Ρ€Π°Π²Π½Π° qEd ΠΎΡ‚Π½ΠΎΡΠΈΡ‚Π΅Π»ΡŒΠ½ΠΎ Ρ‚ΠΎΡ‡ΠΊΠΈ A.

Рисунок 5: ΠŸΡ€ΠΈ отпускании ΠΈΠ· B (T = 0) ΠΎΠ½ Π±ΡƒΠ΄Π΅Ρ‚ ΡƒΡΠΊΠΎΡ€ΡΡ‚ΡŒΡΡ ΠΊ Π½ΠΈΠΆΠ½Π΅ΠΉ пластинС. Как ΠΎΠ½ двиТСтся ΠΏΠΎ Π½Π°ΠΏΡ€Π°Π²Π»Π΅Π½ΠΈΡŽ ΠΊ Π½ΠΈΠΆΠ½Π΅ΠΉ пластинС Π΅Π³ΠΎ ΠΏΠΎΡ‚Π΅Π½Ρ†ΠΈΠ°Π»ΡŒΠ½Π°Ρ энСргия ΡƒΠΌΠ΅Π½ΡŒΡˆΠ°Π΅Ρ‚ΡΡ, Π° кинСтичСская энСргия увСличиваСтся. Когда ΠΎΠ½ достигаСт Π½ΠΈΠΆΠ½Π΅Π³ΠΎ пластина (Π³Π΄Π΅ ΠΌΡ‹ ΠΌΠΎΠΆΠ΅ΠΌ Π²Ρ‹Π±Ρ€Π°Ρ‚ΡŒ ΠΏΠΎΡ‚Π΅Π½Ρ†ΠΈΠ°Π»ΡŒΠ½ΡƒΡŽ ΡΠ½Π΅Ρ€Π³ΠΈΡŽ Ρ€Π°Π²Π½ΠΎΠΉ Π½ΡƒΠ»ΡŽ), Π΅Π΅ ΠΏΠΎΡ‚Π΅Π½Ρ†ΠΈΠ°Π» энСргия Π² Ρ‚ΠΎΡ‡ΠΊΠ΅ A ΠΏΠΎΠ»Π½ΠΎΡΡ‚ΡŒΡŽ прСобразуСтся Π² ΠΊΠΈΠ½Π΅Ρ‚ΠΈΡ‡Π΅ΡΠΊΡƒΡŽ ΡΠ½Π΅Ρ€Π³ΠΈΡŽ Π² Ρ‚ΠΎΡ‡ΠΊΠ΅ B:

ΠžΠ±Ρ€Π°Ρ‚ΠΈΡ‚Π΅ Π²Π½ΠΈΠΌΠ°Π½ΠΈΠ΅, Ρ‡Ρ‚ΠΎ qEd — это Ρ€Π°Π±ΠΎΡ‚Π°, выполняСмая ΠΏΠΎΠ»Π΅ΠΌ Π² качСствС заряда двиТСтся ΠΏΠΎΠ΄ дСйствиСм силы qE ΠΎΡ‚ B ΠΊ A.Π—Π΄Π΅ΡΡŒ m — масса заряда q, Π° v — Π΅Π³ΠΎ ΡΠΊΠΎΡ€ΠΎΡΡ‚ΡŒ ΠΏΡ€ΠΈ достиТСнии Ρ‚ΠΎΡ‡ΠΊΠ° А. Π—Π΄Π΅ΡΡŒ ΠΌΡ‹ ΠΏΡ€Π΅Π΄ΠΏΠΎΠ»ΠΎΠΆΠΈΠ»ΠΈ, Ρ‡Ρ‚ΠΎ элСктричСская ΠΏΠΎΠ»Π΅ ΠΎΠ΄Π½ΠΎΡ€ΠΎΠ΄Π½ΠΎΠ΅! Π Π°Π±ΠΎΡ‚Ρ‹ ΠΏΠΎ E field:

Π”Π°Π²Π°ΠΉΡ‚Π΅ вспомним Ρ‚Π΅ΠΎΡ€Π΅ΠΌΡƒ ΠΎ кинСтичСской энСргии-Ρ€Π°Π±ΠΎΡ‚Π΅ (Work Energy ΠΏΡ€ΠΈΠ½Ρ†ΠΈΠΏ):

, Π³Π΄Π΅ ΠΌΡ‹ Π²Π²Π΅Π»ΠΈ понятиС ΠΏΠΎΡ‚Π΅Π½Ρ†ΠΈΠ°Π»ΡŒΠ½ΠΎΠΉ энСргии ΠΈ консСрвативная сила (сила, ΠΏΠΎΠ΄ ΠΊΠΎΡ‚ΠΎΡ€ΠΎΠΉ ΠΌΠΎΠΆΠ½ΠΎ ΠΎΠΏΡ€Π΅Π΄Π΅Π»ΠΈΡ‚ΡŒ ΠΏΠΎΡ‚Π΅Π½Ρ†ΠΈΠ°Π»ΡŒΠ½ΡƒΡŽ ΡΠ½Π΅Ρ€Π³ΠΈΡŽ Ρ‚Π°ΠΊ Ρ‡Ρ‚ΠΎ продСланная Ρ€Π°Π±ΠΎΡ‚Π° зависит Ρ‚ΠΎΠ»ΡŒΠΊΠΎ ΠΎΡ‚ Ρ€Π°Π·Π½ΠΈΡ†Ρ‹ ΠΏΠΎΡ‚Π΅Π½Ρ†ΠΈΠ°Π»ΡŒΠ½ΠΎΠΉ энСргии функция оцСниваСтся Π² ΠΊΠΎΠ½Π΅Ρ‡Π½Ρ‹Ρ… Ρ‚ΠΎΡ‡ΠΊΠ°Ρ…).

ΠŸΡ€Π°ΠΊΡ‚ΠΈΡ‡Π΅ΡΠΊΠΎΠ΅ ΠΏΡ€Π°Π²ΠΈΠ»ΠΎ опрСдСлСния Ρ‚ΠΎΠ³ΠΎ, являСтся Π»ΠΈ EPE ΡƒΠ²Π΅Π»ΠΈΡ‡Π΅Π½ΠΈΠ΅:

Если заряд двиТСтся Π² Ρ‚ΠΎΠΌ Π½Π°ΠΏΡ€Π°Π²Π»Π΅Π½ΠΈΠΈ, ΠΏΡ€ΠΈ Π½ΠΎΡ€ΠΌΠ°Π»ΡŒΠ½ΠΎΠΌ Π΄Π²ΠΈΠΆΠ΅Π½ΠΈΠΈ Π΅Π³ΠΎ элСктричСская ΠΏΠΎΡ‚Π΅Π½Ρ†ΠΈΠ°Π»ΡŒΠ½Π°Ρ энСргия ΡƒΠΌΠ΅Π½ΡŒΡˆΠ°Π΅Ρ‚ΡΡ. Если заряд двиТСтся Π² ΠΏΡ€ΠΎΡ‚ΠΈΠ²ΠΎΠΏΠΎΠ»ΠΎΠΆΠ½ΠΎΠΌ Π½Π°ΠΏΡ€Π°Π²Π»Π΅Π½ΠΈΠΈ ΠΊ Ρ‚ΠΎΠΌΡƒ, Ρ‡Ρ‚ΠΎ ΠΎΠ½ ΠΎΠ±Ρ‹Ρ‡Π½ΠΎ двигался Π±Ρ‹, Π΅Π³ΠΎ элСктричСская ΠΏΠΎΡ‚Π΅Π½Ρ†ΠΈΠ°Π»ΡŒΠ½Π°Ρ энСргия увСличиваСтся. Π­Ρ‚Π° ситуация ΠΏΠΎΡ…ΠΎΠΆΠ° Π½Π° ΡΠΈΡ‚ΡƒΠ°Ρ†ΠΈΡŽ постоянноС Π³Ρ€Π°Π²ΠΈΡ‚Π°Ρ†ΠΈΠΎΠ½Π½ΠΎΠ΅ ΠΏΠΎΠ»Π΅ (g = 9,8 ΠΌ / с 2 ). Когда Π²Ρ‹ ΠΏΠΎΠ΄Π½ΠΈΠΌΠ°Π΅Ρ‚Π΅ ΠΏΡ€Π΅Π΄ΠΌΠ΅Ρ‚, Π²Ρ‹ увСличивая Π΅Π³ΠΎ Π³Ρ€Π°Π²ΠΈΡ‚Π°Ρ†ΠΈΠΎΠ½Π½ΡƒΡŽ ΠΏΠΎΡ‚Π΅Π½Ρ†ΠΈΠ°Π»ΡŒΠ½ΡƒΡŽ ΡΠ½Π΅Ρ€Π³ΠΈΡŽ.Аналогичным ΠΎΠ±Ρ€Π°Π·ΠΎΠΌ, ΠΊΠΎΠ³Π΄Π° Π²Ρ‹ опускаСтС ΠΎΠ±ΡŠΠ΅ΠΊΡ‚, Π΅Π³ΠΎ гравитационная энСргия Ρ€Π°Π²Π½Π° ΡƒΠΌΠ΅Π½ΡŒΡˆΠ°Π΅Ρ‚ΡΡ.

ΠžΠ±Ρ‰Π°Ρ Ρ„ΠΎΡ€ΠΌΡƒΠ»Π° ΠΏΠΎΡ‚Π΅Π½Ρ†ΠΈΠ°Π»ΡŒΠ½ΠΎΠΉ Ρ€Π°Π·Π½ΠΈΡ†Ρ‹:

Π Π°Π±ΠΎΡ‚Π°, выполняСмая ΠΏΠΎΠ»Π΅ΠΌ E, ΠΊΠΎΠ³Π΄Π° ΠΎΠ½ΠΎ дСйствуСт Π½Π° заряд q для пСрСмСщСния. ΠΎΠ½ ΠΎΡ‚ Ρ‚ΠΎΡ‡ΠΊΠΈ A Π΄ΠΎ Ρ‚ΠΎΡ‡ΠΊΠΈ B опрСдСляСтся ΠΊΠ°ΠΊ Ρ€Π°Π·Π½ΠΎΡΡ‚ΡŒ элСктричСских ΠΏΠΎΡ‚Π΅Π½Ρ†ΠΈΠ°Π»ΠΎΠ² ΠΌΠ΅ΠΆΠ΄Ρƒ Ρ‚ΠΎΡ‡ΠΊΠΈ A ΠΈ B:

Ясно, Ρ‡Ρ‚ΠΎ ΠΏΠΎΡ‚Π΅Π½Ρ†ΠΈΠ°Π»ΡŒΠ½ΡƒΡŽ Ρ„ΡƒΠ½ΠΊΡ†ΠΈΡŽ V ΠΌΠΎΠΆΠ½ΠΎ ΡΠΎΠΏΠΎΡΡ‚Π°Π²ΠΈΡ‚ΡŒ ΠΊΠ°ΠΆΠ΄ΠΎΠΌΡƒ Ρ‚ΠΎΡ‡ΠΊΠ° Π² пространствС, ΠΎΠΊΡ€ΡƒΠΆΠ°ΡŽΡ‰Π΅ΠΌ распрСдСлСниС заряда (Π½Π°ΠΏΡ€ΠΈΠΌΠ΅Ρ€, ΠΏΠ°Ρ€Π°Π»Π»Π΅Π»ΡŒΠ½ΠΎΠ΅ Ρ‚Π°Ρ€Π΅Π»ΠΊΠΈ).ΠŸΡ€ΠΈΠ²Π΅Π΄Π΅Π½Π½Π°Ρ Π²Ρ‹ΡˆΠ΅ Ρ„ΠΎΡ€ΠΌΡƒΠ»Π° обСспСчиваСт простой Ρ€Π΅Ρ†Π΅ΠΏΡ‚ для расчСта Ρ€Π°Π±ΠΎΡ‚Ρ‹, ΠΏΡ€ΠΎΠ΄Π΅Π»Π°Π½Π½ΠΎΠΉ ΠΏΡ€ΠΈ ΠΏΠ΅Ρ€Π΅ΠΌΠ΅Ρ‰Π΅Π½ΠΈΠΈ заряда ΠΌΠ΅ΠΆΠ΄Ρƒ двумя Ρ‚ΠΎΡ‡ΠΊΠ°ΠΌΠΈ Π³Π΄Π΅ ΠΌΡ‹ Π·Π½Π°Π΅ΠΌ Π·Π½Π°Ρ‡Π΅Π½ΠΈΠ΅ разности ΠΏΠΎΡ‚Π΅Π½Ρ†ΠΈΠ°Π»ΠΎΠ². ΠŸΡ€ΠΈΠ²Π΅Π΄Π΅Π½Π½Ρ‹Π΅ Π²Ρ‹ΡˆΠ΅ утвСрТдСния ΠΈ Ρ„ΠΎΡ€ΠΌΡƒΠ»Π° Π΄Π΅ΠΉΡΡ‚Π²ΠΈΡ‚Π΅Π»ΡŒΠ½Ρ‹ нСзависимо ΠΎΡ‚ ΠΏΡƒΡ‚ΡŒ, ΠΏΠΎ ΠΊΠΎΡ‚ΠΎΡ€ΠΎΠΌΡƒ пСрСмСщаСтся заряд. ΠžΡΠΎΠ±Ρ‹ΠΉ интСрСс прСдставляСт ΠΏΠΎΡ‚Π΅Π½Ρ†ΠΈΠ°Π» Ρ‚ΠΎΡ‡Π΅Ρ‡Π½ΠΎΠ³ΠΎ заряда Q. Π•Π³ΠΎ ΠΌΠΎΠΆΠ½ΠΎ Π½Π°ΠΉΡ‚ΠΈ, просто Π²Ρ‹ΠΏΠΎΠ»Π½ΠΈΠ² ΠΈΠ½Ρ‚Π΅Π³Ρ€ΠΈΡ€ΠΎΠ²Π°Π½ΠΈΠ΅ ΠΏΠΎ простому ΠΏΡƒΡ‚ΠΈ (Π½Π°ΠΏΡ€ΠΈΠΌΠ΅Ρ€, ΠΏΠΎ прямой) ΠΈΠ· Ρ‚ΠΎΡ‡ΠΊΠΈ A расстояниС ΠΎΡ‚ Q ΠΊΠΎΡ‚ΠΎΡ€ΠΎΠ³ΠΎ Ρ€Π°Π²Π½ΠΎ r Π΄ΠΎ бСсконСчности.ΠŸΡƒΡ‚ΡŒ выбираСтся ΠΏΠΎ Ρ€Π°Π΄ΠΈΠ°Π»ΡŒΠ½ΠΎΠΉ Π»ΠΈΠ½ΠΈΠΈ, Ρ‚Π°ΠΊ Ρ‡Ρ‚ΠΎ становится просто Edr. ΠŸΠΎΡΠΊΠΎΠ»ΡŒΠΊΡƒ элСктричСскоС ΠΏΠΎΠ»Π΅ Q Ρ€Π°Π²Π½ΠΎ kQ / r 2 ,


Π­Ρ‚ΠΎΡ‚ процСсс опрСдСляСт элСктричСский ΠΏΠΎΡ‚Π΅Π½Ρ†ΠΈΠ°Π» Ρ‚ΠΎΡ‡Π΅Ρ‡Π½ΠΎΠ³ΠΎ Π·Π°Ρ€ΡΠΆΠ°Ρ‚ΡŒ. ΠžΠ±Ρ€Π°Ρ‚ΠΈΡ‚Π΅ Π²Π½ΠΈΠΌΠ°Π½ΠΈΠ΅, Ρ‡Ρ‚ΠΎ ΠΏΠΎΡ‚Π΅Π½Ρ†ΠΈΠ°Π»ΡŒΠ½Π°Ρ функция скалярная Π²Π΅Π»ΠΈΡ‡ΠΈΠ½Π° Π² ΠΎΡ‚Π»ΠΈΡ‡ΠΈΠ΅ ΠΎΡ‚ элСктричСского поля, ΡΠ²Π»ΡΡŽΡ‰Π΅Π³ΠΎΡΡ Π²Π΅ΠΊΡ‚ΠΎΡ€Π½ΠΎΠΉ Π²Π΅Π»ΠΈΡ‡ΠΈΠ½ΠΎΠΉ. Π’Π΅ΠΏΠ΅Ρ€ΡŒ ΠΌΡ‹ ΠΌΠΎΠΆΠ΅ΠΌ ΠΎΠΏΡ€Π΅Π΄Π΅Π»ΠΈΡ‚ΡŒ элСктричСский ΠΏΠΎΡ‚Π΅Π½Ρ†ΠΈΠ°Π» энСргия систСмы зарядов ΠΈΠ»ΠΈ зарядовых распрСдСлСний.ΠŸΡ€Π΅Π΄ΠΏΠΎΠ»ΠΎΠΆΠΈΠΌ, ΠΌΡ‹ вычисляСм ΠΏΡ€ΠΎΠ΄Π΅Π»Π°Π½Π½ΡƒΡŽ Ρ€Π°Π±ΠΎΡ‚Ρƒ ΠΎΡ‚Π½ΠΎΡΠΈΡ‚Π΅Π»ΡŒΠ½ΠΎ элСктричСскиС силы ΠΏΡ€ΠΈ ΠΏΠ΅Ρ€Π΅ΠΌΠ΅Ρ‰Π΅Π½ΠΈΠΈ заряда q ΠΈΠ· бСсконСчности Π² Ρ‚ΠΎΡ‡ΠΊΡƒ Π½Π° расстоянии r ΠΎΡ‚ заряд Q. Π Π°Π±ΠΎΡ‚Ρƒ Π²Ρ‹Π΄Π°Π»:

ΠžΠ±Ρ€Π°Ρ‚ΠΈΡ‚Π΅ Π²Π½ΠΈΠΌΠ°Π½ΠΈΠ΅, Ρ‡Ρ‚ΠΎ Ссли q ΠΎΡ‚Ρ€ΠΈΡ†Π°Ρ‚Π΅Π»ΡŒΠ½ΠΎΠ΅, Π΅Π³ΠΎ Π·Π½Π°ΠΊ Π΄ΠΎΠ»ΠΆΠ΅Π½ ΠΈΡΠΏΠΎΠ»ΡŒΠ·ΠΎΠ²Π°Ρ‚ΡŒΡΡ Π² ΡƒΡ€Π°Π²Π½Π΅Π½ΠΈΠ΅! Π‘Π»Π΅Π΄ΠΎΠ²Π°Ρ‚Π΅Π»ΡŒΠ½ΠΎ, систСма состоящий ΠΈΠ· ΠΎΡ‚Ρ€ΠΈΡ†Π°Ρ‚Π΅Π»ΡŒΠ½ΠΎΠ³ΠΎ ΠΈ ΠΏΠΎΠ»ΠΎΠΆΠΈΡ‚Π΅Π»ΡŒΠ½ΠΎΠ³ΠΎ Ρ‚ΠΎΡ‡Π΅Ρ‡Π½ΠΎΠ³ΠΎ заряда, ΠΈΠΌΠ΅Π΅Ρ‚ ΠΎΡ‚Ρ€ΠΈΡ†Π°Ρ‚Π΅Π»ΡŒΠ½Ρ‹ΠΉ ΠΏΠΎΡ‚Π΅Π½Ρ†ΠΈΠ°Π»ΡŒΠ½Π°Ρ энСргия.

ΠžΡ‚Ρ€ΠΈΡ†Π°Ρ‚Π΅Π»ΡŒΠ½Π°Ρ ΠΏΠΎΡ‚Π΅Π½Ρ†ΠΈΠ°Π»ΡŒΠ½Π°Ρ энСргия ΠΎΠ·Π½Π°Ρ‡Π°Π΅Ρ‚, Ρ‡Ρ‚ΠΎ Ρ€Π°Π±ΠΎΡ‚Π° Π΄ΠΎΠ»ΠΆΠ½Π° Π±Ρ‹Ρ‚ΡŒ Π²Ρ‹ΠΏΠΎΠ»Π½Π΅Π½Π° ΠΏΡ€ΠΎΡ‚ΠΈΠ² элСктричСского поля ΠΏΡ€ΠΈ Ρ€Π°Π·Π΄Π²ΠΈΠΆΠ΅Π½ΠΈΠΈ зарядов!

Π’Π΅ΠΏΠ΅Ρ€ΡŒ рассмотрим Π±ΠΎΠ»Π΅Π΅ ΠΎΠ±Ρ‰ΠΈΠΉ случай, связанный с ΠΏΠΎΡ‚Π΅Π½Ρ†ΠΈΠ°Π» Π² окрСстности ряда зарядов, ΠΊΠ°ΠΊ ΠΏΠΎΠΊΠ°Π·Π°Π½ΠΎ Π½Π° рисункС Π½ΠΈΠΆΠ΅:

ΠŸΡƒΡΡ‚ΡŒ r 1 , r 2 , r 3 Π±ΡƒΠ΄Π΅Ρ‚ расстояния зарядов Π΄ΠΎ Ρ‚ΠΎΡ‡ΠΊΠΈ поля А, Π° r 12 , r 13 , r 23 ΠΏΡ€Π΅Π΄ΡΡ‚Π°Π²Π»ΡΡŽΡ‚ собой расстояниС ΠΌΠ΅ΠΆΠ΄Ρƒ зарядами.ЭлСктричСский ΠΏΠΎΡ‚Π΅Π½Ρ†ΠΈΠ°Π» Π² Ρ‚ΠΎΡ‡ΠΊΠ΅ А Ρ€Π°Π²Π΅Π½:

.

ΠŸΡ€ΠΈΠΌΠ΅Ρ€:

Если ΠΌΡ‹ принСсСм заряд Q ΠΈΠ· бСсконСчности ΠΈ помСстим Π΅Π³ΠΎ Π² Ρ‚ΠΎΡ‡ΠΊΡƒ A продСланная Ρ€Π°Π±ΠΎΡ‚Π° Π±ΡƒΠ΄Π΅Ρ‚:

Буммарная элСктричСская ΠΏΠΎΡ‚Π΅Π½Ρ†ΠΈΠ°Π»ΡŒΠ½Π°Ρ энСргия этой систСмы обвинСния, Π° ΠΈΠΌΠ΅Π½Π½ΠΎ, Ρ€Π°Π±ΠΎΡ‚Π°, нСобходимая для Ρ‚ΠΎΠ³ΠΎ, Ρ‡Ρ‚ΠΎΠ±Ρ‹ привСсти ΠΈΡ… Π½Π° свои Π½Ρ‹Π½Π΅ΡˆΠ½ΠΈΠ΅ долТности, ΠΌΠΎΠΆΠ΅Ρ‚ Π±Ρ‹Ρ‚ΡŒ рассчитываСтся ΡΠ»Π΅Π΄ΡƒΡŽΡ‰ΠΈΠΌ ΠΎΠ±Ρ€Π°Π·ΠΎΠΌ: сначала ΠΏΡ€ΠΈΠ²Π΅Π΄ΠΈΡ‚Π΅ q1 (нулСвая Ρ€Π°Π±ΠΎΡ‚Π°, Ρ‚Π°ΠΊ ΠΊΠ°ΠΊ Π½Π΅Ρ‚ заряда Π΅Ρ‰Π΅), Π·Π°Ρ‚Π΅ΠΌ Π² ΠΏΠΎΠ»Π΅ q1 вывСсти q2, Π·Π°Ρ‚Π΅ΠΌ Π² поля q1 ΠΈ q2 принСсти q3.Π”ΠΎΠ±Π°Π²ΡŒΡ‚Π΅ всю Ρ€Π°Π±ΠΎΡ‚Ρƒ, Π½Π΅ΠΎΠ±Ρ…ΠΎΠ΄ΠΈΠΌΡƒΡŽ для Π²Ρ‹Ρ‡ΠΈΡΠ»ΠΈΡ‚ΡŒ ΠΎΠ±Ρ‰ΡƒΡŽ Ρ€Π°Π±ΠΎΡ‚Ρƒ. Π Π΅Π·ΡƒΠ»ΡŒΡ‚Π°Ρ‚ Π±ΡƒΠ΄Π΅Ρ‚:

ΠžΠ±Π½Π°Ρ€ΡƒΠΆΠ΅Π½ΠΈΠ΅ элСктричСского поля ΠΏΠΎ элСктричСскому ΠΏΠΎΡ‚Π΅Π½Ρ†ΠΈΠ°Π»Ρƒ:

ΠšΠΎΠΌΠΏΠΎΠ½Π΅Π½Ρ‚ E Π² любом Π½Π°ΠΏΡ€Π°Π²Π»Π΅Π½ΠΈΠΈ являСтся ΠΎΡ‚Ρ€ΠΈΡ†Π°Ρ‚Π΅Π»ΡŒΠ½Ρ‹ΠΌ ΡΠΊΠΎΡ€ΠΎΡΡ‚ΡŒ измСнСния ΠΏΠΎΡ‚Π΅Π½Ρ†ΠΈΠ°Π»Π° с расстояниСм Π² этом Π½Π°ΠΏΡ€Π°Π²Π»Π΅Π½ΠΈΠΈ:

Π‘ΠΈΠΌΠ²ΠΎΠ» называСтся Π“Ρ€Π°Π΄ΠΈΠ΅Π½Ρ‚. ЭлСктричСскоС ΠΏΠΎΠ»Π΅ — это Π³Ρ€Π°Π΄ΠΈΠ΅Π½Ρ‚ элСктричСского ΠΏΠΎΡ‚Π΅Π½Ρ†ΠΈΠ°Π»Π°.Π›ΠΈΠ½ΠΈΠΈ элСктричСского поля всСгда пСрпСндикулярно ΡΠΊΠ²ΠΈΠΏΠΎΡ‚Π΅Π½Ρ†ΠΈΠ°Π»ΡŒΠ½Ρ‹ΠΌ повСрхностям.

ΠŸΠΎΠ²Π΅Ρ€Ρ…Π½ΠΎΡΡ‚ΠΈ Ρ€Π°Π²Π½Ρ‹Ρ… хвостов:

Π­Ρ‚ΠΎ Π²ΠΎΠΎΠ±Ρ€Π°ΠΆΠ°Π΅ΠΌΡ‹Π΅ повСрхности, ΠΎΠΊΡ€ΡƒΠΆΠ°ΡŽΡ‰ΠΈΠ΅ заряд. распрСдСлСниС. Π’ частности, Ссли распрСдСлСниС заряда сфСричСскоС (Ρ‚ΠΎΡ‡Π΅Ρ‡Π½Ρ‹ΠΉ заряд ΠΈΠ»ΠΈ ΠΎΠ΄Π½ΠΎΡ€ΠΎΠ΄Π½ΠΎ заряТСнная сфСра), повСрхности сфСричСскиС, концСнтричСскиС с Ρ†Π΅Π½Ρ‚Ρ€ΠΎΠΌ заряда распрСдСлСниС. Π‘ΠΈΠ»ΠΎΠ²Ρ‹Π΅ Π»ΠΈΠ½ΠΈΠΈ элСктричСского поля всСгда пСрпСндикулярно ΡΠΊΠ²ΠΈΠΏΠΎΡ‚Π΅Π½Ρ†ΠΈΠ°Π»ΡŒΠ½Ρ‹ΠΌ повСрхностям.Π£Ρ€Π°Π²Π½Π΅Π½ΠΈΠ΅ ΠΏΠΎΠ΄Ρ€Π°Π·ΡƒΠΌΠ΅Π²Π°Π΅Ρ‚, Ρ‡Ρ‚ΠΎ ΠΈΠ·-Π·Π° ΠΎΡ‚Ρ€ΠΈΡ†Π°Ρ‚Π΅Π»ΡŒΠ½Ρ‹ΠΉ Π·Π½Π°ΠΊ, Π½Π°ΠΏΡ€Π°Π²Π»Π΅Π½ΠΈΠ΅ E ΠΏΡ€ΠΎΡ‚ΠΈΠ²ΠΎΠΏΠΎΠ»ΠΎΠΆΠ½ΠΎ Π½Π°ΠΏΡ€Π°Π²Π»Π΅Π½ΠΈΡŽ, Π² ΠΊΠΎΡ‚ΠΎΡ€ΠΎΠΌ V увСличиваСтся; E Π½Π°ΠΏΡ€Π°Π²Π»Π΅Π½ с Π±ΠΎΠ»Π΅Π΅ высоких ΡƒΡ€ΠΎΠ²Π½Π΅ΠΉ V Π½Π° Π±ΠΎΠ»Π΅Π΅ Π½ΠΈΠ·ΠΊΠΈΠ΅ (с Π±ΠΎΠ»Π΅Π΅ высоких ΠΏΠΎΡ‚Π΅Π½Ρ†ΠΈΠ°Π» для сниТСния ΠΏΠΎΡ‚Π΅Π½Ρ†ΠΈΠ°Π»Π°). Π”Ρ€ΡƒΠ³ΠΎΠΉ словами, Π³Ρ€Π°Π΄ΠΈΠ΅Π½Ρ‚ скаляра (Π² Π΄Π°Π½Π½ΠΎΠΌ случаС E-поля) Π½ΠΎΡ€ΠΌΠ°Π»Π΅Π½ ΠΊ повСрхности постоянного значСния (ΡΠΊΠ²ΠΈΠΏΠΎΡ‚Π΅Π½Ρ†ΠΈΠ°Π»ΡŒΠ½Π°Ρ ΠΏΠΎΠ²Π΅Ρ€Ρ…Π½ΠΎΡΡ‚ΡŒ) скаляра ΠΈ Π² Π½Π°ΠΏΡ€Π°Π²Π»Π΅Π½ΠΈΠΈ максимальная ΡΠΊΠΎΡ€ΠΎΡΡ‚ΡŒ измСнСния постоянного скаляра. Π—Π°ΠΏΠΎΠΌΠ½ΠΈΡ‚Π΅ это ΡƒΡ‚Π²Π΅Ρ€ΠΆΠ΄Π΅Π½ΠΈΠ΅, ΠΊΠΎΠ³Π΄Π° ΠΌΡ‹ ΠΏΡ€ΠΎΠ²ΠΎΠ΄ΠΈΠΌ экспСримСнт.

ЭлСктричСский ΠΏΠΎΡ‚Π΅Π½Ρ†ΠΈΠ°Π» Π² ΠΎΠ΄Π½ΠΎΡ€ΠΎΠ΄Π½ΠΎΠΌ элСктричСском ΠΏΠΎΠ»Π΅ — College Physics

Π¦Π΅Π»ΠΈ обучСния

  • ΠžΠΏΠΈΡˆΠΈΡ‚Π΅ взаимосвязь ΠΌΠ΅ΠΆΠ΄Ρƒ напряТСниСм ΠΈ элСктричСским ΠΏΠΎΠ»Π΅ΠΌ.
  • Π’Ρ‹Π²Π΅Π΄ΠΈΡ‚Π΅ Π²Ρ‹Ρ€Π°ΠΆΠ΅Π½ΠΈΠ΅ для элСктричСского ΠΏΠΎΡ‚Π΅Π½Ρ†ΠΈΠ°Π»Π° ΠΈ элСктричСского поля.
  • РассчитайтС Π½Π°ΠΏΡ€ΡΠΆΠ΅Π½Π½ΠΎΡΡ‚ΡŒ элСктричСского поля с ΡƒΡ‡Π΅Ρ‚ΠΎΠΌ расстояния ΠΈ напряТСния.

Π’ ΠΏΡ€Π΅Π΄Ρ‹Π΄ΡƒΡ‰Π΅ΠΌ Ρ€Π°Π·Π΄Π΅Π»Π΅ ΠΌΡ‹ исслСдовали взаимосвязь ΠΌΠ΅ΠΆΠ΄Ρƒ напряТСниСм ΠΈ энСргиСй.Π’ этом Ρ€Π°Π·Π΄Π΅Π»Π΅ ΠΌΡ‹ исслСдуСм взаимосвязь ΠΌΠ΅ΠΆΠ΄Ρƒ напряТСниСм ΠΈ элСктричСским ΠΏΠΎΠ»Π΅ΠΌ. НапримСр, ΠΎΠ΄Π½ΠΎΡ€ΠΎΠ΄Π½ΠΎΠ΅ элСктричСскоС ΠΏΠΎΠ»Π΅ создаСтся ΠΏΡƒΡ‚Π΅ΠΌ размСщСния разности ΠΏΠΎΡ‚Π΅Π½Ρ†ΠΈΠ°Π»ΠΎΠ² (ΠΈΠ»ΠΈ напряТСния) Π½Π° Π΄Π²ΡƒΡ… ΠΏΠ°Ρ€Π°Π»Π»Π΅Π»ΡŒΠ½Ρ‹Ρ… мСталличСских пластинах, ΠΎΠ±ΠΎΠ·Π½Π°Ρ‡Π΅Π½Π½Ρ‹Ρ… A ΠΈ B. Π½Π°ΠΏΡ€ΡΠΆΠ΅Π½Π½ΠΎΡΡ‚ΡŒ поля; это Ρ‚Π°ΠΊΠΆΠ΅ ΠΏΠΎΠΊΠ°ΠΆΠ΅Ρ‚ Π±ΠΎΠ»Π΅Π΅ Ρ„ΡƒΠ½Π΄Π°ΠΌΠ΅Π½Ρ‚Π°Π»ΡŒΠ½ΡƒΡŽ взаимосвязь ΠΌΠ΅ΠΆΠ΄Ρƒ элСктричСским ΠΏΠΎΡ‚Π΅Π½Ρ†ΠΈΠ°Π»ΠΎΠΌ ΠΈ элСктричСским ΠΏΠΎΠ»Π΅ΠΌ. Π‘ Ρ‚ΠΎΡ‡ΠΊΠΈ зрСния Ρ„ΠΈΠ·ΠΈΠΊΠ°, любой ΠΈΠ»ΠΈ ΠΌΠΎΠΆΠ΅Ρ‚ Π±Ρ‹Ρ‚ΡŒ использован для описания любого распрСдСлСния заряда.Π½Π°ΠΈΠ±ΠΎΠ»Π΅Π΅ тСсно связано с энСргиСй, Ρ‚ΠΎΠ³Π΄Π° ΠΊΠ°ΠΊ Π½Π°ΠΈΠ±ΠΎΠ»Π΅Π΅ тСсно связано с силой. являСтся скалярной Π²Π΅Π»ΠΈΡ‡ΠΈΠ½ΠΎΠΉ ΠΈ Π½Π΅ ΠΈΠΌΠ΅Π΅Ρ‚ направлСния, Π° являСтся Π²Π΅ΠΊΡ‚ΠΎΡ€Π½ΠΎΠΉ Π²Π΅Π»ΠΈΡ‡ΠΈΠ½ΠΎΠΉ, ΠΈΠΌΠ΅ΡŽΡ‰Π΅ΠΉ ΠΊΠ°ΠΊ Π²Π΅Π»ΠΈΡ‡ΠΈΠ½Ρƒ, Ρ‚Π°ΠΊ ΠΈ Π½Π°ΠΏΡ€Π°Π²Π»Π΅Π½ΠΈΠ΅. (ΠžΠ±Ρ€Π°Ρ‚ΠΈΡ‚Π΅ Π²Π½ΠΈΠΌΠ°Π½ΠΈΠ΅, Ρ‡Ρ‚ΠΎ Π²Π΅Π»ΠΈΡ‡ΠΈΠ½Π° напряТСнности элСктричСского поля, скалярная Π²Π΅Π»ΠΈΡ‡ΠΈΠ½Π°, прСдставлСна ​​ниТС.) Π’Π·Π°ΠΈΠΌΠΎΡΠ²ΡΠ·ΡŒ ΠΌΠ΅ΠΆΠ΄Ρƒ ΠΈ выявляСтся ΠΏΡƒΡ‚Π΅ΠΌ вычислСния Ρ€Π°Π±ΠΎΡ‚Ρ‹, ΡΠΎΠ²Π΅Ρ€ΡˆΠ°Π΅ΠΌΠΎΠΉ силой ΠΏΡ€ΠΈ ΠΏΠ΅Ρ€Π΅ΠΌΠ΅Ρ‰Π΅Π½ΠΈΠΈ заряда ΠΈΠ· Ρ‚ΠΎΡ‡ΠΊΠΈ A Π² Ρ‚ΠΎΡ‡ΠΊΡƒ B. Π² «ЭлСктричСская ΠΏΠΎΡ‚Π΅Π½Ρ†ΠΈΠ°Π»ΡŒΠ½Π°Ρ энСргия: Ρ€Π°Π·Π½ΠΎΡΡ‚ΡŒ ΠΏΠΎΡ‚Π΅Π½Ρ†ΠΈΠ°Π»ΠΎΠ²Β», это слоТно для ΠΏΡ€ΠΎΠΈΠ·Π²ΠΎΠ»ΡŒΠ½Ρ‹Ρ… распрСдСлСний заряда, Ρ‚Ρ€Π΅Π±ΡƒΡŽΡ‰ΠΈΡ… расчСтов.ΠŸΠΎΡΡ‚ΠΎΠΌΡƒ ΠΌΡ‹ рассматриваСм ΠΎΠ΄Π½ΠΎΡ€ΠΎΠ΄Π½ΠΎΠ΅ элСктричСскоС ΠΏΠΎΠ»Π΅ ΠΊΠ°ΠΊ интСрСсный частный случай.

Π Π°Π±ΠΎΡ‚Π°, ΡΠΎΠ²Π΅Ρ€ΡˆΠ°Π΅ΠΌΠ°Ρ элСктричСским ΠΏΠΎΠ»Π΅ΠΌ Π½Π° (Рис.) По ΠΏΠ΅Ρ€Π΅ΠΌΠ΅Ρ‰Π΅Π½ΠΈΡŽ ΠΏΠΎΠ»ΠΎΠΆΠΈΡ‚Π΅Π»ΡŒΠ½ΠΎΠ³ΠΎ заряда ΠΎΡ‚ A, ΠΏΠΎΠ»ΠΎΠΆΠΈΡ‚Π΅Π»ΡŒΠ½ΠΎΠΉ пластины, Π±ΠΎΠ»Π΅Π΅ высокий ΠΏΠΎΡ‚Π΅Π½Ρ†ΠΈΠ°Π», ΠΊ B, ΠΎΡ‚Ρ€ΠΈΡ†Π°Ρ‚Π΅Π»ΡŒΠ½ΠΎΠΉ пластины, Π±ΠΎΠ»Π΅Π΅ Π½ΠΈΠ·ΠΊΠΈΠΉ ΠΏΠΎΡ‚Π΅Π½Ρ†ΠΈΠ°Π», составляСт

Π Π°Π·Π½ΠΈΡ†Π° ΠΏΠΎΡ‚Π΅Π½Ρ†ΠΈΠ°Π»ΠΎΠ² ΠΌΠ΅ΠΆΠ΄Ρƒ Ρ‚ΠΎΡ‡ΠΊΠ°ΠΌΠΈ A ΠΈ B Ρ€Π°Π²Π½Π°

.

Если ввСсти это Π² Π²Ρ‹Ρ€Π°ΠΆΠ΅Π½ΠΈΠ΅ для Ρ€Π°Π±ΠΎΡ‚Ρ‹, получится

Π Π°Π±ΠΎΡ‚Π° Π΅ΡΡ‚ΡŒ; здСсь, Ρ‚Π°ΠΊ ΠΊΠ°ΠΊ ΠΏΡƒΡ‚ΡŒ ΠΏΠ°Ρ€Π°Π»Π»Π΅Π»Π΅Π½ полю ΠΈ Ρ‚Π°ΠΊ.Π’Π°ΠΊ ΠΊΠ°ΠΊ ΠΌΡ‹ это Π²ΠΈΠ΄ΠΈΠΌ. ΠŸΠΎΠ΄ΡΡ‚Π°Π½ΠΎΠ²ΠΊΠ° этого выраТСния для Ρ€Π°Π±ΠΎΡ‚Ρ‹ Π² ΠΏΡ€Π΅Π΄Ρ‹Π΄ΡƒΡ‰Π΅Π΅ ΡƒΡ€Π°Π²Π½Π΅Π½ΠΈΠ΅ Π΄Π°Π΅Ρ‚

Заряд отмСняСтся, ΠΈ поэтому напряТСниС ΠΌΠ΅ΠΆΠ΄Ρƒ Ρ‚ΠΎΡ‡ΠΊΠ°ΠΌΠΈ A ΠΈ B оказываСтся Ρ€Π°Π²Π½Ρ‹ΠΌ

.

, Π³Π΄Π΅ — это расстояниС ΠΎΡ‚ A Π΄ΠΎ B ΠΈΠ»ΠΈ расстояниС ΠΌΠ΅ΠΆΠ΄Ρƒ пластинами Π½Π° (Рисунок). ΠžΠ±Ρ€Π°Ρ‚ΠΈΡ‚Π΅ Π²Π½ΠΈΠΌΠ°Π½ΠΈΠ΅, Ρ‡Ρ‚ΠΎ ΠΏΡ€ΠΈΠ²Π΅Π΄Π΅Π½Π½ΠΎΠ΅ Π²Ρ‹ΡˆΠ΅ ΡƒΡ€Π°Π²Π½Π΅Π½ΠΈΠ΅ ΠΏΠΎΠ΄Ρ€Π°Π·ΡƒΠΌΠ΅Π²Π°Π΅Ρ‚, Ρ‡Ρ‚ΠΎ Π΅Π΄ΠΈΠ½ΠΈΡ†Ρ‹ измСрСния элСктричСского поля — Π²ΠΎΠ»ΡŒΡ‚ Π½Π° ΠΌΠ΅Ρ‚Ρ€. ΠœΡ‹ ΡƒΠΆΠ΅ Π·Π½Π°Π΅ΠΌ, Ρ‡Ρ‚ΠΎ Π΅Π΄ΠΈΠ½ΠΈΡ†Ρ‹ измСрСния элСктричСского поля — Π½ΡŒΡŽΡ‚ΠΎΠ½Ρ‹ Π½Π° ΠΊΡƒΠ»ΠΎΠ½; Ρ‚Π°ΠΊΠΈΠΌ ΠΎΠ±Ρ€Π°Π·ΠΎΠΌ, Π²Π΅Ρ€Π½ΠΎ ΡΠ»Π΅Π΄ΡƒΡŽΡ‰Π΅Π΅ ΡΠΎΠΎΡ‚Π½ΠΎΡˆΠ΅Π½ΠΈΠ΅ ΠΌΠ΅ΠΆΠ΄Ρƒ Π΅Π΄ΠΈΠ½ΠΈΡ†Π°ΠΌΠΈ:

НапряТСниС ΠΌΠ΅ΠΆΠ΄Ρƒ Ρ‚ΠΎΡ‡ΠΊΠ°ΠΌΠΈ A ΠΈ B

, Π³Π΄Π΅ — это расстояниС ΠΎΡ‚ A Π΄ΠΎ B ΠΈΠ»ΠΈ расстояниС ΠΌΠ΅ΠΆΠ΄Ρƒ пластинами.

КакоС максимально Π²ΠΎΠ·ΠΌΠΎΠΆΠ½ΠΎΠ΅ напряТСниС ΠΌΠ΅ΠΆΠ΄Ρƒ двумя пластинами?

Π‘ΡƒΡ…ΠΎΠΉ Π²ΠΎΠ·Π΄ΡƒΡ… ΠΏΠΎΠ΄Π΄Π΅Ρ€ΠΆΠΈΠ²Π°Π΅Ρ‚ ΠΌΠ°ΠΊΡΠΈΠΌΠ°Π»ΡŒΠ½ΡƒΡŽ Π½Π°ΠΏΡ€ΡΠΆΠ΅Π½Π½ΠΎΡΡ‚ΡŒ элСктричСского поля ΠΎΠΊΠΎΠ»ΠΎ. Π’Ρ‹ΡˆΠ΅ этого значСния ΠΏΠΎΠ»Π΅ создаСт Π΄ΠΎΡΡ‚Π°Ρ‚ΠΎΡ‡Π½ΡƒΡŽ ΠΈΠΎΠ½ΠΈΠ·Π°Ρ†ΠΈΡŽ Π² Π²ΠΎΠ·Π΄ΡƒΡ…Π΅, Ρ‡Ρ‚ΠΎΠ±Ρ‹ ΡΠ΄Π΅Π»Π°Ρ‚ΡŒ Π²ΠΎΠ·Π΄ΡƒΡ… ΠΏΡ€ΠΎΠ²ΠΎΠ΄Π½ΠΈΠΊΠΎΠΌ. Π­Ρ‚ΠΎ допускаСт разряд ΠΈΠ»ΠΈ искру, ΡƒΠΌΠ΅Π½ΡŒΡˆΠ°ΡŽΡ‰ΠΈΠ΅ ΠΏΠΎΠ»Π΅. Каково ΠΆΠ΅ максимальноС напряТСниС ΠΌΠ΅ΠΆΠ΄Ρƒ двумя ΠΏΠ°Ρ€Π°Π»Π»Π΅Π»ΡŒΠ½Ρ‹ΠΌΠΈ проводящими пластинами, Ρ€Π°Π·Π΄Π΅Π»Π΅Π½Π½Ρ‹ΠΌΠΈ 2,5 см сухого Π²ΠΎΠ·Π΄ΡƒΡ…Π°?

БтратСгия

Нам Π΄Π°Π½ΠΎ максимальноС элСктричСскоС ΠΏΠΎΠ»Π΅ ΠΌΠ΅ΠΆΠ΄Ρƒ пластинами ΠΈ расстояниС ΠΌΠ΅ΠΆΠ΄Ρƒ Π½ΠΈΠΌΠΈ.Π’Π°ΠΊΠΈΠΌ ΠΎΠ±Ρ€Π°Π·ΠΎΠΌ, ΡƒΡ€Π°Π²Π½Π΅Π½ΠΈΠ΅ ΠΌΠΎΠΆΠ½ΠΎ ΠΈΡΠΏΠΎΠ»ΡŒΠ·ΠΎΠ²Π°Ρ‚ΡŒ для расчСта максимального напряТСния.

РСшСниС

Π Π°Π·Π½ΠΎΡΡ‚ΡŒ ΠΏΠΎΡ‚Π΅Π½Ρ†ΠΈΠ°Π»ΠΎΠ² ΠΈΠ»ΠΈ напряТСниС ΠΌΠ΅ΠΆΠ΄Ρƒ пластинами

Π’Π²ΠΎΠ΄ Π·Π°Π΄Π°Π½Π½Ρ‹Ρ… Π·Π½Π°Ρ‡Π΅Π½ΠΈΠΉ для ΠΈ Π΄Π°Π΅Ρ‚

ΠΈΠ»ΠΈ

(ΠžΡ‚Π²Π΅Ρ‚ состоит Ρ‚ΠΎΠ»ΡŒΠΊΠΎ ΠΈΠ· Π΄Π²ΡƒΡ… Ρ†ΠΈΡ„Ρ€, ΠΏΠΎΡΠΊΠΎΠ»ΡŒΠΊΡƒ максимальная Π½Π°ΠΏΡ€ΡΠΆΠ΅Π½Π½ΠΎΡΡ‚ΡŒ поля являСтся ΠΏΡ€ΠΈΠ±Π»ΠΈΠ·ΠΈΡ‚Π΅Π»ΡŒΠ½ΠΎΠΉ.)

ΠžΠ±ΡΡƒΠΆΠ΄Π΅Π½ΠΈΠ΅

Одним ΠΈΠ· слСдствий этого Ρ€Π΅Π·ΡƒΠ»ΡŒΡ‚Π°Ρ‚Π° являСтся Ρ‚ΠΎ, Ρ‡Ρ‚ΠΎ трСбуСтся ΠΎΠΊΠΎΠ»ΠΎ 75 ΠΊΠ’, Ρ‡Ρ‚ΠΎΠ±Ρ‹ ΡΠΎΠ²Π΅Ρ€ΡˆΠΈΡ‚ΡŒ скачок искры Ρ‡Π΅Ρ€Π΅Π· 2,5 см (1 дюйм.) ΠΏΡ€ΠΎΠΌΠ΅ΠΆΡƒΡ‚ΠΎΠΊ, ΠΈΠ»ΠΈ 150 ΠΊΠ’ для искры 5 см. Π­Ρ‚ΠΎ ΠΎΠ³Ρ€Π°Π½ΠΈΡ‡ΠΈΠ²Π°Π΅Ρ‚ напряТСния, ΠΊΠΎΡ‚ΠΎΡ€Ρ‹Π΅ ΠΌΠΎΠ³ΡƒΡ‚ ΡΡƒΡ‰Π΅ΡΡ‚Π²ΠΎΠ²Π°Ρ‚ΡŒ ΠΌΠ΅ΠΆΠ΄Ρƒ ΠΏΡ€ΠΎΠ²ΠΎΠ΄Π½ΠΈΠΊΠ°ΠΌΠΈ, Π²ΠΎΠ·ΠΌΠΎΠΆΠ½ΠΎ, Π½Π° Π»ΠΈΠ½ΠΈΠΈ элСктропСрСдачи. МСньшСС напряТСниС Π²Ρ‹Π·ΠΎΠ²Π΅Ρ‚ искру, Ссли Π½Π° повСрхности Π΅ΡΡ‚ΡŒ Ρ‚ΠΎΡ‡ΠΊΠΈ, ΠΏΠΎΡΠΊΠΎΠ»ΡŒΠΊΡƒ Ρ‚ΠΎΡ‡ΠΊΠΈ ΡΠΎΠ·Π΄Π°ΡŽΡ‚ большиС поля, Ρ‡Π΅ΠΌ Π³Π»Π°Π΄ΠΊΠΈΠ΅ повСрхности. Π’Π»Π°ΠΆΠ½Ρ‹ΠΉ Π²ΠΎΠ·Π΄ΡƒΡ… Ρ€Π°Π·Ρ€ΡƒΡˆΠ°Π΅Ρ‚ΡΡ ΠΏΡ€ΠΈ Π±ΠΎΠ»Π΅Π΅ Π½ΠΈΠ·ΠΊΠΎΠΉ напряТСнности поля, Π° это ΠΎΠ·Π½Π°Ρ‡Π°Π΅Ρ‚, Ρ‡Ρ‚ΠΎ мСньшСС напряТСниС заставит искру ΠΏΡ€ΠΎΡΠΊΠΎΡ‡ΠΈΡ‚ΡŒ Ρ‡Π΅Ρ€Π΅Π· Π²Π»Π°ΠΆΠ½Ρ‹ΠΉ Π²ΠΎΠ·Π΄ΡƒΡ…. Π‘Π°ΠΌΡ‹Π΅ большиС напряТСния ΠΌΠΎΠ³ΡƒΡ‚ ΡΠΎΠ·Π΄Π°Π²Π°Ρ‚ΡŒΡΡ, Π½Π°ΠΏΡ€ΠΈΠΌΠ΅Ρ€, статичСским элСктричСством Π² Π·Π°ΡΡƒΡˆΠ»ΠΈΠ²Ρ‹Π΅ Π΄Π½ΠΈ.

Π˜ΡΠΊΡ€ΠΎΠ²Π°Ρ ΠΊΠ°ΠΌΠ΅Ρ€Π° ΠΈΡΠΏΠΎΠ»ΡŒΠ·ΡƒΠ΅Ρ‚ΡΡ для отслСТивания Ρ‚Ρ€Π°Π΅ΠΊΡ‚ΠΎΡ€ΠΈΠΈ частиц высоких энСргий.Π˜ΠΎΠ½ΠΈΠ·Π°Ρ†ΠΈΡ, создаваСмая частицами ΠΏΡ€ΠΈ ΠΏΡ€ΠΎΡ…ΠΎΠΆΠ΄Π΅Π½ΠΈΠΈ Ρ‡Π΅Ρ€Π΅Π· Π³Π°Π· ΠΌΠ΅ΠΆΠ΄Ρƒ пластинами, позволяСт искрС ΠΏΡ€Ρ‹Π³Π½ΡƒΡ‚ΡŒ. Π˜ΡΠΊΡ€Ρ‹ располоТСны пСрпСндикулярно пластинам, слСдуя силовым линиям элСктричСского поля ΠΌΠ΅ΠΆΠ΄Ρƒ Π½ΠΈΠΌΠΈ. Π Π°Π·Π½ΠΎΡΡ‚ΡŒ ΠΏΠΎΡ‚Π΅Π½Ρ†ΠΈΠ°Π»ΠΎΠ² ΠΌΠ΅ΠΆΠ΄Ρƒ сосСдними пластинами нСдостаточно высока, Ρ‡Ρ‚ΠΎΠ±Ρ‹ Π²Ρ‹Π·Π²Π°Ρ‚ΡŒ искры Π±Π΅Π· ΠΈΠΎΠ½ΠΈΠ·Π°Ρ†ΠΈΠΈ, создаваСмой частицами ΠΈΠ· экспСримСнтов Π½Π° ускоритСлях (ΠΈΠ»ΠΈ космичСскими Π»ΡƒΡ‡Π°ΠΌΠΈ). (Π˜ΡΡ‚ΠΎΡ‡Π½ΠΈΠΊ: Π”Π°Π΄Ρ€ΠΎ, Wikimedia Commons)

ПолС ΠΈ сила Π²Π½ΡƒΡ‚Ρ€ΠΈ элСктронной ΠΏΡƒΡˆΠΊΠΈ

(a) ЭлСктронная ΠΏΡƒΡˆΠΊΠ° ΠΈΠΌΠ΅Π΅Ρ‚ ΠΏΠ°Ρ€Π°Π»Π»Π΅Π»ΡŒΠ½Ρ‹Π΅ пластины, Ρ€Π°Π·Π΄Π΅Π»Π΅Π½Π½Ρ‹Π΅ Ρ‡Π΅Ρ‚Ρ‹Ρ€ΡŒΠΌΡ.00 см ΠΈ Π΄Π°Π΅Ρ‚ элСктронам ΡΠ½Π΅Ρ€Π³ΠΈΡŽ 25,0 кэВ. Какая Π½Π°ΠΏΡ€ΡΠΆΠ΅Π½Π½ΠΎΡΡ‚ΡŒ элСктричСского поля ΠΌΠ΅ΠΆΠ΄Ρƒ пластинами? Π±) ΠšΠ°ΠΊΡƒΡŽ силу это ΠΏΠΎΠ»Π΅ Π±ΡƒΠ΄Π΅Ρ‚ ΠΎΠΊΠ°Π·Ρ‹Π²Π°Ρ‚ΡŒ Π½Π° кусок пластика с зарядом, ΠΊΠΎΡ‚ΠΎΡ€Ρ‹ΠΉ ΠΏΡ€ΠΎΡ…ΠΎΠ΄ΠΈΡ‚ ΠΌΠ΅ΠΆΠ΄Ρƒ пластинами?

БтратСгия

ΠŸΠΎΡΠΊΠΎΠ»ΡŒΠΊΡƒ напряТСниС ΠΈ расстояниС ΠΌΠ΅ΠΆΠ΄Ρƒ пластинами Π΄Π°Π½Ρ‹, Π½Π°ΠΏΡ€ΡΠΆΠ΅Π½Π½ΠΎΡΡ‚ΡŒ элСктричСского поля ΠΌΠΎΠΆΠ΅Ρ‚ Π±Ρ‹Ρ‚ΡŒ вычислСна нСпосрСдствСнно ΠΈΠ· выраТСния. Как Ρ‚ΠΎΠ»ΡŒΠΊΠΎ Π½Π°ΠΏΡ€ΡΠΆΠ΅Π½Π½ΠΎΡΡ‚ΡŒ элСктричСского поля извСстна, сила, Π΄Π΅ΠΉΡΡ‚Π²ΡƒΡŽΡ‰Π°Ρ Π½Π° заряд, опрСдСляСтся с ΠΏΠΎΠΌΠΎΡ‰ΡŒΡŽ. ΠŸΠΎΡΠΊΠΎΠ»ΡŒΠΊΡƒ элСктричСскоС ΠΏΠΎΠ»Π΅ ΠΈΠΌΠ΅Π΅Ρ‚ Ρ‚ΠΎΠ»ΡŒΠΊΠΎ ΠΎΠ΄Π½ΠΎ Π½Π°ΠΏΡ€Π°Π²Π»Π΅Π½ΠΈΠ΅, ΠΌΡ‹ ΠΌΠΎΠΆΠ΅ΠΌ Π·Π°ΠΏΠΈΡΠ°Ρ‚ΡŒ это ΡƒΡ€Π°Π²Π½Π΅Π½ΠΈΠ΅ Π² Ρ‚Π΅Ρ€ΠΌΠΈΠ½Π°Ρ… Π²Π΅Π»ΠΈΡ‡ΠΈΠ½,.

РСшСниС для (a)

Π’Ρ‹Ρ€Π°ΠΆΠ΅Π½ΠΈΠ΅ для Π²Π΅Π»ΠΈΡ‡ΠΈΠ½Ρ‹ элСктричСского поля ΠΌΠ΅ΠΆΠ΄Ρƒ двумя ΠΎΠ΄Π½ΠΎΡ€ΠΎΠ΄Π½Ρ‹ΠΌΠΈ мСталличСскими пластинами Ρ€Π°Π²Π½ΠΎ

.

ΠŸΠΎΡΠΊΠΎΠ»ΡŒΠΊΡƒ элСктрон являСтся однозарядным ΠΈ ΠΏΠΎΠ»ΡƒΡ‡Π°Π΅Ρ‚ ΡΠ½Π΅Ρ€Π³ΠΈΡŽ 25,0 кэВ, Ρ€Π°Π·Π½ΠΎΡΡ‚ΡŒ ΠΏΠΎΡ‚Π΅Π½Ρ†ΠΈΠ°Π»ΠΎΠ² Π΄ΠΎΠ»ΠΆΠ½Π° ΡΠΎΡΡ‚Π°Π²Π»ΡΡ‚ΡŒ 25,0 ΠΊΠ’. Вводя это Π·Π½Π°Ρ‡Π΅Π½ΠΈΠ΅ ΠΈ расстояниС ΠΌΠ΅ΠΆΠ΄Ρƒ ΠΏΠ»ΠΈΡ‚Π°ΠΌΠΈ 0,0400 ΠΌ, ΠΏΠΎΠ»ΡƒΡ‡Π°Π΅ΠΌ

РСшСниС для (b)

Π’Π΅Π»ΠΈΡ‡ΠΈΠ½Π° силы, Π΄Π΅ΠΉΡΡ‚Π²ΡƒΡŽΡ‰Π΅ΠΉ Π½Π° заряд Π² элСктричСском ΠΏΠΎΠ»Π΅, получаСтся ΠΈΠ· уравнСния

ΠŸΠΎΠ΄ΡΡ‚Π°Π½ΠΎΠ²ΠΊΠ° извСстных Π·Π½Π°Ρ‡Π΅Π½ΠΈΠΉ Π΄Π°Π΅Ρ‚

ΠžΠ±ΡΡƒΠΆΠ΄Π΅Π½ΠΈΠ΅

ΠžΠ±Ρ€Π°Ρ‚ΠΈΡ‚Π΅ Π²Π½ΠΈΠΌΠ°Π½ΠΈΠ΅, Ρ‡Ρ‚ΠΎ Π΅Π΄ΠΈΠ½ΠΈΡ†Ρ‹ измСрСния — Π½ΡŒΡŽΡ‚ΠΎΠ½Ρ‹, Ρ‚.ΠΊ.Π‘ΠΈΠ»Π°, Π΄Π΅ΠΉΡΡ‚Π²ΡƒΡŽΡ‰Π°Ρ Π½Π° заряд, ΠΎΠ΄ΠΈΠ½Π°ΠΊΠΎΠ²Π° нСзависимо ΠΎΡ‚ Ρ‚ΠΎΠ³ΠΎ, Π³Π΄Π΅ находится заряд ΠΌΠ΅ΠΆΠ΄Ρƒ пластинами. Π­Ρ‚ΠΎ ΠΏΠΎΡ‚ΠΎΠΌΡƒ, Ρ‡Ρ‚ΠΎ элСктричСскоС ΠΏΠΎΠ»Π΅ ΠΌΠ΅ΠΆΠ΄Ρƒ пластинами ΠΎΠ΄Π½ΠΎΡ€ΠΎΠ΄Π½ΠΎ.

Π’ Π±ΠΎΠ»Π΅Π΅ ΠΎΠ±Ρ‰ΠΈΡ… ситуациях, нСзависимо ΠΎΡ‚ Ρ‚ΠΎΠ³ΠΎ, являСтся Π»ΠΈ элСктричСскоС ΠΏΠΎΠ»Π΅ ΠΎΠ΄Π½ΠΎΡ€ΠΎΠ΄Π½Ρ‹ΠΌ, ΠΎΠ½ΠΎ ΡƒΠΊΠ°Π·Ρ‹Π²Π°Π΅Ρ‚ Π² Π½Π°ΠΏΡ€Π°Π²Π»Π΅Π½ΠΈΠΈ ΡƒΠΌΠ΅Π½ΡŒΡˆΠ΅Π½ΠΈΡ ΠΏΠΎΡ‚Π΅Π½Ρ†ΠΈΠ°Π»Π°, ΠΏΠΎΡ‚ΠΎΠΌΡƒ Ρ‡Ρ‚ΠΎ сила, Π΄Π΅ΠΉΡΡ‚Π²ΡƒΡŽΡ‰Π°Ρ Π½Π° ΠΏΠΎΠ»ΠΎΠΆΠΈΡ‚Π΅Π»ΡŒΠ½Ρ‹ΠΉ заряд, Π½Π°ΠΏΡ€Π°Π²Π»Π΅Π½Π° ​​как Π² Π½Π°ΠΏΡ€Π°Π²Π»Π΅Π½ΠΈΠΈ, Ρ‚Π°ΠΊ ΠΈ Π² Π½Π°ΠΏΡ€Π°Π²Π»Π΅Π½ΠΈΠΈ Π±ΠΎΠ»Π΅Π΅ Π½ΠΈΠ·ΠΊΠΎΠ³ΠΎ ΠΏΠΎΡ‚Π΅Π½Ρ†ΠΈΠ°Π»Π°. ΠšΡ€ΠΎΠΌΠ΅ Ρ‚ΠΎΠ³ΠΎ, Π²Π΅Π»ΠΈΡ‡ΠΈΠ½Π° Ρ€Π°Π²Π½Π° скорости ΡƒΠΌΠ΅Π½ΡŒΡˆΠ΅Π½ΠΈΡ с расстояниСм. Π§Π΅ΠΌ быстрСС ΡƒΠ±Ρ‹Π²Π°Π΅Ρ‚ с расстояниСм, Ρ‚Π΅ΠΌ большС элСктричСскоС ΠΏΠΎΠ»Π΅.Π’ Ρ„ΠΎΡ€ΠΌΠ΅ уравнСния общая связь ΠΌΠ΅ΠΆΠ΄Ρƒ напряТСниСм ΠΈ элСктричСским ΠΏΠΎΠ»Π΅ΠΌ Ρ€Π°Π²Π½Π°

.

Π³Π΄Π΅ — расстояниС, Π½Π° ΠΊΠΎΡ‚ΠΎΡ€ΠΎΠΌ происходит ΠΈΠ·ΠΌΠ΅Π½Π΅Π½ΠΈΠ΅ ΠΏΠΎΡ‚Π΅Π½Ρ†ΠΈΠ°Π»Π°,,. Π—Π½Π°ΠΊ минус Π³ΠΎΠ²ΠΎΡ€ΠΈΡ‚ Π½Π°ΠΌ, Ρ‡Ρ‚ΠΎ ΡƒΠΊΠ°Π·Ρ‹Π²Π°Π΅Ρ‚ Π² Π½Π°ΠΏΡ€Π°Π²Π»Π΅Π½ΠΈΠΈ ΡƒΠΌΠ΅Π½ΡŒΡˆΠ΅Π½ΠΈΡ ΠΏΠΎΡ‚Π΅Π½Ρ†ΠΈΠ°Π»Π°. ЭлСктричСскоС ΠΏΠΎΠ»Π΅ называСтся Π³Ρ€Π°Π΄ΠΈΠ΅Π½Ρ‚ΠΎΠΌ (ΠΏΠΎ высотС ΠΈΠ»ΠΈ Π½Π°ΠΊΠ»ΠΎΠ½Ρƒ) элСктричСского ΠΏΠΎΡ‚Π΅Π½Ρ†ΠΈΠ°Π»Π°.

Бвязь ΠΌΠ΅ΠΆΠ΄Ρƒ напряТСниСм ΠΈ элСктричСским ΠΏΠΎΠ»Π΅ΠΌ

Π’ Ρ„ΠΎΡ€ΠΌΠ΅ уравнСния общая связь ΠΌΠ΅ΠΆΠ΄Ρƒ напряТСниСм ΠΈ элСктричСским ΠΏΠΎΠ»Π΅ΠΌ Ρ€Π°Π²Π½Π°

.

Π³Π΄Π΅ — расстояниС, Π½Π° ΠΊΠΎΡ‚ΠΎΡ€ΠΎΠΌ происходит ΠΈΠ·ΠΌΠ΅Π½Π΅Π½ΠΈΠ΅ ΠΏΠΎΡ‚Π΅Π½Ρ†ΠΈΠ°Π»Π°,,.Π—Π½Π°ΠΊ минус Π³ΠΎΠ²ΠΎΡ€ΠΈΡ‚ Π½Π°ΠΌ, Ρ‡Ρ‚ΠΎ ΡƒΠΊΠ°Π·Ρ‹Π²Π°Π΅Ρ‚ Π² Π½Π°ΠΏΡ€Π°Π²Π»Π΅Π½ΠΈΠΈ ΡƒΠΌΠ΅Π½ΡŒΡˆΠ΅Π½ΠΈΡ ΠΏΠΎΡ‚Π΅Π½Ρ†ΠΈΠ°Π»Π°. ЭлСктричСскоС ΠΏΠΎΠ»Π΅ называСтся Π³Ρ€Π°Π΄ΠΈΠ΅Π½Ρ‚ΠΎΠΌ (ΠΏΠΎ высотС ΠΈΠ»ΠΈ Π½Π°ΠΊΠ»ΠΎΠ½Ρƒ) элСктричСского ΠΏΠΎΡ‚Π΅Π½Ρ†ΠΈΠ°Π»Π°.

Для Π½Π΅ΠΏΡ€Π΅Ρ€Ρ‹Π²Π½ΠΎ ΠΌΠ΅Π½ΡΡŽΡ‰ΠΈΡ…ΡΡ ΠΏΠΎΡ‚Π΅Π½Ρ†ΠΈΠ°Π»ΠΎΠ², ΠΊΠΎΡ‚ΠΎΡ€Ρ‹Π΅ становятся бСсконСчно ΠΌΠ°Π»Ρ‹ΠΌΠΈ, Π½Π΅ΠΎΠ±Ρ…ΠΎΠ΄ΠΈΠΌΠΎ ΠΈΡΠΏΠΎΠ»ΡŒΠ·ΠΎΠ²Π°Ρ‚ΡŒ Π΄ΠΈΡ„Ρ„Π΅Ρ€Π΅Π½Ρ†ΠΈΠ°Π»ΡŒΠ½ΠΎΠ΅ исчислСниС для опрСдСлСния элСктричСского поля.

ΠšΠΎΠ½Ρ†Π΅ΠΏΡ‚ΡƒΠ°Π»ΡŒΠ½Ρ‹Π΅ вопросы

ΠžΠ±ΡΡƒΠ΄ΠΈΡ‚Π΅, ΠΊΠ°ΠΊ связаны Ρ€Π°Π·Π½ΠΎΡΡ‚ΡŒ ΠΏΠΎΡ‚Π΅Π½Ρ†ΠΈΠ°Π»ΠΎΠ² ΠΈ Π½Π°ΠΏΡ€ΡΠΆΠ΅Π½Π½ΠΎΡΡ‚ΡŒ элСктричСского поля. ΠŸΡ€ΠΈΠ²Π΅Π΄ΠΈΡ‚Π΅ ΠΏΡ€ΠΈΠΌΠ΅Ρ€.

Какова Π½Π°ΠΏΡ€ΡΠΆΠ΅Π½Π½ΠΎΡΡ‚ΡŒ элСктричСского поля Π² области постоянного элСктричСского ΠΏΠΎΡ‚Π΅Π½Ρ†ΠΈΠ°Π»Π°?

Π‘ΡƒΠ΄Π΅Ρ‚ Π»ΠΈ ΠΎΡ‚Ρ€ΠΈΡ†Π°Ρ‚Π΅Π»ΡŒΠ½Ρ‹ΠΉ заряд, ΠΏΠ΅Ρ€Π²ΠΎΠ½Π°Ρ‡Π°Π»ΡŒΠ½ΠΎ находящийся Π² состоянии покоя, Π΄Π²ΠΈΠ³Π°Ρ‚ΡŒΡΡ Π² сторону Π±ΠΎΠ»Π΅Π΅ высокого ΠΈΠ»ΠΈ Π½ΠΈΠ·ΠΊΠΎΠ³ΠΎ ΠΏΠΎΡ‚Π΅Π½Ρ†ΠΈΠ°Π»Π°? ΠžΠ±ΡŠΡΡΠ½ΠΈΡ‚ΡŒ, ΠΏΠΎΡ‡Π΅ΠΌΡƒ.

Π—Π°Π΄Π°Ρ‡ΠΈ ΠΈ упраТнСния

ΠŸΠΎΠΊΠ°ΠΆΠΈΡ‚Π΅, Ρ‡Ρ‚ΠΎ Π΅Π΄ΠΈΠ½ΠΈΡ†Ρ‹ измСрСния напряТСнности элСктричСского поля Π’ / ΠΌ ΠΈ Н / К Π΄Π΅ΠΉΡΡ‚Π²ΠΈΡ‚Π΅Π»ΡŒΠ½ΠΎ эквивалСнтны.

Какова Π½Π°ΠΏΡ€ΡΠΆΠ΅Π½Π½ΠΎΡΡ‚ΡŒ элСктричСского поля ΠΌΠ΅ΠΆΠ΄Ρƒ двумя ΠΏΠ°Ρ€Π°Π»Π»Π΅Π»ΡŒΠ½Ρ‹ΠΌΠΈ проводящими пластинами, Ρ€Π°Π·Π΄Π΅Π»Π΅Π½Π½Ρ‹ΠΌΠΈ Π½Π° 1,00 см ΠΈ ΠΈΠΌΠ΅ΡŽΡ‰ΠΈΠΌΠΈ Ρ€Π°Π·Π½ΠΎΡΡ‚ΡŒ ΠΏΠΎΡ‚Π΅Π½Ρ†ΠΈΠ°Π»ΠΎΠ² (напряТСниС) ΠΌΠ΅ΠΆΠ΄Ρƒ Π½ΠΈΠΌΠΈ?

ΠΠ°ΠΏΡ€ΡΠΆΠ΅Π½Π½ΠΎΡΡ‚ΡŒ элСктричСского поля ΠΌΠ΅ΠΆΠ΄Ρƒ двумя ΠΏΠ°Ρ€Π°Π»Π»Π΅Π»ΡŒΠ½Ρ‹ΠΌΠΈ проводящими пластинами, Ρ€Π°Π·Π΄Π΅Π»Π΅Π½Π½Ρ‹ΠΌΠΈ расстояниСм 4.00 см. Π°) Какова Ρ€Π°Π·Π½ΠΈΡ†Π° ΠΏΠΎΡ‚Π΅Π½Ρ†ΠΈΠ°Π»ΠΎΠ² ΠΌΠ΅ΠΆΠ΄Ρƒ пластинами? (b) БчитаСтся, Ρ‡Ρ‚ΠΎ пластина с самым Π½ΠΈΠ·ΠΊΠΈΠΌ ΠΏΠΎΡ‚Π΅Π½Ρ†ΠΈΠ°Π»ΠΎΠΌ находится ΠΏΠΎΠ΄ Π½ΡƒΠ»Π΅Π²Ρ‹ΠΌ Π²ΠΎΠ»ΡŒΡ‚. Каков ΠΏΠΎΡ‚Π΅Π½Ρ†ΠΈΠ°Π» 1,00 см ΠΎΡ‚ этой пластины (ΠΈ 3,00 см ΠΎΡ‚ Π΄Ρ€ΡƒΠ³ΠΎΠΉ)?

(Π°)

(Π±)

Как Π΄Π°Π»Π΅ΠΊΠΎ Π΄Ρ€ΡƒΠ³ ΠΎΡ‚ Π΄Ρ€ΡƒΠ³Π° находятся Π΄Π²Π΅ проводящиС пластины, ΠΌΠ΅ΠΆΠ΄Ρƒ ΠΊΠΎΡ‚ΠΎΡ€Ρ‹ΠΌΠΈ сущСствуСт Π½Π°ΠΏΡ€ΡΠΆΠ΅Π½Π½ΠΎΡΡ‚ΡŒ элСктричСского поля, Ссли ΠΈΡ… Ρ€Π°Π·Π½ΠΎΡΡ‚ΡŒ ΠΏΠΎΡ‚Π΅Π½Ρ†ΠΈΠ°Π»ΠΎΠ² составляСт 15,0 ΠΊΠ’?

(a) Π‘ΡƒΠ΄Π΅Ρ‚ Π»ΠΈ Π½Π°ΠΏΡ€ΡΠΆΠ΅Π½Π½ΠΎΡΡ‚ΡŒ элСктричСского поля ΠΌΠ΅ΠΆΠ΄Ρƒ двумя ΠΏΠ°Ρ€Π°Π»Π»Π΅Π»ΡŒΠ½Ρ‹ΠΌΠΈ проводящими пластинами ΠΏΡ€Π΅Π²Ρ‹ΡˆΠ°Ρ‚ΡŒ ΠΏΡ€Π΅Π΄Π΅Π» прочности для Π²ΠΎΠ·Π΄ΡƒΡ…Π° (), Ссли пластины Ρ€Π°Π·Π΄Π΅Π»Π΅Π½Ρ‹ Π½Π° 2.00 ΠΌΠΌ ΠΈ ΠΏΡ€ΠΈΠ»ΠΎΠΆΠ΅Π½Π° Ρ€Π°Π·Π½ΠΎΡΡ‚ΡŒ ΠΏΠΎΡ‚Π΅Π½Ρ†ΠΈΠ°Π»ΠΎΠ²? (Π±) Насколько Π±Π»ΠΈΠ·ΠΊΠΎ Π΄Ρ€ΡƒΠ³ ΠΊ Π΄Ρ€ΡƒΠ³Ρƒ ΠΌΠΎΠ³ΡƒΡ‚ Π±Ρ‹Ρ‚ΡŒ пластины ΠΏΡ€ΠΈ ΠΏΡ€ΠΈΠ»ΠΎΠΆΠ΅Π½Π½ΠΎΠΌ напряТСнии?

(Π°) НСт. ΠΠ°ΠΏΡ€ΡΠΆΠ΅Π½Π½ΠΎΡΡ‚ΡŒ элСктричСского поля ΠΌΠ΅ΠΆΠ΄Ρƒ пластинами Π½ΠΈΠΆΠ΅, Ρ‡Π΅ΠΌ пробивная ΠΏΡ€ΠΎΡ‡Π½ΠΎΡΡ‚ΡŒ Π²ΠΎΠ·Π΄ΡƒΡ…Π° ().

(Π±) 1,7 ΠΌΠΌ

НапряТСниС Π½Π° ΠΌΠ΅ΠΌΠ±Ρ€Π°Π½Π΅, ΠΎΠ±Ρ€Π°Π·ΡƒΡŽΡ‰Π΅ΠΉ ΠΊΠ»Π΅Ρ‚ΠΎΡ‡Π½ΡƒΡŽ стСнку, составляСт 80,0 ΠΌΠ’, Π° Ρ‚ΠΎΠ»Ρ‰ΠΈΠ½Π° ΠΌΠ΅ΠΌΠ±Ρ€Π°Π½Ρ‹ составляСт 9,00 Π½ΠΌ. Какая Π½Π°ΠΏΡ€ΡΠΆΠ΅Π½Π½ΠΎΡΡ‚ΡŒ элСктричСского поля? (Π—Π½Π°Ρ‡Π΅Π½ΠΈΠ΅ Π½Π° ΡƒΠ΄ΠΈΠ²Π»Π΅Π½ΠΈΠ΅ Π²Π΅Π»ΠΈΠΊΠΎ, Π½ΠΎ Π²Π΅Ρ€Π½ΠΎ. ΠœΠ΅ΠΌΠ±Ρ€Π°Π½Ρ‹ ΠΎΠ±ΡΡƒΠΆΠ΄Π°ΡŽΡ‚ΡΡ Π² Ρ€Π°Π·Π΄Π΅Π»Π΅ ΠšΠΎΠ½Π΄Π΅Π½ΡΠ°Ρ‚ΠΎΡ€Ρ‹, диэлСктрики ΠΈ нСрвная ΠΏΡ€ΠΎΠ²ΠΎΠ΄ΠΈΠΌΠΎΡΡ‚ΡŒ — Π­Π»Π΅ΠΊΡ‚Ρ€ΠΎΠΊΠ°Ρ€Π΄ΠΈΠΎΠ³Ρ€Π°ΠΌΠΌΡ‹.) МоТно ΡΡ‡ΠΈΡ‚Π°Ρ‚ΡŒ, Ρ‡Ρ‚ΠΎ элСктричСскоС ΠΏΠΎΠ»Π΅ ΠΎΠ΄Π½ΠΎΡ€ΠΎΠ΄Π½ΠΎΠ΅.

ΠœΠ΅ΠΌΠ±Ρ€Π°Π½Π½Ρ‹Π΅ стСнки ΠΆΠΈΠ²Ρ‹Ρ… ΠΊΠ»Π΅Ρ‚ΠΎΠΊ ΠΈΠΌΠ΅ΡŽΡ‚ Π½Π° сСбС ΡƒΠ΄ΠΈΠ²ΠΈΡ‚Π΅Π»ΡŒΠ½ΠΎ большиС элСктричСскиС поля ΠΈΠ·-Π·Π° раздСлСния ΠΈΠΎΠ½ΠΎΠ². (ΠœΠ΅ΠΌΠ±Ρ€Π°Π½Ρ‹ Π±ΠΎΠ»Π΅Π΅ ΠΏΠΎΠ΄Ρ€ΠΎΠ±Π½ΠΎ ΠΎΠ±ΡΡƒΠΆΠ΄Π°ΡŽΡ‚ΡΡ Π² Ρ€Π°Π·Π΄Π΅Π»Π΅ «НСрвная ΠΏΡ€ΠΎΠ²ΠΎΠ΄ΠΈΠΌΠΎΡΡ‚ΡŒ — элСктрокардиограммы».) Каково напряТСниС Π½Π° ΠΌΠ΅ΠΌΠ±Ρ€Π°Π½Π΅ Ρ‚ΠΎΠ»Ρ‰ΠΈΠ½ΠΎΠΉ 8,00 Π½ΠΌ, Ссли Π½Π°ΠΏΡ€ΡΠΆΠ΅Π½Π½ΠΎΡΡ‚ΡŒ элСктричСского поля Π½Π° Π½Π΅ΠΉ составляСт 5,50 ΠœΠ’ / ΠΌ? Π’Ρ‹ ΠΌΠΎΠΆΠ΅Ρ‚Π΅ ΠΏΡ€Π΅Π΄ΠΏΠΎΠ»ΠΎΠΆΠΈΡ‚ΡŒ ΠΎΠ΄Π½ΠΎΡ€ΠΎΠ΄Π½ΠΎΠ΅ элСктричСскоС ΠΏΠΎΠ»Π΅.

Π”Π²Π΅ ΠΏΠ°Ρ€Π°Π»Π»Π΅Π»ΡŒΠ½Ρ‹Π΅ проводящиС пластины Ρ€Π°Π·Π΄Π΅Π»Π΅Π½Ρ‹ расстояниСм 10,0 см, ΠΏΡ€ΠΈ этом ΠΎΠ΄Π½Π° ΠΈΠ· Π½ΠΈΡ… ΠΏΡ€ΠΈΠ½ΠΈΠΌΠ°Π΅Ρ‚ Π½ΡƒΠ»Π΅Π²ΠΎΠ΅ напряТСниС.(Π°) Какова Π½Π°ΠΏΡ€ΡΠΆΠ΅Π½Π½ΠΎΡΡ‚ΡŒ элСктричСского поля ΠΌΠ΅ΠΆΠ΄Ρƒ Π½ΠΈΠΌΠΈ, Ссли ΠΏΠΎΡ‚Π΅Π½Ρ†ΠΈΠ°Π» 8,00 см ΠΎΡ‚ Π½ΡƒΠ»Π΅Π²ΠΎΠΉ пластины (ΠΈ 2,00 см ΠΎΡ‚ Π΄Ρ€ΡƒΠ³ΠΎΠΉ) составляСт 450 Π’? Π±) КакоС напряТСниС ΠΌΠ΅ΠΆΠ΄Ρƒ пластинами?

НайдитС ΠΌΠ°ΠΊΡΠΈΠΌΠ°Π»ΡŒΠ½ΡƒΡŽ Ρ€Π°Π·Π½ΠΎΡΡ‚ΡŒ ΠΏΠΎΡ‚Π΅Π½Ρ†ΠΈΠ°Π»ΠΎΠ² ΠΌΠ΅ΠΆΠ΄Ρƒ двумя ΠΏΠ°Ρ€Π°Π»Π»Π΅Π»ΡŒΠ½Ρ‹ΠΌΠΈ проводящими пластинами, Ρ€Π°Π·Π΄Π΅Π»Π΅Π½Π½Ρ‹ΠΌΠΈ Π½Π° 0,500 см Π²ΠΎΠ·Π΄ΡƒΡ…Π°, с ΡƒΡ‡Π΅Ρ‚ΠΎΠΌ максимально допустимой напряТСнности элСктричСского поля Π² Π²ΠΎΠ·Π΄ΡƒΡ…Π΅.

Двухзарядный ΠΈΠΎΠ½ ускоряСтся Π΄ΠΎ энСргии 32,0 кэВ элСктричСским ΠΏΠΎΠ»Π΅ΠΌ ΠΌΠ΅ΠΆΠ΄Ρƒ двумя ΠΏΠ°Ρ€Π°Π»Π»Π΅Π»ΡŒΠ½Ρ‹ΠΌΠΈ проводящими пластинами, Ρ€Π°Π·Π΄Π΅Π»Π΅Π½Π½Ρ‹ΠΌΠΈ расстояниСм 2.00 см. Какая Π½Π°ΠΏΡ€ΡΠΆΠ΅Π½Π½ΠΎΡΡ‚ΡŒ элСктричСского поля ΠΌΠ΅ΠΆΠ΄Ρƒ пластинами?

Π­Π»Π΅ΠΊΡ‚Ρ€ΠΎΠ½ Π΄ΠΎΠ»ΠΆΠ΅Π½ Π±Ρ‹Ρ‚ΡŒ ускорСн Π² ΠΎΠ΄Π½ΠΎΡ€ΠΎΠ΄Π½ΠΎΠΌ элСктричСском ΠΏΠΎΠ»Π΅ Π½Π°ΠΏΡ€ΡΠΆΠ΅Π½Π½ΠΎΡΡ‚ΡŒΡŽ. (Π°) Какая энСргия Π² кэВ пСрСдаСтся элСктрону, Ссли ΠΎΠ½ ускоряСтся Π½Π° 0,400 ΠΌ? (Π±) На ΠΊΠ°ΠΊΠΎΠ΅ расстояниС Π΅Π³ΠΎ Π½ΡƒΠΆΠ½ΠΎ Π±ΡƒΠ΄Π΅Ρ‚ ΡƒΡΠΊΠΎΡ€ΠΈΡ‚ΡŒ, Ρ‡Ρ‚ΠΎΠ±Ρ‹ ΡƒΠ²Π΅Π»ΠΈΡ‡ΠΈΡ‚ΡŒ Π΅Π³ΠΎ ΡΠ½Π΅Ρ€Π³ΠΈΡŽ Π½Π° 50,0 ГэВ?

(Π°)

(Π±)

Глоссарий

скаляр
физичСская Π²Π΅Π»ΠΈΡ‡ΠΈΠ½Π° с Π²Π΅Π»ΠΈΡ‡ΠΈΠ½ΠΎΠΉ, Π½ΠΎ Π±Π΅Π· направлСния
Π²Π΅ΠΊΡ‚ΠΎΡ€
физичСская Π²Π΅Π»ΠΈΡ‡ΠΈΠ½Π° с Π²Π΅Π»ΠΈΡ‡ΠΈΠ½ΠΎΠΉ ΠΈ Π½Π°ΠΏΡ€Π°Π²Π»Π΅Π½ΠΈΠ΅ΠΌ
.

alexxlab

Π”ΠΎΠ±Π°Π²ΠΈΡ‚ΡŒ ΠΊΠΎΠΌΠΌΠ΅Π½Ρ‚Π°Ρ€ΠΈΠΉ

Π’Π°Ρˆ адрСс email Π½Π΅ Π±ΡƒΠ΄Π΅Ρ‚ ΠΎΠΏΡƒΠ±Π»ΠΈΠΊΠΎΠ²Π°Π½. ΠžΠ±ΡΠ·Π°Ρ‚Π΅Π»ΡŒΠ½Ρ‹Π΅ поля ΠΏΠΎΠΌΠ΅Ρ‡Π΅Π½Ρ‹ *